Sunteți pe pagina 1din 276

RELEASED EXAMINATION

ANATOMIC SCIENCES (11)


BIOCHEMISTRY-PHYSIOLOGY (12)

PART l
MONDAY - A.M.
DECEMBER 1998

TEST BOOKLET

DO NOT OPEN TEST BOOKLET UNTIL INSTRUCTED TO BEGIN.

1.

In addition to this test booklet, you should have an answer sheet.

2.

Use a No. 2 pencil on all test materials.

3.

Enter your name and reference number below.

Name
Last

First

Middle

Reference Number

4.

Enter your name and reference number on the side of the answer sheet numbered 1 to 100 (SIDE 1).
Blacken the circle that corresponds to each digit of your reference number.

5. Enter the name of this test (ANATOMIC SCIENCES) and the number of this test (11) on the answer sheet.
Blacken the circles that correspond to the test number.

6. Check to be sure you have completed each step above.


YOU ARE READY TO BEGIN. For each test item, decide which choice is correct and blacken the corresponding
circle on the answer sheet. Record only one answer for each test item; there is no penalty for selecting an
incorrect response. You may write in the test booklet; however, your score is based on the total number of
correct answers recorded on your answer sheet. You are allowed 3 112 hours to complete this test booklet.
All test materials must be returned before or immediately upon the request of the Test Administrator. Use of a
magnifjing glass is permitted. Use of other study aids is not permitted. No test items are to be copied or notes
taken. Infraction of these rules will result in your scores being voided.

COPYRIGHT 1998
JOINT COMMISSION ON NATIONAL DENTAL EXAMINATIONS
AMERICAN DENTAL ASSOCIATION
211 EAST CHICAGO AVENUE, CHICAGO, ILLINOIS 60611

1.

Which of the following does a human normally


possess before birth but not after?
A.
B.
C.
D.
E.

7.

A.
B.
C.
D.

Fossa ovalis
Hepatic vein
Ligamentum teres
Ductus arteriosus
Crista terminalis

E.

8.

2.

The core of a microvillus consists of which of the


following?
A.
B.
C.
D.
E.

3.

A.
B.
C.
D.
E.

4.

Rough endoplasmic reticulum


Membrane bound vesicles
Sarcoplasmic reticulum
Mitochondrion
Golgi complex

In adults, the thyroid gland's point of origin is


seen as the

E.

9.

5.

Which of the following triangles is formed by the


superior belly of the omohyoid muscle, the
anterior border of the sternocleidomastoid
muscle, and the posterior belly of the digastric
muscle?
A.
B.
C.
D.
E.

6.

copula.
tuberculum impar.
sulcus terminalis.
palatine tonsil.
foramen cecum.

Superclavicular
Submandibular
Digastric
Muscular
Carotid

Sensations from the left face and teeth are


interpreted in which of the following lobes?
A.
B.
C.
D.
E.

Left frontal
Right frontal
Left parietal
Right parietal
Right temporal

occipital somites.
first pharyngeal (branchial) arch.
second pharyngeal (branchial) arch.
third pharyngeal (branchial) arch.
fourth pharyngeal (branchial) arch.

After a right side radical mastoid operation, a


patient has facial distortion and the mouth is
drawn upward to the left. He is unable to close
his right eye. Saliva tends to accumulate in his
right cheek and dribble out of the corner of his
mouth. Which of the following nerves MOST
likely was damaged during the operation?
A.
B.
C.
D.

E.
A.
B.
C.
D.
E.

colon.
jejunum.
appendix.
duodenum.
fundus of the stomach.

The sensory innervation to the posterior third of


the tongue reflects the embryonic origin of its
covering epithelium from the
A.
B.
C.
D.

9 + 2 arrangement of microtubules
9 triplets of microtubules
Intermediate filaments
Vimentin filaments
Microfilaments

Which of the following organelles is the site of


protein synthesis?

Submucosal glands are usually located in the

Vagus
Facial
Trigeminal
Hypoglossal
Glossopharyngeal

10. The blood-testis barrier is maintained by


A.
B.
C.
D.
E.

spermatogonia.
spermatocytes.
Sertoli cells.
Leydig cells.
myoepithelial cells.

11. A patient's pain from the temporomandibular


joint was referred to the skin over the parotid
region and to the side of the head. This referral
pattern is based on the distribution of which of
the following nerves?
A.
B.
C.
D.
E.

Auriculotemporal
Greater auricular
Temporal branch of the facial
Superior alveolar
Inferior alveolar

12. The temporomandibularjoint capsule is supplied


by several nerves. Two of these are the
auriculotemporal and the
A.
B.
C.
D.
E.

facial.
buccal.
maxillary.
masseteric.
inferior alveolar.

13. If the needle is advanced too far posteriorly


during an inferior alveolar block injection,
anesthesia of the mandibular teeth will NOT
occur, because the needle has entered the
A.
B.
C.
D,
E.

parotid gland.
pterygoid space.
inferior alveolar canal.
medial pterygoid muscle.
superior pharyngeal constrictor muscle.

17. Infections or neoplasms that spread by


lymphatics from the skin of the angle of the
mouth MOST likely pass to which of the
following lymph nodes?
A.
B.
C.
D.
E.

Lingual
Submental
Submandibular
Anterior cervical
Superficial cervical

18. Preganglionic sympathetic fibers to the head


have their cell bodies in the
A.
B.
C.
D.
E.

paravertebral ganglia.
cervicothoracic ganglia.
superior cervical ganglia.
gray rami communicantes of the thoracic
region.
intermediolateral horns of the thoracic
spinal cord.

14. Meiosis occurs in


A.

ductus epididymis.

8. uriniferous tubules.
C.
D.
E.

seminiferous tubules.
stratum germinativum.
germinal epithelium of ovary.

15. The cell bodies of proprioceptive nerves carrying

19. In the posterior mediastinum, the thoracic duct


usually lies
A.
B.
C.

anterior to the phrenic nerves.


posterior to the esophagus.
on the anterolateral surface of trachea.

information from the periodontal ligaments are


located in the
A.
B.
C.
D.
E.

nucleus ambiguus.
trigeminal ganglion.
spinal nucleus of V.
main sensory nucleus of V.
mesencephalic nucleus of V.

20. Which of the following veins is formed by the


union of the internal jugular and subclavian
veins?
A.
B.
C.
D.
E.

Superior vena cava


Brachiocephalic
Pulmonary
Cephalic
Azygos

16. Which of the following are two muscles that


prevent food from entering the nasopharynx
during swallowing?
A.
B.
C.
D.
E.

Mylohyoid and the palatoglossus


Styloglossus and the palatoglossus
Tensor tympani and the stylopharyngeus
Tensor veli palatini and the levator veli
palatini
Musculus uvulae and the inferior
pharyngeal constrictor

21. Which of the following is the MOST numerous


cell type in the normal dental pulp?
A.
B.
C.
D.
E.

Adipocyte
Mast cell
Fibroblast
Macrophage
Odontoblast

22. The middle meningeal artery enters the cranial

28. The hepatic triad consists of which of the

vault through the

following?

A.

A.
B.
C.
D.

3.

C.
D.
E.

foramen ovate.
foramen lacerum.
foramen rotundum.
foramen spinosum.
superior orbital fissure.

E.

Central vein, hepatic vein, and bile duct


Portal vein, bile duct, and hepatic artery
Portal vein, bile canaliculi, and hepatic vein
Central vein, bile canaliculi, and hepatic
artery
Hepatic artery, hepatic vein, and bile duct

23. Which of the following structures is


retroperitoneal?
A.
B.
C.
D.
E.

Spleen
Kidney
Stomach
Sigmoid colon
Transverse colon

24. The hamular process is an extension of the

29. Which of the following represents a popular


theory for the force that is considered to be the
MOST responsible for active tooth eruption?
A.
B.
C.

A.
B.
C.
D.
E.

ethmoid bone.
palatine bone.
maxillary tuberosity.
medial pterygoid plate.
lateral pterygoid plate.

D.
E.

Cells and fibers in the developing PDL pull


the tooth toward the oral cavity.
Dentin formation in the root pushes the
tooth into the oral cavity.
Vascular pressure within the pulp pushes
the tooth intraorally.
Apical fundal bone formation pushes the
tooth.
Crestal bone formation pulls the tooth.

25. The muscularis externa of the esophagus in the


lower one third consists of
A.
B.
C.
D.

mostly skeletal muscle, but some smooth


muscle.
equal amounts of smooth and skeletal
muscles.
smooth muscle only.
skeletal muscle only.

30. Which of the following ligaments helps resist


posterior movement of the mandibular
condyles?
A.
3.
C.
D.
E.

Pterygomandibular raphe
Temporomandibular
Anterior cruciate
Sphenomandibular
Stylomandibular

26. The dermis may be classifed as which of the


following types of connective tissue?
A.
B.
C.
D.
E.

Adipose
Modified elastic
Reticular connective
Dense regular collagenous connective
Dense irregular collagenous connective

31. The MOST commonly accepted theory for pain


reception in the dental pulp suggests that
A.

B.

27. Which of the following structures travels just


posterior to the arch of the azygos vein?
A.
B.
C.
D.
E.

Hemiazygos vein
Right vagus nerve
Right phrenic nerve
Right lymphatic duct
Greater splanchnic nerve

C.

D.
E.

nerves in the dentinal tubules have


receptors for pain.
odontoblastic processes serve as the only
receptors for pain.
there are numerous Meissner's corpuscles
in the pulp that respond to pain stimuli.
nerve endings in the periodontal ligament
enter the pulp through accessory canals to
provide pain reception.
hydrodynamic phenomena involving fluid
flux in the tubules result in stimuli that
activate pain receptors within the pulp.

32. In a healthy temporomandibularjoint the


articular surfaces are covered with which of the
following?
A.
B.
C.
D.
E.

Fat
Bone
Hyaline cartilage
Synovial membrane
Dense fibrous connective tissue

38. Oxyphil cells are present in which of the


following?
A.
B.
C.
D.
E.

Thymus
Spleen
Fundic stomach
Neurohypophysis
Parathyroid gland

39. Which of the following represents the MOST


33. The hyoid bone is attached by muscles or
ligaments to the

vascular portion of the articular disc of the


temporomandibular joint?

A.
B.
C.
D.

A.
B.
C.
D.

palatine bone and the soft palate.


epiglottis and the cricoid cartilage.
mandible, the first rib, and the clavicle.
mandible, the styloid process, and the
tongue.

Anterior thick zone (band)


Posterior thick zone (band)
Intermediate thin zone (band)
Bilaminar zone

40. The spleen, the thymus, and lymph nodes are


34. The pterygoid plexus is usually formed by veins

similar in that each

that are tributaries of which of the following?


A.
B.
C.
D.
E.

Maxillary vein
Angular vein
Facial vein
Occipital vein
Anterior jugular vein

35. Which of the following represents the primary


supinator at the radioulnar joint?
A.

6.
C.
D.
E.

Supinator
Brachialis
Biceps brachii
Brachioradialis
Coracobrachialis

36. Which of the following veins are ldcated within

A.
6.
C.
D.
E.

filters blood.
contains lymphocytes.
has a medulla and a cortex.
serves as a filter for tissue fluid.
has afferent and efferent lymphatic
vessels.

41. Which of the following represents the MOST


common cartilage found in the embryo and in
the adult?
A.
6.
C.
D.

Fibrocartilage
Hyaline cartilage
Elastic cartilage
Calcified cartilage

42. The ossicles of the ear are located in the

the bones of the calvarium?


A.
B.
C.
D.
E.

Carotid
Diploic
Meningeal
lnfrahyoid
Subarachnoid

A.
B.
C.
D.
E.

utricle.
labyrinth.
inner ear.
middle ear.
external auditory meatus.

43. A terminal branch of the mandibular nerve


emerges through which of the following
foramina?

37. Loss of the gag reflex suggests damage to


which of the following cranial nerves?
A.
B.
C.
D.

V
VII
IX
XI1

A.

6.
C.
D.
E.

Supraorbital
lnfraorbital
Stylomastoid
Rotundum
Mental

44. Bilateral contraction of which of the following

50. The thinnest portion of the bony orbit is located

muscles acts to protrude the mandible?

in the

A.
B.
C.
D.
E.

A.
6.
C.
D.
E.

Lateral pterygoid
Digastric
Buccinator
Temporalis
Mylohyoid

45. Which of the following organs is the NEAREST

roof.
floor.
medial wall.
lateral wall.
posterior wall.

51. The nerve of the pterygoid canal contains which

to the right kidney?

of the following fibers?

A.
B.
C.
D
E.

A.
B.
C.
D.
E.

Colon
Spleen
Stomach
Pancreas
Duodenum

Taste and vision


Pressure and touch sensation
Pain and temperature sensation
Sympathetic and parasympathetic
Proprioception and touch sensation

46. Which of the following connective tissue types is

52. If there is an organ that needs to stretch and

primarily found in the fetus, but not in the adult?

return to its original shape, then which of the


following epithelia would MOST likely be
associated with it?

A.
6.
C.
D.
E.

Loose
Mesenchyme
Areolar
Dense regular
Dense irregular

47. Antidiuretic hormone is PRODUCED by cells


that reside in which of the following structures,
cells or areas?
A.
6.
C.
D.
E.

Pars nervosa
Pars intermedia
Basophils of the pars distalis
Acidophils of the pars distalis
Supraoptic nucleus of the hypothalamus

48. Which of the following structures directly


develops from the cells remaining in the
remnants of the preovulatory follicle after
ovulation?
A.
B.
C.
D.
E.

Theca interna
Corpus luteum
Corpus albicans
Atretic follicle
Interstitial gland

49. The heart is contained in which mediastinum?


A.
B.
C.
D.

Middle
Superior
Anterior
Posterior

A.
B.
C.
D.
E.

Transitional
Simple cuboidal
Simple columnar
Simple squamous
Pseudostratified columnar

53. Each of the following laminae help to form a


tooth EXCEPT one. Which one is this
EXCEPTION?
A.
B.
C.

Vestibular
Successional
Dental

54. Which of the following anterior primary rami form


the medial cord of the brachial plexus?
A.
B.
C.
D.
E.

C-4 and C-5


C-6 and C-7
C-5, C-6, and C-7
C-8 and T-1
T-2 and T-3

55. The only purely serous minor salivary gland is


located in the lamina propria of the
A.
B.
C.
D.
E.

tongue.
hard palate.
soft palate.
buccal mucosa.
attached gingiva.

56. The maxillary artery is MOST often a branch of

61. Which of the following cells forms a Howship's

which of the following arteries?

lacuna?

A.
B.
C.
D.
E.

A.
B.
C.
D.
E.

Facial
External carotid
Internal carotid
Common carotid
Superficial temporal

57. Which of the following structures runs directly


adjacent to, and parallel with, the median nerve
in the arm?
A.
B.
C.
D.
E.

Ulnar artery
Radial artery
Cephalic vein
Brachial artery
Medial brachial cutaneous nerve

Osteocyte
Osteoblast
Osteoclast
Chondrocyte
Chondroblast

62. Which portion of dentin is the MOST highly


mineralized?
A.
B.
C.
D.

lntratubular (peritubular)
Intertubular
lnterglobular
Mantle

63. Which of the following is the posterior limit of the


vestibular side of the cheek?

58. Postganglionic fibers from the superior cervical


ganglion innervate which of the following
muscles?
A.
B.
C.
D.
E.

Ciliary
Inferior oblique
Dilator pupillae
Sphincter pupillae
Levator palpebre superioris

59. Connective tissue proper is characterized as


having
A.
B.
C.
D.
E.

poor vascularization.
poor reparative ability.
sensitivity as its main function.
more intercellular material than cells.
mast cells as the predominant cell type.

Which of the following represents a segment of


the bronchial tree that is less than 1 mm in
diameter and lined by simple columnar ciliated
epithelium with no cartilage plates in the wall?
A.
B.
C.
D.
E.

Bronchiole
Alveolus
Bronchus
Alveolar sac
Bronchial septum

A.
B.
C.
D.
E.

Retromolar pad
Sulcus terminalis
Pterygomandibular raphe
Stylomandibular ligament
Sphenomandibular ligament

64. When a developing carious lesion reaches the


dentinoenamel junction, this will often initiate the
formation of which of the following at the pulp
interface of the dentin?
A.
B.
C.
D.
E.

Mantle dentin
Primary dentin
False pulp stones
lnterglobular dentin
Tertiary (reparative) dentin

65. Which nerve enters the pharynx in the gap


between the superior and middle constrictors?
A.
B.
C.
D.
E.

Lingual
Hypoglossal
Glossopharyngeal
Inferior laryngeal
Superior laryngeal

66. Certain nerve trunks are composed of several


fascicles. Which of the following entirely
encloses these trunks?
A.
B.
C.

Epineurium
Endoneurium
Perineurium

67. Pain in the temporomandibular joint would be


perceived by endings of the
A.

B.
C.
D.
E.

auriculotemporal nerve in the capsule and


periphery of the disc.
lateral pterygoid nerve in the capsule, disc
and retrodiscal pad.
auriculotemporal nerve throughout the
meniscus.
maxillary branch of the trigeminal in the
retrodiscal pad and capsule.
inferior alveolar nerve.

68. Which of the following lymphoid aggregations is


(are) the MOST likely to be associated with
pseudostratified columnar ciliated epithelium?
A.
B.
C.
D.

Palatine tonsil
Lingual tonsil
Pharyngeal tonsil
Peyer's patches

72. Which of the following represents the main


arterial supply to the brain?
A.
B.
C.
D.

73. Which of the following is the nerve that supplies


the mimetic muscles?
A.
B.
C.
D.
E.

associated with increasing age?


A.
B.
C.

E.

provides the GREATEST resistance to


substances attempting to move between cells?
A.
B.
C.
D.
E.

Gap junction
Zonula occludens
Zonula adherens
Macula adherens
Fascia adherens

70. How is a large vein different from its companion


artery? The vein has
A.
B.
C.
D.
E.

a smaller lumen.
a less extensive vasa vasorum.
a more prominent tunica media.
an indistinct endothelial lining.
a more prominent tunica adventitia.

A.
B.
C.
D.

E.

Corpus callosum
Internal capsule
Medial lemniscus
Lateral lemniscus
Anterior commissure

pain fibers.
sensor fibers.
upper motor neuron fibers only.
lower motor neuron fibers only.
upper and lower motor neuron fibers.

76. The prochordal plate consists of

B.

cortex are found in which of the following?


A.
B.
C.
D.
E.

Increased cellularity
Increased collagenous components
Development of more extensive
subodontoblastic nerve plexus
Development of more extensive
subodontoblastic capillary plexus
Development of more extensive lymphatic
plexus

75. The pyramids contain

A.

71. Fiber tracts passing from the thalamus to the

Vagus
Facial
Trigeminal
Hypoglossal
Spinal accessory

74. Which of the following is a pulpal change

D.

69. Which of the following intercellular junctions

Internaljugular
lnternal carotid
External carotid
Middle meningeal

C.
D.
E.

embryonic endoderm, mesoderm, and


ectoderm.
a circular area anterior to the notochord in
which the endoderm is fused to the
embryonic mesoderm.
the cloaca1 membrane and the overlying
amnion.
endoderm of the roof of the yolk sac and
embryonic ectoderm.
the cardiogenic mesoderm and the
neurectoderm.

77. Which of the following structures are found in

82. Accessory root canals can be formed by

the infratemporalfossa?
A.
A.

B.
C.
D.

Temporalis muscle and parotid gland


Masseter and lateral pterygoid muscles
Superficial temporal artery and parotid
lymphatic nodes
Mandibular division of V and chorda
tympani branch of VII

78. Which of the following glands is purely mucous?


A.
B.
C.
D.
E.

Parotid
Palatine
Sublingual
von Ebnet's
Submandibular

B.
C.
D.

a break in the epithelial diaphragm prior to


dentin formation.
a break in the epithelial root sheath
(Hertwig) prior to dentin formation.
adherence of the epithelial root sheath
(Hertwig) to the dentinal surface.
an epithelial rest that lies in contact with
the dentin of the root.

83. In the mid-palatal raphe region of the hard


palate, which of the following represents the
layer of the epithelium of the oral mucosa which
is deep to the stratum spinosum?
A.
B.
C.
D.
E.

Stratum lucidum
Stratum basale
Lamina propria
Stratum granulosum
Stratum corneum

79. A patient bites the tip of his tongue. The pain


that ensues is carried by way of which of the
following cranial nerves?
A.
B.
C.
D.
E.

Vagus, X
Facial, VII
Trigeminal, V
Hypoglossal, XI1
Glossopharyngeal, IX

84. Information from the hypothalamus is carried to


the anterior pituitary (pars distalis) by the
A.
B.
C.
D.
E.

gamma efferent system.


hypothalamic radiation.
hypothalamo-hypophyseal tract.
hypothalamic association fibers.
hypothalamo-hypophyseal portal system.

80. To expose the submandibular duct by an


intraoral approach, one must cut through which
of the following?
A.
B.
C.
D.

E.

Mucous membrane only


Mucous membrane and the genioglossus
muscle
Mucous membrane and the mylohyoid
muscle
Mucous membrane and the geniohyoid
muscle
Mucous membrane and the anterior
digastric muscle

85. On its way to the brain, the vertebral artery


passes through which of the following?
A.
B.
C.

D.

Foramen magnum
Foramen lacerum
Jugular foramen
Foramen spinosum

86. In addition to the common carotid arteries, which


81. The submandibular ganglion is associated with
which of the following nerves?

of the following blood vessels provides


circulation to the brain?

A.
B.
C.
D.
E.

A.
B.
C.
D.
E.

Inferior alveolar
Glossopharyngeal
Maxillary
Mylohyoid
Lingual

Vertebral artery
Thyrocervical trunk
Costacervical trunk
Superior thyroid artery
Internal thoracic artery

87. On a comparative basis, bone and cellular


cementum are similar in that both
A.

B.
C.

D.

are difficult to resorb, heal well, and


contain cement lines.
contain concentric patterns, have
Volkmann's canals, and can be cancellous.
are deposited throughout life, acquire
blood vessels during aging, and contain
similar collagenous fibers.
contain cells in lacunae with canaliculi that
extend primarily toward the nutritional
source.

88. Which of the following cranial nerves supplies


the muscles derived from the first pair of
branchial arches?
A.
B.
C.
D.
E.

Vagus
Facial
Trigeminal
Hypoglossal
Glossopharyngeal

92. Which of the following bones is formed primarily


by intramembranous ossification?
A.
B.
C.
D.
E.

93. Gnarled enamel is MOSTfrequently found


A.
B.
C.
D.
E.

of

B.

A.
B.
C.
D.
E.

pons.
midbrain.
diencephalon.
medulla oblongata.
cerebral peduncles.

90. Which of the following structures may be found

in cusps.
near the cervical line.
around pits and fissures.
adjacent to contact areas.
equally on all surfaces of the crown.

94. An oblique facial cleft follows the li


A.

89. Cranial nerve V emerges from the

Femur
Tibia
Stapes
Humerus
Mandible

C.
D
E.

ry processes.
lateral nasal an
medial nasal
axillary processes.
al nasal processes.
medial an
maxJfk?&nd
mandibular processes.
asal and mandibular processes.

&Y""-

11%

95. A patient bleeds from the anterior septa1 region


of his nose. A break in the vessel walls has
caused this bleeding. These vessels originate
from which of the following arteries?

within the posterior mediastinum?


A.
A.
B.
C.
D.
E.

Heart
Phrenic nerve
Thoracic duct
Arch of the aorta
Recurrent laryngeal nerve

B.
C.

D.

Angular
Posterior superior alveolar
Sphenopalatine
Greater palatine

96. Maturation of enamel is characterized by a

91. Which group of fibers of the periodontal ligament


offer the MOST resistance to movement of the
tooth in an apical direction?
A.
B.
C.
D.
E.

Alveolar crest
lnterradicular
Horizontal
Oblique
Apical

percentage increase in inorganic content and a


percentage
A.

B.
C.

D.
E.

increase in water.
increase in organic content.
decrease in water and decrease in organic
content.
decrease in water and increase in organic
content.
increase in water and decrease in organic
content.

97. Para-keratinized oral mucosa is often found on


which of the following?
A.

Oral surface of the soft palate


Ventral surface of the tongue
Skin surface of the lips
Floor of the mouth
Attached gingiva

100. In the photomicrograph below of a ground


section of a tooth viewed by transmitted light,
some dentinal tubules appear black because
they

98. Which of the following represents the major


fibrillar component of mature dentin?
A.
B.
C.
D.
E.

Elastin
Reticulin
Type I collagen
Type II collagen
Type Ill collagen

99. Each of the following terms applies to a


discussion of enamel structure EXCEPT one.
Which one is this EXCEPTION?
A.
B.
C.
D.
E.

Prisms
Perikymata
Striae of Retzius
Contour lines of Owen
lnterprismatic substance

A.
B.
C.
D.
E.

are filled with blood.


are filled with minerals.
are filled with bacteria.
have become filled with air during
sectioning.
contain deeply stained odontoblastic
processes.

NATIONAL BOARD DENTAL EXAMINATIONS


PART 1
TEST: ANATOMIC SCIENCES
FORM: 11 (All)
DATE: D98

Item

Key,

Item

Key

Item

Key

Item

Key

1
2
3
4
5

D
E

26
27
28
29
30

E
B
B
A
B

51
52
53
54
55

D
A
A
D
A

76
77
78
79
80

D
D
B
C
A

E
E
D
A

56
57
58
59
60

B
D

81
82
83
84
85

E
B

E
E

6
7
8
9
10

D
D
D
B
C

31
32
33
34
35

11
12
13
14
15

A
D
A
C
E

36
37
38
39
40

E
D
B

61
62
63
64
65

16
17
18
19
20

41
42
43
44
45

B
D
E
A
A

66
67
68
69
70

A
A
C
B

46
47
48
49
50

B
E
B
A
C

71
72
73
74
75

21
22
23
24
25

E
B
B
C

D
B
D
C

D
A

B
E
A

86
87
88
89
90

A
D

D
E
A

91
92
93
94
95

B
B
B
B
C

96
97
98
99
100

C
E
C
D
D

A
C
E
C

A
C

BIOCHEMISTRY-PHYSIOLOGY (12)

BEFORE PROCEEDING
1.

Turn your answer sheet over to the side numbered 101 - 200 (SIDE 2).

2.

Enter your name on the second side of the answer sheet.

3.

Enter the name of this test (BIOCHEMISTRY-PHYSIOLOGY) and the number of this
test (12) on the answer sheet. Blacken the circles that correspond to the test number.

4.

Check to be sure you have completed each step above.

5.

Proceed with the examination.

101.

Which of the following substances, under


physiologic conditions, is essential to the
transformation of G-actin to F-actin?
A.
B.
C.
D.
E.

102.

B.
C.
D.

103.

Cell walls of caries-related streptococci


decomposition end products of oral
lactobacilli
Byproducts of carbohydrate metabolism
by oral streptococci
Enzymes which break down grampositive cell walls
Components of extracellular matrix

107.

B.
C.
D.
E.

108.

109.
104.

Which of the following BEST explains the


primary action of antidiuretic hormone?
A.
B.
C.
D.
E.

105.

It decreases the activity of the Na-K


pump in the distal tubule.
It increases the Hz0 permeability of the
collecting ducts and the distal tubules.
It decreases the pore size of the distal
tubules and the collecting ducts.
It decreases the glomerular filtration rate.
It inhibits the action of glutaminase.

Which of the following takes place as


proinsulin is converted to insulin?
A.
B.
C.
D.

Disulfide bonds are formed.


Disulfide bonds are broken.
The polypeptide chain is lengthened.
A segment of the polypeptide chain is
removed.

Rectum
Internal anal sphincter
Antrum of the stomach
Upper esophagus
Gastro-esophageal sphincter

The movement of ~ a + across


+
a membrane is
characterized by which of the following?
A.
B.
C.
D.

110.

Preparing enzymes that synthesize


nucleic acids
Isolating the genome of an organism
Amplifying a specific DNA sequence
Separating polyclonal antibodies
Synthesizing RNA from DNA

Each of the following segments of the


gastrointestinaltract consists of smooth
muscle under autonomic nervous control
EXCEPT one. Which one is this EXCEPTION?
A.
B.
C.
D.
E.

0 mglmin
50 mglmin
100 mglmin
180 mglmin
350 mglmin

Sodium
Calcium
Chloride
Magnesium
Potassium

The polymerase chain reaction is MOST


useful for which of the following?
A.

Which of the following represents the normal


clearance of glucose?
A.
B.
C.
D.
E.

GABA increases the permeability of


postsynaptic membranes to which of the
following ions?
A.
B.
C.
D.
E.

ATP
Chloride
Manganese
Pyrophosphate
Creatine phosphate

Chondroitin sulfate and hyaluronic acid are


characterized by which of the following?
A.

106.

A facilitated diffusion
An example of a cotransport system
In many cell membranes involves a
~ a + -+~ a countertransport
+
system
Maintains ~ a + concentration
+
very much
higher in the cell than in the extracellular
fluid

The enzyme catalyzing the rate-controlling


step in the de novo synthesis of fatty acids is
regulated allosterically by the positive
modulator
A.
B.
C.
D.
E.

ATP.
NADPH.
citrate.
cyclic AMP.
oxaloacetate.

111.

Which of the following is required for


vitamin Dg synthesis?
A.
B.
C.
D.

2 .

Intensity of the stimulus


Diameter of the nerve fiber
Duration of the absolute refractory period
Duration of the relative refractory period

Fibrin
Thrombospondin
Prothrombin
Thromboplastin
Fibrinogen

B.
C.
D.
E.

119.

120.
115.

The anticaries effect of fluoride is LEAST


related to which of the following?
A.
B.
C.
D.
E.

116.

A.
B.
C.
D.
E.

Oleic
Lactic
Sialic
Stearic
Linoleic

Amide
Anomeric
Epimeric
Hydrogen
Glycosidic

In skeletal muscle, excitation-contraction


coupling is mediated by which of the
following?
A.
B.

Binding of calcium and calmodulin


Efflux of potassium from transverse
tubules
C. A sudden rise in permeability to chloride
ions
D. Release of acetylcholinesterase from
nerve terminals
E. Release of calcium from the
sarcoplasmic reticulum

Ion exchange
Decreased solubility
Facilitation of remineralization
Its presence during enamel formation
Activation of plaque polysaccharide
hydrolysis

Which of the following represents a


polyunsaturated fatty acid that is commonly
found in animal cell membranes?

Shivering
ATP hydrolysis
Exothermic reactions
Peripheral vasodilation
Increased thyroxine release

Which of the following types of bonds link


amino acid residues to form proteins?
A.
B.
C.
D.
E.

Ketoacidosis
Severe metabolic alkalosis
A deficiency in prostaglandinformation
An inability to synthesize ascorbic acid

Total G-C content of total DNA


increasing
UV light absorption increasing
Complementary strands becoming
random coils
Base stacking becoming disrupted
Hydrogen bonds breaking

Each of the following is a mechanism in


humans helping to raise body temperature
EXCEPT one. Which one is this
EXCEPTION?
A.
B.
C.
D.
E.

A sustained, severe carbohydrate deficiency


in the diet will result in which of the following?
A.
B.
C.
D.

As DNA is denatured, each of the following


events takes place EXCEPT one. Which event
is this EXCEPTION?
A.

Which of the following represents the normal


substrate of thrombin?
A.
B.
C.
D.
E.

114.

Decarboxylation in the liver


UV activation of precursors in skin
Metabolism by gut bacteria
Deamination in the kidney

The maximal frequency of impulses that can


be carried by a nerve fiber is limited by which
of the following?
A.
B.
C.
D.

113.

117.

121.

A mucin is which of the following types of


proteins?
A.
B.
C.
D.
E.

Simple protein
Phosphoprotein
Chromoprotein
Nucleoprotein
Glycoprotein

122.

The tricarboxylic acid cycle is initiated by the


condensation of which of the following two
molecules?
A.
B.

Pyruvate and malate


NAD' and oxaloacetate

C.
D.
E.

NAD' and oxalosuccinate


Acetyl coenzyme A and oxaloacetate
Acetyl coenzyme A and oxalosuccinate

128.

A.
B.
C.
D.

129.
123.

Compared to skeletal muscle contraction,


smooth muscle contraction
A.
B.
C.
D.

requires more energy for a given tension.


occurs several times more rapidly.
can be maintained for a longer time.
is unaffected by the autonomic nervous
system.

Androgens are produced in the testis and


A.
B.
C.
D.
E.

125.

131.

A number of catabolic pathways are


allosterically inhibited by an increase in the
concentration of which of the following?
A.
B.
C.
D.
E.

127.

Entropy
Enthalpy
Free energy
Potential energy
Activation energy

ADP
AMP
ATP
NAD+
Pyruvate

Both systemic and pulmonary circulations


have the same
A.
B.
C.
D.
E.

pulse pressure.
total capacitance.
diastolic pressure.
resistance.
flow rate.

Nonpolar
Carbon-containing
Amphipathic
Hydrophilic

Renal blood flow


Total number of nephrons
Glomerular filtration rate
Length of the loop of Henle
Diameter of the distal tubule

Each of the following appears in the


glomerular filtrate in concentrations
approximately equal to those in plasma
EXCEPT one. Which one is this
EXCEPTION?
A.
B.
C.
D.
E.

132.

4 ATP and 2 lactic acid molecules.


2 ATP and 2 pyruvic acid molecules.
4 ATP and 2 pyruvic acid molecules.

The ability to concentrate urine varies among


animal species. The maximum urine
concentration that can be produced by an
animal is MOST closely related to which of the
following?
A.
B.
C.
D.
E.

What thermodynamic parameter is a measure


of randomness or disorder in a system?
A.
B.
C.
D.
E.

126.

adrenal cortex.
thyroid.
adrenal medulla.
pituitary.
hypothalamus.

2 ATP and 2 lactic acid molecules.

Which of the following is LEAST descriptive of


lipids?
A.
B.
C.
D.

130.
124.

Under strict anaerobic conditions, the


catabolism of one glucose molecule would
yield a net of

Urea
Glucose
Amino acids
Steroid hormones
Plasma electrolytes

If an axonal membrane transiently becomes


very permeable to Na+ ions, then the
membrane potential of the cell will approach
A.
B.
C.
D.
E.

-70 mV.
-60 mV.
-50 mV.
OmV.
+60 mV.

133.

The clinical symptoms of dermatitis, diarrhea,


and dementia suggest a deficiency in which of
the following vitamins?
A.
B.
C.
D.
E.

138.

Retinol
Riboflavin
Tocopherol
Ascorbic acid
Niacin

A.
B.
C.
D.
E.

139.
134.

Streptomycin is an antibiotic which inhibits the


process of
A.
B.
C.
D.
E.

translation in eukaryotes.
translation in prokaryotes.
transcription in eukaryotes.
transcription in prokaryotes.
DNA replication in prokaryotes.

136.

Glucose
Steroids
Cholic acid
Oxaloacetate
Ketone bodies

Insulin INCREASES the activity of which of


the following?

Which of the following could be beneficial in


reducing edema in the arms of women who
have had radical mastectomy with removal of
axial lymph nodes?

A.
B.
C.
D.
E.

A.
B.
C.
D.
E.

Enolase
Phosphorylase a
Phosphofructokinase
Glucose-6-phosphatase
Fructose 1-6 diphosphatase

lncreased parasympathetic activity results in


A.
B.
C.
D.

137.

carotid bodies.
carotid sinuses.
expiratory center.
inspiratory center.
alveolar nerve endings.

Most endogenous cholesterol in the liver is


usually converted into which of the following?
A.
B.
C.
D.
E.

140.
135.

The stimulating effect of low arterial 0 2


tension on respiration is usually produced
through an effect on the

decreased salivary secretion.


increased cardiac contractility.
decreased gastric motility and tone.
increased bronchiolar smooth muscle
contraction.

A patient who has emphysema is MOST likely


to exhibit which of the following changes in
functional residual capacity (FRC) and
compliance?
FRC
A.
B.
C.
D.
E.

141.

Increased
Decreased
lncreased
Decreased
Decreased

Compliance
Increased
No change
No change
lncreased
Decreased

Which of the following substances is LEAST


polar?
A.
B.
C.
D.

142.

Hypertensiveagents
Avoidance of all diuretic agents
Increase interstitial oncotic pressure
Decrease interstitial hydrostatic pressure
Administration of a plasma volume
expander

Ethanol
Cholesterol
Palmitic acid
Glycocholic acid

Which of the following molecules would likely


form a micelle when mixed with water and
agitated?
A.
B.
C.
D.

Serine
Glycerol
Phospholipid
Triglyceride

143.

Which of the following is MOST likely to result


from increased vagal activity?
A.
B.
C.
D.
E.

144.

D.

A.
B.
C.
D.

149.

Increases
Increases
Decreases
Decreases

Increases
Decreases
Increases
Decreases

Which of the following is the major protein


component of cementum?
A.
B.
C.
D.
E.

Elastin
Keratin
Collagen
Amelogenin
Osteonectin

a reducing agent.
an oxidizing agent.
both a reducing agent and an oxidizing
agent.
neither a reducing agent nor an oxidizing
agent.

150.

Which of the following do elastin and collagen


have in common?
A.
B.
C.
D.
E.

Easily stretched
Absence of proline
Disulfide crosslinking
Triple helix structure
About one-third glycine

Coenzyme A participates in
A.
B.
C.
D.

147.

Laminin
Vimentin
Phosphophoryn
Osteonectin
Fibronectin

In respiratory acidosis, arterial C02 content


and pH become abnormal. Which of the
following BEST describes their respective
changes?

2+.
Some carbohydrates convert Cu Ions to CU+
ions. This property is related to their ability to
act as
A.
B.
C.

146.

lncreased heart rate


lncreased stroke volume
lncreased cardiac output
Decreased cardiac oxygen consumption
Decreased transit time through the AV
node

Which of the following noncollagenous protein


components BEST characterizes dentin
matrix?
A.
B.
C.
D.
E.

145.

148,

formylation.
protein synthesis.
methionine activation.
activation of carboxyl groups.

151.

A.
B.
C.
D.
E.

Which of the following is the FIRST step in the


catabolism of many amino acids?
Formation of a dipeptide with glutamate
Conjugation of the alpha amino to
glucuronate
C.. Transamination of the alpha amino to a
keto acid
D. Conjugation of the alpha carboxyl group
to glucuronate
E. Decarboxylation of the alpha carboxyl
group to form a primary amine

A.
B.

The plasma osmolality decreases after


infusion of

152.

aldosterone.
vasopressin (ADH).
angiotensin II.
parathyroid hormone.
isotonic saline solution.

Which of the following features distinguishes


active transport from facilitated diffusion?
A.
B.
C.
D.

Specificity
Carrier-mediated
Requires metabolic energy
Presence of a transport maximum (Tm)

153.

Each of the following secretes H C a - into the


gastrointestinaltract EXCEPT one. Which
one is this EXCEPTION?
A.
B.
C.
D.

154.

C.
D.

Excess H+ in the matrix


An H+ gradient across the inner
membrane
The formation of thioesters in the matrix
A conformational change in the inner
membrane

NA+
NADH
NADP+
NADPH
FADH2

The function of which of the following types of


nucleic acid is to activate and select specific
amino acids for protein synthesis?

Which of the following changes in lung


compliance and vital capacity would be
observed in the absence of pulmonary
surfactant?
Lung Compliance
A.
B.
C.
D.
E.

159.

160.

E.

Which of the following is a pyrimidine base


that is present in RNA but is NOT present in
DNA?
A.
B.
C.
D.
E.

Uracil
Guanine
Thymine
Adenine
Cytosine

162.

Refraction
Potentiation
Perception
Adaptation
Accommodation

Hematocrit of 45%
Venous blood with pH of 7.2
3
White blood cell count of 10,000/mm
Red blood cell count of 7,000,000/mm3
Pulse pressure equivalent to 80 mm Hg

Earaches may develop as a result of blockage


of the eustachian tube because
A.
B.
C.
D.

157.

Increase
Decrease
No change
Increase
Decrease

Which of the following is a typical finding for a


normal 23-year-old man?
A.
B.
C.
D.
E.

161.

Increase
Increase
Increase
Decrease
Decrease

Vital Capacity

Which of the following represents a decrease


in the frequency of action potentials in an
afferent neuron despite a constant stimulus
intensity?
A.
B.
C.
D.
E.

Which of the following is necessaryfor


de novo synthesis of cholesterol?
A.
B.
C.
D.
E.

156.

Chief cells
Colon mucosa
Salivary glands
Stomach mucosa

The chemical energy generated by


mitochondria1electron transport results from
which of the following?
A.
B.

155.

158.

of anvil misalignment.
the ossicle can no longer vibrate.
fluid in the middle ear cannot escape.
pressure on the round window distorts
the basement membrane.
pressure in the middle ear is not
equalized with atmospheric pressure.

The activities of each of


one contribute to calculus form
one is this EXCEPTION?

163.

Which of the following stimulates vagal nerve


endings in the lung parenchyma and inhibits
inspiration?
A.
B.
C.
D.

164.

B.

C.
D.

165.

B.

C.
D.

167.

169.

170.

Motor activity and pain and temperature


sensations on the same side
Motor activity on the same side and pain
and temperature sensations on the
opposite side
Motor activity and pain and temperature
sensations on the opposite side
Motor activity on the opposite side and
pain and temperature sensations on the
same side

A.
B.
C.
D.
E.

A band
I band
Muscle sarcomeres
Two consecutive Z lines
Series elastic elements

Urea
Ammonia
Uric acid
Creatinine
Aspartic acid

Which of the following are the immediate


effects of calcitonin on serum levels of calcium
and phosphate?
Serum calcium

If the molar percentage of A (adenine) in a


native DNA specimen is 22%, then what is the
molar content of G (guanine)?

During an isotonic contraction, which of the


following bands or structures do NOT change
in width or length?

ketone-body formation.
cholesterol synthesis.
fatty-acid synthesis.
gluconeogenesis.
the Krebs cycle.

After prolonged acidosis, which of the


following represents the nitrogenous product
that is excreted in high amounts in the urine?
A.
B.
C.
D.
E.

171.

vasoconstriction of coronary vessels.


vasodilation of coronary vessels.
stimulation of chemoreceptors.
inhibition of chemoreceptors.

In relative insulin insufficiency, acetyl CoA is


usually channeled into
A.
B.
C.
D.
E.

Right-sided lesions of the spinal cord result in


loss of which of the following (below the level
of the lesion)?
A.

166.

Must be able to replicate synchronously


with the host chromosome
Several unique recognition sequences
for one restriction enzyme
Two genes conferring resistance to
different antibiotics
Large size to facilitate plasmid's entry
into cells

A diminished oxygen tension in the


myocardium causes immediate

A,
B.
C.
D.

Decreased arterial pH
Expansion of the lungs
Decreased alveolar 0 2 tension
lncreased alveolar C02 tension

Plasmid vectors suitable for cloning have


which of the following characteristics?
A.

168.

A.
B.
C.
D.
E.

172.

Increased
Increased
Decreased
No change
Decreased

Serum phosphate
Increased
Decreased
Decreased
Increased
No change

Atoms are isotopes of each other only if


A.

B.
C.
D.

their nuclei contain the same number of


neutrons.
their atomic numbers are the same, but
their mass numbers differ.
their mass numbers are the same, but
their atomic numbers differ.
one is a beta emitter, but the other is an
alpha emitter.

173.

Sodium fluoride inhibits glycolysis by affecting


which of the following?
A.
B.
C.
D.

178.

Amylase
Enolase
Phosphatase
Phosphorylase

The Henderson-Hasselbalch equation shows


that
A.
B.
C.

D.

174.

Hyperventilation alters the acid-base balance


of arterial blood by
A.
B.
C.
D.

179.

A.
B.
C.
D.
E.

increasing C02 and decreasing pH.


decreasing C02 and decreasing pH.
decreasing C02 and increasing pH.

Which of the following collagen synthesis


post-translational events occurs
extracellularly?
A.
B.
C.
D.

Glycosylation
Hydroxylation
Intermolecular cross-linkage
Disulfide bond formation

A.
B.
C.
D.
E.

177.

become denatured.
migrate to the negative pole.
migrate to the positive pole.
remain stationary and unchanged.
separate into its different monomeric
forms.

The level of nonprotein nitrogen in the blood is


due principally to the level of which of the
following?
A.

Ferritin

6. Myoglobin
C.
D.
E.

182.

Which of the following sweeteners is


non-nutritive as well as non-cariogenic?
A.
B.
C.
D.
E.

183.

Hemoglobin
Transferrin
Cytochrome-c

D-fructose
Saccharin
Galactose
Sorbitol
Maltose

Epinephrine causes an elevation in CAMP


levels in muscle cells which in turn activate

Urea

6. Ammonia
C.
D.
E.

Prolactin
Oxytocin
ACTH
TSH

Which of the following represents the plasma


protein that transports ferrous iron?
A.

If the pH becomes lower than the isoelectric


point of a protein, then how will the protein
respond in an electrophoreticsystem? It will

Very low density lipoprotein


Low density lipoprotein
Chylomicrons
Liposomes
Micelles

Blocking the hypothalamic-hypophyseal


venous portal system increases the secretion
of which of the following?
A.
B.
C.
D.

181.

176.

Triglyceride absorbed into the lymphatic


system is transported to the liver as which of
the following?

increasing C02 and increasing pH.

180.
175.

dilution of a buffer increases its pH.


pH = pka when an acid is 0.1 N.
pH = pka when an acid is half
neutralized.
pH is independent of the dissociation
constant of the acid.

Creatine
Arginine
Uric acid

A.
B.
C.
D.
E.

ATPase.
adenyl cyclase.
glycogen synthetase.
glycogen phosphorylase.
glycogen phosphorylase phosphatase.

184.

Acromegaly is due to an excessive production


of which of the following?
A.
B. .
C.
D.

185.

186.

Thyrotropin
Gonadotropin
Somatotropin
Adrenocorticotropin

Intravenous injections of KC1 solution would


increase the secretion of which of the
following?
A.
B.
C.
D.
E.

189.

Pieces of dog parotid gland and pancreas are


transplanted under the skin of the donor
animal. After re-establishment of circulation,
ingestion of food will result in secretion from
which of the following?
A.
B.
C.
D.

190.

lnsulin
Cortisol
Calcitonin
Aldosterone
Parathyroid hormone

Which of the following conditions is indicated


during isovolumetric ventricular contraction of
the heart?
A.
B.
C.

Frank-Starling's law of the heart states that the


D.
A.
B.
C.
D.

187.

191.

B.
C.
D.

increase cardiac rate and strength of


contraction.
cause vasoconstriction throughout the
peripheral circulatory system.
excite sympathetic nerves and inhibit
parasympathetic nerves.
inhibit tonic activity of sympathetic
nerves and excite parasympathetic
nerves.

The rate of diffusion across the alveolar wall is


inversely proportional to which of the
following?
A.
B.
C.
D.

The surface area for gaseous exchange


The thickness of the alveolar wall
The difference in the partial pressures of
the gas
The solubility of the gas

192.

Load on the muscle


Gamma efferent system
Alpha efferent discharge
Length of the extrafusal fibers

In the denervated heart, adjustments to


increased workload are mediated by
mechanisms associated with
A.
B.
C.
D.

193.

All valves are open.


All valves are closed.
The aortic and pulmonary valves are
open and the mitral and tricuspid valves
are closed.
The aortic and pulmonary valves are
closed and the mitral and tricuspid
valves are open.

Which of the following controls the excitability


of the muscle spindle?
A.
B.
C.
D.

When arterial pressure increases,


pressoreceptors discharge and
A.

188.

initial length of cardiac muscle fibers


affects the strength of contraction.
strength of contraction depends on the
strength of the ventricular muscle.
heart rate depends directly on thickness
of the ventricle.
strength of contraction depends on the
strength of stimulus.

Parotid and pancreatic transplants


The fragment of the pancreas only
The fragment of the parotid only
Neither fragment

increased end systolic volume.


decreased end systolicvolume.
decreased end diastolic volume.
increased end diastolic volume.

Which of the following is secreted more during


the absorptive state than during the
postabsorptive state?
A.
B.
C.
D.
E.

lnsulin
Glucagon
Cortisol
Thyroxine
Epinephrine

194.

Bulging veins in the neck result from which of


the following?
A.
B.
C.
D.
E.

195.

Pulmonary edema
Hemolytic anemia
Systemic hypotension
Congestive heart failure
Intermittent claudication

A parathyroidectomized animal will exhibit


which levels of calcium and phosphate ion
concentration in the plasma?
Calcium

A.
B.
C.
D.
E.

196.

Low
Normal
High
Normal
Low

Phosphate ion
concentration
High
Low
Low
Normal
Low

Which term of the Michaelis-Menten equation,

can be determined from the y intercept of a


Lineweaver-Burk plot?

197.

Absence of which of the following blood


enzymes drastically reduces blood CO2
carrying capacity?
A.
B.
C.
D.
E.

198.

Carbonic anhydrase
Alkaline phosphatase
Pyruvate carboxykinase
Histidine decarboxylase
Serum glutamic-oxaloacetate
transaminase

Glucosuria with hyperglycemia usually occurs


in which of the following?
A.
B.
C.
D.
E.

199.

Pellagra
Addison's disease
Diabetes mellitus
Diabetes insipidus
Parkinson's disease

The largest amount of glomerular filtrate is


reabsorbed in the
A.
B.
C.
D.
E.

200.

proximal convoluted tubule.


descending loop of Henle.
ascending loop of Henle.
distal convoluted tubule.
collecting duct.

How many anomeric carbons are present in a


fructose molecule?

NATIONAL BOARD DENTAL EXAMINATIONS


PART 1
TEST:
FORM:
DATE:

BIOCHEMISTRY-PHYSIOLOGY
12 (A12)
D98

Item

Key

Item

Key

Item

Key

Item

Key

101
102
103
104
105

126
127
128
129
130

E
A
D
D

151
152
153
154
155

A
B
D

176
177
178
179
180

B
A
C
C
A

131
132
133
134
135

D
E
E
B
C

156
157
158
159
160

D
A
E
D
A

181
182
183
184
185

D
B
D
D

D
A

161
162
163
164
165

186
187
188
189
190

A
D
B
B
B

191
192
193
194
195

B
D
A
D
A

196
197
198
199
200

B
A

106
107
108
109
110

D
A
B
D
C
C

D
C
C

111
112
113
114
115

E
A
E

136
137
138
139
140

116
117
118
119
120

E
A
D
A
E

141
142
143
144
145

121
122
123
124
125

E
D
C

146
147
148
149
150

A
A

A
C

D
C
A
D
C

B
C
E

166
167
168
169
170
171
172
173
174
175

B
C

B
B
A

B
A
B
C

B
B
D
C

C
A

DENTAL ANATOMY AND OCCLUSION (14)

BEFORE PROCEEDING

- 200 (SIDE2).

1.

Turn your answer sheet over to the side numbered 101

2.

Enter your name on the second side of the answer sheet.

3.

Enter the name of this test (DENTAL ANATOMY and OCCLUSION)and the number
of this test (14) on the answer sheet. Blacken the circles that correspond to the test
number.

4.

Check to be sure you have completed each step above.

5.

Proceed with the examination.

All test items refer to permanent teeth unless


"primary" is specified.

105.

All test items relating t o occlusion refer t o Class I


canine and molar relationship unless otherwise
specified. Tenns such as "normal" or "ideaIJ'are
synonymous with the above definition.

101.

Which of the following are the LASTprimaly


teeth to erupt?
A.
B.
C.
D.
E.

A.
B.
C.
D.

106.

Maxillary canines
Mandibular canines
Mandibular first molars
Maxillary second molars
Mandibular second molars

Which of the following is the correct sequence


of dental tissues from SOFTEST to
HARDEST?
A.
B.
C.
D.

103.

The crown of which premolar is wider


faciolingually than mesiodistally, and has two
cusps that are approximately equal in height?
A.
B.
C.
D.

104.

B.
C.
D.

E.

with the mesial marginal ridge of the


maxillary second premolar and the distal
marginal ridge of the first premolar.
in the lingual embrasure between
maxillary canine and first premolar.
with the lingual surface of the maxillary
first premolar.
in the lingual embrasure between
maxillary premolars.
with no maxillary tooth.

109.

condylar head and the disc.


inferior and superior retrodiscal lamina.
retrodiscal tissue and capsular ligament.
disc and the articular fossa and
eminence.

If a maxillary first molar has a fourth pulp


canal, it is located in which of the following
roots?
A.
B.
C.
D.

Maxillary first
Mandibular first
Maxillary second
Mandibular second

In the intercuspal position, the lingual cusp of


a mandibular first premolar usually occludes
A.

108.

Facio-occlusalline
Central fossa line
Linguo-occlusalline
Curve of Spee
Curve of Wilson

The upper compartment of the


temporomandibular joint is that space
between the
A.
B.
C.
D.

Dentin, cementum, enamel


Cementum, dentin, enamel
Dentin, enamel, cementum
Cementum, enamel, dentin

Facial
Lingual
Central
Distofacial

In an ideal occlusion, the facial cusps of the


posterior mandibular teeth oppose which
landmark of the maxillary dentition?
A.
B.
C.
D.
E.

107.
102.

Through which of the following grooves of the


mandibular first molar does the maxillary
mesiolingual cusp pass in a lateral excursive
movement on the working side?

Mesiofacial
Distofacial
Palatal
Fourth

Which of the following representsthe general


crown form of canines, as viewed from the
labial aspect?
A.
B.
C.
D.
E.

Hexagonal
Triangular
Rhomboidal
Pentagonal
Diamond-shaped

110.

On the crown of the maxillary canine, the


height of contour is normally located in the
cervical third of which of the following
surfaces?
A.
6.
C.
D.

111.

112.

A.
6.
C.
D.

116.

Distal
Facial
Palatal
Occlusal
Proximal

The protrusive pathway of the mandibular


cusps on the maxillary posterior teeth is
toward the

When compared to the maxillary central


incisor, the maxillary canine normally exhibits
which of the following?

A.
B.
C.
D.
E.

Labial
Lingual
Both labial and lingual
Neither labial nor lingual

Which of the following surfaces of a tooth is


always next to an adjacent tooth?
A.
B.
C.
D.
E.

115.

Which of the following teeth have proximal


contact areas at approximately the same
levels cervicoincisally,or cervico-occlusally,
on the mesial and the distal?
A.
B.
C.
D.

117.

mesial.
facial.
distal.
lingual.

The non-working pathway of the maxillary


cusps on the mandibular posterior teeth is
toward the

A.
6.
C.
D.

A.
6.
C.
D.

114.

118.

distofacial.
distolingual.
mesiofacial.
mesiolingual.

Maxillary central incisors


Mandibular central incisors
Maxillary canines
Mandibular canines

In the intercuspal position, the facial aspect of


the mesial cusp ridge of a mandibular canine
opposes which of the following maxillary
structures?

E.

113.

Shorter root
Wider crown mesiodistally
Thicker crown labiolingually
Longer crown incisogingivally
Shorter crown and shorter root length

DL of a canine
DF of a canine
ML of a lateral incisor
DL of a lateral incisor
DL of a central incisor

For an amalgam restoration, it will be MOST


difficult to obtain close matrix-band adaptation
on which of the following surfaces?
A.
B.
C.
D.
E.

Mesial of a maxillary first premolar


Distal of a maxillary first premolar
Mesial of a maxillary second premolar
Distal of a mandibular second premolar
Mesial of a mandibular first molar

Which of the following illustrations represents


the distal view of a maxillary first molar?

119.

From an occlusal view, the arrangement of


permanent teeth of the maxillary and
mandibular arches are parabolic in shape. In
one segment of the dentition, however, four
teeth are aligned in a straight line. In what
region is this segment located?
A.
6.
C.
D.

Maxillary anterior
Maxillary posterior
Mandibular anterior
Mandibular posterior

120.

A posterior tooth has a faciolingual crown


dimension greater than its mesiodistal crown
dimension. In addition, it has two roots. This
tooth is MOST likely a
A.
B.
C.
D.
E.

121.

A.
B.
C.
D.
E.

123.

Anterior border
Posterior border
Bilaminar zone
Intermediate zone
Retrodiscal area

Palatal, mesiobuccal,distobuccal
Palatal, distobuccal, mesiobuccal
Mesiobuccal, palatal, distobuccal
Distobuccal, mesiobuccal, palatal

In an ideal intercuspal relation, which of the


following maxillary cusps will oppose the
mesiofacial groove of the mandibular first
molar?
A.
B.
C.
D.
E.

Maxillary first
Maxillary second
Mandibular first
Mandibular second

129.

Facial cusp of the second premolar


Mesiofacial cusp of the first molar
Distofacial cusp of the first molar
Mesiolingual cusp of the first molar
Distolingual cusp of the first molar

Maxillary central incisor


Maxillary lateral incisor
Mandibular lateral incisor
Maxillary canine
Mandibular canine

Which of the following periodontal ligament


fibers provide the major support to the tooth
during function?
A.
B.
C.
D.
E.

Apical
Oblique
Horizontal
Alveolar crest
lnterradicular

When proximal surfaces of adjacent teeth


diverge from an area of contact, an
embrasure is formed
A.
B.
C.
D.

Which of the following anterior teeth exhibits


the MOST deviation in crown morphology?
A.
B.
C.
D.
E.

125.

Which of the following ranks the roots of the


maxillary first molar in order from LARGEST to
SMALLEST?
A.
B.
C.
D.

128.

Mandibular first
Mandibular second
Maxillary first
Maxillary second

Which of the following premolars usually has


two lingual cusps?
A.
B.
C.
D.

124.

127.

Maxillary first
Maxillary second
Mandibular first
Mandibular second

In the sagittal plane, which of the following


represents the thickest section of the articular
disc?

Which premolar poses the GREATEST


problem when root canal therapy or extraction
are being considered?
A.
B.
C.
D.

Which primary molar typically has a


transverse ridge, an oblique ridge, and a
distolingual groove?
A.
B.
C.
D.

122.

maxillary first premolar.


mandibular first premolar.
mandibular second premolar.
mandibular first molar.
maxillary second molar.

126.

130.

lingually only.
lingually and facially only.
lingually and occlusally only.
lingually, facially, occlusally, and
cervically.

The pulp chamber of a mature tooth contains


A.
B.
C.
D.
E.

cells that continue to form primary


dentin.
an enamel lining for thermal protection.
blood vessels and nerves.
cernentoblasts.
ameloblasts.

131.

Between which of the following teeth in the


primary dentition is the mandibular "primate"
space usually found?
A.

6.
C.
D.

E.

132.

134.

How many pulp horns would be expected on a


newly erupted Tooth #18?

138.

In which of the following areas of a healthy


mouth is the alveolar process the thinnest?

During a lateral movement of the mandible.


the articular disc is tightly attached to the head
of the condyle by which of the following
ligaments?
A.
B.
C.
D.
E.

133.

Central incisors
Central and lateral incisors
Lateral incisor and canine
Canine and first molar
First and second molars

137.

Superior retrodiscal
Inferior retrodiscal
Sphenomandibular
Stylomandibular
Collateral

A.

6.
C.
D.
E.

139.

On which of the following surfaces of


permanent teeth is pit and fissure caries
MOST likely to occur?

Which of the following teeth has its distal


proximal contact area located entirely in the
incisal or the occlusal one-third?

A.
6.
C.

A.
B.
C.
D.
E.

D.

Maxillary canine
Mandibular canine
Maxillary lateral incisor
Mandibular lateral incisor
Mandibular second premolar

E.

140.

The occlusal outline of a mandibular first


molar is usually similar to a

6.
C.
D.

Which of the following teeth is the MOST likely


to have a bifurcated root?
A.
6.
C.
D.
E.

136.

C.
D.

mesially and lingually.


distally and lingually.

Assuming occlusion and alignment are


normal, the arrow on the illustration below
represents the path taken by which cusp of a
second molar?

Maxillary central incisor


Maxillary lateral incisor
Mandibular lateral incisor
Maxillary canine
Mandibular canine

The occlusal table of any posterior tooth


makes up what percent of the total
buccolingual dimension of the tooth?

Cusp

A.
A.
B.
C.
D.

distally and facially.

8. mesially and facially.

circle.
square.
rhomboid.
pentagon.

141.
135.

Facial surfaces of maxillary first molars


Lingual surfaces of maxillary first molars
Facial surfaces of mandibular first
premolars
Lingual surfaces of mandibular first
molars
Proximal surfaces of mandibular incisors

When in its proper position relative to the


plane of occlusion, the crown of a mandibular
second molar inclines
A.

A.

Lingual to the maxillary central incisors


Facial to the mandibular central incisors
Lingual to the maxillary canines
Lingual to the mandibular first molars
Facial to the maxillary second molars

25-35 percent
40-50 percent
55-65 percent
70-80 percent

6.
C.
D.
E.

Mesiofacial
Mesiolingual
Distofacial
Distolingual
Cusp of Carabelli

142.

In an ideal intercuspal relation, which of the


following maxillary teeth have single
antagonists?
A.
B.
C.
D.
E.

147.

A.
B.
C.
D.

Central incisors
Lateral incisors
First premolars
First molars
Third molars

148.
143.

Which of the following represents the structure


on the crown of a maxillary canine which is
located immediately to the mesial of the
mesiolingualfossa?
A.
B.
C.
D.
E.

Lingual ridge
Mesial marginal ridge
Distal marginal ridge
Distolingual fossa
Mesial developmental groove

The epithelial attachment is always an actual


part of the tooth's
A.
B.
C.
D.
E.

145.

The dentition of a normally developed


6 ID-year-old child usually consists of which
of the following teeth?
A.
B.
C.
D.

46.

8 primary and 8 permanent


10 primary and 10 permanent
18 primary and 6 permanent
20 primary

Which of the following represents the


movement of a tooth through the surrounding
tissue so that the clinical crown gradually
appears longer?
A.
B.
C.
D.
E.

150.

Eruption
Attrition
Mastication
Exfoliation
Longitudinal development

Anterior guidance
Posterior guidance
Bennett side shift
Intercondylar distance

Which of the following represents the


EARLIEST age by which the roots of the
maxillary first premolar are completely
formed?
A.
B.
C.
D.

151.

Maxillary first
Maxillary second
Mandibular first
Mandibular second

Which of the following plays the greatest role


in disoccluding the posterior teeth in lateroprotrusive movements?
A.
B.
C.
D.

anatomical crown.
clinical crown.
gingival line.
cervical line.
periodontium.

#22 and #23


#23 and #24
#24 and #25
#25 and #26

Which primary molar has a crown somewhat


resembling a permanent premolar, but the
root form is typical of a permanent molar?
A.
B.
C.
D.

149.
144.

In a protrusive movement, the maxillary left


central incisor can potentially come in contact
with which of the following mandibular teeth?

8-9 years
10-11 years
12-13 years
14-15 years

Viewed from the occlusal, the basic coronal


outline of a mandibular second premolar is
A.
B.
C.
D.
E.

square.
circular.
rhomboidal.
pentagonal.
rectangular.

Which of the following BEST describes the


location of the cervical line on a mandibular
lateral incisor?
A.
B.
C.
D.

E.

156.

A.

On the distal, it is more apical than on


the mesial.
On the lingual, it is more incisal than on
the facial.
On the mesial, it is more apical than on
the distal.
On the facial, it is more apical than on
the lingual.
On the facial and the lingual, it is at the
same level.

B.
C.
D.

157.

153.

A.
B.
C.
D.

Tooth

Distal
Mesial
Central
Mesial

Second premolar
Second premolar
First molar
First molar

158.

E.

Maxillary central incisor


Mandibular central incisor
Maxillary lateral incisor
Mandibular lateral incisor
Maxillary canine

Ligaments associated with the TMJ serve to

D.
E.

155.

Maxillary first
Mandibular first
Maxillary second
Mandibular second

Which of the following teeth would MOST


likely be congenitally absent?
A.
B.
C.
D.

154.

from the dentinoenamel junction apically


in the cervical third of the crown.
from the dentinoenameljunction
occlusally in the cervical third of the
crown.
in the same direction as in a permanent
tooth in the cervical third, but not in the
incisal or occlusal thirds of the crown.
in a diffused and gnarled manner in the
cervical third of the crown.

Which of the following molars has the


LARGEST mesiodistal measurement of its
crown?
A.
B.
C.
D.

In a normal occlusion, the lingual cusp of the


maxillary second premolar occludes with
which fossa of which mandibular tooth?
Fossa

Enamel rods in a primary tooth extend

inhibit the normal masticatory cycle.


protect surrounding and supporting
tissues from damages.
assist musculature in producing
movement.
allow for retrusive movement to occur.
provide elastic potential to allow for
border movements.

From a facial view, where would the


cementoenamel junction be MOST apically
positioned on the crown of a primary
mandibular first molar?
A.
B.
C.
D.

Mesial one-third
Middle one-third
Distal one-third
Equally positioned in all thirds

159.

Which of the following posterior teeth normally


exhibit two root canal orifices?
A.
B.
C.
D.
E.

160.

Mandibular first premolars


Maxillary first premolars
Maxillary second premolars
Maxillary first molars
Mandibular first molars

Which of the following describes the proximal


contact relationship between a maxillary
central incisor and a maxillary lateral incisor?

A.
B.
C.
D.

Contact is offset to the lingual.


Contact is centered incisocervically.
Lingual embrasure is larger than the
facial embrasure.
lncisal embrasure is the largest of all the
embrasures.

161.

A crown concavity, which is confluent with a


longitudinal groove of the root, is a rather
common feature of which surface of which
premolar?

A.
B.
C.
D.
E.

162.

Mesial
Distal
Mesial
Distal
Mesial

Maxillary first
Maxillary first
Mandibular first
Mandibular first
Mandibular second

167.

Which of the following characteristics is


common to maxillary first and second
premolars?
The lingual cusp tip is offset to the
mesial.
B. The lingual cusp tip is offset to the distal.
C. The facial and lingual cusps are equal in
size and prominence.
D. The mesial root concavity extends
across the cervical line onto the cervical
third of the crown.
E. The lingual height of contour is in the
cervical third of the crown.

168.

In a molar, where do root canals usually join


the pulp chamber?
A.
B.

At the level of the furcation


At varying levels, dependent upon age
C. Within the middle third of the crown
D. Within the cervical third of the crown
E. Apical to the cementoenamel junction

169.

The lingual aspect of a mandibular first molar


presents with
A.

A.
B.
C.
D.
E.

Number of roots
Number of lingual cusps
Presence of a lingual groove
Position of the lingual cusps
Number of developmental grooves

B.

C.
D.
E.

165.

When compared with the incisal embrasure


between the maxillary central and lateral
incisors, the incisal embrasure between the
maxillary central incisors is
A.
B.
C.
D.
E.

larger.
smaller.
the same size.
determined by the position of the
epithelial attachment.
determined by the height of curvature of
the cervical line.

Maxillary first
Maxillary second
Mandibular first
Mandibular second

A.

widermesiodistally.
narrower mesiodistally.
more symmetrical.
having a smoother lingual surface.

Which of the following characteristics of a


mandibular first molar distinguishes this tooth
from a mandibular second molar?

From the facial view, which premolar has a


mesial cusp ridge longer than its distal cusp
ridge?
A.
B.
C.
D.

Maxillary central
Maxillary lateral
Mandibular central
Mandibular lateral

The crown of a mandibular lateral incisor,


compared to the crown of a mandibular central
incisor in the same mouth, is BEST described
as
A.
B.
C.
D.

4 64.

Premolar

A peg-shaped crown form is an occasional


anomalous occurrence of which of the
following incisors?
A.
B.
C.
D.

163.

Surface

166.

170.

a lingual groove extending onto the


lingual surface and ending in the cervical
third of the crown.
a large mesiolingual cusp and much
smaller distolingual cusp.
the lingual surface of each cusp
possessing a slightly convex shape in
the occlusal third.
three lingual cusps.
two lingual grooves.

On the crown of a maxillary canine, which of


the following structures is located immediately
to the mesial of the mesiolingual fossa?
A.
B.
C.
D.
E.

Lingual ridge
Distal marginal ridge
Mesial marginal ridge
Distolingual fossa
Mesial developmental groove

171.

The transseptal group of periodontal fibers


travel from one root to an adjacent root
surface.

176.

This pathway occurs via the facial aspect of


the alveolus.
A.
B.
C.
D.

Both statements are TRUE.


Both statements are FALSE.
The first statement is TRUE, the second
is FALSE.
The first statement is FALSE, the second
is TRUE.

When the posterior teeth are in a crossbite


relationship, which of the following cusps are
considered supporting cusps?
A.
B.
C.
D.

177.

The central pit of the maxillary first molar is


formed at the bases of the triangular ridges of
which cusps?
A.

When the mandible moves to the right side,


which of the following represents the
excursive tooth contacts which could occur on
the right molars?
A.
B.
C.
D.

173.

Non-working side contacts


Working side contacts
Lateral contacts
Occlusion

B.
C.
D.

178.

Which of the following representsthe eighth


tooth from the midline in each mandibular
quadrant at age 26?
A.
B.
C.
D.
E.

179.
174.

The usual order of eruption of primary teeth


following the central incisors is

B.
C.
D.

175.

first molars, lateral incisors, second


molars, canines.
lateral incisors, first molars, second
molars, canines.
lateral incisors, first molars, canines,
second molars.
lateral incisors, canines, first molars,
second molars.

During an endodontic procedure on


Tooth #21, clinical symptoms lead the dentist
to suspect the presence of a second pulp
canal. Which direction is the MOST likely
location for the suspected canal?
A.
B.
C.
D.

Facial
Distal
Lingual
Mesial

B.
C.
D.

180.

Mesiolingual developmental
Distolingual developmental
Mesial marginal
Linguogingival

The physiologic rest position of the mandible


is established when the
A.

A.

Mesiobuccal, mesiolingual,and
distobuccal
Mesiobuccal, mesiolingual, and
distolingual
Mesiobuccal, distobuccal, and
distolingual
Mesiolingual, distobuccal, and
distolingual

Which of the following grooves on a


mandibular first premolar originates in an
occlusal pit and extends onto a proximal
surface?
A.
B.
C.
D.

Second molar with incomplete root


Second molar with complete root
Third molar with incomplete root
Third molar with complete root
Not erupted

Maxillary facial and mandibular facial


Maxillary facial and mandibular lingual
Maxillary lingual and mandibular facial
Maxillary lingual and mandibular lingual

muscles of mastication are in tonic


equilibrium.
maxillary and mandibular teeth make the
greatest occlusal contact.
condyles are in their most retruded
position.
condyles are in their most protruded
position.

In an ideal intercuspal relation, the


mesiolingualcusps of maxillary molars
occlude with which of the following anatomic
structures of mandibular molars?
A.
B.
C.
D.

Mesial fossae
Distal fossae
Central fossae
Mesial marginal ridges

181.

Which of the following occurs in a right lateral


movement?

A.

B.
C.
D.

186.

The right condyle primarily rotates.


The right condyle moves down the
eminentia.
The facial cusps of the mandibular left
side pass under the maxillary left facial
cusps.
The facial cusps of the mandibular right
side pass under the maxillary right
lingual cusps.

A.
B.
C.
D.
E.

187.
182.

Some degree of protection for lips, cheeks,


and tongue is afforded by the

A.
B.
C.
D.

183.

188.

Cingulum
Cervical ridge
Marginal ridge
lncisal ridge
Root apex

190.
A.
B.
C.
D.

185.

larger distofacial cusp.


shorter distolingual groove.
greater mesiodistal crown width.
greater occlusocervical crown width.

Which of the following are the principal muscle


fibers that retrude the mandible?

A.
B.
C.
D.

Superficial rnasseter
Posterior belly of the digastric
Posterior fibers of the temporal
Posterior fibers of the internal pterygoid

Lateral
Lateral
Lateral
Central
Central

Concrescence
Enamel pearls
Hypercementosis
Cementa1 pearls

The distofacial of a maxillary


The mesiofacial of a maxillary
The lingual of a maxillary
The mesial of a mandibular
The distal of a mandibular

Maxillary first premolar


Maxillary first molar
Mandibular second premolar
Mandibular first molar
Mandibular second molar

Which of the following teeth normally exhibits


five major cusps?
A.
B.
C.
D.
E.

191.

Facio-incisal
Mesial and distal
Distal
Mesial
Distal

Which of the following teeth usually has the


steepest cusp inclines?

A.
B.
C.
D.
E.

When compared to a maxillary first molar, the


crown of a second molar normally exhibits a

Maxillary incisor

In the midroot cross-section, which root of a


first molar has the largest area?
A.
B.
C.
D.
E.

189.

Marginal ridge(s)

Excessive calcified tissue formation at the root


apices is known as which of the following?

A.
B.
C.
D.

Which of the following structures calcifies first


in an anterior tooth?
A.
B.
C.
D.
E.

184.

contact of adjacent teeth.


deflecting function of triangular ridges.
facial and lingual heights of contour of
the teeth.
horizontal and vertical overlapping of the
teeth.

In the intercuspal position, the facio-incisal


aspect of a mandibular central incisor opposes
which of the following structure(s) of which
maxillary incisor?

Primary mandibular first molar


Primary maxillary second molar
Maxillary first molar
Mandibular first molar
Mandibular second molar

In the diagram of the envelope of motion, the


maximum opening position is the MOST

A.
B.
C.
D.
E.

centrally-located.
anterior.
superior.
posterior.
inferior.

192.

The primary mandibular first molar normally


exhibits which of the following?
A.

196.

The third tooth from the midline normally


erupts prior to the fourth tooth from the midline
in

An oblique ridge

6. A facial pit

A.

C.
D.
E.

B.

Three roots
A distal groove
A distal triangular fossa

C.
D.

193.

Which of the following BEST characterizes a


of mandibular second molar?
A.
B.
C.
D.
E.

197.

Exhibits four cusps


Exhibits a DF groove
Exhibits a distal cusp
Exhibits one fused root
Similar to the five-cusp variety
mandibular third molar

194.

Assuming occlusion and alignment are


normal, the arrow in the illustration below
represents the path taken by a

195.

cusp of canine.
facial cusp of first premolar.
lingual cusp of first premolar.
facial cusp of second premolar.
lingual cusp of second premolar

When a patient moves from maximum


intercuspal position to place the anterior teeth
into an edge-to-edge relationship, the
condyles of the mandible have moved
A.
B.
C.
D.
E.

backward.
forward.
downward.
forward and downward.
backward and downward.

Transverse of the oblique ridge


Distal oblique
Distolingual
Faciolingual
Central

The maxillary lateral incisor is usually equal to


or larger than the maxillary central incisor in
which dimensions?
A.
B.
C.
D.
E.

200.

6 to 7 years
8 to 9 years
10 to IIyears
12tol3years
14 to I 5 years

Which of the following grooves on a maxillary


first molar runs from the mesial pit to the
central pit on the occlusal surface?
A.
B.
C.
D.
E.

199.
A.
B.
C.
D.
E.

At what age is a primary maxillary canine


usually exfoliated?
A.
B.
C.
D.
E.

198.

the mandibular arch of the primary


dentition.
the maxillary arch of the primary
dentition.
the mandibular arch of the permanent
dentition.
the maxillary arch of the permanent
dentition.

Root width (faciolingually)


Crown width (mesiodistally)
Crown width (faciolingually)
Root length (cervicoapically)
Crown length (cervicoincisally)

Which of the following canines is the last


anterior tooth to erupt and at what age does it
erupt?
A.
B.
C.
D.

Mandibular; 9-10 years


Maxillary; 9-10 years
Mandibular; 11-12 years
Maxillary; 11-12 years

NATIONAL BOARD DENTAL EXAMINATIONS


PART 1
TEST:
FORM:
DATE:

DENTAL ANATOMY-OCCLUSION
14 (A14)
D98
h

Key

Item

Key

Item

Key

126
127
128
129
130

C
A
B
D
C

151
152
153
154
155

D
A
A
B
A

176
177
178
179
180

B
A
A
A
C

B
D
A
D
C

131
132
133
134
135

D
E
D
D
E

156
157
158
159
160

B
B
C
B
C

181
182
183
184
185

A
D
D
B
C

111
112
113
114
115

E
A
A
D
C

136
137
138
139
140

C
C
B
B
C

161
162
163
164
165

A
B
A
E

186
187
188
189
190

D
C
D
A
D

116
117
118
119
120

B
D
A
D
A

141
142
143
144
145

D
E
B
E
C

166
167
168
169
170

A
A
E
C
C

191
192
193
194
195

E
E
A
D
D

121
122
123
124
125

B
B
D
B
B

146
147
148
149
150

A
B
A
A
C

171
172
173
174
175

C
B
D
C
C

196
197
198
199
200

C
C
E
D
D

Item

Key

Item

101
102
103
104
105

D
B
C
E
B

106
107
108
109
110

..
Anatom ic Sciences
-

6.

~:'\M1id1 of the following represents the secretory

gland?

A
.

8.

C.
D.
E.
)2.'.'.
.

B.
C.
D.

Iodine
Protease
Thyroxine
Calcitonin
Thyroglobulin

E.

7.
V, VII, IX,X, andXII
V, VII, IX, andX
V, IX, and X
VII, IX,andXI
VII and IX

A contralateral
musculature

B.

An ipsilateral flaccid paralysis of the facial


musculature

C.

A contralateral paralysis of only the lower


facial muscles

D.

A contralateral paralysis of only the upper


facial muscles
An ipsilateral flaccid paralysis of only the
lower facial muscles

E.

Hypophysis
Adrenals
Testes
Parathyroids
Islets of Langerhans

paralysis of the facial

8.

The dentist incises the mucous membrane of


the floor of the mouth. This incision extends
from the molar region to the sublingual caruncle
(papilla). Which of the following structural
groups will be exposed first?
A

B.
The temporalis

A
B.
C.
D.
E.

muscle inserts into the

c.

coronoid process.
condylar process.
fovea of the mandible.

D.
E.

lateral aspect of the mandibular angle.


articular disk of the temporomandibular
joint.

Which of the following is surrounded partly by


connective. tissue and epithelium, contains
lymphoid follicles, has no sinuses, and is
penetrated by a number of crypts?

9.

A Spleen
B. Thymus
C. Lymph node
D. Palatine tonsil
E. Bursa of Fabricius
3

Sublingual gland, lingual nerves, and


submandibular duct
Hyoglossus and mylohyoid muscles, and
hypoglossal nerve
.
Lingual nerve, lingual artery, and anterior
belly of the digastric muscle
Lingual nerve, hypoglossal nerve, and
submental branch of the facial artery
Anterior belly of the digastric muscle,
mylohyoid nerve, and submental branch of
the facial artery

The trigeminal nerve innervates exclusively


which of the following muscles or muscle parts?

A
B.
C.
D.
E.

.r.

The growth rate in the cartilage of the epiphyseal


plate of a long bone is markedly retarded when
there is a lack of hormone from which of the
following?

A
B.
C.
D.
E.

Peripheral (lower motor neuron) lesions of


aanial nerve VII will cause which of the
following?

5.

a means for communication between


cells.
a means for attachment of adjacent cells.
an outlet for secretory products of a cell.
a bridge for continuity of cytoplasm
between adjacent cells.
a means by which tonofibrils can pass
from one cell into another.

Parasympathetic innervation controlling


salivation originates with which of the following
aanial nerves?

A
B.
C.
D.
E.

4.

Desmosomes function by serving as


."

product of the parafollicular cells of the thyroid -

Buccinator and masseter


Mylohyoid and geniohyoid
Medial and lateral pterygoid
Tensor and levator veli palatini
Anterior and posterior bellies of digastric

MICROBIOLOGY-PATHOLOGY (13)
DENTAL ANATOMY AND OCCLUSION (14)

PART l
MONDAY - P.M.
DECEMBER 1998

TEST BOOKLET
DO NOT OPEN TEST BOOKLET UNTIL INSTRUCTED TO BEGIN.

1.

In addition to this test booklet, you should have an answer sheet.

2.

Use a No. 2 pencil on all test materials.

3.

Enter your name and reference number below.

Name
Last

First

Middle

Reference Number

4.

Enter your name and reference number on the side of the answer sheet numbered 1 to 100 (SIDE 1).
Blacken the circle that corresponds to each digit of your reference number.

5. Enter the name of this test (MICROBIOLOGY-PATHOLOGY) and the number of this test (13) on the
answer sheet. Blacken the circles that correspond to the test number.

6. Check to be sure you have completed each step above.


YOU ARE READY TO BEGIN. For each test item, decide which choice is correct and blacken the corresponding
circle on the answer sheet. Record only one answer for each test item; there is no penalty for selecting an
incorrect response. You may write in the test booklet; however, your score is based on the total number of
correct answers recorded on your answer sheet. You are allowed 3 112 hours to complete this test booklet.
All test materials must be returned before or immediately upon the request of the Test Administrator. Use of a
magnifying glass is permitted. Use of other study aids is not permitted. No test items are to be copied or notes
taken. Infraction of these rules will result in your scores being voided.

COPYRIGHT 1998
JOINT COMMISSION ON NATIONAL DENTAL EXAMINATIONS
AMERICAN DENTAL ASSOCIATION
211 EAST CHICAGO AVENUE, CHICAGO, ILLINOIS 60611

1.

A patient who has anemia, poorly localized


abdominal pain, and wrist and foot drop probably
is manifesting a toxic state induced by which of
the following?
A.
B.
C.
D.
E.

2.

3.

A.
B.
C.
D.
E.

4.

Protooncogenes
Viral oncogenes
Retroviral genes
Analogous chromosomes
Homologous chromosomes

Which of the following bacterial vaccines is given


routinely to children in the U.S.A.?
A.
B.
C.
D.
E.

7.

Measles
Roseola
Chickenpox
Viral meningitis
Infectious mononucleosis

Normal human cells contain gene sequences


homologous to virus genome sequences known to
induce cancer in animals. What term applies to
these gene sequences when found in human
cells?

BCG
Cholera
Polyvalent pneumococcus
Measles, mumps, rubella
Diptheria, pertussis, tetanus

Which of the following commonly produces a


positive heterophile antibody test?
A.
B.
C.
D.
E.

Carbon tetrachloride
Carbon monoxide
Bismuth
Mercury
Lead

A patient with shingies (herpes zoster) is MOST


likely to have had a previous episode of which of
the following?

A.
B.
C.
D.
E.

6.

Which of the following inflammatory responses is


associated with an infection caused by pyogenic
pathogens?
A.
B.
C.
D.

8.

Catarrhal
Granulomatous
Pseudomembranous
Acute suppurative

Which of the following bacterial genera is LEAST


likely to be found among normal anaerobic flora of
the gingival sulcus?
A.
B.
C.
D.
E.

9.

Adenovirus
Cytomegalovirus
Epstein-Barrvirus
Parainfluenza virus
Herpes simplex virus type 1

Mycobactenurn
Fusobactenum
Prevotella
Actinobacillus
Potphyrornonas

If Type A blood is accidentally transfused into a


Type B recipient, the immediate hemolytic
reaction would be the result of which of the
following?
A.
B.
C.
D.
E.

IgM against the A antigen


IgA against the A antigen
IgG against the B antigen
T cytotoxic cell activation
Sensitization to antigens other than A or B

10. Which of the following is the MOST appropriate

5.

Which of the following organisms is commonly


located on the dorsum of the tongue?
A.
B.
C.
D.
E.

Actinomyces israelii
Streptococcus salivanus
Lactobacillus acidophilus
Prevotella rnelaninogenica
Streptococcusmutans

alternative for treating a patient allergic to


penicillin who has an infection caused by bacteria
sensitive to penicillin?
A.
B.
C.
D.
E.

Rifampin
Amoxicillin
Erythromycin
Tetracycline
Cephalosporin

11. In the past 30 years, the mortality rate of which of

16. A patient with rheumatic heart disease is MOST

the following forms of cancer in women has


shown a marked increase?

likely to develop congestive heart failure due to


which of the following?

A.
B.
C.
D.
E.

A.
B.
C.
D.
E.

Uterine cervix
Colorectal
Stomach
Breast
Lung

12. Normal clotting time and normal platelet count


accompanied by prolonged bleeding time
suggest which of the following?
A.
B.
C.
D.
E.

Hemophilia
Steatorrhea
Polycythemia Vera
Thrombocytopenic purpura
Excessive aspirin ingestion

13. Dry heat sterilization of dental instruments


requires
A.
B.
C.
D.
E.

3 minutes at 2 7 0 ' ~ (132'~).


20 minutes at 2 5 0 ' ~ (121'~).
30 minutes at 3 2 0 ' ~ (160'~).
60 minutes at 3 4 0 ' ~ (171'~).
6 hours at 1 0 0 ' ~(38'~).

14. Successful immunization with a hepatitis B


vaccine results in the development of serum
antibodies to which of the following?
A.
B.
C.
D.

HBsAg
HBcAg
HBeAg
HBV-DNA polymerase

17. Which of the following represents the phenotypic


expression of both alleles in a gene pair?
A.
B.
C.
D.
E.

autosomal dominant disorder?


A.
B.
C.
D.
E.

Type 1 diabetes mellitus


Type 2 diabetes mellitus
Gardner syndrome
Cleft palate
Gout

Penetrance
Lyonization
Codominance
Hybridization
Heterozygosity

18. Hereditary angioedema is the result of which of


the following?
A.
B.
C.
D.
E.

Lack of a thymus
Deficiency in C1 esterase inhibitor
Developmental arrest of lymphocytes
Defective VH gene recombination to DJH
Bare lymphocytes (no Class llll antigens)

19. Legionella pneumophilia is commonly transmitted


by
A.
B.
C.
D.
E.

15. Which of the following represents a simple

Heart murmur
Valvular insufficiency
Digitalis toxicity
Cor pulmonale
Aschoff bodies in coronary arteries

sexual contact.
water contaminated with human waste.
aerosolization of organisms in contaminated
soil.
direct contact with respiratory droplets from
a contaminated person.
aerosolization of organisms in air
conditioning cooling towers.

20. Which of the following BEST represents the


pathogenesis of jaundice in patientswith
hepatitis A?
A.
B.
C.
D.
E.

Massive hemolysis
Portal hypertension
Damage to liver cells
Massive fibrosis of the liver
Obstruction of main bile ducts

21. Which of the following pathologic changes is


irreversible?

A.
B.
C.

D.

Fatty change in liver cells


Karyolysis in myocardial cells
Glycogen deposition in hepatocyte nuclei
Hydropic vacuolization of renal tubular
epithelial cells

22. Which of the following is the major VIRAL cause


of birth defects in infants in developed countries?
A.
B.
C.

D.
E.

Measles
Cytomegalovirus
Herpes simplex
Rubella
Varicella-zoster

23. Warts are caused by which of the following?


A.
B.
C.
D.
E.

Papillomaviruses
Herpesviruses
Rhinoviruses
Adenoviruses
Rotaviruses

24. To which of the following molecules does HIV


bind?
A.
B.
C.
D.
E.

CD4
CD8
MHC Class I
MHC Class II
immunoglobulin

27. Uncomplicated healing of a wound by secondary


intention, observed microscopically at three days,
is MOST likely to show evidence of which ofthe
following?

A.
B.
C.
D.
E.

Mature cicatrix
Keloid formation
Granulomatous inflammation
Lack of acute inflammation
Ulceration of the epithelial surface

28. Which of the following represents the MOST


common autopsy finding in instances of sudden
death as a result of myocardial infarction?
A.
B.
C.
D.
E.

Valvular prolapse
Coronary thrombosis
Fibrinous pericarditis
Aneurysm of the heart wall
Rupture of the coronary vessel

29. Which of the following represents the


characteristic cellular infiltrate in delayed type
hypersensitivity?
A.
B.
C.
D.
E.

T cells and macrophages


Eosinophils
Neutrophils
B cells and plasma cells
Mast cells and basophils

30. Which type of oral candidiasis is likely to produce


a firmly adherent white plaque on the oral
mucosa?

25. Which of the following viral diseases has the


LONGEST incubation period?
A.
B.
C.
D.

E.

Herpetic gingivostomatitis
Common cold
Influenza
Measles
Rabies

A.
B.
C.
D.

Acute pseudomembranous
Chronic hyperplastic
Chronic atrophic
Acute atrophic

31. If a foreign antigen enters the body through the


26. Which of the following typically results in sudden
death?
A.
B.
C.
D.
E.

Bronchiectasis
Pneumoconiosis
Pulmonary emphysema
Pulmonary saddle embolus
Idiopathic pulmonary fibrosis

skin, which of the following portions of the


lymphoid system is the antigen MOST likely to
reach FIRST?

A.
B.
C.
D.
E.

Liver
Spleen
Thymus
Lymph nodes
Mucosal-associated lymphoid tissue

32. Each of the following is commonly associated with


congestive heart failure EXCEPT one. Which
one is this EXCEPTION?
A.
B.
C.
D.
E.

Dyspnea
Cyanosis
Anasarca
Ankle edema
Passive congestion of the liver

38. Which of the following genera of bacteria is MOST


commonly found on the skin?
A.
B.
C.
D.
E.

Candida
Brucella
Bacillus
Streptococcus
Staphylococcus

39. Which of the following is unique to gram-negative

33.

The appearance of new strains of human


immunodeficiency virus are primarily the result of
A.
B.
C,
D.

natural selection.
genomic recombination.
errors in translation.
errors in transcription.

34. Bleeding esophageal varices are a common

bacteria?
A.
B.
C.
D.
E.

Flagella
Capsules
Ribosomes
Endotoxins
Endospores

40. Inadequate immobilization after a bone fracture is

complication of which of the following conditions?

MOST likely to result in which of the following?

A.
B.
C.
D.
E.

A.
B.
C.
D.
E.

Cirrhosis
Hiatal hernia
Pancreatic carcinoma
Chronic active hepatitis
Hepatocellular carcinoma

Osteornyelitis
Osteoarthritis
Pseudoarthrosis
Heberden's nodes
Rheumatoid arthritis

41. Phage conversion is responsible for which of the


following?

35. Which of the following is the primary site of origin


of neuroblastoma?
A.
B.
C.
D.
E.

Brain
Kidney
Stomach
Mandible
Adrenal gland

36. In the process of cell death, lysosomal enzymes


function mainly to
A.
B.
C.
D.

autolyze necrotic cells.


mediate cell degeneration.
act as a major target for cell injury.
activate the complement sequence.

A.
B.
C.
D.

Transformation of bacteria
Production of beta-galactose
Production of pyrogenic toxin
Ability of Rhizobium species to fix nitrogen

42. Which of the following represents the BESTway


to verify heat sterilization?
A.
B.
C.
D.
E.

Using a chemical monitor


Recording sterilizer pressure
Recording duration of exposure
Recording sterilizer temperature
Testing with biological indicator

43. Which of the following diseases is MOST often


characterized by hyphae growing in and around
vessels?

37. The antigen-binding site of antibody molecules is


localized in the
A.
B.
C.
D.

Fab fragment.
Fc receptor.
J-chain.
constant region.

A.
B.
C.
D.
E.

Candidiasis
Mucormycosis
Blastomycosis
Histoplasmosis
Coccidioidomycosis

44. In either an allergic reaction or a parasitic

49. The process of gene transfer between bacterial

infection, which of the following cells increase in


number in the bloodstream?

cells that involves the uptake of naked DNA


molecules is classified as which of the following?

A.
B.
C.
D.
E.

A.
B.
C.
D.
E.

Basophils
Eosinophils
Plasma cells
Atypical lymphocytes
Nucleated erythrocytes

45. Which of the following viruses is characterized by

Conjugation
Transduction
Translocation
Transformation
Gene conversion

50. Obesity, striae, moon face, buffalo hump, and

latency and by the clinical symptoms that can


follow trauma, fever, or nerve damage?

osteoporosis are manifestations of which of the


following?

A.
B.
C.
D.
E.

A.
B.
C.
D.
E.

Variola
Influenza
Rubella
Herpes simplex
Coxsackievirus A

Myxedema
Acromegaly
Addison's disease
Cushing's syndrome
Diabetes insipidus

51. The capsule of Streptococcus rnutans is an


important virulence factor that

46. Which of the following is the MOST important


function of bacterial pili in causing human
infectious disease?
A.
B.
C.
D.

Making the bacteria motile


Allowing bacteria to adhere to human cells
Transfering DNA from one bacterium to
another
Destroying phagocytic macrophages and
neutrophils

47. Host responses against encapsulated


Streptococcuspneurnoniae are chiefly mediated
by which of the following?

A.
B.
C.
D.
E.

has proteolytic activity.


enhances oral accumulation.
causes circulatory collapse.
prevents phagocytic digestion.
is necessary to cause bacteremia.

52. Which of the following genera form endospores?


A.
B.
C.
D.
E.

Proteus and Escherichia


Clostridiurn and Bacillus
Porphyromonas and Streptococcus
Mycobacteriurn and Actinornyces
Staphylococcus and Corynebactenurn

53. Which of the following chemical agents has the


broadest antimicrobial spectrum of activity?

A.
B.
C.
D.
E.

IgE
IgD
Opsonins
Cytotoxic T lymphocytes
Delayed hypersensitivity

A.
B.
C.
D.
E.

Phenol
Ethanol
lsopropanol
Glutaraldehyde
Quaternary amine

54. Which of the following viruses does NOT establish


48. MOST rickettsia1 diseases can produce severe

a latent infection?

illness in humans because rickettsiae


A.
B.
C.
D.

produce potent exotoxins.


cause extensive CNS damage.
are destructive for epithelial cells.
are destructive for endothelial cells.

A.
B.
C.
D.
E.

Poliovirus
Epstein-Barr
Herpes simplex
Varicellazoster
Cytomegalovirus

55. Cor pulmonale is usually the direct result of


A.
B.
C.
D.
E.

viral pneumonia.
myocardial ischemia.
systemic hypertension.
resistance to blood flow through the lungs.
streptococcal hypersensitization.

60. A patient states that he immediately begins


wheezing and has difficulty breathing when
exposed to penicillin. Which ofthe following
classes of antibodies is responsible for the
difficulty in breathing?
A.
B.
C.
D.
E.

IgA
IgE
IgG
IgM
IgD

56. Which of the following represents one function of


macrophages in immunity?
A.
B.
C.
D.

Produce IL-2
Produce antibody
Present antigen to antigen-specific T cells
Mediators of antibody dependent cellular
cytotoxicity

61. Multiple drug resistance is related MOST closely


to bacterial
A.
B.
C.
D.
E.

plasmids.
chromosomes.
transduction.
transformation.
viruses in the bacterial wall.

57. What is the role of lactobacilli in coronal caries?


A.
B.
C.
D.
E.

Primary pathogen in most cases


Highly pathogenic in animal models
Causes progression of existing lesions
Highly adherent colonizer of smooth
surfaces
Consumes lactic acid produced by
streptococci

62. An enterotoxin formed by Staphylococcus aureus


causes
A.
B.
C.
D.

food poisoning.
a rash on the skin.
neuromuscular paralysis.
spasms of voluntary muscles.

63. The basic chemical structure of endotoxins


includes which of the following?

58. Which of the following represents the formation of


an intravascular blood clot?
A.
B.
C.
D.
E.

Thrombosis
Hemorrhage
Infarction
Embolization
Transudation

A.
B.
C.
D.
E.

Capsular polysaccharide
Acid stable peptidoglycan
H and L chain glycoproteins
Lipoteichoic acid, polysaccharide, protein
Somatic 0 polysaccharide, core
polysaccharide, lipid A

64. Which of the following causes whooping cough?

59. Which of the following statements is CORRECT

A.
B.
C.
D.
E.

Haemophilus intluenzae
Streptococcuspneumoniae
Bordetella pertussis
Brucella melitensis
Klebsiella pneumoniae

regarding glioblastoma multiforme?


A.
B.
C.
D.
E.

The tumor is most common before puberty.


It is classified as a type of meningioma.
It is the most common tvpe of astrocvtoma.
Its prognosis is generaliy more favorable
than Grade I astrocytoma.
It is derived from the epithelial lining of the
ventricles.

65. Which of the following disorders is LEAST likely to


be included in the differential diagnosis of a
patient with acute appendicitis?
A.
B.
C.
D.
E.

Crohn's disease
Duodenal peptic ulcer
Meckel's diverticulitis
Pelvic inflammatory disease
Gastroenteritis with mesenteric adenitis

66. Dimorphism in microorganisms is characterized


A.
B.
C.
D.

71. Penicillin is usually non-toxic to human cells


because human cells LACK which of the
following?

by the capability to produce


both sexual and asexual spores.
both a yeast phase and a mycelial phase.
two distinct types of clinical infections.
both by budding and by production of
spores.

A.
B.
C.
D.

Mitochondria
Peptidoglycans
Topoisomerases
Nuclear membranes

72. Anaerobic bacteria obtain their energy


67. During a Type I hypersensitivity reaction,
leukotrienes and prostaglandin
from which of the following?
A.
B.
C.
D.
E.

are generated

Thymus
Histamine
Arachidonic acid
Bradykinin and other kinins
Granules of mast cells and basophils

requirements from which of the following


reactions?
A.
B.
C.
D.
E.

Respiration
Fermentation
Atmospheric oxygen
Oxidative phosphorylation
Catalase and superoxide dismutase

73. In order for a virus to infect a host cell, it must first


a graft versus host reaction occur?

adsorb onto the cell surface. This process


necessitates

A.
B.

A.
B.

68. Under which of the following circumstances does

C.

D.
E.

When nonvital tissues are grafted


When viable lymphoid cells are present in
the graft
When the recipient and the donor are
syngeneic
When the graft is contaminated with gramnegative microorganisms
When the donor and the recipient are of
different races

69. Which of the following is associated with an

C.
D.

phagocytic activity by the cell.


an inhibition of the cellular secretion of
interferon.
an insertion of virally specified glycoproteins
into the host-cell membrane.
a specific binding of the virus to a
cell-membrane receptor.

74. Neisseria gonorrheae has affinity for which of the

increased risk of breast cancer?

following structures?

A.

A.
B.
C.
D.
E.

B.
C.
D.
E.

History of bearing multiple children


Estrogen deficiency
Silicone implants
Fibroadenoma
Obesity

Skin
Nerve cells
Plasma cells
Muscle fibers
Mucous membrane

75. Which of the following is MOST likely caused by


70. Which of the following are directly associated with
destruction of glomerular basement membranes
in a patient with glomerulonephritis?
A.

B.
C.
D.

Eosinophils
Lymphokines
IgE antibodies
Polymorphonuclear leukocytes

chronic blood loss due to a long-standing peptic


ulcer?
A.
B.
C.

D.
E.

Aplastic anemia
Hemolytic anemia
Pernicious anemia
Myelophthisic anemia
Iron deficiency anemia

76. An afebrile patient with conjugated


hyperbilirubinemia and an absence of
urobilinogen MOST likely has which of the
following conditions?
A.
B.
C.
D.
E.

Cholelithiasis
Aplastic anemia
Hemolytic anemia
Acute hepatitis
Alcoholic cirrhosis

81. Individuals with severe, uncomplicated pulmonary


emphysema might be expected to show which of
the following?
A.
B.
C.
D.

Cough
Chest pain
Normal or increased total lung capacity
Difficulty when breathing in (inspiration)

82. Which of the following is an oncogenic RNA


virus?

77. Which of the following genera of fungi is MOST


frequently recovered from healthy mucous
membranes?
A.
B.
C.
D.
E.

Candida
Aspergillus
Histoplasma
Blastomyces
Coccidioides

A.
B.
C.
D.
E.

Rotavirus
Retrovirus
Herpesvirus
Paramyxovirus
Papillomavirus

83. Candidiasis in the adult oral cavity may signify a


change in the balance of oral microbiota. This
particular change is often seen in persons who
are taking which of the following drugs?

Which of the following disinfectants is effective


against herpes simplex viruses but NOT
rhinoviruses?
A.
B.
C.
D.
E.

Phenolics
lodophores
Glutaraldehydes
Sodium hypochlorite
70% isopropyl alcohol

A.
B.
C.

Antiviral
Antifungal
Antibacterial

84. In addition to Kaposi's sarcoma, which other


malignant neoplasm is often observed in AIDS?
A.
B.
C.
D.
E.

Mycosis fungoides
Testicular carcinoma
Neuroblastoma
Non-Hodgkin's lymphoma
Rhabdomyosarcoma

79. The chemotactic accumulation of inflammatory


cells that occurs at the sites where immune
complexes are deposited is MOST probably due
to the presence of
A.
B.
C.
D.
E.

C5a.
Factor B.
IL-2.
IgA.
IgE.

80. In which of the following organs or tissues are


ascending infections common?
A.
B.
C.
D.
E.

Bone
Heart
Kidney
Peritoneum
Subcutaneous connective tissue

85. Which of the following represents a malignant


tumor arising from mesenchymal tissue?
A.
B.
C.
D.
E.

Sarcoma
Adenoma
Carcinoma
Hamartoma
Choristoma

86. Certain enzymes are responsible for suppuration.


These enzymes derive chiefly from
A.
B.
C.
D.
E.

serum.
tissue.
neutrophils.
lymphocytes.
plasma cells.

87. Which of the following represents the classic

92. Which of the following represents the epithelial

microscopic lesion of rheumatoid arthritis?

change MOST predictive of cancer?

A.
B.
C.
D.

A.
B.
C.
D.
E.

E.

Tophus
Pannus
Aschoff body
Heberden node
Wire-loop lesion

88. A 43-year-old woman exhibits radiographic


evidence of an osteolytic lesion of the humerus.
Her serum calcium level is elevated.
Microscopically, the bone lesion shows numerous
giant cells. Which of the following represents the
MOST probable diagnosis?
A.
B.
C.
D.

Renal rickets
Fibrous dysplasia
Osteitis deformans
Hyperparathyroidism

Dysplasia
Metaplasia
Acanthosis
Parakeratosis
Hyperkeratosis

93. Acute leukemias are MOSToften seen in which of


the following age groups?
A.
B.
C.
D.
E.

Under 20 years
20-40 years
40-60 years
60-80 years
Over 80 years

94. Which of the following is a major component of


nephrotic syndrome?

89. Adult respiratory distress syndrome might be


caused by each of the following EXCEPT one.
Which one is this EXCEPTION?
A.
B.
C.
D.
E.

Shock
Heroin overdose
Viral pneumonia
Cigarette smoking
Breathing 100 percent 0 2

A.
B.
C.
D.
E.

95. Which of the following is the MOST common skin


cancer in humans?
A.
B.

90. Which of the following is thought to be of MOST


significance in the
an uncontrolled diabetic?
A.

Genetics

endothelium

Anemia
Hematuria
Hypertension
Proteinuria
Red cell casts in the urine

C.
D.

E.

Malignant melanoma
Basal cell carcinoma
Squamous cell carcinoma
Sebaceous adenocarcinoma
Transitional cell carcinoma

96. Which of the following represents the MOST


probable cause for prolonged bleeding time in a
patient with leukemia?

A.
B.
C.
D.

E.

Decreased factor Vlll


lncreased factor IX
lncreased megakaryocytes
Decreased level of serum calcium
Decreased number of blood platelets

91. Rheumatic fever can be a sequelae to which of


the following streptococcal infections?
A.
B.
C.
D.

E.

Pyoderma
Diphtheria
Scarlet fever
Dental caries
Streptococcal gangrene

97. An abscess in the oral cavity with central necrosis


extruding pus containing sulfur granules would be
which of the following?
A.
B.
C.
D.

Furuncle
Tuberculosis
Actinomycosis
Vincent's infection

98. Patients with which of the following malignancies


have the poorest prognosis?
A.
6.
C.
D.
E.

Lung cancer
Malignant melanoma
Pancreatic carcinoma
Carcinoma of the colon
Squamous carcinoma of the tongue

99. Asymptomatic carriers are a major hazard of


which of the following?
A.
6.
C.
D.
E.

Plague
Shigellosis
Salmonellosis
Typhoid fever
Legionnaire's disease

100. Acid phosphatase is elevated in which ofthe


following malignancies?
A.
6.
C.
D.
E.

Multiple myeloma
Carcinoma of the breast
Squamous cell carcinoma
Carcinoma of the thyroid
Metastatic carcinoma of the prostate

NATIONAL BOARD DENTAL EXAMINATIONS


PART 1
TEST:
FORM:
DATE:

MICROBIOLOGY-PATHOLOGY
13 (A13)
D98

Item

Key

Item

Key

Item

Key

Item

Key

1
2
3
4
5

E
C
A
E
B

26
27
28
29
30

D
E
B
A
B

51
52
53
54
55

B
B
D
A
D

76
77
78
79
80

A
A
E
A
C

6
7
8
9
10

C
D
A
A
C

31
32
33
34
35

D
C
D
A
E

56
57
58
59
60

C
C
A
C
B

81
82
83
84
85

C
B
C
D
A

11
12
13
14
15

E
E
D
A
C

36
37
38
39
40

A
A
E
D
C

61
62
63
64
65

A
A
E
C
B

86
87
88
89
90

C
B
D
D

16
17
18
19
20

B
C

B
E
C

41
42
43
44
45

C
E
B
B
D

66
67
68
69
70

B
C
B
E
D

91
92
93
94
95

C
A
A
D
B

21
22
23
24
25

B
B
A
A
E

46
47
48
49
50

B
C
D
D
D

71
72
73
74
75

B
B
D
E
E

96
97
98
99
100

E
C
C
D
E

.f
.

10.

15.

As the subclavian vein crosses the first rib, it


lies

Which of the following nerves is the MOS'


likelyto become injured in fractures of th

mid-humeralshaft?

A
B.
C.
D.
E.

anterior to the anterior scalene muscle.


posterior to the anterior scalene
muscle.
posterior to the posterior scalene
muscle.
between the anterior and the posterior
scalene muscles.
between the scalene posterior and the
levator scapulae muscles.

A
B.
C.
D.
E.

16.

11. Which of the following BEST describes the

A
B.
C.

A
B.

D.
E.

C.

D.
E.

12.

13.

14.

A
B.
C.'
D.
E.

Secretory
Morphogenic
Organizing
Maturative
Protective

18.

Histologically,the osteoclasts of bone


resorption are typically
A
B.
C.
D.

anuclear.
mononuclear.
multinuclear.
polymorphonuclear.

19.

submucosal glands in the duodenum..~


only.'
simple columnqr epithelium lining the'
stomach only.
.
muscularis mucosa in the stomach
only.
smooth muscle in the external
musculature of the duodenum only.
..

frontal vein.
cavernous sinus.
anterior facial vein.
internal jugular vein.
superior petrosal sinus.

Each of the followingdevelops as an


outpocketing of the gut tube EXCEPT one.
Which one is this EXCEPTION?
A
B.
C.
D.
E.

inferior to the mandibular lingula.


superior to the auriculotemporal nerve.
lateral to the neck of the mandible.
lateral to the medial pterygoid muscle.
lateral to the stylomandibular ligament.

mucosal glands in the stomach only..

The superior and inferior ophthalmic veins


drain directly or indirectly into the
A
B.
C.
D.
E.

During an intraoral injection to the mandibular


foramen, the needle passes through the
mucous membrane and the buccinator
muscle. As it does so, the needle lies
A
B.
C.
D.
E.

Simple columnar
Simple squamous
Stratified cuboidal
Stratified squamous
Psuedostratified columnar

17.> It is possible to distinguish histologically \;1


between the stomach and the duodenum
because of the presence of

In the life cycle of an ameloblast, there are


cells that contain Tomes' processes. These
cells are in which of the followingstages?
A
B.
C.
D.
E.

Ulnar
Radial
Median
Axillary
Musculocutaneous

Which of the followingtypes of epithelium


well adapted for secretory or absorptive
functions?

passage of material through the hepatic


sinusoids?
Blood passes toward the central vein.
Bile passes peripherally toward the.
portal canal.
Lymph moves centrally to join the
sublobular duct.
Lymph moves peripherally toward the
space of Disse.
Blood passes peripherally away from.
the central vein.

Lung
Liver
Spleen
Pancreas
Gallbladder

24.

Which of the following processes can be seen


in the photomicrograph below?

of the following nerves contributes to


motor innervation of intrinsic muscles of

"

tongue?
.,
:.

"

it
,0
:1

..

A
B.
C.
D.

.,'

Osteoclasia
Endochondral bone formation
Intramembranous bone formation
Secondary center of ossification

Each of the following muscles receives motor


innervation from the ansacervicalis EXCEPT
. one. \Nhich one is this EXCEPTION?

!i

.,1

"~

:1

25.

A Omohyoid
B. Thyrohyoid
C. Geniohyoid
O. Sternohyoid
E. Sternothyroid

Which of the following structures leaves an


impression on the right lung?

A Azygos vein
B. Right vagus nerve
C. Right phrenic nerve
D. Descending thoracic aorta
E. Right common carotid artery

The three divisions of the trigeminal nerve


pass through openings in which of the
following bones?

26.

Which of the following represents the main


support of the wall of a bronchus?

A
B.
C.
D.
E.

Frontal
Sphenoid
Temporal
Parietal
Occipital

Smooth muscle
Hyaline cartilage
Elastic membranes
Dense irregular connective tissue
Elastic and collagenous connective
tissue

.,

66.

Melanocytes are derived from which of the


following?
A
B.
C.
D.
E.

67.

70.

Ectoderm
Endoderm
Mesoderm
Dermatomes
Neural crest

A
B.
C.
D.

71.

When both dentitions of the human are


considered, which of the followingrepresents,
in years, the life span of the dental lam~~

A
B..

31

C.

D. 9 ~~
E. 12\-t~

4' c:>C.o~

~ 0 ..'.

Levator veli palatini


Tensor veli palatini
Palatopharyngeus
Musculus Uvulae
Palatoglossus.

68. Cementum differs from dentin in that


cementum
A

8.

C.

D.

E.
,

72.

contains more inorganic material than


dentin.
is not formed followingeruption of the
tooth.
can contain cells, whereas dentin
contains cells as well as cell
processes.
is produced by cells of the periodontal
ligament, but dentin is produced by pulp
cells.
contains some elastic fibers, whereas
dentin contains only collagenous fibers.

From which of the following is the periodontal


ligament derived?
A
8.
C.
D.
E.

73.

69.

A dentist willuse screw-type implants to


replace the maxillary incisors (Teeth #s 7, 8,
9, and 10). If these implants pass through the
bone in this region, then which of the following
regions willthey be entering?
A
B.
C.
D.
E.

74.

Orbit
Mental foramen
Nasal cavity
Maxillarysinus
Pterygopalatine fossa

Greater stainability
Higher quantity of fluids
Lesser content of inorganic salts
Greater content of inorganic salts
Greater content of large collagen fibers

Which of the following represent fan-shaped,


hypocalcified areas that originate at the
dentinoenameljunctionand extend into
enamel for part of its thickness?
A
8.
C.
D.
E.

10

Dental sac
Enamel organ
Dental papilla
Epithelial root sheath
Outer enamel epithelium

Compared with intertubular dentin, peritubular


dentin is characterized by which of the
following?
A
B.
C.
D.
E.

Transverse temporal gyrus


Angular gyrus of the parietal lobe
Precentral gyrus of the frontal lobe
Paracentral lobule on the medial
surface

During swallowing, muscular contraction


results in movements that seal off the
oropharynx from the nasopharynx. Which of
the following muscles cause movements that
result in a fold in the posterior wall of the
pharynx?
A
B.
C.
D.
E.

----

Highlyskilled, discrete motor activity of the


hand is dependent on which of the following
cortical areas of the hemisphere?

Tufts
Spindles
Lamellae
Hunter-Schreger bands
Contour lines of Owen

ji
!
I

,
75.

The calcified bodies sometimes found in the


POL are BEST described as which of the
following?

A
B.
C.
D.
E.

80. The lingula of the mandible serves as an


attachment for which of the following?

A Temporalis
8. Stylomandibular ligament
C. Sphenomandibular ligament
D. Temporomandibular ligament

Cementicles
Denticles
Bone
Enamel pearls
Mineralized interstitial tissue

_.

81.

76.

Cell bodies of the somatic motor system lie in


which of the following locations?

A
B.
C.
D.
E.

D.

83.

79.

Facial
Accessory

Trigeminal
Hypoglossal
Glossopharyngeal

Optic nerve fibers from the nasal half of the


retina cross the midline and enter the optic
tract of the opposite side by way of the

A
B.
C.
D.
E.

optic chiasma.
lateral geniculate body.
bipolar cells of the retina.
brachium of the superior colliculus.
geniculocalcarine tract (optic
radiations).

Ameloblasts
Odontoblasts
Stellate reticulum
Stratum intermedium
Reduced enamel epithelium

In a newly erupted tooth, the junction between


teoth surface and the crevicular epithelium
c:Jnsists of which of the following?

, Interstitial crevicular fluid


1"'1.
S. Basal lamina-like structure between
enamel and epithelium
c. Basal lamina-like structure between
cementum and epithelium
D. Basal lamina-like structure between
dentin and epithelium
_. Keratin fibers, running from the
epithelium deeply into the enamel

nerve that supplies derivatives of the third


branchial arch?

'I/hich of the following is derived from


A
S.
C.
D.

78. Which of the following represents the cranial

A
B.
C.
D.
E.

primordium of the buccal frenula.


vestibular lamina.

ectomesenchyme7

Which of the following represents the location


of the cell bodies of pain fibers. in the
glossopharyngeal nerve?
A
Otic ganglion
B. Nucleus ambiguus
C.- Trigeminal-. (semilunar) ganglion
D. Spinal nucleu~ of cranial nerve V
E. Superior ganglion of cranial nerve IX

dental lamina for primary teeth.

s. dental lamina for permanent teeth.


C. primordium of the parotid gland.

82.
77.

In the developing embryo the palate is


separated from the lip by a shallow sulcus in
the depths of which two epithelial laminae
arise. The outer lamina is the
A.

Dorsal horn
Ventral horn
Autonomic ganglia
Dorsal root ganglia
Intermediolateral horn

Tendon of the digastric muscle

84.

vVhichgroup of fibers of the periodontal


ligament is the first to offer resistance to
movement of the tooth in an occlusal
direction?

A
8.
C.
D.
~
c:.

Alveolar crest
Interradicular
Horizontal
Oblique
Apical

-.it".:
.~~;

.,

85. Which of the followingBEST characterizes the

89.

alveolar mucous membrane?


A
B.
C.
D.
E.

A
B.
C.
D.
E.

Has no melanocytes
Firmly bound to underlying bone
Well developed epithelial ridges
Separated from the gingiva by the free
gingival groove
.
Appears red due to high vascularity and
thinness of epithelium

90.

innervation, which of the followingbranchial


arches are concerned in development of the
tongue?

C.
D.
E.

91.

First and second only


First, second, and third
Second and third only
Second, third, and fourth
Third and fourth only

Simple squamous
Simple cuboidal
Stratified squamous
Simple low columnular
Psuedostratified ciliated columnar

Which of the following muscles insert(s) onto


the neck of the condyle?
A
B.
C.
D.

86. As demonstrated by the pattern of sensory

A
B.

Salivary gland striated ducts are composed of


which of the followingtypes of epithelium?

Masseter
Temporalis
Lateral pterygoid
Medial pterygoid

Odontoblasts are characterized by


A
B.
C.
D.
E.

87. The apical cytoplasm of active serous

being located on external surfaces of


roots.
being shed from the tooth at the time of
eruption.
differentiating first at the cervical region
of a forming tooth.
secreting a non-fibrous matrix
'composed of chains of amino acids.
possessing long cytoplasmic
processes which lie within dentinal
tubules.

glandular cells is typically filledwith which of


the following?
A
B.
C.
D.
E.

92.

Large amount of DNA


Abundance of ribosomes
Abundance of mitochondria
Abundance of lipid droplets
Abundance of zymogen granules

Which of the following are pure serous


glands?
A
B.
C.
D.
E.

Sublingual glands
Glands of Brunner
Submandibular glands
Glands of van Ebner
Glands of Blandin-Nuhn

88.

93.

Each of the followingembryologic structures


is derived from the first branchial arch
EXCEPT one. Which one is this EXCEPT/ON?
A
B.
C.
D.
E.

Tuberculum impar
Maxillaryprocess
Mandibular process
Intermaxillary process
Lateral lingual swelling

Which of the following structures contacts


posteriorly with the isthmus of the thyroid
gland?
A
B.
C.
D.
E.

12

Larynx
Pharynx
Trachea
Esophagus
Carotid sheath

I'

94.

The infrahyoid muscles receive their motor


innervation from which of the following?
A .
B.
C.
D.
E.

95.

Vagus nerve
Supraclavicular nerves
Brachial plexus
Pharyngeal plexus
Branches of the cervical plexus

Hyaline cartilage
Fibrocartilage
Fibrous connective tissue
Articular cartilage
Elastic cartilage

A
B.
C.
D.
E.

99.

Dentinal tubules are S-shaped in the crown of

the toothdue to the


A
B.
C.
D.
E.

97.

The crescents or demilunes of the mucous


alveoli of the sublingual gland are composed
of which of the following cells?
Mucous
Serous
Neural
Striated
Myoepithelial

Which of the following types of tissues can be


demonstrated on the posterior slope of the
articular eminence?

A
B.
C.
D.
E.

96.

98.

incremental pattern.
epithelial diphragm.
crowding of odontoblasts.
.

A fracture of the hamulus affects the action of


which of the following muscles?

A
B.
C.
D.
E.

Superior constrictor of the pharynx


Levator veli palatini
Tensor veli palatini
Salpingopharyngeus
Buccinator

formationof peritubulardentin.
calcification pattern of maturing dentin.

Hertwig's epithelial root sheath is derived


from which of the following?
A

B.
C.
D.
E.

Inner dental epithelium and stellate


reticulum
Inner dental epithelium and stratum
intermedium
Outer dental epithelium and stellate
reticulum
Outer dental epithelium and stratum
intermedium
Inner dental epithelium and outer dental
epithelium

100. Cell bodies of proprioceptive fibers in V are


located in the

A
B.
C.
D.
E.

chief nucleus.
spinal nucleus.
semilunar ganglion.
geniculate ganglion.
mesencephalic nucleus.

.,

RELEASED

NATIONAL BOARD DENTAL EXAMINATIONS


PART 1
TEST:

ANATOMIC SCIENCES

FORM: 11
DATE: D96

Item

Key

Item

Key

Item

Key

1
2
3
4
5

D
E
B
A
D

26
27
28
29
30

B
B
C
C
D

51
52
53
54
55

A
C
D
A

6
7
8
9
10

B
A
A
C
A

31
32
33
34
35

D
A
A
E
D

11
12
13
14
15

A
A
C
D
B

36
37
38
39
40

16
17
18
19
20

A
B
B
C
E

21
22
23
24
25

D
C
B
B
A

'Item

Key

Do

76
77
78
79
80

B
E
E
A
C

56
57
58
59
60

B
D
A
C
D

81
82
83
84
85

E
B
B
E
E

B
E
B
A
A

61
62
63
64
65

B
D
E
C
A

86
87
88
89
90

B
E
D
D
C

41
42
43
44
45

E
C
D
D
C

66
67
68
69
70

E
D
C
C

91
92
93
94
95

E
D
C
E
C

46
47
48
49
50

B
D
E
B
B

71
72
73
74
75

C
A
D
A
A

96
97
98
99
100

C
E
B
C
E

- indicates item not scored


14

,,';

Part I

Biochem istry-Physiology Exam


.

December 1996
101. Which of the following vitamins is the LEAST
likely to be invoived in tooth development and
calcification?

A
B.
C.
D.

A
B1
C
0

106. Which of the following represents the chemical


substance that is the immediate source of
energy for muscle contraction?
A
Glycogen
B. Acetyl CoA
C. ..Lactic acid
D. Creatine phosphate
E. Adenosine triphosphate

102. The major function of serum LDL is to transport


which of the following? .
A
B.
C.
D.
E.

Bile salts from the. intestine


Triglycerides from the intestine
Free fatty acids from adipose tissue
Cholesterol and phospholipids from
peripheral tissues
Cholesterol, cholesterol esters, and
phospholipids from the liver

107. Intracellular and interstitial body fluids have


similar

A
B.
C.
D.
E.

total osmotic pressures.


colloid osmotic pressures.
sodium ion concentrations.
chloride ion concentrations.
potassium ion concentrations.

103. The fluid-mosaic model for membrane structure


proposes that
A

B.
C.
D.

104.

the outer and inner faces of the


membrane are identical.
peripheral proteins are situated only on
the outer face of the plasma membrane.
integral proteins are associated with the
hydrophobic phase of the bilayer.
both polar and nonpolar ends of
membrane phospholipids are within the
hydrophobic phase of the bilayer.

A protein in solution is at its isoelectric point


when the

A
B.
C.
D.
E.

pH is the same as that of the blood.


logarithm of the concentration is zero.
pH allows for maximum solubility.
pH is such that no migration occurs during
electrophoresis.
pH is produced by a 1:1 mixture of the
protein solution with isotonic saline.

105. The absolute refractory period of a nerve action


potential is determined by the duration of which
of the following?
A
B.
C.
D.

Sodium activation gate opening


Sodium inactivation gate closure
Potassium activation gate opening
Potassium inactivation gate closure

108. Breathing a gas mixture with 5 percent CO2


ultimately leads to which of the foHowing?

A
B.
C.
D.
E.

Hypoventilation
A decrease in heart rate
A stimulation of central chemoreceptors
An inhibition of peripheral chemoreceptors
A decrease in cerebrospinal fluid
hydrogen-ion concentration

109. Ovulation is triggered by a marked increase in


which of the following?
A
B.
C.
D.
E.

Estrogen
.Estradiol
Progesterone
Luteinizing hormone
Follicle-stimulating hormone

110. Which of the following must be digested before


being in a form that can be absorbed by
enterocytes?

A
B.
C.
D.
E.

Monoglycerides
Fatty acids
Fructose
Glycine
Maltose

"

,
116. Where in the autonomic nervous system is
norepinephrine stored?

111. Alpha-ketoglutarate, oxygen, and ascorbic


acid are essential for which of the following
processes?

A
A
8.
C.
D.
E.

Incorporation of proline
Hydroxylation of proline
Gamma-Carboxylation of proline
Oxidative deamination of lysine
Activation of procollagen peptidase

B.
C.

D.

Preganglionic sympathetic nerve


endings
Postganglionic sympathetic nerve
endings.
- - -Preganglionic parasympathetic nerve
endings
Postganglionic parasympathetic nerve
endings

112. Low Density Lipoprotein (LDL)particles get


into cells by
A
8.
C.
D.
E.

117. Which of the followingwill inactivate human


immune deficiency virus (HIV)reverse
transcriptase?

simple diffusion.
cell-cell fusion;
active transport.
facilitated diffusion.
receptor-mediated endocytosis.

A ATP
B. p.zr
C. dATP
D. Fluorouracil
E. Methotrexate

113. Which of the following represents the major


pathway for metabolism of excessive
intraneuronal free norepinephrine?
A
8.
C.
D.

E.

118. Which of the following hormones conserves


body protein, carbohydrate, and fat stores?

Hydrolysis by cholinesterase
Deamination by monoamine oxidase
Hydroxylation by - monoamine oxidase
Hydroxylation by dopamine beta
hydroxylase
Methylation by catechol-a-methyl
transferase

A
8.
C.
D.

114. Assumingthat P50 =26 torrs, underconditions

119. Each of the followingis involved in gene


cloning EXCEPT one. Which one is this
EXCEPTION?

where p02 =30 torrs, the average number of


2 molecules bound per hemoglobin
molecule is closest to
A
8.
C.
D.
E.

A
B.
C.
D.
E.

0.5.
less than 1.
almost 2.
greater than 2.
greater than 3.

DNA ligase
RNA polymerase
DNApolymerase I
Restriction nucleases
Reverse transcriptase

120. Which of the followinghas a high affinityfor


binding calcium and collagen in the calcifying
matrix?

115. In the DNA molecule, guanine on 1 strand is


joined to cytosine on the complementary
strand by which of the followingbonds?

A Amide
8. 1 hydrogen
C. 2 hydrogen
D. 3 hydrogen
E. Phosphodiester

Cortisol
Glucagon
Insulin
Somatotropin

A
8.
C.
D.
E.
16

Calcitonin
Osteogenin
Osteonectin
Amelogenin
Fibronectin

...
...

121.

In the glycolytic sequence, the enzyme ~hat


brings about the transition from 6-carbon
metabolites to 3-carbon metabolites is

A
B.
C.
O.
E.

122.

phosphoglucoisomerase.
phosphofructokinase.
phosphorylase.
hexokinase.
aldolase.

126.

During oxidative phosphorylation, the


energetically unfavorable proton gradient is
created using energy from which of the
following?

A
B.
C.
D.
E.

ATP hydrolysis
ATP synthesis
ADP transport
Electron transfers
The reduction of NAD+

Which of the following represents a striated


muscle that contains transverse tubules, a
slow rate of calcium sequestration, and is
inhibited by acetylcholine?

127. Which of the following explains why enamel is


A
B.
C.
D.

harder than bone?

Cardiac
Skeletal
Multi-unit smooth
Single unit smooth

B.

Enamel crystals are larger and more


firmly packed.
Enamel contains amelogenins in its
organic

C.
123. Tay-Sachs disease is associ~ted with an
inborn error of metabolism involving a specific
enzyme which normally degrades a particular
molecule in the gray matter. This enzyme acts
on which of the following?

A
B.
C.
D.
E.

D.
E.

""

Enamel contains more magnesium and


carbonate.
Enamel crystals have more surface
area.
Enamel contains more collagen.

"

Polysaccharides
Acylglycerols
Gangliosides
Fatty acids
Proteins

128. Which of the following represent(s) the matrix


proteins of enamel?
A

124. Vascular smooth muscle relaxes in response


to

A
B.
C.
D.
E.

matrix.

hyperoxia.
adenosine.

B.
C.
D.
E.

Carboxylglutamic acid containing


proteins
Type I collagen
Amelogenins
Proteoglycans
Elastin

vasopressin.
angiotensin.
norepinephrine.

125. Which of the following is MOST often


associated with free fatty acid transport in
human blood?

A Albumin
B. Globulin
C. Cholesterol
D. Sphingqlipid
E. Mucopolysaccharide

129. ADH receptors in the nephron are located on


the tubular membrane of which of the
following?
A
B.
C.
D.

"-

Distal tubule
Proximal tubule
Ascending loop of Henle
Descending loop of Henle

130. Which of the following changes promotes the


formation of extracellular edema?
A
8.
C.
D.

E.

131.

135. What single substance is effective in


reversing ketosis in a non-diabetic patient?

Increase in tissue fluid hydrostatic


pressure
Increase in plasma protein
concentration
Decrease in capillary hydrostatic
pressure
Capillary.filtration exceeds capillary
absorption
Capillary absorption exceeds capillary
filtration

A
8.
C.
D.
E.

136.

In addition to phosphoric acid, which of the


following are the products of hydrolysis of
lecithin?

A
B.
C.
D.
E.

D.

133.

134.

A Allosteric
8. Competitive
C. Irreversible
O. Uncompetitive
E. Noncompetitive

Has an amphiphilic surface


Contains 12 ions per unit cell
As found in bone and enamel contains
no ion substitutions
Has a higher solubility product constant
than fluoroapatite

138.

Polyuridylic acid in a cell-free system capable


of protein synthesis results in production of
polyphenylalanine. In this system, polyuridylic
acid functions as

A
B.
C.
D.
E.

Which of the following BEST explains why


proteins are able to buffer .physiologic
solutions over a wide range of pH?
A
B.

DNA.
transfer RNA.
messenger RNA.
ribosomal RNA.
mitochondrial RNA.

C.
D.

A derivative of vitamin K is the coenzyme for


which of the following?

A
8.
C.
D.
E.

Frequency
Amplitude
Wavelength
Secondary waves
Sympathetic vibrations

137. ATP inhibits phosphofructokinase even


though ATP also is a substrate for the
enzyme. Which of the following types of
inhibitiol1s BEST explains. this phenomenon?

132. Which of the following BEST characterizes


hydroxyapatite?

A
8.
C.

Intensity of sound is dependent mainly on


which of the following physical characteristics
of sound waves?

A
B.
C.
D.
E.

Glycerol, fatty acids, serine


Glycerol, fatty acids, choline
Sphingosine, acetic acid, inositol
Glyceraldehyde, fatty acids, choline
Glyceraldehyde, fatty acids,
ethanolamine

Urea
Glucose
Insulin
Leucine
Palmitic acid

They are macromolecules of high


molecular weight.
They contain many functional groups
with differing pKs.
They have unique tertiary structures that
sequester hydrogen ions.
They have peptide bonds that are
resistant to hydrolysis.

139. The intrinsic factor for vitamin 812absorption


is produced in the

Production of menadiol
Esterification of retinol
Hydrolysis of peptide bonds
Cross-linking of fibrinogen
Carboxylation of glutamate side chains

A
B.
C.
D.
E.
18

liver.
stomach.
pancreas.
duodenum.
lacteals.

....

140. Which of the following bonds link the


monomeric units of nucleic acids?
A
B.
C.
D.
E.

141.

A
B.
C.
D.
E.

occlusion.
summation.
adaptation.
facilitation.
sensory deprivation.

bone.
skin.
heart muscle.
nervous tissue.
connective tissue.

147. The largest amount.of body water can be


found in which of the following?
A
B.
C.
D.
E.

Emotional feelings are MOST closely related


to which area of the brain?
A
B.
C.
D.
E.

143.

Ionic
Peptide
Thioester
Glycosidic
Phosphodiester

Decreased response of sense organs when


exposed' to a constant stimulus is called

A
B.
C.
D.
E.

142.

146. Glutamate decarboxylase, an enzyme that


catalyzes the formation of gamma-amino
butyric acid (GABA), is unique to

Thalamus
Brain stem
Cerebellum

148. MOST fluid reabsorption by the kidney occurs


in which of the following?

Hypothalamus
Limbic system

A
B.
C.
D.
E.

Each of the following is a glycosaminoglycan


EXCEPT one. Which one is this EXCEPTION?
A

B.
C.
D.
E.

Urine
Blood plasma
Intracellular fluid
Interstitial fluid
Stomach and intestines

Chondroitin sulfate
Dermatan sulfate
Hyaluronic acid
Heparan sulfate
Keratin

Distal tubule
Proximal tubule
Collecting duct
Ascending loop of Henle.
Descending loop of Henle

149. Which of the following portions of the


cardiovascular system contains the greatest
volume of blood?

144. Which of the following minerals are


considered to be cariostatic?
A
Fluoride Phos hfA(.O~
B. Fluoride,
iu~

A
B.
C.
D.
E.

~\)

C. Co~~r,
phate
D. ~rt6~m,
Copper
E.' \:ruoride, Lead

145.

Each of the following describes hyaluronate


EXCEPT one. Which one is this EXCEPTION?
A
B.
C.
D.
E.

Polyanion
Highly polar
Glycosaminoglycan
Compact, folded structure
Extracellular matrix component

150.

Arterioles
Capillaries
Systemic veins
Chambers of the heart
Pulmonary vasculature

Each of the following enzymes functions in


association with a membrane EXCEPT one.
Which one is this EXCEPTION?

A Succinate dehydrogenase
B. Na+/K+ATPase
C. Adenylate cyclase
D. Phosphofructokinase
E. Coenzyme Q reductase

.,

151. Which of the following generates a slowly

156.

developing long-term response in target


tissues by binding to an intracellular receptor?
A
8.
C.
D.
E.

Hydrolysis of sucrose by the enzyme sucrase


yields

A
8.
C.
D.
E.

Glucagon
Estrogen
Prolactin
Growth hormone
Parathyroid hormone

glucose
glucose
glucose
glucose
fructose

only.
and maltose.
and fructose.
and galactose.
and maltose.

152. Which of the following functions as part of the


extracellular matrix?

A
8.
C.
D.
E.

157.

Mucin
Heparin
Collaginase
Chondroitin sulfate
Dolichol phosphate

Calcium ions initiate contraction in skeletal


muscle when they

A
8.
C.
D.
E.

bind to T tubules.
bind to troponin.
interact with actin.
interact with myosin.
bind to sarcoplasmic reticulum.

153. Which of the following represents the pH of a


solution that has a 10-5M concentration of OHion?

A
8.
C.
D.
E.

154.

5
7
9
Determinable only if the pkais known.
Determinable only if the base
composition is known.

158.

A
8.
C.
D.

Compared to slow-twitch (Type I) muscle


fibers, each of the following is more
characteristic of fast-twitch (Type II) muscle
fibers EXCEPT one. Which one is this
EXCEPT/Ow? .
A
8.

C.
D.
E.

159.

They contain more mitochondria.


They have higher myosin ATPase
activity.
They have fewer capillaries surrounding
them.
They have a more extensive
sarcoplasmic reticulum.
They contain higher concentrations of
glycolytic enzymes.

155. Which of the following is MOST likely to cause


respiratory acidosis?
Moving from high altitude to sea level
Untreated diabetes mellitis
Hypoventilation
Kidney failure
Vomiting

Cytoplasm
Membrane
Cell wall
Nucleus
Mitochondria

Each of the following characterizes a peptide


hormone EXCEPT one. Which one is this
EXCEPT/Ow?

A
8.
C.
D.
E.
20

Enterokinase
Peptidase
Secretin
Pepsin

Enzymes that catalyze the anaerobic


processes of carbohydrate metabolism are
found predominantly in which part of a cell?

A
8.
C.
D.
E.

160.

A
8.
C.
D.
E.

Which of the following enzymes converts


trypsinogen to trypsin?

Stored in secretory granules


Synthesized in a precursor form
8inds to intracellular receptors
Acts by generating a second
messenger
Usually transported unbound in plasma

161. Which of the following substances represents


an unsaturated fatty acid?

A
B.
C.
D.
E.

162.

Cholesterol
Palmitate
Stearate
Choline
Oleate

Which of the following acids represents both a


product of bacterial glycolysis and is
cariogenic?

A
B.
C.
D.
E.

166.

Lactic
Acetic
Succinic
Propionic
Phosphoric

Each of the following represents an amino


acid found in proteins and used directly in the
reactions of protein synthesis EXCEPT one.
Which one is this EXCEPTION?

A
B.
C.
D.
E.

167.

Proline
Arginine
Tryptophan
Asparagine
Hydroxylysine

For a reaction catalyzed by an enzyme with a


Km= 1 mM, which of the following represents
the effect on the velocity if [S] is changed from
10 mM to 20 mM? (Assume that the enzyme
obeys Michaelis-Menten kinetics.)

A
B.
C.
.D.
E.

Small decrease
Small increase
Twofold decrease
Twofold increase
Twentyfold increase

163. What linkages occur in glycogen at branch


points between glucose units?
A
B.
C.
D.
E.

Alpha-1,4
Alpha-1,6
Beta-1,3
Beta-1,4
Beta-1,6

168. Each of the following is an effect of parathyroid


hormone EXCEPT one. Which one is this
EXCEPTION?
A
B.

164. Protein kinase regulate the activities of key


enzymes through which of the following?

A
B.
C.
D.
E.

165.

Oxidation
Hydrolysis
Acetylation
Phosphorylation
Dephosphorylation

Regulation of each of the following


mechanisms is associated with the
hypothalamus EXCEPT one. Which one is
this EXCEPTION?

A
B.
C.
D.
E.

Sleep
Water balance
Body temperature
Pupillary diameter
Carbohydrate metabolism.

C.
D.

E.

Stimulation of 1-alpha-hydroxylase in
kidney
Stimulation of osteoclastic activity in
bo'ne
Stimulation of calcium reabsorption by
kidney
Inhibition of phosphate reabsorption by
kidney
Inhibition of intestinal absorption of
calcium

169. Which of the following liver enzymes, absent


from other tissues, gives the liver an
advantage over other cells in taking up
glucose after a meal?

A
B.
C.
D.
E.

Glucokinase
Aldolase
Hexokinase
Enolase
Glucose-6-phosphatase

,-

.,

170.

175. Which of the following enzymes or processes

In which of the following conditions might the


systolic blood pressure be abnormally high?
.A
B.
C.
D.
E.

ensures that the correct amino acid is


incorporated for a particular codon during
protein synthesis?

Cardiac shock
Heart failure
Anaphylactic shock
Decreased arterial compliance
Ventricular fibrillation

A
B.
C.
D.
E.

171. Which of the following occurs primarily in


adults and is characterized by increased
airway resistance, decreased diffusing
capacity of the lung, and chronicJperman~

A
B. Asthma
Em
C. Pn hYSe

D. C~
E.

176. Ifproteinis catabolized for energy,then MOST


of the energy is derived from which of the
following?

~~\J.

hypoxia?

Amino Acyl-t-RNA synthetase"


Ribosomal protein synthesis
Post-transcription splicing
RNA synthetase
Helicase

~ 0~ S(;O

A
B.
C.
D.
E.

a lung.

Hyalin membrane disease

Urea production
Oxidative deamination
Transamination reactions
Cleavage of peptide bonds
Oxidation of ex-ketoacids derived from
amino acids

172. The concentration of which of the following


amino acids can be used as an estimation of
the amount of collagen present in a tissue?
A
B.
C.
D.

173.

177. Each of the following lipid classes is

Hydroxyproline
Aspartic acid
Proline
Serine'

incorporated into membranes EXCEPT one.


Which one is this EXCEPTION?

A
B.
C.
D.
E.

If the air temperature is 105F and the relative


humidity is 10 percent, then which of the
following represents the primary cause for
loss of body heat?

A
B.
C.
D.
E.

178. The amplitude of an action potential can be


increased MOST easily by increasingwhich of
the following concentrations?

Evaporation
Conduction
Convection
Radiation
Hyperventilation

A
B.
C.
D.
E.

174. Which of the following hormones is released


by the zona fasciculata and exerts permissive
actions to allow insulin, glucagon and
epinephrine to work more effectively at their
target tissues?
A
B.
C.
D.
E.

179.

Androstenedione
Somatomedin
Aldosterone
Thyroxine
Cortisol

Intracellular sodium
Extracellular sodium
Intracellular chloride
Intracellular potassium
Extracellular potassium

Decreased arterial pressure upon standing is


compensated by
A
B.
C.
D.

22

~-

Cholesterol
Ganglioside
Triglyceride
Sphingomyelin
Phosphatidylcholine

decreased heart rate.


dilation of mesenteric vessels.
constriction of systemic arterioles.
dilation of venules.

184. Each of the following combinations lists the


name of the hormone, its chemical type, and
its major tissue of origin EXCEPT one. Which
one is this EXCEPTION?

180. Synaptic vesicle contents are released at the


neuromuscular junction when which of the
following occurs?
A
B.
C.
D.
E.

Hyperpolarization of motor end-plate


Shortening of skeletal muscle
Release of ATP from nerve terminal
Entry of calcium at nerve terminal
Release of acetylcholine from motor
end-plate

Hormone

181. Which of the followingcharacterizes exergonic


reactions?
A
B.
C.
D.
E.

Decreased entropy
Increased enthalpy
Decreased enthalpy
Negativefree energychange
Positive free energy change

Chemical
Type

Tissue
Adrenal
cortex
Posterior
pituitary
Adrenal
medulla
Anterior
pituitary
Hypothalamus

ACTH

Steroid

B.

Vasopressin

Peptide

C.

Epinephrine Catecholamine

D.

Thyrotropin

E.

Somatostatin Peptide

Glycoprotein

185. Which of the following causes activation of the


pyloricpump,relaxationof the pylorus,and
contractionof the loweresophageal sphincter?
A
8.
C.
D.
E.

182. In an aqueous solution at pH 7, a peptide,


containing 1 amino group side chain and 2
carboxyl group side chains would have which
of the followingnet charges?
A
. B.
C.
D.
E.

2+
1+
0
12-

186. An animal is in negative nitrogen balance if its


A
8.
C.
D.
E.

183. A subject consumes 250 ml of oxygen per


minute with a tidal volume of 400 ml and a
respiratory rate of 18 per minute. Which of the
following represents this subject's respiratory
minute volume in liters?

'"
~i

.,

~i
~i

A
B.
C.
D.
E.

Gastrin
Secretin
Pepsinogen
Acetylcholine
Cholecystokinin

urine is nitrogen-free.
nitrogen intake equals output.
nitrogen intake exceeds output.
nitrogen output exceeds intake.
new tissue is being synthesized.

187. Each of the following can cause glucosuria


EXCEPT one. Which one is this EXCEPT/Ow?
A
8.
C.
D.
E.

1.8
4.5
5.4
7.2
10.0
23

Low insulin level


High blood sugar level
Impaired tubular reabsorption
High glomerular filtration rate
High renal threshold for glucose

.-

--

"

,
188. The clearance rate for a substance that is

192. Which of the following represents the major


force that causes glomerular filtration?

completely removed from the blood during


one pass through the kidney is equal to which
of the following?
A
B.
C.
D.
E.

A
B.
C.

Renal plasma flow


Filtration fraction
Urinary excretion rate
Glomerular filtration rate
Tubular transport maximum

D.

Tubular hydrostatic pressure


Tubular colloid osmotic pressure
Glomerular capillary hydrostatic
pressure
Glomerular capillary colloid osmotic
pressure

193. If the plasma clearance of a substance which


is freely filtered is less than that of inulin, then
189. A marked fall from normal in the oxygen
tension in arterial blood would stimulate the
receptors in the
A
B.
C'.
D.
E.

A
B.

central nervous system


chemoreceptors.
aortic arch and the carotid sinus.
aortic and carotid bodies.
walls of the great veins.
respiratory center.

c.
D.
E.

the substance becomes bound to


protein in the tubules.
there is net secretion of the substance
in the tubules.
there is a net reabsorption of the
substance in the tubules.
the substance is neither secreted nor
reabsorbed in the tubules.
the substance is secreted in the
proximal tubule to a greater degree than
it is in the distal tubule.

190. Oxygen tension is GREATEST inwhich of the


following blood vessels?
A
B.
C.
D.
E.

194. Hypoactivity of the posterior pituitary gland


leads to

Aorta
Pulmonary vein
Pulmonary artery
Coronary artery
Coronary vein

A
B.
C.
D.

195. The metabolite, 25-hydroxycholecalciferol, is


derived MOST immediately from

191. Which of the followinggroups includes only


amino acids essential for humans?
A
B.
C.
D.
E.

dwarfism.
cretinism.'
acromegaly.
diabetes insipidus.

A
B.
C.
D.
E.

Valine, serine, leucine


Leucine, lysine, glycine
Tyrosine, threonine, tryptophan
Phenylalanine, methionine, proline
Tryptophan, methionine, isoleucine
24

ergosterol.
cholesterol.
7-dehydro-sitosterol.
7-dehydro-cholesterol.
22-dihydro-ergosterol.

"
196.

In a complete heart block, the


electrocardiograph shows which of the
following?

A
B.
C.
D.
E.

197.

Cardiac output is expressed as a product of


A
B.
C.
D.
E.

198.

An increased PO intelVal
An increase in height of the P wave
One ORS complex for every three P
waves
An increase in amplitude of the ORS
complex
Dissociation of the P wave and the ORS
complex

stroke volume and heart rate.


venous pressure and heart rate.
stroke volume and respiratory rate.
stroke volume and diastolic filling.
venous pressure and coronary blood
flow.

MOST of the CO2in blood is combined as


A
B.
C."
D.
E.

H2CO3.
HCO3-.
GH3-COOH.
carbonic acid.
carbaminohemoglobin.

199. A deficiency of choline in the diet can cause


abnormalities in the metabolism of
A
B.
C.
D.
E.

200.

lipids.
proteins.
minerals.

'

carbohydrates.
nucleoproteins.

Which of the following is a function of the


enterogastric reflex?
A

Decreases the opening of the

B.
C.
D.
E.

gastroesophageal
sphincter
Increases motility of the esophagus
Decreases motility of the stomach
Increases motility of the ileum
Increases gallbladder emptying

-=

'..
RELEASED

NATIONAL BOARD DENTAL EXAMINATIONS


PART 1
TEST:

BIOCHEMISTRY-PHYSIOLOGY

FORM: 12

DATE: D96
.

Item

Key

Item

Key

Item

Key.

Item

,Key

101
102
103
104
105

B
E
C
D
B

126
127
128
129
130

D
A
C
A
D

151
152
153
154
155

B
D
C
A
C

176
177
178
179
180

E
C
B
C
D

106
107
108
109
-110

E
A
C
D

131
132
133
134
135

B
D
C
E
B

156
157
158
159
160

C
B
A
A
C

181
182
183
184
18.5

D
D
D
A
A

E
A
B
D

186
187
188
189
190

D
E
A
C
B

.E

111"
112
113
114
115

B
E
B
D
D

136
137
138
139
140

B
A
B
B
E

161
162
163
164
165

116
117
118
119
120

B
B
C
B
C

141
142
143
144
145

C
E
E
D

166
167
168
169
170

E
B
E
A
D

191
192
193
194
195

E
C
C
D
D

121
122
123
124
125

E
A
C
B
A

146
147
148
149
150

D
C
B
C
D

171
172
173
174
175

A
A
E
A

196
197
198
199
200

E
A
B
A
C

.D

- indicates item not scored


26

.."
-~
,;~

Part I

Microbiology-Pathology

Exam

Decef11ber J 996

1.

6.

Which of the following is CORRECT for obligate


anaerobic microorgansims in the oral cavity?

A They do not exist in this area.


B. Only gram-positive organisms are found.
C. They are normal flora and opportunistic.
D. They are seldom isolated in the laboratory.
E. They can be completely controlled by

A
B. -.
C.
D.
E.

using antibiotics.

2.

'Iroooo

-.

Malignant epithelial cells have an increased


number and wider distribution of which of the
following receptors?

A
B.
C.
D.
E.

3.

\000&

7.

4~

Each of the following is a risk factor in


.
atherosclerosis EXCEPT one. Which one is this
EXCEPTION?

8.

influenza virus is difficultto transmit.


RSVcan be prevented with effective
vaccines.
secretory IgAis not effective in preventing
disease.

Which of the followingrepresent(s) the MOST


commonsource of pulmonary embolism?

._-

Which of the following is the single MOST

A
B.
C.
D.
E.

5.

RSV can be treated with amantadine.

RSVcauses disease primarily in infants.

A Esophageal varices
B. Endarteritis'
C. lymphangitis
.
D. Buerger's disease
E. Thrombophlebitis

numerous group of microorganisms in the oral


cavity?

Respiratory syncytial virus (RSV) infection differs


A
B.
C.
D.

----

0
25
33
50
75

frominfluenzainfectionsin that

Il-1
laminin
Histamine
Complement
Immunoglobulin

A Heredity
B. Alcoholism
c. Hypertension
D. Diabetes mellitus
E. Hyperlipoproteinemia

An autosomal dominant trait showing 50


percent penetrance will be phenotypically
expressed in what percent of the offspring?

9.

Enterococci
Staphylococci
Anaerobic streptococci
Facultative streptococci
Beta-hemolytic streptococci

Aflatoxin is produced by

A
B.
C.
D.
E.

Mucor.
Candida.
Tricophyton.
Penicillium.
Aspergillus.

!
\I
I

I
l

Each of the following characterizes a chlamydiaI


infection EXCEPT one. Which one is this
EXCEPTION?

10. Which of the following conditions predisposes


to lung cancer by causing squamous
metaplasia of bronchial epithelium?

A large numbers of asymptomatic carriers


B. Frequent co-infection with gonorrhea
C. The ability of the organism to survive in the
D.

A
B.
C.
D.
E.

host extracellularly
The greater likelihood that younger
women will acquire salpingitis

Bronchiectasis
Bronchial asthma
Chronic bronchitis
Bronchial carcinoid
Pulmonary emphysema

g;

2
~,

..
..

11. . The likelihood that oral bacteria play an

16.

important role in gingival inflammation is


evidenced by which of the following?

A
B.
C.
D.

12.

An increase in salivary hyaluronidase


An increased number of bacteria in
saliva
An increase of neutralizing antibodies in
saliva
A reduction of inflammation with
reduction of plaque

In addition to Neisseriameningitidis which of


the following is a significant cause of
meningitis?

A
B.
C.
D.
E.

A
B.
C.
D.
E.

involved in bacillary dysentery?

A
B.
C.
D.
E.

Streptococcus faecalis
Streptococcus pyogenes
Streptococcus pneumoniae
Staphylococcus aureus
Staphylococcus epidermidis'

Which of the following are MOST antigenic?

A
B..
C.
D.
E.

14.

Lipids
Haptens
Proteins
Nucleic acids
Carbohydrates

Which of the following characterizes victims of


fatal, acute carbon monoxide poisoning?

A
B.
C.
D.
E.

Vibrio
Shigella
Entamoeba
Salmonella
Escherichieae

Prostacyclin and thromboxane are products of


which pathway?
A

Complement cascade

B..

:..Cycl.ooxyg!:!nase

C.
D.

Lipoxygenase
Fibrinolytic

19. The MOST important viral cause of


gastroenteritis in children less than 2 years
old is

Cherry red blood


Acute renal failure
Massive liver necrosis
A hypercoagulability state
Anemia and generalized white cell
depletion

A
B.
C.
D.
E.

20.

15. Which of the following represents the chief

Microsporum canis.
Mycoplasma hominis.
Leptospira pomona.
Actinomyces israelii.
Histoplasma capsulatum.

17. Which of the following genera is MOST likely

18.

13.

A fungus that causes systemic disease, most


commonly of the lungs, and is characterized
by its production of tuberculate
chlamydospores in culture is

rotavirus.
echovi(1Js.
rhinovirus.
coxsackievirus.
cytomegalovirus.

MOST bacterial endotoxins are composed of


which of the following?

complication of mumps in the adult male?


A
B.
C.
D..

Orchitis
Prostatitis
Glomerulonephritis
Chronic nonspecific sialadenitis

A
B.
C.
D.
E.

Pure proteins
Pure carbohydrates
Heteropolysaccharides
Mucoprotein complexes
Lipo prote in-polysaccharid e complexes

..
"

21.

Which of the following is the principal factor


underlying localized edema in inflammation?
A
B.
C.
D.
E.

22.

24.

27.

A Subdural hematoma
B. Cerebral concussion
C. Cerebral infarction
D. Acute encephalitis
E. Alzheimer's disease

28.

Actinobacillus actinomycetemcomitans
Bacteroides forsythus
Fusobacterium nucJeatum
Porphyromonas gingivaJis
Prevotella intermedia

Which chemical substance is usually


secreted by pheochromocytomas?
,A
B.
C.
D.
E.

Which of the following represents the MOST


reliable postmortem indicator of left venticular
cardiac failure?

29.

Ascites
Venous congestion
Enlargement of the spleen
Peripheral edema of the ankles
Chronic passive congestion of the lungs

30.

I
I

II
I

are potentially teratogenic.


primarilyaffect the elderly.
are prevented by vaccinations.
have non-human animal reservoirs.
can not be treated with antibiotics.

Which of the followingprevent(s) the


synthesis of peptidoglycans?
A
B.
C.
D.
E.

Sarcoma
Fibroadenoma
Adenocarcinoma
Fibrocystic disease
Intraductal papilloma

Catecholamine
Aldosterone
Cortisone
Insulin
Renin

Rubella and toxoplasmosis are similar in that


both
A
B.
C.
D.
E.

Which of the following represents the MOST


frequent cause of a clinically palpable breast
mass in an adult woman?
A
B.
C.
D.
E.

Enhance secretion of IgA


Enhance antibody response
Stimulate complement synthesis
Desensitize to a given antigen
Activate mast cell degranulation

A patient lapses into a state of fluctuating


levels of consciousness or coma several
hours after sustaining blunt trauma to the
head. This finding is consistent with which of
the following conditions?

- ,,'-

A 17-year -old patient has periodontitis


involvingthe anterior teeth with sparse
plaque. Which of the followingis the probable
primary pathogen?

A
B.
C.
D.
E.

25.

A
B.
C.
D.
E.

The MOST common source of massive


hematemesis in alcoholics is (are)

A
B.
C.
D.
E.

Which of the followingdescribes the function


of adjuvants?

Lymphatic obstruction
Obstruction of venous outflow
Reduced intravascular osmotic
pressure
Protein leakage into tissue spaces
Increased capillary permeability

A peptic ulcer.
B. acute gastritis.
C. esophageal varices.
.Q:,,:;.Mallory-Weiss syndrome.
E. acute hemorrhagic pancreatitis.

23.

26.

Lysozymes
Penicillin
Tetracyclines
Erythromycin
Teichoic acids
~
j

"

31.

36.

Which of the following is MOST likely to cause


a sudden arrest of heart function?

A
B.
C.
D.
E.

Mitral stenosis
Angina pectoris
Constrictive pericarditis
Cardiac tamponade
Subacute bacterial endocarditis

Which of the following do striated muscle,


smooth muscle, and cardiac muscle have in
common?

Hyperplasia of these elements is


common.

B.

They have a limited capacity to


regenerate.
Hypertrophy is a common response to

C.

inJury. .

D.

32.

If a bacteria were susceptible to both penicillin


and erythromycin, then it would not be
appropriate to treat the patients with both
antibiotics at the same time because

37.
A
B.
C.

D.
E.

penicillin is inactivated by erythromycin.


erythromycin exerts its effect
extracellularly.
erythromycin enhances beta-Iactamase
activity.
erythromycin is primarily an anti-fungal
agent.
penicillin is only effective against
growing cells.

38.

GO~~

r\ c:

A deficiency in which of the following cells can


predispose to candidiasis?

A Basophils
B. Eosinophils
C. Macrophages
D. Plasma cells
E. T lymphocytes

39.
Which of the following BEST describes the
similarity between Streptococcus pneumoniae
and Cryptococcus neoformans?

T lymphocytes are MOST directly involved in


which of the following?

A
B.
C.
D.
E.

Commonly found in soil samples


Readily prevented by a vaccine
Readily treated by tetracycline
Antiphagocytic polysaccharide capsules

40.
The presence of which of the following in a
patient's serum affords protection agaiAst
hepatitis B?
A
B.
C.
D.

B cells
Macrophages

C. Killer(K)cells ~O
D. Cytotox~"i-!tlhh\C\ttes
E. NaturC1\~P<"iK)cells

A 'Polymyxin
B. Rifampicin
C. Ethambutol
D. Cycloserine
E. Ciprofloxacin

35.

Which of the following cells are thought to be

A
B.

gyrase inhibitor with a broad spectrum of


activity?

A
B.
C.
D.

They need a constant high 2


concentration to function.

MOST important in the control of metastase~

33. Which of the following represents a DNA

34.

.-

Cerebral embolism occurring as a


complication of myocardial infarction is MOST
indicative of which of the following?
A
B.
C.
D.
E.

Anti-HBcAg
Anti-HB sAg

Anti-HAV
Anti-HBeAg
5

Immediate hypersensitivity
Immune complex injury
Complement synthesis
Contact dermatitis
Antibody formation

Mural thrombosis
Phlebothrombosis
Decreased stroke volume
Lack of collateral circulation
Disseminated intravascular coagulation

'..
j]

41.

A
B.
C.
D.
E.

42.

The major cariogenic property of


Streptococcus mutans is associated with its
ability to produce which of the following
enzymes?

44.

47.

Their
Their
Their
Their

48.

physical appearance
intraoral locations
density and number
gradual confluence

49.

Platelets
Mast cells
Lymphocytes
Plasma cells
Activated macrophages

Sarcomere
Myofascia
Myocyte nucleus
Sarcolemmal membrane
Acetylcholine'receptor

Each of the following represents a common


opportunistic infection associated with HIV
EXCEPT one. Which one is this EXCEPTION?

A
B.
C.
D.
E.

Interleukin-1 and tumor necrosis factor in


inflammation are produced by which of the
following?

A
B.
C.
D.
E.

In myasthenia gravis, autoantibodies are


directed against which of the following
structures?

A
B.
C.
D.
E.

Hyaluronidase
Chondroitinase
Aminopeptidase
Glucosyltransferase
Fructosyltransferase

The oral lesions of herpangina can often be


distinguished from hand-foot-and-mouth
disease by which of the following?
A
B.
C.
D.

In the process of necrosis, a reduction in the


size of the nucleus and a condensation of
nuclear material is known as

A pyknosis.
B. karyolysis.
C. karyorrhexis.
D. metachromasia.
E. hyperchromatism.

IL-2
Kinin
Tryposin
Complement
Histamine

A
B.
C.
D.
E.

43.

46.

Natural killer cells capable of destroying


malignant cells MUST be first activated with
which of the following?

Candidiasis
Hairy leukoplakia
Adenovirus conjunctivitis
Cryptosporidium enterocolitis
Pneumocystic carinii pneumonia

Acute biliary obstruction produces a rise in

A urobilinogen in urine.
B. urobilinogen in stool.
C. conjugated bilirubin in urine.
D. conjugated bilirubin in serum.
E. unconjugated bilirubin in serum.

';~1

45.

A viremia is characterized by which of the


following?
A
B.

c.
D.

50.

Infected cells are transformed.


Infected cells are no longer releasing
virus.
The virus is most easily spread via
gastrointestinal secretions.
The virus is most susceptible to
circulating antibodies.

Which of the following is the usual cause of


chronic pyelonephritis?

A
B.
C.
D.
E.
6

Systemic hypertension
Type IV hypersensitivity
Chronic glomerulonephritis
Bacteremic seeding of the kidneys
Infection resulting from urinary reflux

.~

,.,.
51.

55.

If serial dilutions of human saliva are


inoculated on blood agar plates, then
incubated in air for 24 hours at 37C, which of
the following groups of microorganisms will'
grow in the greatest number?

A
B.
C.
D.
E.

cell to appear in large numbers at the site of


injured tissue?

A
B.
C.
D.
E.

Actinomycetes
Streptococcus
Lactobacillus
Staphylococcus
Fusobacterium

56.
52.

High levels of Porphyromonas


gingivalis-specific antibodies are seen in
MOST of the serum samples from patients
with adult periodontitis. These antibodies are
which of the following?

A
B.
C.
D.
E.

53.

54.
'i

IgM
IgG
IgE
IgA
IgD

57.

58.

Phenylketonuria
Wilson's disease
Diabetes mellitus
T ay-Sachs

MOST antibiotic resistance in bacteria is


caused by which of the following?
A
B.

C.
D.
E.

59.

Genes that are carried on plasmids


Depression of a previously existing
gene pool
An ever increasing mutation rate in
bacteria
Unequal distribution of antibiotics into
body spa~es
Induction of specific mutations by
specific antibiotics

Oral yeast
Hepatitis B virus
Bacterial endospore
Mycobacterium tuberculosis
Human immunodeficiency virus

motile.
sensitive to H2O2.
able to ferment lactate.
resistant to penicillin.
producers of 2 and H2Ofrom H2O2'

A 6-year-old boy has minute white specks on


the oral mucosa adjacent to his first molars. A
bluish-red ring surrounds these spots. He
appears to have a cold and his eyes are red
and runny. There is a blotchy reddish rash
behind his ears and on his face. This child
has which of the following?

A
B.
C.
D.
7

Phenols
Chlorhexidine
Ethylene oxide
70 percent isopryl alcohol
Alkaline glutaraldehyde

Anaerobic bacteria lacking catalase are


generally

A
B.
C.
D.
E.

disease

Neutrophil
Lymphocyte
Monocyte

Which of the following is MOST resistant to


sterilizing conditions?

A
B.
C.
D.
E.

Galactosemia

Plasma cell

Eczema
Measles
Chickenpox
Scarlet fever

.-

Macrophage

Which of the following is an antimicrobial


agent that primarily inactivates cellular DNA?

A
B.
C.
D.
E.

Excessive deposits of copper in liver cells,'


degenerative changes in the brain, and a
greenish-brown ring at the outer margin of the
cornea characterize which of the following?

A
B.
C.
D.
E.

Which of the following is the first inflammatory

""".

-"

.,

60.

A
B.
C.
D.
E.

61.

65.

In anemia resulting from drug-induced bone


marrow suppression, the peripheral blood
smear shows erythrocytes to be

A
B.
C.
D.

hypochromic - microcytic.
hypochromic - normocytic.
normochromic - normocytic.
normochromic-microcytic.
hyperchromic - macrocytic.

E.

66.

A hemorrhagic tendency is seldom seen in


which of the following conditions?

A
B.
C.
D.
E.

The spores of Bacillus anthracis are


destroyed by

Scurvy
Acute leukemia
Renal insufficiency
Hepatic insufficiency
Secondary thrombocytopenia

refrigerating (-7C for 48 hours).


autoclaving (121C for 20 minutes).
pasteurizing (61.7C for 30 minutes).
immersing in boiling water (100C for
10 minutes).
placing in a hot air oven (121C for 20
minutes).

Congestion in the early stages of


inflammation is caused by which of the
following?

A
B.
C.
D.
E.

Ischemia
Venous dilation
Active hyperemia
Venous constriction
Lymphatic obstruction

,
t

J
I
i
I

62.

67~

In which of the following is the largest amount


of genetic information transferred from one
cell to another?

A
B.
C.
D.
E.

A
B.
C.
D.
E.

F-dudion
Conjugation
Transduction
Transformation
Recombination

68.

63.

In a dry-heat oven, which of the following


temperatures is sufficient for achieving
sterilization in 1-2 hours?
A
B.
C.
D."

81C
100C
121C

-I

I;

Candida albicans
Histoplasma capsulatum
Blastomyces dermatitidis
Cryptococcus neoformans
Trichophyton mentagrophytes

Deletions
Inversions
Frameshifts
Transitions
Transversions

160C

Which of the following is the site of oxidative


phosphorylation in bacteria?

A
B.
C.
D.
E.

Which of the following kinds of mutations is


caused by base analogues?

A
B.
C.
D.
E.

69.
64.

The finding of yeast cells and


chlamydospores in the oral mucosa suggests
which of the following?

Which of the following acids is the chief


product of carbohydrate metabolism of
Streptococcus mutans?

A
B.
C.
D.
E.

Nucleus
Ribosome
Cytoplasm
Cell membrane
Mitochondrion
8

Acetic
Formic
Lactic
Butyric
Propionic

..
70.

A
B.
C.
D.

71.

Bacillus subtilis
Lactobacillus casei
Streptococcus mutans
Staphylococcus aureus

,i

E.

Renin
Histamine
C3and Cs
Bradykinin
Prostaglandins

76.

.\

'.

Gout results from a metabolic defect in which


of the following?
A
B.

72.

73.

Delayed hypersensitivity
Tuberculin reaction
Contact dermatitis
Arthus reaction
Anaphylaxis

77.

Fat
Purine
Pigment
Glucose
Calcium

Which of the followingviral-associated


enzymes is unique to RNA tumor viruses?
A
B.
C.
D.

Neuramidase
Capping enzyme
Reverse transcriptase
DNA-dependent RNA polymerase

Which of the following neoplasms appears


the MOST often in children?

A
B.
C.
D.
E.

74.

C~
D.
E.

The histamine that is released by mast cells


is.responsible for the principal symptoms of
which of the following?
A
B.
C.
D.
E.

Iron deficiency
Chronic external loss of blood
Frequent episodes of hemolysis
Inhibition of hematopoiesis by
sulfonamides
Inabilityof the stomach to form intrinsic
factor
'

Laboratory studies reveal megaloblasts in


bone marrow, and a hyperchromic, macrocytic
anemia in peripheral blood. Oral examination
reveals atrophic glossitis. Which of the
followingis the MOST likely cause of this
condition?
A
B.
C.
D.

Inflammatoryvasodilation can be evoked by


each of the followingEXCEPT one. Which one
is this EXCEPTION?
A
B.
C.
D.
E.

75.

Which of the followingorganisms represents


a significant secondary invader of carious
lesions?

78.

Neuroblastoma
Chondrosarcoma
Adenocarcinoma
Multiple myeloma
Basal cell carcinoma

A
B.
C.
D.
E.

When horse serum is injected intravenously


into a rabbit and again into the skin two or
three weeks later, what is the necrotizing
reaction that occurs at the site of the second
injection?
A
8.
C.
D.
E.

Which of the following bowel diseases is


characterized by noncaseating
granulomatous inflammation in the gut wall?

79.

Each of the followingcells is an


antigen-specific cell EXCEPT one. Which one
is this EXCEPTION?
A
B.
C.
D.
E.

Atopy
Anaphylaxis
Serum sickness
Arthus phenomenon
Prausnitz-Kustner reaction
9

Diverticulitis
Peptic ulcer
Ulcerative colitis
Celiac disease
Crohn's disease

B cells
Macrophages
T helper cells
Langerhans cells
Dendritic cells

,-

"

,
80.

86. A 53-year-old patient has an indurated.

Streptococcus pyogenes, group A, is


subdividedinto specificantigenictypes
principally on the basis of immunologic
differences in its

chronic ulcer near the inner canthus. The


MOST likely diagnosis is

'

A
B.
C.
D.
E.

A
B.
C.
D.
E.

Mprotein.
streptolysin O.
streptolysin S.
C polysaccharide.
hyaluronic acid capsule.

verruca vulgaris.
malignant melanoma.
basal cell carcinoma.
squamous cell carcinoma.
adenocarcinoma of the lacrimal duct.

81. Which of the followingbacterial exotoxins


87.

converts plasminogen to plasmin?


A
B.
C.
D.
E.

M-protein
Coagulase
Plasmalysin
Hyaluronidase
Streptokinase

Which of the following bacteria causes


epidemics of meningitis among adults? i-Q
A Staphylococcus aureus c...f;;O~
B. Klebsiella pneumoniif' -;;I
C. Haemophilus iQ~)e
D. Neisseri~ ~~~s
E.

82. Which of the followingis the LEAST likely

Stre\~~~r

pneumoniae

complication of nodular prostatic hyperplasia?


A
B.
C.
D.
E.

Dysuria
Hydronephrosis
Pyelonephritis
Development of carcinoma
Urinary tract obstruction

88.

A Bacterial infections
B. Viral infections
C. Fungal infections
D. Parasitic infestations

83. A positive tuberculin test always indicates


which of the following?
A
B.
C.
D.

Active tuberculosis
Chronic tuberculosis
No exposure to tuberculosis
Hypersensitivity to tuberculoproteins

89.

84. An endocrine disorder affecting the skeleton

hyperadrenalism.
hyperthyroidism.
hyperpituitarism.
hyperparathyroidism.
hyperfunction of the thymus.

90.

85. Which of the followingtumors is of connective


tissue origin?
A
B.
C.
D.
E.

Bone pain, osteolytic lesions, plasma cell


infiltrationof marrow and synthesis of
abnormal immunoglobulins describe
A
B.
C.
D.
E.

and kidney, that promotes metastatic


calcification, is
A
B.
C.
D.
E.

Eosinophils are characteristically seen in


which of the following conditions?

Which of the followingcharacterizes edema


associated with the nephrotic syndrome?
A
B.
C.
D.
E.

Myxoma
Adenoma
Melanoma
Carcinoma
Papilloma
10

systemic lupus erythematosus.


selective IgAdeficiency.
polyclonal gammopathy.
Paget's disease of bone.
multiple myeloma.

Polyuria
Hematuria
Hyperlipidemia
Hypoalbuminemia
Hypoprothrobinemia

c,

....

91.

A
B.
C.
D.
E.

92.

96.

Blood in the sputum is characteristic of each


of the following EXCEPT one. Which one is
this EXCEPTION?

A
B.
C.
D.
E.

Emphysema
Tuberculosis
Lobar pneumonia
Pulmonary embolism
Bronchogenic carcinoma

risk of developing osteosarcoma?

A
B.
C.
D.
E.

alpha-hemolytic streptococci.
Staphylococcus aureus.
a variety of viruses.
herpes simplex virus.
hemophilus influenzae.

98.

Which of th~ following components of


hypersensitivity, and causes platelet injury?

A
B.
C.
D.
E.

Necrosis
Exudation
Epithelioid cells
Langhans' giant cells
Spreading of the initial focus

99.
Administration of tetanus toxoid provides what
type of immunity?

A
B.
C.
D.
E.

Which of the following represents the MOST


potent carcinogen?

A
B.
C.
D.

11

Aplasia
Anaplasia
Pleomorphism
Hyperchromatism
Abnormal mitosis

Immunological resistance to MOST


intracellular pathogens is manifested as
which of the following?

A
B.
C.
D.
E.

Estrogen
Benzpyrene
Folic acid
Cholic acid

Protein A
Coagulase
Beta toxin
Teichoic acid
Polysaccharide capsule

Each of the following is a histologic feature of


malignant growth EXCEPT one. Which one is'
this EXCEPTION?

A
B.
C.
D.
E.

Innate
Natural active
Natural passive
Artificial active
Artificial passive

100.

95.

Osteomalacia
Osteoporosis
Osteoblastoma
Osteitis deformans
Osteogenesis imperfecta

Staphylococcusis antiphagocytic,, elicits


Hypersensitivity to M. Tuberculosis is
manifested by which of the following?
A
B.
C.
D.
E.

94.

nonencapsulation.
rapid growth rate.
ability to metastasize.
lack of differentiation.
excessive mitotic activity.

97. Which of the following conditions increase the


The majority of cases of pharyngitis are
caused by

A
B.
C.
D.
E.

93.

The MOST important characteristic of


malignant neoplasms, which distinguishes
them from benign neoplasms, is their

Lymphoid .atrophy
Humoral immunity
Cellular immunity
Wheal and flare reactions
Non-specific serum protection

.
-

,; .

..

RELEASED

NATIONAL BOARD DENTAL EXAMINATIONS


PART 1
TEST: lVIICROBIOLOGY-PATHOLOGY
FORM: 13
DATE: D96

Key

Item

Key

Item

Key

Item

Key.

C
B
B
D
C

26
27
28
29
30

B
A
A
A
B

51
52
53
54
55

B
B
C
A
C

76
77
78
79
80

B
C
E
C
A

6
7
8
9
10

B
B
E
E
C

31
32
33
34
35

D
E
E
D
B

56
57
58
59
60

C
C
B
B
C

81
82
83
84
85

E
D
D
D
A

11
12
13
14
15

D
C
C
A
A

36
37

38
39
40

E
D
A

61
62
63
64
65

C
B
D
D
B

86
87
88
89
-90

C
D
E
D

16
17
18
19
20

E
B
B
A
E

41
42
43
44
45

A
D
B
E
D

66
67
68
69
70

C
A
D
C
B

91
92
93
94
95

A
C
A
D
B

21
22
23
24
25

E
C
A
E
D

46
47
48
49
50

A
E
C
D
E

71
72
73
74
75

A
E
A
D
E

96
97
98
99
100

C
D
A
A
C

Item
1
2
3
4
5

.-

- indicates item not scored


12

Part I
December 1996

Dental Anatomy and Occlusion.

105. 'tl11ichof the following incisors

All test items refer to permanent teeth unless


"primary" is specified.

has its mesial


and distal contact areas at the same
incisocervical level?

All test items relating to occlusion refer to a Class I


canine and molar relationship unless otherwise
specified. Terms such as "norma'" or "ideal" are
synonymous with the above definition.

A
B.
C.
D.

101. In an ideal intercuspal relation, the lingual cusp


of the maxillary second premolar contacts the
A
B.
I
C.
I
I

D.

106. 't'hich of the following is correct regarding the


contact relationship of maxillary central incisors
and maxillary lateral incisors?

distal fossa of the mandibular second


premolar.
mesial marginal ridge of the mandibular
first molar.
distal marginal ridge of the mandibular
second molar.
distal marginal ridge of the mandibular
first premolar.

A
6.
C.

Contacts are centered faciolingually.


Contacts are centered incisocervically.
Facial embrasures are wider than lingual
embrasures."'~

D.

Incisal embrasures are larger than


gingival embrasures.

107. Tne distal contact area -ofa maxillary lateral


incisor with ideal alignment is located

102. Which of the followingincisal angles of maxiJIary


teeth exhibits the.greatest convexity?
A
B.
C.
D.

Maxillarycentral
Mandibular central
Maxillarylateral
Mandibular lateral

A
8.
C.
D.

Mesioincisal of the central


Distoincisal of the central
Mesioincisal of the lateral
Distoincisalof the lateral

E.

near the incisal edge.


in the middle third.
in the incisal third.
at the junction of the incisal and middle
thirds.
at the junction of the middle and cervical
thirds.

103. Which of the followingrepresents the geometric


form of anterior teeth when viewed from the
mesial or distal aspect?
A
B.
C.
D.
E.

108. Which of the followingteeth have long axes


positioned with their root apices facial and their
crowns lingual?

Rhomboidal
Trapezoidal
Elliptical
Triangular
Square

A B.
C.
D.
E.

Maxillaryincisors
Mandibular incisors
Maxillary premolars
Maxillarymolars
Mandibular molars

.,

104. Each of the followingcusps of the maxillary


molars is part of the maxillarymolar primary
cusp triangleEXCEPTone. Whichone is this
EXCEPTION?
A
B.
C.
D.

109. In (at) which of the following is the mesial


contact area of a maxillary canine located?

Mesiofacial
Mesiolingual
Distofacial
Distolingual

A
B.
C.
D.
13

.-

The incisal third


The junction of incisal and middle thirds
The middle third
The junction of middle and cervicalthirds

.,

110. Which of the following represents the tissue


formed in response to stimuli produced by
carious penetration of a tooth?

115. Maxillarytooth crowns exhibit concavities on


which of the followingsurfaces?
A

A
B.
C.
D.

Predentin
Primary dentin
Secondary dentin
Interglobular dentin

B.
C.

D.
111. Calcification of the mandibular third molars
generally begins at
A
B.
C.
D.
E.

E.

The mesial of central incisor and first


premolar
The mesial of canine and first molar
The distal of first premolar and the
mesial surface of second molar
The mesial of first premolar and the
distal of first molar
The distal of the first molar and the
mesial of the second molar

3-4 years.
5-7 years.
8-1 0 years.

116. The cross-sectional view at midroot illustrated


belowis MOSTprobablythat of a

11-13 years.
14-16 years.

112. Each of the following morphologic structures


can be seen on any incisor EXCEPT one.
Which.one is this EXCEPTION?
A
B.
C.
D.

Cingulum
Mesial marginal ridge
Lingual fossa
Transverse ridge

A
B.
C.
D.
E.

113. In contrast to maxillary canine crowns,


mandibular canine crowns have which of the
following anatomically?
A
B.
C.
D.
E.

Greater measurements, mesiodistally


Greater measurements, faciolingually
More accentuated marginal ridges
Well-defined lingual pits
Less-pronounced cingula

117.

Postural
Intercuspal
Retruded contact
Protruded contact

~
118.

A
B.
C.
D.

Which of the following jaw positions is


determined almost exclusively by the behavior
of the musculature?

A
B.
C.
D.

114. Illustrated below is a frontal border tracing of a


patient with a canine-guided occlusion. Which
letter on the diagram defines the only point
where posterior tooth contact occurs?

mandibular canine.
maxillary first premolar.
mandibular first premolar.
lingual root of a maxillary molar.
distal root of a mandibular first molar.

In an ideal intercuspal relation, the


mesiolingual cusp of-the maxillary first molar
contacts the mandibular first molar in (on) the

A
B.
C.
D.
E.

A
B
C
0
14

central fossa.
distal fossa.
mesial fossa.
mesial marginal ridge.
distal marginal ridge.

."'.
.~

.
~

"

119.

124.The

In an ideal intercuspal relation in a normal


dentition, each of the following teeth contacts

each other EXCEPTone. Whichone is this

periodontal

ligament

fibers are primarily

EXCEPTION?
A

8.
C.
D.

120.

C.
D.
E.
121.

125. On the non-working side in an ideal


occlusion, interfering contacts on posterior
teeth will be located on which inclines of
which cusps?

Distal
Mesiofacial
Mesiolingual
Distofacial
Distolingual

A
8.
C.
D.

8.
C.
D.
E.

Cusps

Outer
Inner
Cuter
Inner

Guiding
Guiding
Supporting
Supporting

126. Which premolar has a facial cusp with a


triangular ridge so uniquely prominent as to
frequently separate its mesial pit from its
distal pit?

A
8.
C.
D.

Facial groove of the mandibular second


molar
Distofacial developmental groove of the
first molar
Embrasure between the mandibular
first and second molars
Embrasure between the mandibular
second and third molars
Central fossa

Maxillaryfirst
Mandibular first
Maxillary second
Mandibular second

127. Which of the following ligaments has an outer


oblique portion which limits the extent of jaw
opening and initiates translation of the
condyle down the articular eminence?

Which of the following primary teeth would


exhibit a prominent cervical ridge on both the
facial and lingual surfaces?

A
8.
C.
D.
E.

Inclines

triangular.
roughly conical.
flattened in a mesiodistal direction.
broader mesiodistally on the lingual
than on the facial.

In an ideal intercuspal relation, the tip of the


mesiofacial cusp of the maxillary second
molar opposes which of the following?
A

123.

Maxillary first premolar and mandibular


second premolar
Maxillary first molar and mandibular
second premolar
Maxillary second premolar and
mandibular first molar
Maxillary second molar and mandibular
third molar

In cervical cross-section, the root of a


mandibular canine is described as

A
8.
C.
-D.

122.

A
Hyaline
8.
Elastic
C..90Ilagenous
D. Fibrocartilagenous

Which of the following represents the largest


cusp of the mandibular first molar?
A
8.

--

composed of which of the fol/owing


connective tissues?

A
F
J
L
T

A
8.
C.
D.
15

Capsular
Collateral
Stylomandibular
Temporomandibular

.,

iiI
.~

128. The arrows on the illustration below represent the


path taken by the opposing contacting cusps. The
mandibular movement indicated is

A
8.
C.
D.
E.

133. The drawing below illustrates the occlusal


view of which premolar?

protrusive.
right lateral; working side.
left lateral; working side.
right lateral; non-working side.
left lateral; non-working side.

A
8.
C.
D.

129. Which of the followingrepresents the norl1!al


eruption age (years) of the maxillarylateral
incisors?

134. Which of the following illustrates the occlusal


view of a primary maxillary right first molar?

.,~~

A
8.
C.
D.

_.,.,.,~:'-

-. ..-.

4-5 years
6-7 years
8-9 year~
10-11 years

&tJ
~~:
2
A
B.
C.
D.
E.

First premolar
Second premolar
First molar
Second molar

:
~.:.:t<)
4

W..J

to'.,--',
:

1
2
3
4
5

135. The drawing below illustrates a cross section


at the cementoenamel junction. This section
is typical of which of the following teeth?

131. The largest incisal/occlusal embrasure is located


between which of the followingteeth?
A
8.
C.
D.
E.

~~

:"';~;;:j;
L. -'::~...:' "'qL..~/

130. Which of the followingmaxillaryteeth has the


largest cervico-occlusal crown height?
A
B.
C.
D.

Maxillaryfirst
Mandibular first
Maxillarysecond
Mandibular second

Maxillarycentral and lateral incisors


Mandibular central and lateral incisors
Maxillarylateral incisor and canine
Maridibularlateral incisor and canine
Maxillarycanine and first premolar

A
B.
C.
D.

132. In an ideal intercuspal relation, the arrow on the


illustration below represents the path taken by
which cusp of which molar?

(~

Maxillaryfirst premolar
Mandibular second premolar
Maxillarymolar
Mandibular molar

136. When a protrusive mandibular movement


(anterior teeth edge-to- edge) is achieved, the
mandibular first molar has the potential to
contact which of the following maxillaryteeth?

A
8.
C.
D.
E.

Cusp

Molar

Mesiofacial
Mesiolingual
Distofacial
Distolingual
Mesiolingual

Second
Second
Second
Second
Third

A
B.
C.
D.
E.
16

First and second premolars


Second premolar only
Second premolar and first molar
First and second molars
Second molar only

.~'

-137. Which of the following is characteristic of


primary maxillary canines?
A
B.
C.

D.

Crown height < mesiodistal width.


Cusp is low and rounded.
The mesioincisal cusp ridge is shorter
than the distoincisal cusp ridge.
The mesioincisal cusp ridge is longer
than the distoincisal cusp ridge.

138. Which of the following can adversely affect the


self-cleaning quality of a dentition in no,rmal
alignment?

A
B.
C.
D.

139.

Central
lateral
Central
Canine

incisor
incisor
and lateral incisors
and lateral incisors

140. Which incisor is MOST frequently markedly


concave on the lingual surface?
A
B.
C.
D.

A
B.
C.
D.

central incisor.
canine.
first premolar.
first molar.

143. The outline below of a pulp cavity indicates


that this tooth probably

Contact of adjacent teeth


Efficient use of a toothbrush
Friction of food material during
mastication
Too great a contour of the cervical
enamel ridge

In ~n acquired Class III crossb1terelationship,


as the mandible retrudes, the maxillary lateral
incisor contacts which of the following teeth?
A
B.
C.
D.

--

142. The root tip MOST likely to be forced into the


maxillary sinus during surgical removal is that
of a maxillary

A
B.
C.
D.

144.

=--

has 3 distinct cusps.


has experienced trauma.
is that of an old person.
is that of a young person.

On permanent

teeth, the greatest

incisal

curvature of a cervical line is on which surface


of which incisor?

A
B.
C.
D.
E.

Surface

Incisor

Mesial
Distal
Mesial
Distal
Mesial

Mandibular central
Mandibular central
Mandibular lateral
Maxillary lateral
Maxillary central

Maxillary central
Mandibular central
Maxillary lateral
Mandibular lateral
145. Which of the following illustrations represents
the mesial view of a mandibular right lateral
incisor?

141.

On which of the following teeth does the


occlusal anatomy often require the placement
of separate MO and DO restorations?

u ij
2

A
B.

C.
D.

1
2
3
4

\j

fJ
2

A
B.
C.
D.

1
2
3
4

..'

'S

..

'~"f

146.

151. The midroot cross-sectional diagram below

In a Class II occlusal relationship, the tip of


the facial cusp of a mandibular first premolar
lies directly below the contacting area
between which maxillary teeth?
A
B.
C.
D.

illustratesthe root of which maxillarymolar?


Viewed from the occlusal

Canine and lateral incisor


Canine and first premolar
First and second premolars
Second premolar and first molar

()G

(0
A
B.
C.
D.

147. Which extrinsic muscle of the tongue


functtons to retract the tongue?

A
B.
C.
D.

Hyoglossus
Styloglossus
Genioglossus
Palatoglossus

152.

148/ When compared with maxillary first


premolars, the central developmental groove
of maxillary seco.nd premolars is

A
B.
C.

'--

D.

149.

D.

E.

Right first
Left first
Right third
Left third

In norma', occlusion, and in a left working


movement, the mesiofacial cusp of the
maxillary left second molar passes through
which of the following mandibular structures?

The embrasure between the left first


-and second premolars'
.
B. The embrasure between the left first
and second molars
C. The facial groove of the left first molar
D. The facial groove of the left second
molar
E. The distofacial groove of the left first
molar

longer with less supplementary


grooves.
shorter with less supplementary
grooves.
longer with multiple supplementary
grooves.
shorter with multiple supplementary
grooves.

153.

Which of the following teeth in the permanent


dentition normally have bifurcations?
A
B.
C.

Mandibular canines
Mandibular second premolars
Maxillary molars and maxillary first
premolars
Mandibular molars and maxillary first
premolars
Maxillary first and second molars

A
B.
C.
D.

Maxillary central incisors


Maxillary lateral incisors
Mandibular lateral incisors
Mandibular canines

cusp of a maxillary first molar opposes which


feature on a mandibular first molar?

At 9 years of age how many primary teeth


remain in the mouth?
A
B.
C.
D.
E.

0
4
8
12
18

A
B.
C.
D.

18

.,

Which of the 'following represents the third


pair of permanent teeth to erupt in normal
sequence?

154. In an ideal intercuspal position, the distofacial


150.

;}

The
The
The
The

distal cusp
distofacial cusp
distofacial developmental groove
mesiofacial developmental groove

}:

~
155.

160.Which premolar

In an ideal intercuspal relation, the


mesiofacial cusps of the mandibular second
molars contact the maxillary molars in (on) the

A
B.
C.

D.

MOST likely possesses a


crescent-shaped central developmental
groove?

distal fossae of the second.


central fossae of the second.
lingual embrasures between the first
and second.
facial embrasures between the first and
second.
mesial marginal ridges of the second.

A
B.
C.
D.

Maxillary first
Mandibular first
Maxillary second
Mandibular second

E.

156.

161. A mandibular canine differs from a maxillary


canine in which of the following?

From the facial view, the maxillary first molar


.

has its lingualroot apex in line withwhichof

A
B.
C.

the following?

A
B.
C.
D.
E.

157.

Facial groove
Oistofacial line angle
Distofacial cusp tip.
Mesiofacial cusp tip
Mesiodistal diameter midpoint

D.

Because of the presence of a fissured groove,


cavity preparations MOST frequently need to
be extended from the occlusal surface to the

A
B.
C.
D.

162. Which of the following molars MOST


frequently have only 3 cusps?

A
B.
C.
D.

facial surface of maxillary molars.


lingual surface of maxillary molars.
lingual surface of mandibular molars.
lingual surface of mandibular first
premolars.

163.
158.

On a maxillary molar, which of the following


ridges is formed by the union of the distal
cusp ridge of the mesiolingual cusp and the
triangular ridge of the distofacial cusp?

A
B.
C.
D.
E.

It is longer.
It has a less pronounced cingulum.
The crown is approximately the same
length.
It has a cusp tip more nearly centered
mesiodistally when viewed from the
facial.

Mesiolingual grooves can be found on which


of the following permanent teeth?

A
B.
C.
D.

Cuspal
Central
Marginal
Oblique
Transverse

Maxillary first
Mandibular first
MaxilJarythird
Mandibular third

Maxillary canines
Mandibular first premolars
Maxillary first molars
Mandibular second molars

164. In an ideal intercuspal relation, the oblique


ridge of the maxilJary first molar opposes
which structure of the mandibular molar?

159.

The interproximal area between first and


second
B. The developmental groove between the
mesiofacial and distofacial cusps of the
first
C. The developmental groove between the
mesiolingual and distolingual cusps of
the first
D. The developmental groove between the
distofacial and distal cusps of the first.

What condylar movement is performed as the


mandible moves from a pure protrusive
movement from maximum intercuspal
position to a maximum protruded position?
A
B.
C.
D.

Translation
Rotation
Hinge
Medial and forward

19

.-

..
.

169.

165. As compared with permanent teeth, the


crowns of the primary teeth are
A
B.
C.

D.

'--

larger.
more bulbous and constricted.
about the same size, but more bellshaped cervically.
narrower mesiodistally in comparison
with their crown length in the anterior
teeth.

Protrusive movement is produced primarily


because of contracture of which of the
following muscles?

A
B.
C.
D.
E.

Masseter

Mylohyoid
Temporalis
Medial pterygoid
Lateral pterygoid

,~

170. Which root surface depression on a maxillary


first molar possesses a concavity that travels
from the cervical third of the crown onto the

166. The smudge mark labeled a, b, C represents


which of the following contacts?

root surface?
~

C!b.
c b

A Lingual
B. Facial

C.
E.
A
B.
C.
D.

"'--

Working side
Non-working side
Protrusive
Lateral protrusive

171.

~
"""'---

--

B.

-----

c.
D.

posteriorly to the pterygomandibular


raphe.
inferiorly to the mylohyoid muscle
insertion.
inferiorly to the masseter muscle
insertion.
inferiorly to the medial pterygoid muscle
insertion.

Distolin~~ ~

Primary maxillary first molar


Primary mandibular first molar
Primary mandibular second molar
Mandibular second premolar
Mandibular first molar

.~
.~
,f'

:,1

;]

:5;1

'1:
71
~

172. The concept of using a lateral checkbite


record to set a respective condylar inclination
implies which of the following?
A

Mandibular lateral translation (Bennett


movement) occurs during the

B.

A
B.
C.
D.
E.

;O
cJ

--------

168.

O~

In which of the following teeth is the mesial


portion MOST distinctly separated from the
remainder of the occlusal table by a
transverse ridge?

A
B.
C.
D.
E.

167. The apices of a mandibular second molar are


located

--

Mesial

D. Distofac:~~~

'--

~~<:)

c (,O

earliest stage of lateral movement.


latter stage of protrusive movement.
middle stage of retrusive movement.
hinge axis movement of the mandible.
ending stage of exhibiting a border
movement.

c.
D.

That the non-working side condyle has


traveled against the posterior wall of the
fossa
That the working side condyle has
traveled down the slope of the articular
eminence
That the non-working side condyle has
moved anteriorly and medially
That the working side condyle has
moved toward the medial wall ofthe
glenoid fossa

'-------

20

.~
:A
"

"

j;~

t:
,;;:

173. Which of the following represents the MOST

178.

common anatomic feature that complicates


periodontal maintenance in a maxillary first
premolar?

'<1

~j
c,l
J

On the occlusal

surface of a maxillary first

molar, the total number of pits is normally the


same as found on the occlusal surface of
which premolar?

<

A
8.
C.
D.

A
B.
C.
D.
E.

Deep concavity on the mesial surface of


the tooth
Intermediate furcational ridge
Mesiolingual developmental groove
Excessive convexity of the facial root

174. The lingual cusp(s) on which of the following

179. From the incisal aspect, the crown of a

mandibular posterior teeth is (are)


approximately 2/3 the height of the respective
facial cusp(s)?
A
B.
C.
D.
E.

maxillary canine normally exhibits which of the


following?
A
B.

First premolar
Second premolar
First molar
Second molar
Third molar

c.

D.
E.

~,

175.

In the comparison of the rhomboidal and


heart-shaped crown outlines of maxillary
molars, the crown portion that differs MOST in
contour and size is the

A
8.
C.
D.

"

r
:f
'J':

.'"

mesiofacial.
distofacial.

180.

f'

mesiolingual.
distolingual.

B.

D.
"

c.

Tetracycline medication in early life


Excessive fluorides in the drinking water
A metabolic disturbance during the
prenatal period
A metabolic disturbance during infancy
and the early childhood periods

A trapezoidal outline
Symmetry between its mesial and distal
portions
A mesial portion that is thinner
faciolingually than the distal portion
A d~stalportion that displays some
concavity in its facial outline
Less faciolingual thickness than the,
crown of a mandibular canine

From a proximal view, which of the following


describes the crown outline on a mandibular
posterior tooth?

176. A hypoplasia of primary teeth which is limited


to ,the incisal thirds of incisors, to the incisal
tips of canines, and to the occlusal portions of
molars, suggests which of the following?
A
B.
C.

Maxillary first
Mandibular first
Maxillary second
U-type mandibular second
Y-type mandibular second

D.

It is designed to protect against root


fracture by having the crown structure
serve as root support.
It is designed to allow for a minimum
amount of chewing efficiency on the
coronal surface of the tooth.
It is usually rhomboidal and has a
design flaw that' encourages cusp
fracture.
It is usually trapezoidal and has a
design flaw that encourages cusp
fracture.

181. Which of the following normally describes the


general crown form of canines when viewed
from the facial or lingual aspect?

177. Which of the following is the first


succedaneous tooth?
A
8.
C.
D.

A
B.
C.
D.
E.

Maxillary central incisor


Mandibular central incisor
Maxillary lateral incisor
Mandibular first molar

21

3-sided
4-sided
5-sided
6-sided
7-sided

...
~

182. Which of the followingteeth is MOST likelyto


have a distal coronal concavity that can pose
special problems in matrix placement?

187.

~
~
'.co

As the mouth is opened widely, the-articular


disk moves in what direction in relation to the
articular eminence?

';

,~

'-

.;.

A
8.
C.
D.

A
8.
C.
D.
E.

Maxillaryfirst premolar
Mandibular first premolar
Maxillaryfirst molar
Mandibular first molar

188. The dentist instructs the patient, who has a


severed left lateral pterygoid muscle, to open
wide. The patient's mandible will move in
which direction?

183. The spacing between anterior teeth in a


5-year-old child is MOST frequently caused by
A

"'---

8.
C.
D.
E.

the presence of an excessive maxillary


anterior frenum.
the pressure from succedaneous teeth.
thumb-sucking.
tongue thrusting.
the growth of the dental arches.

A
8.
C.
D.

184. In a cervical cross section, which premolar(s)


sometimes exhibit a root outline and a pulp
chamber floor outline that are both kidneyshaped?

A
8.

A
8.
C.
D.
E.

To the left
To the right
In a straight protrusive direction
In a retrusive direction

189. During a working movement of the mandible,


the facial cusp ridges of the maxillary first
premolar on the working side oppose 'wnich
of the following mandibular structures?

,-~

----

Laterally
Anteriorly
Posteriorly
Medially
Superiorly

Maxillary first
Mandibular first
Maxillary second
Mandibular second
All premolars

C.
D.

The facial embrasure between the


canine and the first premolar
The distal cusp ridge of the first
premolar and the mesial cusp ridge of
the second premolar
The distal cusp ridge of the second
premolar and the mesial cusp ridge of
the first molar
The mesiofacial groove of the first
molar.

----185. On the crowns of maxillary canines, which'


lobe includes the cusp tip?
-----

A
8.
C.
D.
E.

190. Which of the following BEST describes the


Curve of Spee?

Mesiolingual
Distofacial
Mesiofacial
Lingual
Middlefacial

8.

186. In protrusive movement, the mandibular


canines in a Class II occlusal relationship
articulate with which of the followingmaxillary
teeth?
- ~

A
8.
C.
D.

C.

D.

Canines only
Lateral incisors only
Canines and lateral incisors
Canines and first premolars

E.

--

22

The spherical configuration of the


composite arrangement of the occlusal
surfaces and incisal edges of the teeth
of both dental arches
The facial-lingual curvature resulting
from the facial cusps being the longest
in the mandibular arch, and the lingual
cusps being the longest in the maxillary
arch
The anterior-posterior curvature of the
occlusal surfaces of the teeth, as seen
in a facial view
The inclination of the teeth in relation to
the verticallongaxis ofthe body
The composite arrangement, of the
facia! crown surface heights of contour
of all the teeth in any quadrant

~
:;
~

....

0,

'>'

~.

191.

j
;

.\
,~

Which of the followingnormallydescribesthe


pulp cavity in a mesiodistal section of a
maxillary canine?

A
8.
C.
D.
E.

196. From the facial aspect, the crown of a primary


canine normally has an incisal outline that
exhibits
A
8.
C.

It exhibits 2 pulp horns.


It is widest at its incisal limit.
It is pointed at its incisal limit.
It is widest at the mid root level.

D.

It is generally wider than in a


faciolingual section.

E.
192. Which fiber group of the periodontal ligament
reduces the probability of forceful impaction
into the alveolus because of a blow to the
crown?
A.
8.-'C.
D.
E.

197. A primary molar lacks an identifiable

A
8.
C.
D.
E.

Horizontal
Apical
Oblique
Transseptal
Gingival

194.

The
The
The
The

'.

permanent second molar


permanent first molar
second premolar
primary first molar

On the crowns of maxillary premolars, the


height of contour is normally located in the
cervical third on which of the following
surfaces?
A
8.
C.
D.

D.

E.

A
8.
C.
D.
E.

Mandibular incisor
Mandibular canine
Maxillary first premolar
Mandibular second premolar
Mandibular molar

199. The greatest difficulty in removing calculus


from the root trunk area of molars is seen on
which of the following surfaces?
A
8.
C.
D.
E.

Facial
Lingual
Mesial
Distal

195. Which of the following represents the apex of


the triangular-shaped boundary of the
interproximal space?

A
8.
C.

root trunk.
cervical line.
cervical ridge.
apical foramen.
dentinoenamel junction.

198. The masticatory function of a mandibular first


premolar is MOST similar to that of which of
the following teeth?

193. The occlusal surface of the primary


mandibular second molar closely resembles
the occlusal surface of which of the following
mandibular teeth?

A
8.
C.
D.

2 mamelons.
no slopes, because it is straight.
mesioincisal and distoincisal slopes of
equal length.
notching due to labial developmental
depressions.
a mesioincisal slope that is longer than
the distoincisal slope.

200.

Alveolar bone
Gingival tissue
The marginal ridges of the adjacent
teeth
The proximal surfaces of the adjacent
teeth
The contact area of the adjacent teeth

In viewing the crown of a canine from the


incisal aspect, one normally sees each of the
following structures EXCEPT one. Which one
is this EXCEPTION?

A
8.
C.
D.
E.

23

Mesial of mandibular second


Distal of mandibular second
Lingual of maxillary second
Distal of maxillary first
Mesial of mandibular first

Cingulum
Lingual fossa
Cervical line
Distal cusp ridge
Mesiofacial developmental depression

.,

RELEASED

NATIONAL BOARD DENTAL EXAMINATIONS


PART 1
.

TEST:

..
1f*1Ii
~'r

DENTAL ANATOMY-OCCLUSION

FORM: 14
DATE: D96

..

"--"

Item

Key:.

101
102
103
104
105

A
D
D
D
B

126
127
128
129
130

106
107
108
109
110

A
B
E
B
C

,131
132
133
134
135

,Item

Item

Key:'

B
D
E
C
A

151
152
153
154
155

A
D
C
C
E

176
177
178
179
180

C
B
E
D
C

C,

156
157
158
159
160

A
B
D
A
D

181
182
183
184
185

C
C
E
A
E

Item..... Key

Key.

----

'"'---

'"'--

.~

D
A
C
C

186
187
188"
189
190

C
B
A
B
C

191
192
193
194
195

C
C
B
A
E

B
C
A
A
D

196
197
198
199
200

E
A
B
D
C

111
112
113
114
115

C
D
E
A
D

136
137
138
139
140

C
D
D
D
C

161

116
117
118
119
120

B
A
A
B
B

141
142
143
144
145

A
D
D
E
B

166
167
168
169
170

A
B
A
E

121
122
123
124
125

C
A
B
C
D

146
147
148
149
150

C
B
D
D
D

171
172
173
174
175

162

163
164
165

'B

C
B
D
B

,$

l'.
,

i[
..'"

.,
-

- indicates item not scored


L---

24

27.

A
B.
C.
D.
E.

28.

.29.

--

33.

34.

Radial
Axillary
Accessory
Thotacodorsal
long thoracic

inferiorvena cava.
superior vena cava.
azygos vein.
portal vein.
superior mesenteric vein.

Calcium ions are sequestered by


A
B.
C.
D.
E.

Facial canal
Pterygoid canal
Pharyngeal canal
Sphenopalatine foramen
Pterygomaxillary fissure

The hepatic veins drain blood from the liver


into the
A
B.
C.
D.
E.

Areolar
Reticular
Dense regular
Dense irregular

-which of the followingnerves innervates the


trapezius muscle?
A
B.
C.
D.
E.

Each ofthe followingstructures is an opening


into the pterygopalatine fossa EXCEPT one.
Which one is this EXCEPT/OfV?
A
B.
C.
D.
E.

Meiosis
Mitosis
Budding
Amitosis
Cytokinesis

Tendons are comprised of which of the


followingtypes of collagenous connective
tissue?
A
B.
C.
D.

32.

The diploid number of chromosomes is


maintained in proliferating somatic cells by
which of the followingprocesses?

T-tubules.
sarcoplasm.
sarcosomes.
myofibrils.
sarcoplasmic reticulum.

30. The lateral pterygoid muscle insert~ into


which of the following?

35.
A
B.
C.

;.~

D.
E.
-

31.

Condylar process only


Medialaspect of the mandibularramus
Articular disk of the temporomandibular
joint only
Articular disk of the temporomandibular
joint and neck of the mandible
Articular disk of the temporomandibular
joint and coronoid process

A
B.
C.

Buccinator and palatopharyngeus


Medial pterygoid and palatopharyngeus
Levator veli palatini and
palatopharyngeus
D. Buccinator and superior pharyngeal
constrictor

Ifthe facial nerve were to be cut just after it


exited the stylomastoid foramen, it would
cause loss of innervation to which of the
following?

A
B.
C.
D.
E.

36.

Which of the following features distinguishes


bone from osteoid?
A
B.
C.

Bone is not resorbed.


Bone has a mineralized matrix.
Osteoid contains fewer lacunae per unit
area.
D. Osteoid contains a different type of fiber
in its matrix.

lacrimal gland
Mylohyoidmuscle
Submandibular gland
Orbicularis oculi muscle
Anterior belly of the digastric muscle
6

-....

Which of the followingtwo muscles attach to


the pterygomandibular raphe?

,.,.

37.

Which of the following explains why the Barr


body found in certain epithelial cells is
significant?
A
B.
C.
D.

It suggests mitotic activity.


It indicates protein synthesis.
It indicates a metaplasmic change.

It assists in differentiating
sexes.

Some medications can be absorbed through


the mucosa of the tongue's ventral surface
and through the mucosa of the floor of the
mouth. This absorption can take place in
these areas because the mucosa is
A

It is a symptom of nuclear

disintegration.

E.

42.

covered by pseudostratified squamous


epithelium.
covered by simple squamous
epithelium with a vascular lamina
propria.
covered by thin nonkeratinized stratified
squamous epithelium with a thin lamina
propria.
covered by keratinized stratified
squamous epithelium that contains
numerous capillaries.
pierced by the ducts of numerous minor
salivary glands through which some
types of medication are easily
absorbed.

B.

between the

c.
D.

38.

The vagus nerve supplies parasympathetic


fibers to which of the following?

E.

A Descending colon
B. Ascending colon
C. Sigmoid colon
D. Rectum
E. Anus

I
43.

39.

The lymph vessels that drain both dental


arches connect directly with which of the
following nodes?

A
B.
C.
D.
E.

A
B.
C.
D.
E.

Submandibular
Deep cervical
Sublingual
Retropharyngeal
Superficial cervical

Vagus
Facial
.

Trigeminal
Glossopharyngeal
Hypoglossal

Which of the following strata of the epidermis


is the LEAST cytodifferentiated?

A
B.
C.
D.

I
44.

40.

Pain fibers from the posterior third of the


tongue travel with which cranial nerve?

A patient who has damage to the right


hypoglossal nerve will protrude the tongue
A
B.
C.
D.
E.

Basale
Corneum
Spinosum
Granulosum

upward.
downward.
straight forward.
toward the right side.
toward the left side.

I
~
~

",

J:

45.
41.

The roots of the brachial plexus are derived


from the ventral rami of

A
B.
C.
D.
E.

the cervical plexus.


the spinal accessory nerve.
spinal nerves C3 through C5
spinal nerves C3 through C7.
spinal nerves C5 through T1.

In terms of wall thickness, the relative


proportion of smooth muscle is GREATEST in
which of the following?

A
B.
C.
D.
E.

Veins
Venules
Arterioles
Capillaries
Large arteries

..
.

46.

51.

The superior laryngeal artery pierces the


thyrohyoid membrane in company with which

The buccinator muscle is supplied by which of


the following nerves?

of the following laryngeal nerves?


A
B.
C.
D.
E.

A
B.
c.
D.
E.

Inferior laryngeal
Internal laryngeal
External laryngeal
Superior laryngeal
Recurrent laryngeal

52.

47.

The vertebral artery is a branch of the

A
B.
C.
D.
E.

In which of the following are numerous


afferent lymphatic channels found?

A
R
C.
D.
E.

brachiocephalic artery.
external carotid artery.
internal carotid artery.
subclavian artery.
thyrocervical trunk.

Facial
Lingual
Spinal acessory
Inferior alveolar
Buccal nerve of the trigeminal

Spleen
Thymus
Lymph nodes
Palatine tonsils
Pharyngeal tonsils

-'

48.
-

A
B.
C.
D.
E.

49.

53.

The muscle that is the prime mover in left


lateral excursion is the
right masseter.
left medial pterygoid.
right medial pterygoid.
left lateral pterygoid.
right lateral pterygoid.

A
B.
C.
D.
E.

Each of the following structures lies between


the hyoglossus and the mylohyoid muscles"
EXCEPT one. Which one is this EXCEPTION?

54.

A
B.
C.
D.
E.

50.
-

Lingual nerve
Lingual artery
Sublingual gland
Submandibular duct
Hypoglossal nerve

55.

Nucleolus
Lysos~me
Nucleus
Mitochondrion
Pinocytotic vesicle

The hypophysis is situated in a fossa of which


of the following bones?

A
B.
C.
D.
E.

Form the acrosome


Secrete testosterone
Inhibit spermatogenesis
Supply nutrients to sex cells
Support the germinal epithelium

Colon
Stomach
Jejunum
Gallbladder
Duodenum

Each of the following structures is bound by a


membrane EXCEPT one. Which one is this
EXCEPTION?

A
B.
C.
D.
E.

Which of the following represents the function


of the interstitial cells (of Leydig) in the testis?

A
B.
C.
D.
E.

The submucosa is present in each of the


foHowing EXCEPT one. Which one is this
EXCEPTION?

Ethmoid
Frontal
Temporal
Sphenoid
Palatine

,
56.

White matter of the spinal cord consists


chiefly of which of the following?
A
B.
C.
D.
E.

57.

E.

58.
.

A
B.
C.
D.
E.

59.

63.

Auriculotemporal
Auditory
Temporal
Superior alveolar
Inferioralveolar

C.
D.
E.

left subclavian artery.


arch of the aorta and the left pulmonary
veins.
ligamentum arteriosum and the arch of
the aorta.
pulmonary trunk and the left
brachiocephalic vein.
ligamentum arteriosum and the left
brachiocephalic vein.

Which of the followingcranial nerves contain


parasympathetic preganglionic fibers?
A
B.
C.
D.
E.

II, III, IV, and V


III, IV, V, and VI
III, V, V~~,and IX
III, VII, IX, and X
VII,IX, X, and XI

microvilli.
microtubules.
microfibrils.
tonofilaments.
microfilaments.

Each of the followingenters the orbit by way of


the superior orbital fissure EXCEPT one.
Whjch one is this EXCEPTION?
A Abducens nerve
B. Trochlear nerve
C. Oculomotor nerve
D. Ophthalmic artery
E. Ophthalmic division of the trigeminal
nerve

The capacity of the tongue'for forceful


movement depends on striated muscle
supplied by which of the following cranial
nerves?
A V
B. \111
C. DC
D. X
E. XJI

The left recurrent laryngeal nerve is closely


related at its beginning to the
A
B.

60.

62.

superior vena cava and the azygos vein.


left jugular and brachiocephalic veins.
right jugular and brachiocephalic veins.
left internaljugularand subclavian
veins.
right internal jugular and subclavian
veins.

Pain, touch, temperature and proprioceptive


modalities for the temporomandibular joint
are carried by way of which of the following
nerves?

The core of a ciliumis composed of


A
B.
C.
D.
E.

Perineurium
Myelinated axons
Unmyelinated axons
Nerve cell bodies
Loose connective tissue

The thoracic duct empties directly into the


junction of the
A
B.
C.
D.

61.

64.

How many lobes does the right lung usually


have?
A
B.
C.
D.
E.

65.

1
2
3
4
5

Enlargement of the third ventricle and both


lateral ventricles is caused by obstruction of
the
A
B.
C.
D.

cerebral aqueduct.
foramen of Magendie.
foramina of Luschka.
interventricular foramina of Monro.

.'

1
.

Part I
July 1989

1.

5.

Which of the following papillae of the


tongue are the largest, are the least
numerous, have many taste buds, and are
associated with the ducts of von Ebner's
glands?
1.
2.
3.
4.

Foliate
FiIiform
Fungiform
Circu mvallate

Uniform growth in the infolding portion of


an epithelial root sheath (Hertwig) results in
formation of a
1.

2.
3.
4.

7.

In the photomicrograph below, the letter E


.

2.

3.
4.

8.
1.
2.
3.
4.
5.

4.

dental papilla.
dental sac (follicle).
inner enamel epithelium.
outer enamel epithelium.
bone of the crypt surrounding the tooth
germ.

1.
2.
3.
4.

9.

striated muscle and cartilage.


smooth muscle and elastic fibers.
the basement membrane and collagen
fibers.
areolar connective tissue and cartilage.

root.
enamel matrix.
cementoenamel junction.
dentinoenamel junction.

Which of the following is located at the


opening between the small and the large
intestines?
1.
2.
3.
4.

-2-

medial to the zygomatic arch and inserts


into the coronoid process of the
mandible.
lateral to the zygomatic arch and inserts
into the coronoid process of the
mandible,
medial to the zygomatic arch and inserts
into the medial side of the angle of
the ramus.
lateral to the zygomatic arch and inserts
into the medial side of the angle of
the ramus.

The area at which calcification of a tooth


begins is the
1.
2.
3.
4.

Variationof the size of the lumen of the


bronchiole during inspiration and expiration
is caused primarily by

interatrial septum of the adult.


interatrial septum of embryo and fetus.
interventricular septum of the adult.
interventricular septum of embryo and
fetus.

The temporalis muscle originates on the


lateral surface of the skull and passes
1.

Facial
Buccal
Maxillary
Mylohyoid
Auriculotemporal

The foramen ovale is located in the

1.
2.
3.
4.

single-rooted tooth.
double-rooted tooth.
th ree-rooted tooth.
tooth without a root.

points to (the)

Which of the following nerves innervates the


capsule of the temporomandibular joint?
1.
2.
3.
4.
5.

6.

2.

3.

Anatomic Sciences

The
The
The
The

cardiac sphincter
pyloric sphincter
ileocecal valve
tricuspid valve

I
i

Part I
July 1989

85.

Anatomic Sciences

88.

Which of the following organelles have a


double-unit membrane?
Nucleus
Lysosome
Golgi complex
Mitochondrion
Rough endoplasmic reticulum

(a)
(b)
(c)
(d)
(e)

1.
2.
3.
4.
5.
6.

(a)
(b)
(c)

(a), (b), and (c)


(a) and (c) only
(a) and (d) only
(a), (d), and (e)
(b), (c), and (d)
(b) and (e)

1.
2.
3.
4.
5.
6.
7.

89.

86.

(a) only
(a) and (b) only
(a) and (c) only
(b) only
(b) and (c) only
(c) only
(a), (b), and (c)

(a)
(b)
(c)
(d)
(e)

Portal
Azygos
Splenic
Internal iliac

1.
2.
3.
4.
5.
6.

1. (a) and (b)


2. (a) and (c) only
3. - (a), (c), and (d)
4. (b) and (c) only
5. (b), (c), and (d)
6. (b) and (d) only

90.

\Vhich of the following events may take


place during active inspiration?

if
t

(e)
The facial processes that contribute directly
to the formation
(a)

(b)
i-it

~
I

(c)

(d)
(e)

1.
2.
3.
4.
5.

of the lips are the

1.
2.
3.
4.
5.
6.

maxillaryand mandibular
processes.
maxillaryand lateral nasal
processes.
maxillaryand medial nasal
processes.
medial and lateral nasal processes.
lateral and medial palatine
processes.

(a) and
(a) and
(a) and
(b) and
(b) and

91.

(b)
(c)
(e)
(c)
(d)

The diaphragm descends.


The epiglottis closes.
The uvula swings superiorly.
Rotation occurs at costo-vertebral
articulations.
The lateral diameter of the thorax
increases.

(a), (b), and (d)


(a), (d), and (e)
(a) and (e) only
(b) and (c) only
(b), (c), and (d)
(d) end (e) only

Which of the following arteries is NOT a


branch of the maxillary artery?

1.
2.
3.
4.
5.

-11-

lamellae.
canaliculi.
capillaries.
osseous matrix.
Volkmann's canals.

(a), (b), and (e) only


(a), (c), and (e) only
(a), (d), and (e) only
(b), (c), and (d) only
(b), (c), and (e) only
(a), (b), (c), (d), and (e)

(a)
(b)
(c)
(d)

87.

contains nervous and vascular


elements.
allows for physiologic movement
of the tooth.
provides a cellular source for new
cementum and bone.

Nutrientsand oxygen reach the cells of


compact bone by passing through

Which of the following veins are normally


concerned with drainage of the stomach?
(a)
(b)
(c)
(d)

The periodontal ligament space is vital to the


functional life of a tooth because it

Deep temporal
Middle meningeal
Inferioralveolar
Superficialtemporal
Posteriorsuperior alveolar-

.
Anatomic Sciences

Part I
July 1989

92.

96.

Which or the followingarteryforamen/fissurepairings is NOTcorrect?

Which of the following nerves is NOT


located in the wall of the cavernous sinus?

1.
2.
3.
4.
5.

1. Accessory meningeal-foramen ovale


2. Middle meningeal-foramen spinosum
3. Inferior alveolar-mandibular foramen
4. Anterior tympanic-petrotympanic fissure
5. Posterior superior alveolar-

Abducens
Maxillary
Trochlear
Ophthalmic
Oculomotor

sphenopalatine foramen

93.

'1.
2.
3.
4.
5.

94.

98.

gingiva.
skeletal muscle.
the pulp of a tooth.
the periodontal ligament.
the temporomandibular joint.

The
The
The
The
The

99.

thoracic duct
descending aorta
splanchnic nerve
hemiazygous vein
right recurrent laryngeal nerve

Perikymata
Enamel tuft
Enamel matrix
Enamel lamella

All of the following structures lie superficial


to the hyoglossus muscle EXCEPTthe

1.
2.
3.
4.

-12-

Left gastric
Short gastric,
Gastroduodenal
Left gastroepiploic
Inferior pancreaticoduodenal

Which of the following is never found at the


outer surface of enamel?
1.
2.
3.
4.

Which of the following is NOT contained in


the posterior mediastinum?

1.
2.
3.
4.
5.

Which of the following arteries is NOT


derived from the celiac trunk or its branches?

1.
2.
3.
4.
5.

Cricothyroid
Aryepiglottic
Transverse arytenoid
lateral cricoarytenoid
Posterior cricoarytenoid

Proprioceptors are found in each of the


following EXCEPT

1.
2.
3.
4.
5.

95.

97.

Which of the following laryngeal muscles is


NOT innervated by the recurrent laryngeal
nerve?

lingual nerve.
lingual artery.
hypoglossal nerve.
submandibular duct.

"

~;

..

NATIONAL BOARD DENTAL EXAMINATION PART I


ANSWER KEY
ANATOMIC

SCIENCES

ANS.

NO.

ANS.

NO.

ANS.

NO.

ANS.

26.

51.

1
3
2
5

27.
28.
29.
30.

1
5
1
4

52.
53.
54.
55.

5
3
2
4

77.

31.
32.
33.
34.
35.

5
2
3
5
2

56.
57.
58.
59.
60.

10.

2
1
4
3
5

81.'
82.
83.
-84.
85.

11.
12.
13.
14.
15.

4
2
3
2
5

36.
37.
38.
39.

40.

1
4
3
1
4

61.
62.
63.
64.
65.

4
1
2
3
1

16.
17.
18.
19.
20.

1
4
1
3
2

41.
42.
43.
44.
45.

5
2
3
4
1

66.
67.
68.
69.
70.

4
4
2
3
4

-91.
92.
94.
95.

4
5
1
3
5

21.
22.

5
1
4
3
2

46.
47.
48.
49.
50.

2
3
5
4
1

71.
72.
73.
74.
75.

1
3
1
4
3

96.
97.
98.
99.

1
4
2
2

NO.

1.
2.
3.

4.
5.
6.
7.

8.
9.

23.
24.
25.

,4

3
2
3

. 3

.16.

...-. "-1

78.

4
3
3

79.
80.

86.
87.
88.

89.
90.

93.

--. ..- 4

4
2
4
3
.,.;.

- 2

2
7
5
2

"

'"
'1

~
f

-13-

.
.
Part I

Biochemistry/Physiology

July1989

1.

7.

Which of the following represents a soluble


polysaccharide found in dental plaque and is
formed from the fructose moiety of sucrose?

1. An increased tubular synthesis of NH3


2. A partial reabsorption of the filtered

1. levan
2. Dextran
3. Amylopectin
4. Hyaluronicacid

2.

3.

4.

The maximal number of impulses that a


nerve fiber can carry is determined by which
of the following?

8.

1. The intensity of the stimulus


2. The diameter of the nerve fiber
3. The duration of the absolute refractory
period

2.

period

3.
4.

Preganglionic autonomic nerve fibers are


exclusively.

12.
3.
4.
5.

4.

They are macromolecules of high


molecular weight.
They contain many functional groups
with differing pK's.
They have considerable secondary and
tertiary structures that cause hydrogen
ions to be sequestered.
They have many peptide bonds that are
resistant to hydrolysis by hydrogen or
hydroxyl ions.

What is the effective filtration pressure under


the following conditions?

Capillary blood pressure = 30 mm. Hg.


Plasma osmotic pressure.. 6,000 mm, Hg.

.. 5,975
.. 5 mm. Hg.

TIssue fluid osmotic pressure

TIssue fluid pressure

5.

mm. Hg.

1. Plus 10 mm. Hg.


2. Plus 5 mm. Hg.
3. 0
4. Minus 5 mm. Hg.
5. Minus 10 mm. Hg.

A nystagmus
A change in the force of gravity
A linear acceleration and deceleration
An angular acceleration and deceleration

10.

Two-point discrimination requires that two


sensory receptors are

1.
2.
3.
4.

6.

9.

Which of the following represents an


adequate stimulus for semicircular canals?

1.
2.
3.
4.

5.

somatic.
adrenergic.
sympathetic.
cholinergic.
parasympathetic.

HCO3An elevated production of "new"


bicarbonate
None of these

Which of the foJ/owing explains why proteins


are able to buffer physiologic solutions over a
wide range of pH?
1.

4. The duration of the relative refractory

3.

Renal compensation for chronic metabolic


alkalosis involveswhich of the foJ/owing?

different in threshold.
of two different morphologic types.
innervated by two different axons.
innervated by branches of the same
axon.
separated by a distance of at least
5 mm.

11.

The length-tension diagram shows that the


maximum active tension of a muscle occurs

1.
2.
3.

when a muscle is unstretched.


when sarcomere length increases.
when there is maximum overlap of

4.

at the maximum length that a muscle


may be stretched.

crossbridges.

The most important test for hypoventilation is


the determination of

Various substances are transported in the


blood as complexes associated with plasma
proteins. An example of this is the binding of

1.
2.
3.
4.
5.

1.
2.
3.
4.
5.

arterial
arterial
arterial
venous
venous

pH.
CO2 tension.
oxygen tension.
CO2 tension.
oxygen tension.

-14-

calcium to albumin.
iron to cytochrome f.
cholesterol to globulin.
fatty acids to hemoglobin.
thyroxine to thyroglobulin.

Part I
July 1989

12.

Biochem istry/Physiology

Primary recognition of J3-estradiol by its target


cell depends upon the binding of the
hormone to a specific
1.
2.
3.
4.

17.

A physiologic buffer functions to


1.
2.

nuclear receptor.
cytoplasmic receptor.
cell-membrane receptor.
adenylate cyclase.

3.
4.

13.

Immediately following the production of


Okazaki fragments, the gap between
fragments becomes connected through the
action of.
1.
2.
3.
4.

14.

DNA-ligase.
exonucleases.
DNA polymerase.
DNA-directed RNA polymerase.

Phosphorylation of some enzymes by ATP


converts these enzymes from an active form
into a form that is inactive. Which of the
following conversions exemplifies this
process?

5.

18.

19.

1. Trypsinogen to trypsin
2. Phosphorylase b to phosphorylase a
3. Phosphodiesterase a to
4.

Which of the following amino acids can


most easily be converted to tyrosine in the
human body?
1.
2.
3.
4.
5.

16.

They convert fibrinogen to fibrin.


They agglutinate and plug small,

3.
4.
5.

They initiate fibrinolysIs in thrombosis.


They supply fibrin stabilizing factor.
They supply proconvertin for
thromboplastin activation.

ruptured vessels.

The primary force stabilizing the lipid-protein


complex in cell membranes: is which of the
following?
....
The
The
The
The

covalent linkage
ionic interaction
hydrophilic bonding
hydrophobic interaction

In procaryotic protein synthesis, the


elongation factor G serves to

4.

21~

transport carbon dioxide from the site of


its production to the site of its
elimination.
minimize the increase in hydrogen ion
concentration that accompanies
cellular acid production.
maximize the decrease in hydrogen ion
concentration that accompanies alkali
formation.

1.
2.

1.
2.
3.

Glycine
Arginine
Methionine
Tryptophan
Phenylalanine

elimination to the site of its


production.

Platelets play an important role in


,phemostasis. Which of the following describes
this role?

1.
2.
3.
4.

phosphodiesterase b
Glycogen synthetase I to glycogen
synthetase D

20.
15.

regulate the partial pressure of venous


carbon dioxide.
carry fixed acid from the site of its

form the initiation complex.


facilitate the binding of Fmet tRNA.
translocate the growing peptide chain
and to move the ribosome along the
mRNA.
prevent the larger ribosomal subunits
from binding with those that are
smaller.

Which of the following classes of steroids


contain 18 carbons and an aromatic ring?

Gangliosides are glycolipids found on various


cell surfaces. In addition to sphingosine, their
unit composition contains another
characteristic component. This component is

1. Estrogens
2. Androgens
3. Progestagens
4. G Iucocorticoids
5. Mineralocorticoids

1.
2.
3.
4.
5.

uronic acid.
plasmalogen.
triglyceride.
N-acetylmuramic acid.
N-acetylneuraminic acid.

.
Part I
July 1989

22.

Biochemistry/Physiology

28.

Cholesterol is primarily used by mammalian


cells as

1. an energy storage material.


2. a precursorof ketone bodies.
3. a component of cell membranes.
4. 'a precursorof polyunsaturated fatty
acids.
23.

29.

An amount
An amount
An amount
An amount
mucosal

less than 50%


between 60-70%
between 80-100%
dependent upon the
ferritin level

30.

Rapid filling
Early ejection
Isovolumic relaxation
Isovolumic contraction
None of these

What are the end-products of glycolysis from


glucose produced by L. case; and by other
oral acidogenic bacteria?

31.

results in no change in solubility.


results in no change in isoelectric pH.
is a result of a change in DNA coding.
has no noticeable effect on the O2
transport behavior of the erythrocyte.

Clinical adult onset diabetes mellitus (Type II)


resuIts from

12.
3.
4.
5.

2NADH + 2 ATP

Valine
Histidine
Arginine
Aspartic acid
Glutamic acid

In sickle cell anemia, a variation in an amino


acid of hemoglobin is detected. This
substitution of valine for glutamic acid
1.
2.
3.
4.

1. 2 CH)CH2OH + 2 ATP
2. 2 CH)-CHOH-COOH + 2ATP
3. 2 CH)CHO + 2 CO2 + 2NADH
4. 2 CO2 + 2 CH)-CO-COOH + 2NADH
5. 2 CH )-CHOH-COOH + 2 CO2 +

27.

Niacin
Thiamine
Vitamin A
Vitamin K
Folic acid

Decarboxylation of which of the following


amino acids results in formation of a
vasodilator?

1.
2.
3.
4.
5.

During which of the following phases of the


cardiac cycle are all cardiac valves open?

1.
2.
3.
4.
5.
26.

glycogen.
nucleic acids.
hyaluronidase.
connective tissue. ,

Which of the following represents the


amount of dietary calcium normally absorbed
from the gut of an adult man?

1.
2.
3.
4.

25.

1.
2.
3.
4.
5.

Glycosaminoglycans function as important


structural components of

1.
2.
3.
4.

24.

Which of the following vitamins functions in


coenzymes that are specifically involved in
the transfer and utilization of the single
carbon moiety?

.
;

deficient secretion of insulin.


excess secretion of epinephrine.

failure of the kidneys to retain sugar.

secretion of oxytocin from the posterior


pituitary.
loss of the insulin receptor function in
the target tissues.

For which of the following are the loops of


Henle responsible?
4

1. The acidification of urine


2. The reabsorption of amino acids
3. The reabsorption of most of the
4.

5.

32.

glomerular filtrate
The formation of most of the ammonium
ion excreted in the urine
The establishment of an osmotic
gradient within the medulla of the
kidney

Ammonia helps to reduce the hydrogen ion


concentration in body fluids. This mechanism
involves the
1.
2.
3.
4.

..'
'61
.",

excretion of Na + into urine.


secretion of ammonia into blood.
reabsorption of sodium into blood.
transfer of ammonia from the tubule
lumen to the bloodstream.

."'~

"

-16-

Part I
July 1989

33.

The
The
The
The
The

2.

3.

4.
5.

39.

a shift in anterior pituitary gonadotropin


secretion with LH predominating in
the mixture.
a sudden diminution of anterior pituitary
gonadotropin secretion.
a sudden increase in FSHsetretion by
the anterior pituitary.
all of these.
none of these.

Purine ribonucleoside phosphatesare all

40.

36.

inosine phosphate.
guanosine phosphate.
adenosine phosphate.
guanosine diphosphate.
deoxyadenosine phosphate.

41.

1.
2.
3.
4.

change in synaptic resistance.


greater frequency of sensory end-organ
,discharge.
increased cholinesterase activity at the
synapse.
simultaneous arrival of impulses from a
large number of receptors.

42.

1.

3.

If the presence of a specific compound, C,


increases the Km for an enzyme-substrate
reaction, which of the following would be
true about that enzyme?

1.
2.

3.
4.

5.

4.
5.

C would be a competitive inhibitor of


the enzyme.
C would be a noncompetitive inhibitor
of the enzyme.
The velocity vs. [5] plot for the enzyme
would be the same with or without C.
With C present, the enzyme would
convert substrate to product faster.
With C present, it would take less
substrate to drive the reaction to halfmaximum velocity than without C.

43.

Vitamins C and 0
Vitamins C and K
Phosphorus and iron
Calcium and fluoride

regulate the water permeability of the


distal tubule and the collecting duct.
regulate the sodium transport across the
proximal tubular epithelium.
regulate the osmolality of the proximal
tubular fluid.
reduce the sodium transport across the
collecting duct epithelium.
increase the sodium transport across the
descending limb of the loop of Henle.

The intrinsic factor for vitamin 812absorption


is produced in the
1.
2.
3.
4.
5.

-17-

- VitaminsA and,0

ADH (antidiuretic hormone) functions to

2.
37.

kidneys.
sweat glands.
salivary glands.
gastrointestinal tract.

The lack of wh ich of the followi ng


substances during tooth formation most likely
induces enamel hypoplasia?
'1.
2.
3.
4.
5.

Spatial summation in spinal reflexes is


dependent upon

alveol i.
atmosphere.
tissue fluid.
venous blood.
cells throughout the body.

In the regulation of calcium balance, the


greatest elimination of calcium occurs'in the

1.
2.
3.
4.

--

Urease
Uricase
Xanthine oxidase
Aspartate transcarbamoylase
Carbamoyl-phosphate synthetase

The partial pressures of respiratory gases


found in arterial blood correspond most
closely to those partial pressures found in the

1.
2.
3.
4.
5.

synthesized de novo from the common


intermediate

1.
2.
3.
4.
5.

Which of the following enzymes catalyzes


the formation of uric acid from purines?

1.
2.
3.
4.
5.

H band
Z band
sarcomer~
sarcoplasm
sarcolemma

In the human female, ovulation is believed to


be caused by

1.

35.

38.

Which of the following represents the basic


contractile unit of muscle myofibril?

1.
2.
3.
4.
5.
34.

Biochemistry/Physiology

liver.
stomach.
pancreas.
duodenum.
lacteals.

Part I
July 1989

44.

Which of the following substances represents


the immediate precursor for fatty acid
synthesis?

1.
2.
3.
4.
5.

45.

Biochemistry/Physiology

50.

Which of the following explains why the


biosynthesis of fatty acids requires carbon
dioxide or bicarbonate?

1. 'The bicarbonate/carbonic

Glucose
Citrate
Acetyl-CoA
Acetoacetate
{3-hydroxy-{3-methylgl utaryl-CoA

2.
3.

When a substance is filtered by the


glomerulus and is neither reabsorbed, nor
secreted by the tubules, its clearance is a
.. measure of

4.
1.
2.
3.
4.
5.

46.

51.

2'
2'
3'
3'
5'

to
to
to
to
to

3'.
5'.
1'.
5'.
1'.

3.
4.

52.

53.

It
It
It
It
It

will become denaturated.


will migrate to the negative pole.
will migrate to the positive pole.
will remain stationary and unchanged.
will separate into its different
monomeric forms.

Fluoroapatite can form during hard tissue


formation by a (an)

1. The secretion of steroid hormone


2. The biotransformation of carbohydrate
3. The secretion of epinephrine and

1.
2.
3.

norepinephrine
The regulation of electrolyte and fluid
balance

4.

4.

glands
Cephalic phase of gastric secretion
Pancreaticsecretion of bicarbonate

How will a protein respond in an


electrophoretic system, should the pH
become lower than the isoelectric point of
the protein?

1.
2.
3.
4.
5.

insulin.
fasting.
epinephrine.
a high protein diet.
cirrhosis of the liver.

Which of the following serves as the adrenal


medulla's principal function?

Which of the following is a reflex mediated


by the vagus?

1. Bile flow from the liver


2. Mucus secretion from the Brunner's

A low concentration of glucose in the blood


and a high concentration of liver glycogen
are likely to be caused by

1.
2.
3.
4.
5.

49.

Fats
Minerals
Proteins
Carbohydrates
None of these

In both RNA and DNA, the internucleotide


linkage is a phosphodiester bond that
connects the pentose hydroxyl groups

1.
2.
3.
4.
5.
48.

5.

Ingestion of which of the following most


markedly affects the rate of gastric emptying?

1.
2.
3.
4.
5.

47.

filtration rate.
renal plasma flow.
filtration fraction.
Tm of the substance.
level of the substance in the blood.

acid buffer
system is very efficient at the pH
optimum of this sensitive enzyme
system.
Bicarbonate is a positive effector for this
system and favorably alters the
conformation of its enzymes.
Carbon dioxide is incorporated into
acetyl coenzyme A forming malonyl
coenzyme A (an intermediate in the
synthetic process).
Carbon dioxide is incorporated into
carbamoyl phosphate (a reactive
intermediate in the synthetic process).
Carbon dioxide provides an anaerobic
environment that prevents oxidation of
the sulfhydryl groups present in the
reactive sites of the enzyme system.

substitution of OH ions by F ions.


reaction between CaF2 and CaHPO4.
reaction of F ions with hydroxylysine
residues.
adsorption of F ions onto hydroxyapatite
crystal surfaces.

Part I
July 1989

54.

Biochem istry/Physiolog'

58.

Distribution of fluids between intracellular


and extracellular compartments is mainly
achieved by

1.
2.
3.
4.

5.

bulk flow across the capillary wall.


unrestricted movement of sodium to
achieve osmotic balance.
unrestricted movement of water to
achieve osmotic balance.
unrestricted movement of both sodium
and potassium to achieve osmotic
balance.
movement of potassium out of the
intracellular compartment to achieve
osmotic balance.

Which of the following


feedback regulation?

1.

2.
3.
4.
5.

They activate specific genes.


They activate initiation factors for protein
synthesis.

3.

They increase the intracellular Ca+ +

5.

characterizes

concentration.
They allosterically modify adenylate
cyclase activity.
They bind to cell membrane receptors
and activate adenylate cyclase.

Drinking large quantities of carbonated colatype beverages could lead to


1.
2.
3.

an excessiveCa + + deposition on

4.
5.

enamel decalcification
enamel decalcification

a metabolic alkalosis.
a deposition of pigment in the teeth.
enamel.

The enterohepatic circulation of bile


salts
The inhibition of gastrin secretion by
gastric acid
The absorption of water from the lumen
of the small intestine
The partitioning of fatty acids by chain
length as they are absorbed
The synthesis of triglyceride from
triglyceride c;ligestion products in
intestinal epithelial cells

60.

61.

ovaries but not on testes.


testes but not on ovaries.
both ovaries and testes.
the adrenal cortex in both sexes.

A component
transamination

56.

1.
2.
3.
4.
5.

Which of the following amino acids is


important in the active site of trypsin and
chymotrypsi n?

1.
2.
3.
4.

Serine
leucine
Phenylalanine
Glutamic acid

62.

It is reasonably well-established that the


release of which of the following hormones
is controlled mainly by nerve impulses?

1.
2.
3.
4.
5.

63.

Insulin

-19-

required for a

nIacIn.
thiamine.
folic acid.
pyridoxine.
riboflavin.

Plasma volume.
Intracellular fluid volume
Interstit.al fluid volume

Establishing a tracheostomy

1.
2.
3.
4.
5.

Epinephrine
Testosterone
Parathyroid hormone
Deoxycorticosterone

of the coenzyme
process is

By subtracting the extracellular fluid volume


from the total body water, which of the
following can be estimated?
1.
2.
3.

57.

due to low pH.


due to high pH.

FSH and lH produced by the anterior


pituitary act on
1.
2.
3.
4.

-I
I

1.
2.

4.

59.

55.

Which of the following describes the


principal mechanism by which
glucocorticoids stimulate their target cells?

increased
increased
increased
decreased
decreased

results in

airway resistance.
anatomic dead space.
physiologic dead space.
respiratory work.
effective alveolar ventilation.

Part I

Biochem istry/Physiology

July 1989

64.

1.

2.
3.
4.
5.

65.

69.

Mobilization of fat stored in adipocytes


involves

1. atrial repolarization.
2. ventricular depolarization.
3. ventricular repolarization.
4. atrial depolarization and conduction

activation of fatty acids by synthesis of


acyl CoA.
rearrangement of fatty acids in triglycerides.
phosphorylation of glycerol at the
expense of ATP.
activation of triglyceride lipase by ADP.
activation of triglyceride lipase by a
cAMP dependent protein kinase.

..

5.

70.

Which of the following is solely determined


by the Nernst (equilibrium) potential for
sodium between the inside of the axon and
the surrounding tissue fluid?
1.
2.
3.
4.
5.

The
The
The
The
The

membrane potential
extent of the after-potential
sodium permeability of the nerve
potassium permeability of the nerve
limit of the peak of the action potential

Which of the following important nutrients is


present in milk in low concentration?
1.
2.
3.
4.
5.

67.

Breathing a gas mixture containing 10 per


cent O2 and 90 per cent N2 will stimulate
respiration. This stimulated respiration will
occur because
1.

2.
3.
4.

68.

72.

73.

pH
Peaz
Temperature
Hydrogen ion concentration
2, 3 diphosphoglyceric acid (DPG)

aerobic bacteria.
anaerobic bacteria.
salivary glycoproteins.
mineralized calcium phosphate.

Consider the conversion: alanine


lactic
glucose. This is an example of
acid

1.
2.
3.
4.

nitrogen has an excitatory effect on the


chemoreceptors.
the lack of inspiratory CO2 has an
excitatory effect on the chemoreceptors.
low oxygen tension has an excitatory
effect on the respiratory center in the
brain.
low oxygen tension has an excitatory
effect on the peripheral
chemoreceptors.

through the AV node.


ventricular depolarization plus
ventricular repolarization.

Acquired pellicle is primarily composed of

1.
2.
3.
4.

Iron
Protein
Calcium
Phosphorus
Carbohydrate

glycolysis.
glycogenolysis.
gluconeogenesis.
synthesis of glycerol.

Alanine can be synthesized by


transamination directly from which of the
following acids?
1.
2.
3.
4.
5.

Lactic acid
Pyruvic acid
Glutaric acid
a-ketoglutaric acid
3-phosphoglyceric acid

In the normal cardiac cycle, the volume of


the ventricle is greatest

74.
1.
2.
3.

4.
5.

at end systolic volume.


at end diastolic volume.
between aortic valve closure and AV
valve opening.
before the "a" wave of the atrial pressure
curve.
between "c" and "v" waves of the atrial
pressure curve.

Which of the following is the best method


for determining the three-dimensional
structure of protein?

1.
2.
3.
4.
5.

-20-

The affinity of hemoglobin for oxygen


diminishes as which of the following is
decreased?
1.
2.
3.
4.
5.

71.
66.

The PR interval in the electrocardiogram is


an index of the

Dialysis
Electrophoresis
X-ray diffraction
Ultracentrifugation
None of these

Part I
July 1989

10.

Anatomic Sciences

16.

Cell bodies of primary sensory neurons of


mechanoreceptors in the periodontal
ligament are found in which of the following
brainstem nuclei?
1.
2.
3.
4.
5.

1.
2.
3.

Nucleus so/itarius
Reticular formation
Descending nucleus of V
Chief sensory nucleus of V
Mesencephalic nucleus of V

17.
11.

Salivary glands of the hard palate are located


in the

1.
2.
3.
4.
12.

13.

18.

endochondral ossification.
intramembranous ossification.
expansion of the dental lamina.
the conversion of Meckel's cartilage
directly into bone.

3.

4.

19.

The tooth forms additional apical dentin.


There is continual apposition of bone at
the fundus of the tooth socket.
There is continual apposition of
cementum in the tooth's apical region.
None of these

1.
2.
3.
4.
5.

20.

aberrant sweat glands.


aberrant sebaceous glands.
superficial salivary glands.
aberrant fatty accumulations.
none of these.

1.

The cellular organelle that binds and releases


calcium during relaxation and contraction of
skeletal muscle is the

1.
2.
3.
4.
5.

nucleus.
lysosome.
mitochondrion.
transverse tubule.
sarcoplasmic reticulum.

3.
4.

-3-

Mylohyoid
Thyrohyoid
Hyoglossus
Genioglossus
Posterior digastric

simple.
compound.
exocri ne.
holocrine.
merocrine.

Some medications may be absorbed through


the mucosa of the tongue's ventral surface
and through the mucosa of the floor of the
mouth. This absorption can take place in
these areas because the mucosa is

2.
15.

insulin.
thyroxin.
sex hormone.
growth hormone.

Salivary, sweat, sebaceous, and von Ebner's


glands all have in common the characteristic
of being

1.
2.
3.
4.
5.

Fordyce spots appearing on the mucous


membrane of the cheek result from the
presence of

(alpha cells) of the hypophysis

A child falls on a nail that penetrates the


submental region in the midline. What is the
first muscle to be impaled?
1.
2.
3.
4.
5.

As a tooth undergoes attrition with age and


with use, it maintains contact with its
antagonist. Which of the following describes
how this phenomenon occurs?
1.
2.

14.

median raphe.
gingival lone.
anterolateral lone.
posterolateral lone.

periphery.
superior surface.
inferior surface.

Acidophils
secrete

1.
2.
3.
4.

The mandible, except for the condyle, is


principally formed by
1.
2.
3.
4.

The most vascular portion of the


temporomandibular joint's articular disk is
usually found on the disk's

<=covered
by simple squamous epithelium
with a vascular lamina propria.
covered by. thin nortkeratinized stratified
squamous epithelium with a thin
lamina propria.
covered by stratified squamous
epithelium containing numerous
capillaries.
pierced by the ducts of numerous minor
salivary glands through which some
types of medication are easily
absorbed.

,
~r~

Part I
July 1989

75.

80.

Calcium ions trigger contraction of muscles


by binding to
1.
2.
3.
4.

76.

Biochem istry/Physiology

actin.
myosin.
troponin.
tropomyosin.

1.
2.
3.
4.

Mediated (facilitated) diffusion of substances


across cell membranes differs from simple
diffusion in that mediated diffusion
1.
2.
3.
4.

Negative nitrogen balance (nitrogen excretion


> intake) may be caused by

81.

Which of the following represents unsound


therapeutic protocols?

1. Treating hypothyroidic patients with

requires ATP.
requires another solute.
is a one-directional process.
exhibits saturation kinetics.

2.
3.
4.

77.

5.

The pH of a solution having a 10-5M


concentration of OH - ion is
1.
2.
3.
4.
5.

5.
7.
9.
determinable only if the pKa is known.
determinable only if the base
composition is known.

82.

1.
2.

4.
Which of the following molecular features
contributes to the water-binding properties of
proteoglycans?
1.
2.
3.

4.
5.

The carboxyl groups acting as buffers


Central hyaluronate (a helix) trapping
water within
The space between the core proteins
and the hyaluronate being highly
charged
The large n.umber of alcohol groups on
the polysaccharide chaining H-bond to
water
The large number of serine and
threonine residues in the core protein
offering H-bonding sites

83.

79.
;1<.

~c

CO2 acceptors.
coenzymes in redox reactions.
factors in transamination reactions.
replacements for each other in biologic
reactions.

1.
2.
3.
4.

i
-21-

ATP.
heat.
energy.
oxygen.
electrons.

How are serum calcium and serum


phosphate levels affected in a parathyroid
deficiency state?

Derivatives of riboflavin and niacin function


as

1.
2.
3.
4.

integrate sensory perception.


initiate nerve impulses responsible for
activation of skeletal muscle.
control release of hormones from the
posterior pituitary gland.
modify the pattern of muscular response
in reflex and voluntary contractions.

In respiration, the removal of hydrogen ions


from a substrate is always accompanied by
the removal of
1.
2.
3.
4.
5.

84.

thyroxine
Treating hyperthyroidic patients by
thyroidectomy'
Treating Type I diabetic patients with pig
or cow insulin
Treating acromegalics with biosynthetic
human growth hormone
Treating Type II diabetics with diet and
oral medication that stimulate insulin
production and/or secretion

The cerebellum functions to

3.

78.

growth.
protein synthesis.
dietary lack of essential amino acid.
synthesis and catabolism of equal
amounts of body protein.

Serum
Calcium

Serum
PhosQhate

Increased
Decreased
Increased
Normal

Decreased
Increased
Normal
Increased

Biochemistry/Physiology

Part I
July 1989

85.

1.
2.
3.
4.

86.

89.

How will chronotrophy and inotropy be


affected when there is sympathetic
stimulation to the heart?
Chronotropy

Inotropy

Negatively
Negatively
Positively
Positively

Negatively
Positively
Negatively
Positively

Which of the following types of noncovalent


bonding are found in proteins?
(a)
(b)
(c)
(d)

1.
2.
3.
4.
5.

(a), (c), and (d) only


(a) and (d) only
(b) and (c) only
(b), (c), and (d) only
(a), (b), (c), and (d)

Stimulation of a carotid sinus nerve results in


which of the following changes in heart rate,
arterial blood pressure and venous return?

1.
2.

3.
4.
5.

Heart
Rate

Arterial
Blood
Pressure

Venous
Return

Increased
Increased
Decreased
Decreased
No change

Decreased
Increased
Decreased
Decreased
Decreased

Decreased
Decreased
Decreased
No change
No change

90.

Which of the following can alter the function


of the medullary centers for respiration?
(a)
(b)

87.

Which of the following are likely to result in


highly concentrated urine?
(a)
(b)
(c)
(d)

1.
2.
3.
4.
5.
6.
7.

88.

Ionic
Peptide
Hydrogen
Hydrophobic

(c)

Low sodium diet


Diabetes insipidus
Cellular dehydration
Decreased plasma volume

1.
2.
3.
4.

(a) and
(a) and
(b) and
(a), (b),

(b) only
(c) only
(c) only
and (c)

(a) and (b) only


(a) and (c) only
(a) and (d) only
(b) 'and (c) only
(b) and (d) only
(c) and (d) only
(a), (b), (c), and (d)

91.

Frequencyof impulse activity in the afferent


nerve from a muscle spindle (Group la
afferent fibers) is increased by a (an)
(a)
(b)
(c)

1.
2.
3.
4.
5.
6.

The central and peripheral


chemoreceptors
The apneustic and pneumotaxic
centers in the pons
The central nervous system at the
conscious level

Functions of the hexose monophosphate


shunt include the production of
(a)
(b)
(c)

relaxation of intrafusal muscle


fibers.
increased activity in gammaefferent fibers.
passive stretch of the muscle.

(d)
(e)

(a) only
(a) and (b)
(a) and (c)
(b) only
(b) and (c)
(c) only

1.
2.
3.
4.
5.

-22-

NADP for lactate oxidation.


NADPH for fatty acid synthesis.
glucuronic acid for heparin
synthesis.
D-ribose for nucleic acid synthesis.
ATP for anaerobic muscle
contraction.

(a), (c), and (e)


(a) and (d)
(b), (c), and (e)
(b) and (d)
(c) only

~.
~

Part I
July 1989

92.

Biochemistry/Physiology

95.

In a systemof diffusable ions (Gibbs-Donnan


equilibrium), the presence of impermeant
ions on one side of a-semi-permeable
membrane causes which of the following?
(a)
(b)
(c)

(d)

1.
2.
3.
4.
5.
6.
7.

(a)

An osmotic pressure
A transmembrane potential
An asymmetric distribution of
diffusable ions across the
membrane
A transient diffusion potential only,
that will rapidly disappear

(c)

1.
2.
3.
4.
5.
6.
7.

(a) and (b) only


(a), (b), and (c)
(b) and (c) only
(b) and (d) only
(c) and (d) only
(d) only
(a), (b), (c), and (d)

1.
2.
3.
4.
5.
6.

(a)
(a)
(b)
(b)
(c)
(d)

(a)
(b)
(c)
(d)

Lysyl oxidase
Procollagen proline hydroxylase
Procollagen amino peptidase
Procollagen lysine hydroxylase
Procollagen carboxyl peptidase
and
and
and
and
and
and

(e)

1.
2.
3.
4.
5.
6.

(c)
(e)
(c)
(d)
(e)
(e)

97.

Temperature-regulating mechanisms that


increase heat production include
(a)
(b)
(c)
(d)
(e)

1.
2.
3.
4.
5.

shivering.
cutaneous vasodilation.
increased voluntary activity.
cutaneous vasoconstriction.
increased secretion of epinephrine
from the adrenal medulla.

(a), (b), and


(a), (b), and
(a), (c), and
(a), (c), and
(c), (d), and

(c)

(d)

1.
2.
3.
4.
5.
6.

(c)
(e)
(d)
(e)
(e)

-23-

A
B1
B12
C
K

and (e)
and (d)
(d) only
and (e)
and (e)
(d) only

Which two of the following


smooth muscle?

(b)

94.

Vitamin
Vitamin
Vitamin
Vitamin
Vitamin

(a), (b),
(a), (c),
(a) and
(a), (d),
(b), (c),
(c) and

(a)

~:

(a) only
(a) or (b) only
(a) or (c) only
(b) only
(b) or (c) only
(c) only
(a), (b), and (c)

A person who lacks fruit and vegetables in


his diet is likely to develop a dietary
deficiency in which of the following
vitamins?

Which of the following enzymes require


vitamin C for their activity?
(a)
(b)
(c)
(d)
(e)

The threshold potential becoming


more positive
The rate of spontaneous
depolarization decreasing
The maximum diastolic potential
becoming more negative

(b)

96.

93.

Which of the fol1owing changes in the


electrical properties of a pacemaker cell can
slowdown the rate at which the cell initiates
impulses?

(a)
(a)
(a)
(b)
(b)
(c)

characterize

Spontaneous activity in the


absence of nervous stimulation
Sensitivity to the chemical agents
either released locally from
nerves, or carried in the
circulation
Decreased activity in response to
stimulation of both divisions of
the autonomic nervous system
Decreased tone in response to
passive stretch
and
and
and
and
and
and

(b)
(c)
(d)
(c)
(d)
(d)

Part I
July 1989
98.

Which of the following is NOT a function of


the liver?

1.
2.
3.
4.
5.
99.

Biochemistry/

Detoxification
Gluconeogenesis
Formation of plasma protein.
Secretion of digestive enzymes
Regulation of blood sugar level

Which of the following is NOT a reflex


mediated by the medulla oblongata?

1.
2.
3.
4.
5.

Blinking
Coughing
Vomiting
Patellar
Swallowing

100.

Which of the following are NOT prodl


by the hydrolysis of nucleic acids?
1.
2.
3.
4.
5.

Pentoses
Phosphates
Amino acids
Purine bases
Pyrimidine bases

NATIONAL BOARD DENTAL EXAMINATION PART I


ANSWER KEY
BIOCHEMISTRY-PHYSIOLOGY

- JULY

1989

ANS.

NO.

ANS.

NO.

ANS.

NO.

ANS.

1
-3
4
4
3

26.
27.
28.
29.
30.

2
5
5
2
3

51.
52.
53.
54.
55.

3
2
1
3
2

76.
77.
78.
79.
80.

4
3
4
2
3

31.
32.

10.

2
2
2
3
3

34.
35.

5
3
3
1
1

56.
57.
58.
59.
60.

1
2
1
4
3

81.
82.
83.
84.
85.

4
4
5
2
4

11.
12.
13.
14.
15.

1
2
'1
4
5

36.
37.
38.
39.
40.

4
1
3
1
4

61.
62.
63.
64.
65.

4
2
4
5
5

86.
87.
88.
89.

90.

3
6
5
1
4

16.
17.
18.
19.
20.

1
4
2
4
3

41.
42.
43.
44.
45.

1
1
2
3
1

66.
67.
69.
70.

1
4
2
4
1

91.
92.
93.
94.
95.

4
2
4
4
7

21.

46.

24.

5
3
4
1

48.
49.

1
4
1
3

71.
72.
73.
74.

3
3
2
3

96.
97.
98.
99.

3
1
4
4

25.

50.

75.

NO.

1.
2.
3.

4.
5.
6.
7.
8.
9.

22.
23.

33.

47.

68.

"'.

-25-

100.

Microbiology/Pathology

Part I
July 1989

1.

6.

The primary sources of pathogenic


microorganisms in the dental operatory are

1.
2.
3.
4.
5.

light handles.
dust particles.
patients' charts.
the mouths of patients.
the hands of clinicians.

The ability of certain oral microorganisms to


function as etiologic agents of dental caries
appears to be correlated with the capacity of
these organisms to produce
1.
2.
3.
4.

2.

A disease of childhood characterized by


mental retardation, delayed growth, and
delayed tooth eruption is associated with a
deficiency of

1.
2.
3.
4.
5.

oxytocin.

7.

4.

antibodies.
histamine.
Iymphokines.
leukotrienes.
none of these.

9.

Which of the following microorganisms is


most likely to be cultured from chronic,
bilateral ulcerations at the corners of the
mouth?

1.
2.
3.
4.
5.
6.

The class of antibodies first detected in


serum after primary immunization is usually
1.
2.
3.
4.
5.

Adenocarcinomas of the large intestine are


most common in which segment?

1. Ascending colon
2. Transverse colon
3. Descending colon
4. Rectosigmoid colon

During delayed hypersensitivity reactions,


sensitized small lymphocytes produce
1.
2.
3.
4.
5.

Which of the following is formed in large


quantities during the degradation of glucose
by homofermentativeStreptococcus mutans?
1. Mannitol
2. Lacticacid
3. Acetic acid
4. Butyricacid
5. Propionic acid

growth hormone.
thyroid hormone.
testicular hormone.
mineralocorticoids.

8.
3.

IgA.
IgD.
IgE.
IgG.
IgM.

10.

The rickettsial disease that may have oral


manifestations is
1.
2.
3.
4.

11.

-26-

serum.
saliva.
subgingivalplaque.
supragingivalplaque.

Keloids are composed predominantly of

1.
2.
3.
4.

rickettsialpox.
Brill's disease.
epidemic typhus.
none of these.

Candida
Brucella
Treponema
Aspergillus
Histoplasma
Trichophyton

The most likely source of bacteria found in


diseased periodontal tissue is
1.
2.
3.
4.

5.

proteolytic enzymes capable of


hydrolyzing salivary proteins.
capsules that inhibit the activity of
phagocytic cells present in saliva.
carbonic anhydrase, an enzyme active in
decreasing the CO2 content of saliva.
an extracellular polysaccharide dextranlike substance involved in the
formation of dental plaque.

adipose tissue.
collagen fibers.
granulation tissue.
caseous necrotic debris.

Part I
July 1989

12.

Microbiology/Pathology

17.

CoJ/agen degradation observed in chronic


periodontal disease may result directly from
the action of collagenase enzymes of oral
microbial origin. Such enzymes have been
demonstrated as component systems of

1.
2.
3.
4.

1. Bacteroides species.
2. Leptothrix bucca/is.
3. Entamoeba gingiva/is.
4. Streptococcus faeca/is.
5. Veillonella alcalescens.
18.

13.

14.

19.

20.

are coagulase-positive.
are penicillin-resistant.
are of the same phage type.
are aureus-type staphylococci.
produce hemolysis, liquefy gelatin, and
ferment mannitol.

21.
1.
2.
3.
4.

22.
16.

Sterilization of surgical instruments that are


sensitive to heat can be accomplished by
using

-27-

coxsackievirus.
Epstein-Barr virus.
herpes simplex type I.
varicella-zoster virus.
respiratory syncytial virus.

Disseminated miliary tuberculosis results


from spread of the tubercle bacillus by way
of
1.
2.
3.
4.
5.

1. phenol.
2. an autoclave.
3. ethyl alcohol.
4. ethylene oxide.

Cholera
Gonorrhea
Brucellosis
Gas gangrene
Tuberculosis

The oral vesicular lesions of herpangina and


hand-foot-and-mouth disease are caused by
1.
2.
3.
4.
5.

Anaphylaxis
Wheal and flare
Passive transfer of antibody
Positive tuberculin skin test

Bacillus anthracis.
Clostridium perfringens.
Salmonella schottmiilleri.
Mycobacterium tuberculosis.
Corynebacterium. diphtheriae.

Which of the following diseases is caused by


an agent that produces neither exotoxins nor
endotoxins?
1.
2.
3.
4.
5.

In which of the following is delayed-type


hypersensitivity demonstrated?

an Arthus reaction.
a Shwartzman reaction.
a cell-mediated reaction.
passive cutaneous anaphylaxis.

lecithinases are produced by


1.
2.
3.
4.
5.

The best evidence for a causal relationship


between a nasal carrier of staphylococci and
a staphylococcal infection in a hospital
patient is the demonstration that the
organisms from both individuals
1.
2.
3.
4.
5.

15.

mutation.
sexduction.
conjugation.
recombination.
lysogenic conversion.

lung.
brain.
liver.
kidney.

In skin graft rejection, the major host


response is
1.
2.
3.
4.

The alteration of Corynebacterium


diphtheriae to a virulent strain by the transfer
of DNA temperate bacteriophage is
1.
2.
3.
4.
5.

An embolus originating as a thrombus in the


femoral vein usually occludes a blood vessel
in the

the lymphatics.
the bloodstream.
the air passages.
direct extension.
none of these.

Part I
July 1989
23.

29.

Progressive massive fibrosis of the lung is


characteristic of

1.
2.
3.
4.
5.

24.

Microbiology/Pathology

silicosis.
anthracosis.
actinomycosis.
hemosiderosis.
bronchiectasis.

1.
2.
3.
4.
5.

Bacteremias in patients with heart valve


abnormalities may result in

30.

1. scarlet fever.
2. rheumatic fever.
3. Ludwig's angina.
4. infective endocarditis.
5. streptococcal pharyngitis.

25.

Adenovirus
Cytomegalovi rus
Epstein-Barr"virus
Herpes simplex virus type I
Herpes simplex virus type II

If a susceptible person were given tetanus


antitoxin, what kind of immunity would
result?

1. Innate
2. Natural active
3. Natural passive
4. Artificial active
5. Artificial passive

.'
{;

Shock during infection with gram-negative


bacteria is most likely caused by

1.
2.
3.
4.
5.

26.

Which of the following viruses is strongly


associated with Burkitt's lymphoma and
nasopharyngeal carcinoma?

31.

an exotoxin.
an endotoxin.
ribonuclease.
a capsular antigen.
a flagellar antigen.

Aspirationof material from carious teeth may


cause
1.
2.
3.
4.
5.

lung abscess.
tuberculosis.
lobar pneumonia.
bronchopneumonia.
interstitial pneumonia.

The most common type of epithelial


metaplasia involves
1.
2.
3.
4.

32.

regeneration of epithelium in an area of


cutaneous ulceration.
replacement of squamous cells by
cuboidal cells.
replacement of cuboidal cells by
columnar cells.
repJacement of columnar cells by
stratified squamous epithelium.

Sterilization refers to which of the following?

1. Absence of all living forms


2. Inhibition of bacterial growth
3. Removal of pathogenic bacteria only
4. Removal of pathogenic bacteria, viruses,
and fungi

;1
f~

~:~

,:'
if:

~:

I!'

33.
27.

Which of the following is typically elevated


in the serum of patients with prostate cancer?

1.
2.
3.
4.

28.

1.
2.
3.
4.
5.

Acid phosphatase
AIpha-fetoprotei n
Alkaline phosphatase
Carcinoembryonic antigen

34.

Which of the followingare most


characteristic of cervicofacialactinomycosis?
1.
2.
3.
4.

For the majority of individuals, the initial


infection with herpes simplex virus results in

Multiple ulcers
Enlarged cervical lymph nodes
Multiple cutaneous abscesses with
sinuses
Indurated nodules in the skin of the
neck

Which of the following diseases exhibits ,a


normal prothrombin time with a prolonged
partial thromboplastin time?

1.
2.
3.
4.
5.

-2~-

genital herpes.
a subclinical disease.
herpes labial is in puberty.
a dermal rash in childhood.
encephalitis as a young adult.

Leukemia
Afibrinoginemia
Chronic liverdisease
Athrombocytopenic purpura
Factor VIII deficiency (hemophilia)

~
"

;1
,{

Part I
July 1989

35.

Microbiology/Patholog

1.
2.
3.
4.
5.

36.

41.

Cytopathogenic effect in tissue culture cells


is used specifically for identification of
certain

1.
2.
3.
4.
5.

fungi.
viruses.
anaerobes:
spirochetes.
spore-formers.

42.

The complement system may be activated

1.2.
3.
4.
5.

at C1 and C3.
only by way of Cl
by interferon during viral infection.
by activation of any of the nine
components.
only when body temperature exceeds
101.6oF.

Prolonged administration of streptomycin may


result in damage to which of the following
nerves?

1.
2.
3.
4.

38.

39.

Which type of chromosome is usually


missing from the cells of persons with
Turner's syndrome?

44.

Multiple sclerosis
Alzheimer's disease
Parkinson's disease
Creutzfeldt-Jakob disease

An autosome
Chromosome 21
A sex ch romosome
A "ring" chromosome
A Group A chromosome

Increased functional demand on the heart


produE:esincreased size of the myocardium
by
1.
2.
3.
4.
5.

hyperplasia.
hypertrophy.
calcification.
fatty infiltration.
increased amounts of fibrous connective
tissue.

Bacterial capsules usually consist of

1.
2.
3.
4.

40.

The type of infection most commonly


transmitted by transfusion of properly
screened blood is

1.
2.
3.
4.
5.

Optic
Facial
Auditory
Trigeminal

Which of the followingis a demyelinating


disease of the central nervous system?
1.
2.
3.
4.

conversion.
transversion.
transduction.
transfection.
transformation.

1. cytomegalovirus.
2. type A hepatitis.
3. type B hepatitis.
4. non-A non-B hepatitis.
43.

37.

The process of gene transfer between


bacterial cells that involves the uptake of
naked DNA molecules is

cliitins.
celluloses.
polypeptides.
polysaccharides.

45.

Esophageal varices are a common


complication of

1.
2.
3.
4.

The observations that typify hemolytic


anemias are elevated

portal hypertension.
primary hypertension.
mesenteric thrombosis.
carcinoma of the esophagus.

~'i

'~

,j

~','
',j

,t\

1. conjugated bilirubin, kernicterus, and


choluria.
2. conjugated bilirubin, kernicterus, and

46.

elevated urine urobilinogen.

3. unconjugated bilirubin, kernicterus, and


4.

On the basis of histogenesis and transitions


observed in clinical cases, there appears to
be a relationship between lymphocytic
lymphoma and
1.
2.
3.
4.

elevated urine urobilinogen.


unconjugated bilirubin, decreased blood
hemoglobin, and absent urine
urobilinogen.

-29-

lymphadenitis.
Iymphoepithel ioma.
lymphocytic leukemia.
lymphocytic leukocytosis.

..~

Part I
July 1989

47.

4.

3.

4.

53.

partially dominant.
autosomal dominant.
autosomal recessive.
sex-linked dominant.
sex-linked recessive.

and reacting with antigen.


antigen attached to mast cells and
reacting with antibody.
antibodies (lgE) attached to mast cells,
but no reaction with antigen.
antigen attached to mast cells, but no
reaction with antibody.

~~

Squamous epithelium exhibiting acanthosis,


disorganization of pattern and atypical cells
without invasion is diagnostic of

1. dysplasia.
2. neoplasia.
3. metaplasia.
4. hypoplasia.
5. prosoplasia.

54.

the nerve tissue.


the blood vessels.
smooth muscle tissue.
striated muscle tissue.
fat and fibrous tissue.

Antiseptics differ from disinfectants in that


antiseptics

1.
2.
3.
4.
5.

sterilize.
kiII pathogens.
are bactericidal only.
are bacteriostatic only.
are applied to living tissues.

Which of the following drugs is likely to aid


in treating or preventing influenza in highrisk subjects during epidemics?

1.
2.
3.
4.
5.

51.

2.

mostly aerobic bacteria.


essentially a pure culture.
bacteria not indigenous to the oral
cavity.
essentially the same organisms found in
the healthy sulcus.

'111
,,1
,,;~

A rhabdomyosarcoma is a malignant
neoplasm derived from

1.
2.
3.
4.
5.

50.

Histamine release from mast cells requires


1. antibodies (lgE)attached to mast cells

Classic hemophilia is due to a deficiency of


normal FactorVIIIwhich in turn is due to a
genetic deficiency that is
1.
2.
3.
4.
5.

49.

52.

The bacterial population in the gingival


sulcus or the pocket that influences the
course of periodontal disease involves
1.
2.
3.

48.

Microbiology/pathology

55.

Acyclovir
Amantadine
Griseofulvin
Stilbamidine
Amphotericin B

1. inflammationof a bronchus.
2. organization of alveolar exudate.
3. a patchy, inflammatory distribution.
4. a diffuse, inflammatorydistribution.

A reduction in acid secretion by the stomach,


an increased tendency toward gastric
carcinoma, atrophic glossitis, and myelin
degeneration in the spinal cord are
characteristic of

1.
2.
3.
4.
5.

As opposed to lobar pneumonia,


bronchopneumonia is characterized grossly
and microscopicallyby

56.

thalassemia.
agranulocytosis.
sickle cell anemia.
pernicious anemia.
polycythemia vera.

In children, the bones become bowed in


rickets because of failure of

1.
2.
3.
4.

osteoblasts to develop.
osteoid tissue to calcify.
osteoid tissue to form.
cartilage matrix to form.
if

-30-

{'-.....

Anatomic Sciences

Part I
July 1989

21.

27.

An active fibroblast is characterized by


1.
2.
3.
4.
5.

multiple nucleoli.
a bloated appearance.
a thick plasma membrane.
an unusually large nucleus.
abundant endoplasmic reticulum.

Most of the postganglionic sympathetic nerve


fibers of the head are derived from the
1.
2.
3.
4.

22.

As a result of a mandibular block injection, a


patient has developed a paralysis of the
muscles of facial expression. Where was the
anesthetic solution most likely deposited?
1.
2.
3.
4.

23.

Into the parotid gland


Near the chorda tympani
Into the pterygoid plexus,
Through the mandibular notch

1.
2.
3.
4. .
5.

29.

afferent only.
sympathetic only.
parasympathetic only.
afferent and sympathetic.
afferent and parasympathetic.

Kupffer cells.
the gallbladder.
the hepatic duct.
the common bile duct.
none of these.

Which of the following branches of the


axillary artery usually supplies the latissimus
dorsi muscle?

12.
3.
4.
5.

The white pulp of the spleen is composed


primarily of

1. sinusoids.'
2. macrophages.
3. lymphocytes.
4. connective tissue.

25.

Production of bile takes place in

The nerves in dental pulp are

1.
2.
3.
4.
5.

24.

28.

superior cervical ganglion alone.


superior cervical gangl ion and the vagus
nerve.
superior cervical gangl ion and the
glossopharyngeal nerve.
inferior, middle, and superior cervical
ganglia.

30.

The esophagus is subdivided into three


portions on the basis of a transition in the
1.
2.
3.
4.

Which of the following charaderizes the


alveolar bone proper of a tooth socket?

Thoracodorsal
Thoracoacromial
Lateral thoracic
Circumflex scapular
Posterior humeral circumflex

submucosa.
adventitia.
mucosal layer.
muscularis externa.

1. It consists of compact bone, shows dark


2.

3.
4.

on a radiograph, and contains


Sharpey's fibers.
It resorbs when subjected to pressure,
and contains numerous openings
through which vessels pass.
It contains haversian systems, has
collagen fibers in its matrix, and is
about 96 per cent inorganic salts.
It serves to attach free gingival
periodontal ligament fibers, and is a
part of the total tooth attachment
apparatus.

31.

1.
2.
3.
4.
5.

32.
26.

Basket (myoepithelial) cells are located in the


periodontal ligament.
lumen of secretory acini.
lamina propria of gingiva.
lumen of intercalated ducts.
spaces between the basal lamina and the
secretory cell membrane.

Serotonin is thought to be produced by

Most of the fibers ascending to or


descending from the cerebral cortex traverse
the

1.
2.
3.
4.

1.
2.
3.
4.

chief cells.
Paneth's cells.
mucous neck cells.
enteroendocrine (argentaffin) cells.

-4-

pyramids.
internal capsule.
medial lemniscus.
cerebral peduncles.

a
~
~\

Part I

Microbiology/Pathology

July 1989
'1':

57.

62.

Which of the following is the best


description of the characteristic lesion of
chronic granulomatous inflammation?
1.
2.
3.

4.

A localized collection of liquefied


. cellular debris and dead or dying

1.
2.
3.
4.
5.
6.

neutrophils
A diffuse collection of liquefied cellular
debris and dead or dying neutrophils
A localized area of inflamed connective
tissue exposed to the environment as
a result of denuding of the overlying
epithelium
A circumscribed collection of
lymphocytes, macrophages, and
epithelioid cells with a background of
fibroblasts, capillaries, and delicate
collagen fibers

63.

Which of the following is the single most


numerous group of microorganisms in the
oral cavity?
1.
2.
3.
4.

4.
5.

Enterococci
Anaerobic streptococci
Facultative streptococci
Beta-hemolytic streptococci

64.
59.

1.

Candidiasis

2. Aspergi Ilosis
3. Blastomycosis
4. H is,toplasmosis
65.
60.

1.
2.
3.
4.

1. Neisseria
2. Treponema
3. Actinomyces

5.

5.

Streptococcus
Staphylococcus

66.
61.

The earliest changes in rheumatoid


occur in the

1.
2.
3.
4.

arthritis

-31-

septic emboli.
bacteria in the bloodstream.
traumatic introduction of bacteria.
bacteria ascending from the lower
urinary tract.
bacteria from the large bowel migrating
to the kidney through retroperitoneal
lymphatics.

Which of the following forms of oral


ulcerations involves oral, ocular, and genital
lesions?
1.
2.
3.
4.

synovia.
epiphysis.
attached ligaments.
articular cartilage.

Decreased albumin
Decreased serum amylase
Elevated serum lipase
Elevated serum amylase
Elevated alkaline phosphatase

Infection of the kidney is usually caused by

Which of the following genera most


frequently develops resistance to penicillin?

4.

nuclear division of granulocytes.


atypical regeneration of epithelium.
bone marrow with migration into the
area.
multiplication of nuclei in surrounding
fibrocytes.
fusion or nuclear division of
mononuclear cells.

Which of the following laboratory results is


diagnostic of acute pancreatitis?
1.
2.
3.
4.
5.

In which o(the following mycotic infections


are the organisms characteristically found
within reticuloendothelial cells?

leukemia.
thalassemia.
multiple myeloma.
Hodgk,in's disease.
pernicious anemia.
sickle cell disease.

Multinucleated giant cells of the foreign-body


type originate from

1.
2.
3.

'"

58.

The disease characterized by radiolucent


bone lesions, anemia,
hypergammaglobulinemia,
and one or more
bone marrow tumors containing
predominantly plasma cells is

Beh~et's
Herpetiform
Recurrent aphthous
None of these

~
.

Microbiology/Pathology

Part I
July 1989

83.

(a)
(b)
(c)
(d)

1.
2.
3.
4.
5.
84.

(b)
(c)
(d)
(c)
(d)
(d)

89.

Myeloma
Liposarcoma
Ewing's sarcoma
Osteogenic sarcoma
Metastatic carcinoma

90.

(a) and (c) only


(a), (c), and (e)
(b), (c), and (d)
(c) and (d) only
(c), (d), and (e)
(d) and (e) only

1.
2.
3.
4.
5.

91.

skin.
ovary.
tongue.
stomach.
large bowel.

(a), (b), and


(a), (c), and
(a), (d), and
(b), (c), and
(b), (d), and

A persistent infection may be caused by any


of the following viruses EXCEPT

Each of the following diseases has painless


hematuria as a common early clinical sign
EXCEPT

1.
2.
3.
4.
5.

Seeding or transplantation metastasis is likely


in carcinomas of the
(a)
(b)
(c)
(d)
(e)

Escherichia coli
Salmonella typhi
Proteus- vulgaris
Bacteroides fragilis
Pseudomonas aeruginosa

1. rhinovirus.
2. papilloma virus.
3. cytomegalovirus.
4. herpes simplex virus.
5. varicella-zostervirus.

Which of the following malignant bone


tumors are likely to be encountered in
children or young adults?

1.
2.
3.
4.
5.
6.

asthma.
urticaria.
anaphylaxis.
Arthus reaction.

Which of the following enteric


microorganisms is NOT typically found in
normal human intestinal flora?
1.
2.
3.
4.
5.

Lactobacillus casei
Actinomyces viscosus
Streptococcus mutans
Streptococcus salivarius

(a) and
(a) and
(a) and
(b) and
(b) and
(c) and

(a)
(b)
(c)
(d)
(e)

86.

88.

Which of the following oral bacteria are


aciduric?

1.
2.
3.
4.
5.
6.

The immediate type of hypersensitivityin


which histamine does NOT playa major role
is
1.
2.
3.
4.

Tetralogy of Fallot
Congenital aortic stenosis
Patent ductus arteriosus
Ventricular septal defect

(a), (b), and (c) only


(a), (b), and (d) only
(a), (c), and (d) only
(b), (c), and (d) only
(a), (b), (c), and (d)

(a)
(b)
(c)
(d)

85.

87.

Which of the following cardiac disorders is


subject to infective endocarditis secondary to
a bacteremia from oral microorganisms?

Each of the following is useful in identifying


viruses EXCEPTthe

1.
2.
3.
4.

(d)
(e)
(e)
(d)
(e)

5.

-34-

acute diffuse glomerulonephritis.


carcinoma of the bladder.
focal glomerulonephritis.
renal cell carcinoma.
nephrolithiasis.

nature of the viral nucleic acid.


morphology of the viral protein capsid.
ability of the virus to grow on complex
media.
neutralization of the virus with specific
antisera.
ability of the virus to be inactivated by
certain solvents, such as ether or
chloroform.

Part I

Microbiology/Pathology

July 1989

92.

96.

Common airborne fungi that cause


opportunistic infections in debilitated
individuals include all of the following
EXCEPT

1.
2.
3.
4.

1. Candida.
2. Rhizopus.
3. Aspergillus.
4. Cryptococcus.

93.

94.

97.

Human immunodeficiencyvirus (HIV)can


directly infect all of the followingcells
EXCEPT
1.
2.
3.
-4.

Which of the following is NOT a component


of the body's non-specific defense
mechanism?

98.

Suppuration is mainly the result of the


combined action of four factors. Which of
the following is NOT one of-these factors?

Which of the following is NOT characteristic


of carcinoma in situ?
1.
2.
3.
4.

99.

Which of the following has the least capacity


for regeneration?

1.
2.
3.
4.
5.

Confer conjugal fertility


Carry genetic information
Exist as circular RNA molecules
Exist as extrachromosomal elements in
bacteria

1. Necrosis
2. Presence of lymphocytes
3. Collection of neutrophils
4. Accumulation of tissue fluid
5. Autolysis by proteolytic enzymes

neurons.
macrophages.
CD4 (helper) lymphocytes.
CD8 (suppressor) lymphocytes.

1. Lysozyme
2. Properdin
3. Interferon
4. Complement
5. Fibrinolysin

95.

Which of the following is NOT characteristic


of plasm ids?

Bone
Cardiac muscle
Liverparenchyma
Squamous epithelium
Fibrousconnective tissue

Jaundice is characteristic of all of the


followingconditions EXCEPT
1.
2.
3.
4.
5.

-35-

Pleomorphism
Disorderly maturation
Hyperchromatic nucleus
Disruption of the basement membrane

aplastic anemia.
liver cell damage.
excessive hemolysis.
bile duct obstruction.
carcinoma of the head of the pancreas.

10 ,

q ,1
i

NATIONAL BOARD DENTAL EXAMINATION PART I


ANSWER KEY
MICROBIOLOGY-PATHOLOGY - JULY 1989

NO.

1.
2.

3.

ANS.

NO.

ANS.

NO.

ANS.

NO.

ANS.

4
3

26.
27.

4
1

51.
52.

4
1

76.
77.

2
4

28.

53.

4.
5.

78.

5
1

29.
30.

3
5

54.
55.

5
3

79.
80.

2
6

6.
7.
8.
9.

4
2
4
1

31.
32.
33.

1
1
2

56.
57.
58.

2
4
3

81.
82.
83.

4-

5
2

59.

10.

34.
35.

4
5

84.
85.

2
4

11.
12.
13.
14.
15.

2
1
5
3
4

36.
37.
38.

61.
63.
64.
65.

1
3
5
3
4

86.

40.

1
3
1
4
3

87.
88.
89.
90.

5
4
2
1
5

16.
17.
18.
19.
20.

4
1
3
2
5

41.
42.
43.
44.
45.

5
4
3
2
1

66.
67.
68.
69.
70.

1
3
1
4
1

91.
92.
93.
94.
95.

3
1
4
5
2

21.
22.
23.
24.
25.

1
2
1
4
2

46.
47.
48.
49.
50.

3
4
5
4
2

71.
72.
73.
74.
75.

3
4
4
6
3

96.

3
2
4
1

'3

39.

60.
62.

'

97.
98.
99.

5
5

Part I
July 1989

Dental Anatomy/Occlusion

5.

All test items relating to occlusion refer to a Class 1


canine and molar relationship unless otherwise
specified. Terms such as "normal" or "ideal" are
synonymous with the above definition.

1.

2.

6.

6 and 7 years.
8 and 9 years.
10 and 11 years.
12 and 13 years.

How soon after the eruption of a permanent


tooth is the apex usually fully developed?
1.
2.
3.
4.

The occlusal view of a normal dental arch


segment is shown below. The arrow points to
the

1.
2.
3.
4.

3.

1. fusion.
2. concrescence.
3. dilaceration.
4. dens in dente.
5. hypercementosis.

The primary maxillary canine is usually


exfoliated between the ages of
1.
2.
3.
4.

The cemental union of two fully formed


teeth that were originally separate entities is

Immediately
3 months
2 or 3 years
5 or6 years

Test items 7-9 refer to the following paragraph and to


the cast crown illustrated below.

primary mandibular right first molar.


primary mandibular left first molar.
permanent maxillary right first premolar.
permanent mandibular left second
premolar.
The dentist placed a cast crown on a patient who has
an ideal canine guided occlusion. The maximum
intercuspal position (MIP) contacts are ideal; however,
when the lateral movements were evaluated with an
articulating paper, the following smudge marks were
noted.

In an 'ideal intercuspal position, the cusp


tip of a permanent maxill~rycanine should
contact
1. both mandibular canine and first
2.
3.
4.
5.

4.

premolar.
the mandibular first premolar only.
the mandibular lateral incisor only.
the mandibular canine only.
no other tooth.

7.

1. First molar
2. Second molar
3. First premolar
4. Second premolar

Froma proximal view, which of the following


permanent teeth tends to be positioned in
the arch with its axis most nearly vertical?
1.
2.
3.
4.
5.

Which of the following maxillary teeth is


responsible for the smudge mark labeled
a,b,c?

Maxillary canine
Maxillary lateral incisor
Maxillary central incisor
Mandibular lateral incisor
Mandibular central incisor

8.

The smudge mark labeled a,b,c represents


which of the following contacts?
1.
2.
3.
4.

-37-

Working side
Non-working side
Protrusive
Lateral protrusive

I
July 1989
Part

9.

Dental Anatomy/Occlusion

13.

In order to eliminate the undesirable nature


of this contact (a,b,c), which portions of the
crown should be removed by selective
grinding with a bur?

Which of the following sketches illustrates


the occlusal view of a maxillary right second
premolar?

1. The entire smudge mark labeled a,b,c


2. Only the a and b portions of the
smudge mark
3. Only the band c portions of the
smudge mark
10.

~.
1.
2.
3.
4.
5.

In a left working movement, the mesiofacial


cusp of a maxillary right second molar will
pass through which of the following?

14.

.4

1
2
3
4
5

. In a normal occlusion, the distolingual cusp


of a maxillary second molar occludes with
the
1. central fossa of a mandibular second
molar.
2. distal fossa of a mandibular second
molar..
3. central fossa of a mandibular third
molar.
4. marginal ridges of a mandibular first
molar and a mandibular second molar.
5. marginal ridges of a mandibular second
molar and a mandibular third molar.

15.
The occlusal outline of a permanent
maxillaryfirst molar contains two obtuse and
two acute angles. The obtuse angles are

1.
2.
3.
4.
5.

1. The embrasure space between a


mandibular right third molar and a
mandibular right second molar
2. The embrasure space between a
mandibular right second molar'and a
mandibular right first molar
3. The buccal groove area of a mandibular
right second molar
4. The buccal groove area of a mandibular
first molar
5. None of these

11.

(w

The mesial surface of the crown is almost


parallel to the long axis and the root of a
1.
2.
3.
4.

mesiofacial and distofacial.


mesiofacial and distolingual.
distofacial and distolingual.
mesiolingual and distofacial.
mesiolingual and distolingual.

16.

maxillary first premolar.


mandibular second premolar.
maxillary canine.
mandibular canine.

When a fourth root canal exists in a


maxillaryfirst molar, it is most likely located
in which of the following roots?
~

1.
2.
3.
4.
5.

12.

17.
Which of the followingmuscles is the prime
mover in effectinga left working movement?
1.
2.
3.
4.

The
The
The
The

The smallest cusp of a permanent


mandibular first molar is the
1.
2.
3.
4.
5.

left lateral pterygoid


left medial pterygoid
right lateral pterygoid
right medial pterygoid

-38-

Lingual
Distofacial
Distolingual
Mesiofacial
Mesiolingual

distal.
distofacial.
mesiofacial.
distolingual.
cusp of Carabelli.

Part I
July 1989

18.

Dental Anatomy/Occlusion

23.

The cross-sectional outline at the cervical


level is roughly triangular in a
1.
2.
3.
4.

Occlusocervically,the height of the mesial


marginal ridge of a permanent mandibular
first molar is the same as the height of the

mandibular second premolar.


permanent maxillary second molar.
permanent mandibular first molar.
primary mandibular second molar.

1.
2.
3.
4.

19.

In an ideal centric occlusion, the lingual


cusp tip (or tips) of a mandibular second
premolar contact(s)

5.

triangular ridge of the mesiofacial cusp


of a mandibular first molar.
triangular ridge of the distofacial cusp of
a mandibular first molar.
distal marginal ridge of a mandibular
first molar.
mesial marginal ridge of a mandibular
second molar.
distal marginal ridge of a mandibular
second premolar.

1. the disto-occlusal slope of the buccal


cusp of a maxillary first premolar.

2. the disto-occlusal slope of the lingual


3.
4.
5.

20.

-24.

cusp of a maxillary first premolar.


the mesio-occl usal cusp ridge of a
maxillary second premolar.
the disto-occlusal cusp ridge of a
maxillary second premolar.
no opposing cusps.

In the drawing below, the arrows represent


,

the possiblecontactingareasforwhichof the


followingmandibular cusps?

Which of the followingcharacterizes a


temporomandibular joint?
1.
2.
3.
4.

1.

It is immune to arthritis.
It has one synovial cavity.
It has two synovial cavities.
It has no synovial cavities.

2.
3.
4.

21.

The opening of the nasopalatine canal is


located at the

25.

1.
2.
3.

anterior midline of the palate.


posterior midline of the palate.
posterior palate, bilaterally near the first
molar root apices.
4. posterior,distal to the last erupted
maxillary molar, bilaterally.
5. lateral wall of the oropharynx, just
anterior to tonsillar pillar.

The mandibular movement indicated in the


drawing in the preceding question is
1.
2.
3.
4.
5.

26.

22.

Cusp of canine, facial cusp of first


premolar and distal cusp of first molar
Facial cusps of premolars and distal cusp
of first molar
Facial cusps of premolars and
mesiofacial cusp of second molar
Lingual cusps of premolars and
distolingual cusp of first molar

In a posterior crossbite relationship, which of


the following may contact in a working
movement?

protrusive.
right lateral; working side.
left lateral; working side.
right lateral; non-working side.
left lateral; non-working side.

Primarymolars differ from permanent molars


in that primary molars
1. have heavier root trunks.
2. tend to have less pronounced cervical

1. Outer aspects of mandibular facial cusps


2. Outer aspects of maxillary lingual cusps
3. Outer aspects of mandibular lingual
cusps
4. Inner aspects of mandibular lingual
cusps

ridges.

3. have thicker enamel compared to the


total bulk of crowns.

4. have flatter facial and lingual surfaces


extending from the occlusal to the
cervical ridge.

-39-

Dental Anatomy/Occlusion

Part I
July 1989

27.

33.

The cervical line on adult teeth has the


greatest depth of curvature toward the incisal
on the

1.
2.
3.
4.
5.

1. Maxillary lateral incisor


2. Maxillary canine
3. Mandibular lateral incisor
4. Mandibular central incisor

mesial aspect.
distal aspect
facial aspect.
lingual aspect.
dentinoenamel junction.

34.
28.

In a longitudinal section of a premolar


crown, the enamel is thickest in the
1.
2.
3.
4.

29.

30.

31.

cervical third.
middle third.
occlusal third.
junction of the cervical and middle
thirds.

35.

middle third of the root.


apical third of the root.
incisal third of the crown.
middle third of the crown.
cervical third of the crown.

36.

Assuming'that occlusion and alignment are


normal, the arrow on the sketch below
represents the path taken by the

1. cusp of a canine.
2. facial cusp of a first premolar.
3. lingual cusp of a first premolar.
4. facial cusp of a second premolar.
5. lingual cusp of a second premolar.

Maxillary first
Maxillary second
Mandibular first
Mandibular second

37.
32.

Maxillaryfirst molar
Mandibular first molar
Mandibular second molar
None of these

Maxillary first
Maxillary second
Mandibular first
Mandibular second

When viewed from the facial, which of the


followingpremolars has a mesial cusp ridge
longer than its distal cusp ridge?
1.
2.
3.
4.

Maxillarycanine
Mandibular canine
Maxillarycentral incisor
Mandibular central incisor
Maxillary lateral incisor
Mandibular lateral incisor

In which of the following primary teeth is the


mesial portion most distinctly separated from
the remainder of the occlusal table by a
transverse ridge?
1.
2.
3.
4.

Which of the following permanent molars


has the largest mesiodistal measurement of
its crown?

1.
2.
3.
4.

Which of the following teeth would most


likely be congenitally absent?

1.
2.
3.
4.
5.
6.

A faciolingual, longitudinal section of a


maxillary canine has its widest dimension of
the pulp cavity faciolingually in the
1.
2.
3.
4.
5.

Which of the following primary teeth has the


smallest faciolingual dimension of its crown?

When viewing a maxillarycentral incisor


from the mesial aspect, one normally finds
the incisal ridge of the crown

The mandibular 'movement indicated in the


drawing in the preceding question is
1.
2.
3.
4.
5.

1. facial to the center of the root.


2. lingual to the center of the root.
3. on line with the center of the root.
4. in variable positions.

-40-

protrusive.
right lateral; working side:
left lateral; working side.
right lateral; non-workingside.
left lateral; non-workingside.

r
!

Anatomic Sciences

Part I

July 1989

t
33.
'I

Which of the following represents the


organelle that is typically composed of
vesicles and of curved, flattened cisternae
that are parallel to one another?

1.
2.
3.
4.

5.

37.

The presence of valves characterizes

1.
2.
3.
4.
5.

Centriole
Mitochondrion
Golgi complex
Rough endoplasmic reticulum
Smooth endoplasmic reticulum

38.

veins only.
blood sinusoids only.
lymphatic vessels only.
veins and lymphatic vessels.
blood sinusoids and blood capillaries.

The visceral and parietal pericardia are


continuous at the

1. pulmonary artery only.


2. inferior vena cava only.
3. veins and arteries entering and leaving
the heart.

34.

Long bones of the skeleton increase in length


because of the

1.
2.
3.
4.

5.

39.

mitotic division of osteocytes.


mitotic division of osteoblasts.
resorption of primary bone by
osteoclasts.
appositional growth on the cartilaginous
epiphyseal plate.
interstitial growth in the cartilaginous
epiphyseal plate.

1.
2.
3.
4.

40.

35.

In addition to the bronchus, which other


major structures form the root of the lung?
The pulmonary artery
The pulmonary artery
The bronchial arteries
The bronchial arteries
phrenic nerve

r.',

'..-.-]

36.

,.-....
\../

parasympathetic fibers.
preganglionic visceral efferents.
postganglionic visceral efferents.

The motor division of the trigeminal nerve


leaves the skull through the

1.
2.
3.
4.

foramen ovale.
foramen rotundum.
inferior orbital fissure.
superior orbital fissure.

veins
azygos vein
veins
veins, and the

Please refer to the sketch below in which is


illustrated the head and neck region of a
human embryo. The facial process labeled A
gives rise to which of the following?

The thoracic splanchnic nerves to the celiac


ganglion consist predominantly of

1.
2.
3.

and
and
and
and

1.
2.
3.
4.

The
The
The
The

nasal septum
nasal conchae
incisive papilla
secondary palate

Part I
July 1989
38.

Dental Anatomy/Occlusio

44.

The condyle on the working side generally


rotates about a

1. sagittal axis only.


2. horizontal axis only.
3. horizontal axis and translates laterally.
4. vertical axis and translates laterally.

39.

40.

1.
2.
3.
4.

In a right lateral excursion, the mesiofacial


cusp of the maxillary right first molar passes
through which of the following grooves of
the mandibular right first molar?

1.
2.
3.
4.

45.

When viewing a permanent maxillary 'canine


from the mesial aspect, one would normally
expect a line bisecting the root longitudinally
to

46.

pass facial to the cusp tip.


-

3.

41.

47.
8 primary and 8 permanent teeth.
10 primary and 10 permanent teeth.

1.
2.
3.
4.
5.

Viewed from the occlusal, the greatest


faciolingual diameter of a permanent
mandibular second molar is located in which
third of the crown?

48.

5.

1. single root.
2. mesial marginal developmental groove.
3. facial cusp much larger than the lingual

49.

-41-

Maxillary first
Maxillary second
Mandibular first
Mandibular second
All of these

in the middle third.


in the cervical third.
in the occlusal third.
at the junction of the cervical and
middle thirds.
at the junction of the middle and
occlusal thirds.

Mesial inclination of lingual cusps is present


in which maxillary premolars?

1.
2.
3.
4.

cusp.
developmental depression on the mesial
aspect of the crown including the
cervical area.

The lingual height of contour on a


permanent mandibular second molar is
located
1.
2.
3.
4.

The maxillary second premolar may be


distinguished from the maxillary first
premolar in that the second premolar usually
has a

4.

More occlusal surfaces can be seen from the


mesial than from the distal in which of the

followingpremolars?

18 primary and 6 permanent teeth.


20 primary teeth.
permanent teeth only.

1. Distal
2. Mesial
3. Middle
4. None of these

43.

permanent maxillary first molar.


primary mandibular first molar.
mandibular second premolar.
maxillary second premolar.
maxillary third molar.

The erupted dentition of a normally


developed 7-year-oldchild usually consists of

12.
3.
4.
5.
42.

bisect the cusp tip also.

pulp fibrosis.
morphod ifferentiation.
dentinal dysplasia.
deposition of primary dentin.
deposition of secondary dentin.

The crown of a primary maxillary first molar


bears the closest resemblance to the crown
of a
1.
2.
3.
4.
5.

2. pass lingual to the cusp tip.

middle third.
incisal third.
cervical third.
junction of the middle and cervical
thirds.

Followingthe initial period of functional


activity,an appreciable alteration in size of
the pulp chamber is a direct result of
1.
2.
3.
4.
5.

Lingual groove
Central groove
Facial groove
Distofacial groove

1.

The contact area on the mesial surface of a


mandibular canine is located at the

First premolars only


Second premolars only
Both first and second premolars
Neither first nor second premolars

Part I
July 1989

Dental Anatomy/Occlusion
~

50.

55.

Which of the following teeth has its mesial


contact area located within the incisal or the
occlusal one-third?
1.
2.
3.
4.

The Bennett movement is best described as


the
1.
2.

Maxillary canine
Maxillary first molar
Mandibular second premolar
Mandibular central incisor

3.
4.

51.

Which root of a permanent maxillary first


molar is flattened mesiodistally and has root
depressions on both its mesial and distal
surfaces?
1.
2.
3.
4.
5.

52.

2.
3.
4.

Mesiofacial
Distolingual
Mesiolingual

57.

facial surface of a permanent mandibular


first molar.
lingual surface of a permanent maxillary
first molar.
distal surface of a permanent central
incisor.
mesial surface of a permanent canine.

Perikymataare the result of

59.

The base of the tongue is attached to the


hyoid bone by
1.
2.
3.
4.
5.

2.
3.

4.
5.

A root bifurcation
A mesial concavity
A distolingual groove
An extreme distal cervical line curvature

1. enamel hyperplasia.
2. enamel hypoplasia.
3. interstitial growth.
4. normal enamel apposition.
5. early ameloblastic degeneration.

At the level of the furcation


At varying levels, dependent upon age
Within the cervical third of the crown
Apical to the cementoenamel junction

The sketch below represents the

1.

V
VII
IX
XII

Which of the following anatomic features of


a maxillary lateral incisor will most likely
complicate root planing?
1.
2.
3.
4.

58.

medial shift of the working condyle.


lateral movement of the non-working
condyle.
bodily shift of the mandible in the
direction of the working condyle.
bodily shift of the mandible in the
direction of the non-working condyle.

A branch of which of the following cranial


nerves to the tongue may be anesthetized
during administration of an inferior alveolar
block?

1.
2.
3.
4.

In a molar, where do root canals usually join


the pulp chamber?
1.
2.
3.
4.

54.

Lingual
Distofacial

The height of contour occlusocervically is


located within the middle third of the
1.

53.

56.

;~!
'oJ
!J;

60.

facial view of a permanent mandibular


right second molar.
lingual view of a permanent mandibular
right second molar.
facial view of a permanent mandibular
left second molar.
lingual view of a permanent mandibular
left second molar.
lingual view of a primary mandibular left
second molar.

The root tip most likely to be forced into the


maxillarysinus during surgical removaJis
that of a
1.
2.
3.
4.
5.

-42-

the lingual tonsil.


the hypoglossusmuscle.
the geniohyoid muscle.
three folds of mucous membrane.
both (1)and (2) above.

maxillary first premolar.


permanent maxillarycanine.
permanent maxillaryfirst molar.
permanent maxillarycentral incisor.
primary maxillary second molar at
age.12.

:~

~~

.~

"
~
~
*,
!,
"

11

Part I
July 1989

61.

Dental Anatomy/Occlusion

65.

From an occlusal view, the arrangement of


permanent teeth of the maxiIfary and
mandibular arches are parabolic in shape. In
one segment of the dentition, however, four
teeth are aligned in a straight line. In what

region is this segment located?


1.
2.
3.
4.

Which of the following groups of fibers of


the periodontal ligament is most likely to be
found in the middle third of the root?
1. Apical
2. Obi ique
3. Horizontal
4. Transseptal
5. Alveolar crest

Maxillary anterior
Maxillary posterior
Mandibular anterior
Mandibular posterior

66.

62.

Which of the following best describes the


root lengths of a maxillary molar?

1. The lingual root is the longest and the

The drawing below represents a typical


occlusal outline for amalgam for which of
the following molars?

2.
3.

4.

~~
1.
2.
3.
4.
5.

67.

The proxi mal contact of posterior teeth


creates wear patterns that eventually cause
1.
2.
3.
4.

Mandibular right first molar


Mandibular left first molar
Maxillary right first molar
Maxillary left first molar
Maxillary right third molar

68.
63.

2.

3.
4.

64.

1.
2.
3.
4.

a maxillary lateral incisor and a


maxillary canine at the approximation
of their marginal ridges.
the mesiolingual fossa and the mesial
marginal ridge of a maxillary canine at
the junction of the cervical and
middle thirds.
the incisal embrasure between a
maxillary canine and a lateral incisor.
none of these.

gingival recession.
increased length of clinical crowns.
decreased length of anatomic crowns.
reduced interproximal embrasure spaces.

The non,.working pathway of the maxillary


cusps on the mandibular posterior teeth is
toward the

In the intercuspal position, the incisal third


of the facial surface of a permanent
mandibular canine opposes
1.

distofacial root is the shortest.


The lingual root is the longest and the
mesiofacial root is the shortest.
The mesiofacial root is the longest and
the distofacial root is the shortest.
All roots are the same length.

69.

distofacial.
distolingual.
mesiofacial.
mesiolingual.

Which of the following sagittal views of the


,

Posselt'senvelope is the correct tracing for a


patient with coincidence of centric occlusion
and centric relation?

The spacing between anterior teeth in the


primary dentition is most frequently caused
by

1.
2.
3.
4.

2
1.
2.
3.
4.

thumb-sucking.
tongue thrusting.
the growth of the dental arches.
the pressure from succedaneous teeth.

-43-

1
2
3
4

Part I
July 1989
70.

Dental Anatomy/Occlusion

75.

The healthy free gingiva aids in the selfcleansing processby

1.
2.

3.
4.

When the mandible

slides

in a

protrusive

'contacting movement, which of the following


mandibular
teeth can make contact with the
maxillary lateral incisors?

directing food particles toward the


occlusal surface.
forcing particles away from the proximal
space.
deflecting particles away from the free
gingiva onto the interdental papillae.
adhering closely to the tooth surface
below the height of contour of the
cervical enamel.

1.
2.
3.
4.
76.

Central and lateral incisors


Central incisors and canines
Lateral incisors and canines
All of these

Which of the following positions would yield


the

smallest measurement of vertical

dimension?

71.

1.
2.
3.
4.

72.

Maxillary first
Maxillary second
Mandibular first
Mandibular second

77.

molar is the

1.
2.
3.
4.
5.

maxillary central incisors only.


,
maxillary central and lateral incisors.
maxillary lateral incisors and canines.

78.

2.

3.

4.

79.
distal to the tip of the maxillary canine
cusp.
mesial to the tip of the maxillary canine
cusp.
directly in line with the maxillary canine
cusp tip.
through the embrasure between the
maxillary canine and first premolar.

central groove.
facial groove.
lingual groove.
distal oblique groove.

transverse groove of the oblique ridge.

The lingual cusps of a mandibular first molar


must be restored to accommodate

1.
2.
3.
4.
5.

In an ideal permanent tooth relationship, the


tip of a mandibular canine in lateral
excursion passes

1.

occlusal surface of a maxillary molar,


representsthe working path
of the distofacialcusp of the mandibular
On the

the groove that

The wear facets on the incisal edges of the


mandibular lateral incisors are caused by
occlusion with the

1.
2.
3.

73.

1. Reverse overlap
2. Edge-to-edge
3. Retruded contact
4. Maximum intercuspation

Which of the following premolars has a


mesial marginal ridge that is more cervically
located than its distal marginal ridge?

centric relation.
working movement.
non-working movement.
protrusive position.
maximumintercuspation.

A broad, flat facet existingon the outer


aspect of the mesiolingual cusp of a
maxillary first molar, and running in a
mesiolingual to distofacial direction, was

probably caused

by which of the

following

~,

'It:
"

contacting movements?

1. Working
2. . Non-working
3. Protrusive
4.

Lateral protrusive
.,

74.

80.

Moving the mandible from a maximum


intercuspal position to a retruded contact
position usually results in

1. increasedocclusal vertical dimension.


2. increased horizontal overlap.
3. decreased vertical overlap.
4. all of these.

When compared with the incisal embrasure


between the maxillary central and lateral
incisors,the incisal embrasure between the
maxillary central incisors is

1. larger.
2. smaller.
3. the same size.

-44-

Part I
July 1989

81.

maxillary central incisor.


primary mandibular central incisor.
permanent mandibular central incisor.
permanent mandibular first molar.
primary mandibular first molar.

1.
2.
3.
4.
5.
6.

87.

1. . the initial upward closure of the

3.
4.

84.

85.

mandible.
forward movement of the condyle from
the articular fossa.
posterior displacement of the condyle
from the articular eminence.
the final forceful closure of the molars
through a bolus of food.

(a) and
(a) and
(a) and
(b) and
(b) and
(c) and

(b)
(c)
(d)
(c)
(d)
(d)

In comparison with the primary maxillaryfirst molar, the primary maxillary second
molar
(a)
(b)
(c)
(d)

1.
2.
3.
4.

In a patient with a left canine protection, the


mesiolingual surfaceof the maxillaryright
first molar contacts the distofacialsurface of
the mandibular right first molar during a left
lateral excursion.This contact is
1.
2.
3.
4.
5.

Maxillary central
Mandibular central
Maxi lIary lateral
Mandibular lateral

mesiofacial portion of a first molar.


distofacial portion of a first molar.
distofacial portion of a second molar.
mesiolingual portion of a second molar.
distolingual portion of a second m01ar.

The contraction of the lateral pterygoid


muscle causes

2.

Which of the following permanent incisors


most frequently have concave areas on both
mesial and distal root surfaces?
(a)
(b)
(c)
(d)

Among the primary maxillary teeth, the


cervical ridge would stand out most
prominently as a distinct entity on the
1.
2.
3.
4.
5.

83.

86.

The first succedaneous tooth to erupt is the

1.
2.
3.
4.
5.

82.

Dental Anatomy/Occlusion

normal.
evidence of group function.
a working side interference.
a non-working side interference.
normal, and a non-working side
interference.

88.

(a) and (b) only


(b) and (c) only
(c) and (d) only
(a), (b), (c), and (d)

Mandibular premolars differ from maxillary


premolars in that mandibular premolars have
(a)
(b)
(c)
(d)

During a protrusive excursion, the


mandibular condyles progress in which of
the following directions?

1.
2.
3.
4.
5.

1. Forward and upward


2. Forward and downward
3. Backward and upward
4. Backward and downward

-45-

is less bulky.
is wider mesiodistally.
erupts later.
has two lingual cusps, rather than
three.

more rounded roots.


crowns tilted to the lingual.
less developed lingual cusps.
larger dimensions in all directions.

(a), (b), and (c) only


(a) and (d) only
(b), (c), and (d) only
(c) and (d) only
(a), (b), (c), and (d)

Part I

Dental Anatomy/Occlusion

July 1989

89.

92.

Characteristicsof a permanent mandibular


canine that distinguish it from a permanent
maxillarycanine include

(d)

1.
2.
3.
4.
5.
6.

90.

a more prominent cingulum.


more distinct, lingual fossae.
a comparatively narrower
mesiodistal dimension.
a continuous convex facial surface
from incisal to apical end, when
view proximally.

(b)
(c)
(d)

(a), (b), and (c)


(b) and (c) only
(b) and (d)
(c) only
(c) and (d)
(d) only

1.
2.
3.
4.
5.

93.

In which of the following anterior teeth


would at least one contact area be located
within the middle of the crowns
incisogingivally?
(a)
(b)
(c)
(d)
(e)
(f)

1.
2.
3.
4.
5.

94.
91.

The size of the

pulp cavity within

(b)
(c)
(d)

1.
2.
3.
4.
5.

synovial fluid.
durability of the fibrocartilage.
ligament suspension.
muscles of mastication.

(a) only
(a), (c), and (d) only
(b) and (c) only
(b), (c), and (d) only
(d) only

(a), (b), (c), and (d) .

Which of the followingteeth are


succedaneous?

a tooth is

influenced by the
(a)

During function and in traumatic situations,


the temporomandibular joint is protected by
the

1.
2.
3.
4.
5.
6.

(a), (b), and (c)


(a), (b), (d), and (e)
(b), (c), (e), and (f)
(b), (c), and (f) only
(d), (e), and (f)

They may be found in the cervical


third of the root.
They contain nervous and vascular
tissues.
They may be found in fureation
areas of molars.
They allow the pulp tissue to
communicate with the
periodontal ligament space.

(a) and (b) only


(a), (b), and (c) only
(a), (c), and (d) only
(b) and"{d) only
(a), (b), (c), and (d)

(a)
(b)
(c)
(d)

Maxillary central incisor


Maxillary lateral incisor
Maxillary canine
Mandibular central incisor
Mandibular lateral incisor
Mandibular canine

,j
,

Which of the followingcharacterize the.


accessory pulp canals?
(a)

(a)
(b)
(c)

JI

(a)
(b)
(c)
(d)
(e)
(f)
(g)
(h)

ability of ameloblasts to form new


denti n.
age of the tooth.
parafunctional activity of the tooth.
history of the tooth (abrasion,
erosion, caries, etc.).

Maxillary right first molar


Maxillary right central incisor
Maxillary left canine
Maxillary left first molar
Mandibular left second molar
Mandibular left second premolar
Mandibular left central incisor
Mandibular right second premolar

1. (a), (d), (e), (f), and (h)


2. (a), (d), (e), and (h) only
3. (b), (c), (e), (f), and (g)
4. (b), (c), (f), and (g) only
5. (b), (c), (f), (g), and (h)

(a), (b), and (c) only


(a) and (c) only
(b), (c), and (d) only
(b) and (d) only
(a), (b), (c), and (d)

-46-

1!
iJ
~

i.
~

Part I

Dental Anatomy/Occlusi(

July 1989

95.

(a)
(b)
(c)
(d)

1.
2.
3.
4.
5.
96.

Which of the following


of permanent maxillary
(a)
(b)
(c)
(d)
(e)
(f)

The facial surfaces of permanent


maxillary first molars
The lingual surfaces of permanent
maxillary first molars
The facial surfaces of permanent
mandiburar first molars
The lingual surfaces of permanent
mandibular first molars

1.
2.
3.
4.
5.

(a) and (c)


(a) and (d)
(b) and (c) only
(b), (c), and (d)
(b), and (d) only

(a),
(a),
(a),
(b),
(a),

are anatomic features


central incisors?

Cingula
Mamelons
Cuspal ridges
Marginal ridges
Cervical ridges
Triangular ridges
(b),
(b),
(d),
(c),
(b),

(c), and (f) only


(d), and (e) only
and (e) only
and (f) only
(c), (d), (e), and (f)

The design of a restored occlusal surface is


dependent upon the
(a)
(b)
(c)
(d)

1.
2.
3.
4.
5.
97.

98.

On which of the following surfaces is caries


most likely to occur?

contour of the articular eminence.


position of the tooth in the arch.
amount of lateral shift in the
rotating condyle.
amount of vertical overlap of
anterior teeth.

(a) and
(b), (c),
(b) and
(c) and
(a), (b),

99.

(c) only
and (d) only
(d) only
(d) only
(c), and (d)

1.
2.
3.
4.

When occlusal relationships are normal,


which portions of the mandibular molars
have potential for contacting antagonists
during the eccentric movements of the
mandible?
(a)
(b)
(c)
(d)

1.
2.
3.
4.
5.

The outer
cusps
The inner
cusps
The outer
cusps
The inner
cusps

(a) and
(a), (b),
(b) and
(c) and
(a), (b),

In the mandibular arch, the contact areas


(cervico-incisally or cervico-occlusally) are at
approximately equal levels mesially and
distally on each tooth EXCEPTthe'
mandibular

canine.
first molar..
first premolar.
lateral incisor.

aspects of the buccal


aspects of the buccal
aspects of the lingual

100.

aspects of the lingual

The functions of the dental pulp include all

of the following EXCEPTto

1. form dentin.
2. supply dentin with nutrients.
3. innervate the enamel with nerve fibers.
4. transmit sensory stimuli to the central

(b) only
and (d) only
(d) only
(d) only
(c), and (d)

nervous

;i~

-47-

system.

NATIONAL BOARD DENTAL EXAMINATION PART I


ANSWER KEY
DENTAL ANATOMY & OCCLUSION

JULY 1989

ANS.

NO.

ANS.

NO.

ANS.

NO.

ANS.

1.
2.
3.
4.
5.

3
2
5
1
2

26.
27.
28.
29.
30.

4
.1
3
5
3

51.
52.
53.
54.
55.

3
2
4
1
3

76.
77.,
78.
79.
80.

4
2
2
1
2

6.
7.
8.
9.
10.

3
1
1
3
5

31.
32.
33.
34.
35.

1
3
4
5
2

56.
1
57.
3
58.,
4
59. not scored
60.
3

81.
82.
83.
84.
85.

3
1
2
4
2

11.
12.
13.
14.
15.

4
3
2
5
4

36.
37.
38.
39.
40.

3
3
4
1
2

61.
62.
63.
64.
65.

4
3
1
3
2

86.
87.
88.
89.
90.

5
2
1
5
4

16.
17.
18.
19.
20.

4
1
2
5
3

41.
42.
43.
44.
45.

3
2
1
2
5

66.
67.
68.
69.
70.

1
4
1
3
4

91.
92.
93.
94.
95.

3
5
6
5
3

21.
22.
23.
24.
25.

1
4
5
2
2

46.
47.
48.
49.
50.

4
3
1
3
4

71.
72.
73.
74.
75.

3
2
2
4
3

96.
97.
98.
99.
100.

5
2
2
1
3

NO.

Anatomic Sciences

Part I

July1989
41.

45.

The unpaired vesselscontributing to the


arterial circle of the brain include the

Please refer to the photomicrograph below.


Which letter points to acellular cementum?

1. internal carotid and vertebral arteries.


.2, posterior and anterior cerebral arteries.
3. anterior cerebral and anterior
4.
5.

42.

Which of the following layers is totally


lacking in thin skin?
1.
2.
3.
4.

43.

Stratum
Stratum
Stratum
Stratum

corneum
lucidum
spinosum
germinativum

The anterior interventricular (descending)


artery is accompanied by the
1.
2.
3.
4.

44.

communicating arteries.
anterior cerebral and posterior
communicating arteries.
anterior communicating and basilar
arteries.

small cardiac vein.


middle cardiac vein.
great cardiac vein.
coronary sinus.

In the drculating blood of an adult, which of


the following leukocytes normally appears in
the highest numbers?

1.
2.
,3.
4.
5.

Basophils
Monocytes
Eosinophils
Neutrophils
Lymphocytes

1.
2.
3.
4.

-6-

A
B
C
D

r
,

Part I
July 1989

46.

The jugular foramen transmits which cranial


nerves?
1.
2.
3.
4.

47.

51.

VII, IX, and XII


IX, X, and XI
IX, XI, and XII
X, XI, and XII

Which of the following structures contacts


the thyroid gland?

1. .A
2. B
3. C
4. 0
5. E

52.

Connective tissue proper is characterized as


having
1.
2.
3.
4.

The ribonucleic acid involved in protein,.


synthesis is found primarily in the
1. nucleolus.
2. mitochondria.
3. 'Golgi apparatus.
4. fluid ground substance.
5. granular endoplasmic reticulum.

53.

little tissue fluid.


poor reparative ability.
sensitivity as its main function.
more intercellular material than cells.

Which of the following nerves descends


anteriorly to the root of the right lung and
between the mediastinal pleura and the
pericardium?
1.
2.
3.
4.

50.

The pulmonary vein


The pulmonary artery
The bronchial artery
None of these

1. Lingual nerve
2. Phrenic nerve
3. . Thoracic duct
4. Thyroarytenoid muscle
5. Recurrent laryngeal nerve

49.

Identify the thalamus in the drawing below.

Which of the following supplies oxygenated


blood to the capillary plexus of tissues>
surrounding a secondary bronchus?
1.
2.
3.
4.

48.

Anatomic Science

Metastasizing cells that originate in the tip of


the tongue follow the paths of the tongue's
lymphatic vessels and arrive first in which of
the following nodes?

1.
2.
3.
4.
5.

Submental
Submandibular
Supraclavicu lar
Superficial>parotid
Superior deep cervical

54.

The right vagus nerve


The right greater splanchnic nerve
The nerve that supplies the right half of
the diaphragm.
The nerve that supplies pain fibers to
alveoli of the right lung.

The superficial head of the medial pterygoid


muscle originates on the tuberosity of the
maxilla and on the
1.
2.
3.
4.

pterygoid fossa.
pyramidal process of the palatine bone.
crest of the greater wing of the sphenoid
bone.
lateral surface of the lateral pterygoid
plate.

,-

Anatomic Sciences

Part I
July 1989

55.

1.
2.
3.
4.

56.

59.

In the fetus, the ductus venosus permits


blood to flow from the
pulmonary vein to the aorta.
pulmonary artery to the aorta.
right atrium to the left atrium.
umbilical vein to the inferior vena cava.

The posterior cord of the brachial plexus


gives rise to which of the following nerves?
1.
2.
3.
4.
5.

Radial
Long thoracic
Thoracoacromial
Medial pectoral
Lateral pectoral

1.
2.
3.

60.

57.

In the photomicrograph of enamel below, the


degree of calcification of C, compared to that
of 0, is

1.
2.
3.
4.
5.

The most important organelle or component


of a cell for oxidative processes is the

1.
2.
3.
4.
5.

The postcentral gyrus is located in the


insula.
frontal lobe.
temporal lobe.
parietal lobe.
occipital lobe.

61.

The cell body of the efferent (motor) neuron


of a spinal reflex arc is located in
1.
2.

3.
4.

62.

an autonomic ganglion.
the dorsal root ganglion (spinal
ganglion).
the anterior gray column (ventralhorn)
of the spinal cord.
the posterior gray column (dorsal horn)
of the spinal cord.

nucleus.
nucleolus.
mitochondrion.
Golgi complex.
endoplasmic reticulum.

The primary sensory neurons' nucleus of


termination involved in pain from the
maxillary second molar is the
1.
2.
3.
4.
5.

58.

higher.
lower.
similar.

thalamic nucleus.
nucleus ambiguus.
nucleus of tractus solitarius.
spinal nucleus of the trigeminal nerve.
mesencephalic nucleus of the trigeminal
nerve.

Lymphfrom lungs, bronchi, and trachea


drains chiefly into
1.
2.
3.

mediastinal lymph nodes.


deep cervical lymph nodes and vessels.
superficial cervical lymph nodes and
vessels.
4. lymph nodes and the vessels paralleling
internal mammary blood vessels.

-8-

"'"'

Part I
July 1989
63.

In a faciolingual section through a tooth and


its related gingiva, the demarcation line
between free gingiva and attached gingiva
extends between the
1.

2.
3.
4.

64.

Anatomic Sciences

alveolar crest and the free gingival


groove.
bottom of the gingival sulcus and the
free gingival groove.
apical margin of the epithelial
attachment and the alveolar crest.
apical margin of the epithelial
attachment and the free gingival
groove.

68.

1.
2.
3.
4.

69.

1. The pulmonary and bronchial veins


2. The coronary sinus and pulmonary veins
3. The superior and inferior venae cavae

65.

and coronary sinus


The superior vena cava, coronary sinus,
and hemiazygos vein

2.
3.
4.
5.

facial artery, the superior labial artery,


and angular arteries.
lingual artery, the deep lingual artery,
and internal nasal arteries.
occipital artery, the ascending palatine
artery, and posterior nasal arteries.
maxillary artery, the sphenopalatine
artery, and posterior lateral nasal
arteries.
superficial temporal artery, the transverse
facial artery, and external nasal
arteries.

The branch of the ophthalmic artery that


pierces the optic nerve is the

1.
2.
3.
4.

supraorbital.
anterior ciliary.
anterior ethmoidal.
central artery of the retina.

the highest intercostal artery.


the posterior intercostal arteries.
the bronchial and esophageal arteries.
the superior epigastric and
musculophrenic arteries.

Which of the followingcharacterizes a lymph


node?
1.
2.
3.

The external carotid artery contributes to the


blood supply of the nasal cavity by way of
the
1.

67.

71.

Osteons enlarge.
Cells develop processes.
Water content decreases.
Collagenous fibrils form.
Ground substance aggregates.

The internal thoracic artery gives rise to


1.
2.
3.
4.

The connective tissue sheath that surrounds a


muscle as a whole and is synonymous with
the gross anatomic deep fascia is the

1. epimysium.
2. endomysium.
3. perimysium.
4. periosteum.
5. perichondrium.

66.

70.

Otic
Geniculate Trigeminal
Pterygopalati ne

Which of the following is the major change


in bone matrix during its mineralization?
1.
2.
3.
4.
5.

Which of the followingempty into the right


atrium?

4.

Cell bodies of taste fibers from the anterior


two-thirds of the tongue are located in which
of the following ganglia?

4.

It contains medullary cords.


It lacks afferent lymphatics.
It has crypts lined with stratified
squamous epithelium.
It has efferent lymphatics leaving at
multiple sites from the capsule.

;!
I

72.

The initial epithelial attachment joining the


gingiva to the tooth arises directly from
1.
2.
3.
4.

73.

oral mucosa.
cervical loop epithelium.
reduced enamel epithelium.
the epithelial root sheath (Hertwig).

Which region of dentin is the most highly


mineralized?
1. The peritubular
2. The intertubular
3. The interglobular

Anatomic Sciences

Part I

July1989
74.

1.
2.
3.
4.
5.

75.

The
The
The
The
The

faci~1artery
occipital artery
maxillary artery
superior thyroid artery
posterior auricular artery

Corpus
Corpus
Oocyte
Oocyte

81.

luteum
albicans
in a mature follicle
in an early primary foHicle""-"h~,~M

82.

A tumor of the maxillary sinus may cause-an


overflow of tears by exerting pressure on the

83.

Which of the following differentiate


cementum from enamel?
(a)
(b)
(c)'

The middle pharyngeal constrictor muscle


originates from the

(d)

Foramen ovale
Stylomastoid foramen
Foramen magnum and hypoglossal canal
Superior orbital fissure and
petrotympanic fissure

84.

Which of the following abdominal viscera


are retroperitoneal?
(a)
(b)
(c)
(d)
(e)
(f)

The tendon of the tensor veli palatini muscle


curves around the

1.
2.
3.
4.
5.

The presence of collagen fibers


The absence of incremental lines
The cellular component in the
mature tissue
The derivation from the epithelial
root sheath (Hertwig)

1. (a) and (b) only


2. (a) and (c) only
3. (a) and (d) only
4. (b) and (c) only
5. (b) and (d) 'only
6. (c) and (d) only
7. (a), (b), (c), and (d)

Preganglionic parasympathetic fibers pass


through which of the following structures?
1.
2.
3.
4.

79.

hyaline cartilage.
elastic cartilage.
highly cellular fibrous connective tissue.
fibrous connective tissue containing
chond rocytes.

1. lacrimal gland.
2. lacrimal puncta.
3. conjunctival sac.
4. nasolacrimal duct.

1. hyoid bone.
2. thyroid cartilage.
3. pterygoid hamul us.
4. pterygomandibular raphe.

78.

cortex of the thymus.


medulla of the thymus.
cortex of the suprarenal (adrenal).
medulla of the suprarenal (adrenal).

In the temporomandibular joint of an older


person, the articulating surface of the
temporal bone is covered by
1.
2.
3.
4.

Which of the following muscles serves as the


forearm's principal extensor?

1. Pronator teres
2. Brachioradialis
3. Triceps brachii
4. Coracobrachial is
5. Extensor carpi radialis longus

77.

The zona reticularis is the inner layer of the


1.
2.
3.
4.

The zona pellucida is associated with which,


of the following?
1.
2.
3.
4.

76.

80.

Which of the following branches of the


external carotid artery originates just below
the level of the hyoid bone's greater cornu?

1.
2.
3.
4.
5.
6.

angular spine.
styloid process.
pterygoid hamulus.
lateral pterygoid plate.
spine of the sphenoid.

-10-

Aorta
liver
Kidney
Stomach
Pancreas
Suprarenal (adrenal)

(a), (b), (c), and (d)


(a), (b), (c), and (e)
(a), (b), (d), and (f)
(a), (c), (e), and (f)
(b), (d), (e), and (f)
(c), (d), (e), and (f)

Part I
July 1989
95.

Dental Anatomy/Occlusi(

(a)
(b)
(c)
(d)

1.
2.
3.
4.
5.
96.

Which of the following


of permanent maxillary
(a)
(b)
(c)
(d)
(e)
(f)

The facial surfaces of permanent


maxillary first molars
The lingual surfaces of permanent
maxillary first molars
The facial surfaces of permanent
mandiburar first molars
The lingual surfaces of permanent
mandibular first molars

1.
2.
3.
4.
5.

(a) and (c)


(a) and (d)
(b) and (c) only
(b), (c), and (d)
(b), and (d) only

(a),
(a),
(a),
(b),
(a),

are anatomic features


central incisors?

Cingula
Mamelons
Cuspal ridges
Marginal ridges
Cervical ridges
Triangular ridges
(b),
(b),
(d),
(c),
(b),

(c), and (f) only


(d), and (e) only
and (e) only
and (f) only
(c), (d), (e), and (f)

The design of a restored occlusal surface is


dependent upon the
(a)
(b)
(c)
(d)

1.
2.
3.
4.
5.
97.

98.

On which of the following surfaces is caries


most likely to occur?

contour of the articular eminence.


position of the tooth in the arch.
amount of lateral shift in the
rotating condyle.
amount of vertical overlap of
anterior teeth.

(a) and
(b), (c),
(b) and
(c) and
(a), (b),

99.

(c) only
and (d) only
(d) only
(d) only
(c), and (d)

1.
2.
3.
4.

When occlusal relationships are normal,


which portions of the mandibular molars
have potential for contacting antagonists
during the eccentric movements of the
mandible?

(a)
(b)
(c)
(d)

1.
2.
3.
4.
5.

The outer
cusps
The inner
cusps
The outer
cusps
The inner
cusps

(a) and
(a), (b),
(b) and
(c) and
(a), (b),

In the mandibular arch, the contact areas


(cervico-incisally or cervico-occlusally) are at
approximately equal levels mesially and
distally on each tooth EXCEPTthe'
mandibular
can ine.
first molar..
first premolar.
lateral incisor.

aspects of the buccal


aspects of the buccal
aspects of the lingual

100.

aspects of the lingual

The functions of the dental pulp include all


of the following EXCEPTto

1.
2.
3.
4.

(b) only
and (d) only
(d) only
(d) only
(c), and (d)

""~

-47-

form dentin.
supply dentin with nutrients.
innervate the enamel with nerve fibers.
transmit sensory stimuli to the central
nervous system.

PART I
MONDAY-A.M.
DECEMBER 1987

1.

ANATOMIC SCIENCES (11)

Which of the following may cause dead tracts


in dentin?
1.
2.
3.
4.
5.

7.

Skeletal muscle enlarges with prolonged


activity as a consequence of
1.
2.
3.

Caries
Erosion
Cavity preparation
Odontoblastic crowding
Any of the above

4.
.

2.

The component of bone tissue that gives bone


tensile strength is the
1.
2.
3.
4.
5.

3.

elastic fibers.
calcified ground substance.
interconnecting canaliculi.
periosteal connective tissue.
collagenous fibrils of matrix.

The cerebellar cortex is characterized


histologically by which of the following types
of cells?
1.
2.
3.
4.
5.

4.

1.
2.
3.
4.
5.

9.

10.

Taste
Proprioception
Pain from the ipsilateral side of the face
Light touch from the contralateral side of
the face

The junction between primary and secondary


dentin is characterized by a

11.

5.

12.
6.

The postganglionic sympathetic fibers to the


vessels of the submandibular salivary gland
arise from cells in the
1.
2.
3.
4.

otic ganglion.
submandibular ganglion.
pterygopalatine ganglion.
superior cervical ganglion.

primary cuticle.
secondary cuticle.
inner enamel epithelium.
cementoenamel junction.
dentinoenamel junction.

The thick myofilaments observed in


transmission electron microscopic preparations
of skeletal muscle are
composed mainly of actin.
present only in the A band.
present in both A and I bands.
traversed or crossed by the Z band.

From an anatomic standpoint, an emergency


airNaY may be established most readily by
opening into the trachea
1.
2.
3.
4.

tubules.
sharp change in direction of dentinal
tubules.
Both (1)and (3) above

cracks.
ameloblasts.
odontoblasts.
hypoplastic rods.
hypocalcified rods.

The structure first formed by the tooth bud that


remains in evidence in the formed tooth is the

1.
2.
3.
4.

1. resting line.
2. reversal line.
3. sharp reduction in number of dentinal
4.

Enamel spindles are formed by

1.
2.
3.
4.
5.

If the spinal tract of the fifth cranial nerve were


sectioned at the level of the caudal medulla,
which of the following modalities would be

1.
2.
3.
4.

8.

Basket
Purkinje
Pyramidal
Martinotti
Horizontal

most affected?

5.

differentiation of myoblasts.
mitotic division of muscle fibers.
an increase in endomysial connective
tissue.
an increase in sarcoplasm and in the
number of myofibrils of existing muscle
fibers.

between thyroid carti lages.


at the level of the jugular notch.
through the thyrohyoid membrane.
through the median cricothyroid ligament.

The organelle that binds and releases calcium


during relaxation and contraction of skeletal
muscle is a

1.
2.
3.
4.
5.

nucleus.
lysosome.
mitochondrion.
transverse tubule.
sarcoplasmic reticulum.

"".

. .
"--""'.'..".

':r~~!
.
.
93.

All of the following structures can be seen in a


histologic examination of the adult parotid
gland EXCEPT

1.
2.
3.
4.
5.
94.

Which of the following layers of the epidermis


is least cytodifferentiated?

1.
2.
3.
4.
5.

striated ducts.
serous demilunes.
intercalated ducts.
myoepithelial cells.
granular serous cells.

Stratum basale
Stratum corneum
Stratum spinosum
Prickle-cell layer
Stratum' granulosum

Which of the following arteries does NOT


accompany the corresponding nerve
throughout its course?
1.
2.
3.
4.

95.

97.

Lingual
Infraorbital
Inferioralveolar
Posterior superior alveolar

98.

Which of the following tissues would be least


affected if the anterior lobe of the hypophysis
were destroyed?

1.
2.
3.
4.
5.

Which of the following structural changes is


NOT encountered in passing along the
respiratory system from the trachea to a
respiratory bronchiole?

Thyroid epithelium
Medulla of the adrenal gland
Interstitial cells of the testis
Zona fasciculata of the adrenal gland
Spermatogenic epithelium of the testis

1. Decrease in goblet cells


2. Decrease in ciliated cells
3. Total loss of cartilage from the wall
4. Progressive change from stratified
.

squamous to cuboidal epithelial lining

96.

All of the following structures are concerned


with development of the tongue EXCEPTthe

1.
2.
3.
4.
5.

copula.
macula.
tube~culum impar.
secondbranchial arch.
third branchial arch.

99.

The secretion of which of the following


endocrine glands is NOT essential to liJe?
1.
2.
3.
4.
5.

Parathyroids
Adrenal cortex
Adrenal medulla
Anterior pituitary

Pancreatic islets (Langerhans)

...

.."

NATIONAL BOARD DENTAL EXAMINATION PART I


A NS\i ER

ANATOMIC SCIENCES

.'

KEY

.-

DECEMBER 1987

NO.

ANS.

NO.

ANS.

NO.

ANS.

NO.

ANS.

1.
2.
3.

5
5
2
3
4

26.
27.
28.
2 9.
30.

1
4
1
1
3

51.
52.
53.
54.
55.

4
3
3
4
3

76.
77.
78.
79.
80.

3
5
2
1
2

31.
32.
33.
34.
35.

4
1
4
4
3

56.
57.
58.
59.
60.

2
2
3
2
1

81.
82.
83.
84.
85.

2
4

10.

4
4
3
5
2

11.
12.
13.
14.
15.

4
5
5
2
1

36.
3 7.
38.
3 9.
40.

2
3
3
1
3

61.
62.
63.
64.
6 5.

1
2
2
4
3

86.
8 7.
88.
89.
90.

3
2
4
4
1

16.
17.
18.
19.

2o.

3
2
4
1
3

41.
42.
43.
44.
45.

4
2
2
4
3

66.
6 7.
68.
69.
70.

4
1
2
4
2

91.
92.
93.
94.
95.

5
1
2
1
4

21.
22.
23.
24.
25.

2
1
2
5
3

46.
47.
48.
49.
50.

2
3
4
4
1

71.
72.
73.
74.
75.

4
4
2
4
3

96.
97.
98.
99.

2
1
2
3

4.
5.
6.
7.

8.
9.

'"

7
1
3

.,
,

BIOCHEMISTRY-PHYSIOLOGY(12)

1.

7.

The ~ value of an enzyme is numerically


equal to
1.
2.

3.
4.

2.

1.
2.
3.
4.

half the maximum velocity expressed in


moles/1iter.
velocity of a reaction divideQ by substrate
concentration.
substrate concentration in moles/liter
necessary to achieve half the maximum
velocity of a reaction.
maximum velocity divided by half the
substrate concentration in moles necessary
to achieve maximum velocity.

5.

8.
The major driving force for formation of a
lipid micelle is
1.
2.
3.
4.

4.

What is the product P in the following


reaction?
a-ketoglutarate + alanine~pyruvate

1.
2.
3.
4.
5.

1.
2.
3.
4.
5.

decreased
increased
decreased
increased
decreased

demand for oxygen.


cerebral blood flow.
carbon dioxide tension.
hydrogen ion concentration.
.
oxygenation of carotid bodies.

WIthin physiologic limits, activity of the


respiratory center is decreased directly by
decreased
increased
increased
increased
pressure.

10.

pH.
pH.
hemoglobin concentration.
plasma carbon dioxide partial

relaxation of the masseter muscle.


contraction of the masseter muscle.
contraction of the digastric muscle.
no change in mandibular position.

11.

Chondroitin sulfate occurs primarily in

1.
2.
3.
4.
5.

+ P

Glutamate
Glutamine
Aspartate
Succinate
Pyridoxine phosphate

An action potential in a nerve fiber is related


to

1. entry of sodium ion followed by exit of


potassium ion.
2. flow of electrons acro~sthe membrane
followingchange in membrane
permeability.
3. changed orientation of molecules in the
membrane giving rise to a static potential
difference.
4. a breakdown of metabolic products
resulting in different concentrations of
potassium across the membrane.

Strong stimulation of spindles in the masseter


muscle causes

1.
2.
3.
4.

6.

Lysine
Leucine
Glycine
Tyrosine
Tryptophan

Excluding the effect of drugs, apnea occurring


after hyperventilation of an anesthetized
pati~nt results from.

1.
2.
3.
4.

5.

L-amino acids.
D-monosaccharides.
phosphatidylethanol am ines.
2 fatty acids, 1 glycerol, 1 phosphoric
acid and 1 choline residue.
2 fatty acids, 1 glycerol, 1 phosphoric
acid and 1 cholesterol molecule.

Which of the following amino acids lessens


the need for phenylalanine?
1.
2.
3.
4.
5.

protein-lipid interaction.
hydrophobic interaction between
hydrocarbon tails.
hydrogen bonding between water
molecules of the micellar core.
electrostatic interaction between the
micellar core and polar heads.

9.

3.

Complete hydrolysisof one mole of lecithin


yields

Pantothenic acid is an integral part of


1.
2.
3.
4.
5.

bile.
blood.
liver.
urine.
cartilage.

12

NAD.
cobalamin.
folic acid.
coenzyme A.
pyridoxine phosphate.

.~;..
..<.
'.',~

12.

Spatial summation results from the

18.

1. convergence of several afferent impulses


2.
3.
4.

13.

albumins.
keratins.
histones.
r-globulins.
scleroproteins.

21.

1.
2.
3.
4.
5.

During resting potential


At the firing level
During local depolarization
During negative after-potential
During positive after-potential

actin.
myosin.
troponin.
tropomyosin.

The degenerate nature of the genetic code


implies

1. a common tRNA for at least two amino


acids.

2. that a remarkable degree of inaccuracy


occurs in transcription.

genetic repression.
genetic suppression.
allosteric inhibition.
competitive inhibition.
noncompetitive inhibition.

When are nerve fibers hypoexcitable?

sterol.
glycerol.
lecithin.
sphingosine.
alcohols of high molecular weight.

Calcium ions trigger contraction of muscles by


binding to

1.
2.
3.
4.

22.

DNA.
transfer RNA.
ribosomal RNA.
messenger RNA.
mitochondrial RNA.

Neutral fats contain mixtures of one or more


fattyacids esterified with
1.
2.
3.
4.
s.

ATP inhibits phosphofructokinase even though


ATP is also a substrate for the enzyme. This
phenomenon is an example of

1.
2.
3.
4.
5.

17.

20.

Respiration
Simple reflex
Temperature regulation
Coordinated muscle movement

Addition of synthetic polyuridylic acid to a


cell-free system capable of protein synthesis
results in greatly enhanced incorporation of
phenylalanine into peptide linkages. In this
system, polyuridylic acid is performing a
function normally performed by

1.
2.
3.
4.
5.

glycine.
aspartic acid.
glutamic acid.
acetoacetic acid.
a--ketoglutaric acid.

Proteins rich in basic amino acids, such as


lysine and arginine, which bind strongly to
DNA are

1.
2.
3.
4.
5.
16.

19.

Amide
Anomeric
Epimeric
Hydrogen
Glycosidic

Of the following,the most immediate source


of oxaloaceticacid during metabolism is
1.
2.
3.
4.
5.

15.

1.
2.
3.
4.

What type of bonds link amino acid residues


to form protei ns?

1.
2.
3.
4.
5.

14.

on the same postsynaptic nerve soma.


repetitive firing of discharges (from the
same presynaptic terminal), eventually
producing an action potential.
exhaustion of the stores of transmitter
substance at the synapse.
elaboration of some inhibitory substance,
such as y-aminobutyric acid, at the
synapse.

The spinal cord is the only structure in the


central nervous system necessary for which of
the following body functions?

3. the existence of multiple codons for each


amino acid.

4. the existence of multiple species of


ribosomes for control of messenger
translation.

23.

Vitamin A functions to

1. prevent pellagra.
2. promote absorption of calcium.
3. promote differentiation of epithelial cells.
4. maintain the integrity of connective
tissues.

,L ;

.,

24.

1.
2.
3.
4.

25.

4.

'.r"

memory.
vision.
hearing.
muscular coordination.
both (2) and (3) above.

Which of the following lipids participates in


mitochondrial electron transport?

1.
2.
3.
4.

32.

excreted as urea.
excreted in the form of uric acid.
reused and converted to AT? needed as a
source of energy.
broken down to give NH) and either
malor;ic acid or methylmalonic acid.

Ubiquinone
fJ-carotene
a-tocopherol
Sphingomyelin

4.

5.

Corti's organ is responsible for perception of

1.
2.
3.
4.
5.

33.

light.
sound.
taste.
temperature.
proprioception.

The annulospiral (primary) ending of the


skeletal muscle spindle is stimulated when the

gl uconeogenesis.
the pentose phosphate pathway.
the Krebs' tricarboxyl ic acid cycle.
decarboxylation of aromatic amino acids.
one-carbon transfer from tetrahydrofolate
derivatives.
>...

1.
2.
3.
4.

A submaximal direct stimulus to a skeletal


muscle causes

1.
2.
3.

29.

Ionic bonds
Glycosidic bonds
Phosphodiester bonds
Phosphotriester bonds

Ribose phosphate needed for nucleic acid


synthesis can be derived from

1.
2.
3.
4.
5.

28.

31.

causes loss of

Purine bases taken in the human diet in the


form of DNA or RNA are mostly

1.
2.
3.

27.

Destruction of the cerebellum

1.
2.
3.
4.
5.

Anemia
Polycythemia
Pulmonary AV shunt
Hypovolemia due to hemorrhage.

Which of the following characterize the links


between monomeric units of nucleic acids?

1.
2.
3.
4.

26.

30.

In which of the following conditions is oxygen


partial pressure in arterial blood low?

34.

contraction of no fibers.
contraction of some fibers.
partial contraction of all fibers in the
muscle.
less contraction than with a subminimal
stimulus.
more contraction than with a maxi~1
stimulus.

Functionally, visceral smooth muscle and


cardiac pacemaker cells are similar in that

1.
2.
3.

4.

A deficiency of choline in the diet may cause


abnormalities in metabolism of

35.

1. lipids.
2. proteins.
3. minerals.
4. carbohydrates.
5. nucleoproteins.

muscle is relaxed.
muscle is stretched.
intrafusal fibers are relaxed.
gamma efferent fibers are inhibited.

neither contracts without neural input.


neither has a true "resting" membrane
potential.
both require a precontractile stretch to
initiate contraction.
the "resting" membrane potential of each
is approximately 85 mY.

Failure of the heart to show tetanus when


stimulated is explained by

1.
2.
3.
4.

fatigue.
automaticity.
a long chronaxie.
a long refractory period.

!'-

, ';:

1A

:,~.'t, '
..'!;~t3.

36.

The usual plasma concentrationof glucose is


80-100 mgJdl., yet, normally, glucose does not
appear in the urine. The best explanation for
this fact is

41.

1. Creatine-creatinine
2. ATP-ADP + inorganic phosphate
3. Creatine phosphate + ADP-ATP +

1. glucose is freely filtered, but is removed


by the collecting {juct.

2. glucose is fr-eelyfiltered, but is removed


3.
4.

37.

Niacin
Biotin
Thiamine

43.

Tocopherol
Phylloquinone

creatine
Creatine phosphate-creati!1e
phosphate

+ inorganic

Decreased blood colloidal osmotic pressure


causes fluid to shift from

1.
2.
3.
4.
5.

interstitial space to blood.


blood to interstitial space.
intracellular space to interstitial space.
extracellular space to intracellular space.
None of the above

The

most abundant form of iron in human

blood plasma is found

1.
2.
3.
4.
5.

44.
glucokinase.
adenylate cyclase.
glucose-6-phosphatase.
glycogen phosphorylase.
phosphorylase phosphatase.

as free Fe++.
as free Fe+++.
as ferritin.
associated with transferrin.
associated with ceruloplasmin.

Hydroxylationat the 1 position of


25-hydroxycholecalciferoloccurs primarily in
the
1.
2.
3.
4.

skin.
liver.
kidney.
intestinal mucosa.

Acids found in the citric acid (Krebs1 cycle are


1.
2.
3.
4.
5.

40.

42.

Mobilization of blood sugar from liver


glycogen occurs by way of a series of
enzymatic reactions. The last step in this
sequence is catalyzed by

1.
2.
3.
4.
5.

39.

4.

Avidin influences which of the following


vitamins?

1.
2.
3.
4.
5.

38.

by reabsorption in the proximal


convoluted tubule.
glucose is secreted by the proximal
convoluted tubule and reabsorbed by the
distal convoluted tubule.
the glomerulus prevents glucose from
being filtered into the nephron.

Which of the following reactions is the


immediate source of energy for muscle
contraction?

45.
glutamic acid and succinic acid.
glucuronic acid and aspartic acid.
oxaloacetic acid and aspartic acid.
oxaloacetic acid and pyruvic acid.
oxaloacetic acid and a-ketoglutaric acid.

Albinism is a genetic disease that results in


incomplete metabolism of

1.
2.
3.
4.
5.

alanine.
tyrosine.
cysteine.
histidine.
tryptophan.

Carbonic anhydrase in erythrocytes increases

1. rate of dissociation of H2CO3.


2. rate of formation of oxyhemoglobin.
3. permeability of erythrocytes to HCO3 - .
4. formation of carbamino compounds with
hemoglobin.

46.

Which of the following is present in plasma


but absent in serum?

1.
2.
3.
4.
5.

Albumin
Globulin
Lecithin
Fibrinogen
Prothrombin

47.

53.

Which of the following hormones are


produced by the neurohypophysis and affect
contraction of smooth muscle?
1.
2.
3.
4.

The high ratio of postganglionic to


preganglionic fibers in the sympathetic system
has the physiologic result that

1.
2.

Vasopressin(ADH) and oxytocin


Thyrotropin and intermedin
Adrenotropin and somatotropin
Aldosterone and luteinizing hormone

3.
4.

48.

During extended, strenuous exercise, the


energy requirement for contracting skeletal
muscle is supplied largely from
1.
-

2.
3.
4.

49.

oxidation of fatty acids and acetoacetate.


oxidative-decarboxylation of pyruvic acid
to acetyl coenzyme A.
oxidation of acetyl coenzyme A in the
tricarboxylic acid cycle.
anaerobic transformation of glucose from
glycogen to lactic acid.

54.

Sucrose
Sucrose
Glucose
Glucose

5.

and lipid
and saliva
and saliva
and protein

55.

low flow and low resistance.


equal flow and low resistance.
low flow and equal resistance.
equal flow and equal resistance..
high flow and equal resistance.

56.

-Sounds heard during systole in the antecubital


space are produced by

1.
2.
3.
4.

52.

The PR interval in an electrocardiogram


index of the

1.
2.
3.
4.
5.

2.

3.

closure of AV valves.
closure of the aortic valve.
turbulent blood flow through the artery.
laminar blood flow through the occluded
artery.

4.

57.

is an

length of auricular filling.


period of isometric relaxation.
length of auricular contraction.
conduction time between atria and
ventricles.
period of maximal ejection of ventricular
systole.

16

Hyperventi lation
High fluid intake
Excessive smoking
Severe muscular effort
Ingestion of ammonium

chloride

malonyl CoA is an intermediate in


synthesis.
acyl carrier protein is needed in
catabolism.
no flavoprotein enzymes are required for
catabolism.
propionyl CoA may serve as an
intermediate in synthesis.

Tropic hormone releasing factors produced in


the hypothalamus are transported into the
anterior pituitary through

1.
2.
3.
4.
5.

causes some degree of

The pathway of extramitochondrial synthesisof


, even-numberedfatty acids differs from that or
the catabolism of fatty acids in that
1.

51.

preganglionic

dorsal root ganglia.


sympathetic chain ganglia.
the brain stem and the spinal cord.
thoracic and lumbar segments of the
spinal cord.
none of the above.

Which of the following


temporary alkalosis?

1.
2.
3.
4.
5.

Compared with systemic circulation under


normal conditions, pulmonary circulation is
characterized by low pressure and

1.
2.
3.
4.
5.

Cell bodies of sympathetic


neurons are located in

1.
2.
3.
4.

Dental plaque arises primarily as a result of


enzymatic reactions using which of the
following?

1.
2.
3.
4.

50.

5.

convergence of stimuli occurs.


synaptic transmission is low, leading to a
delay in response.
stimulation of the sympathetic nervous
system leads to widespread effects.
stimulation of the sympathetic nervous
system leads to very localized, discrete
effects.
the presynaptic stimulus must have a
greater amplitude than when the number
of preganglionic and postganglionic fibers
is equal.

nerve impulses.
the ultrashort feedback mechanism.
the peripheral circulatory system.
the microcirculatory
portal system.
long nerve fibers or ducts that also
transport neurophysins.

..
..
58.

Glomerular filtration rate may be determined


by clearance of

63.

1. urea.
2. water.
3. inulin.
4. glucose.
5. para-aminohippuric acid (PAH).

59.

Secretin functions in digestion of proteins by


increasing

1.
2.
3.
4.

Prolonged medication with cortisol produces


atrophy of the adrenal cortex through
1.
2.
3.
4,

64.

flow of bile.
secretion of pepsin.
flow of pancreatic juice.
secretion of carboxypeptidase.

Countercurrent multiplier activity of the kidney


is dependent upon the
1.
2.
3.
4.

60.

The air remaining in alveoli at the end of a


forced maximal expiration is the
1.
2.
3.
4.
5.

61.

tidal volume.
vital capacity.
residual volume.
expiratory reserve volume.
inspiratory reserve volume.

5.

65.

presence of ADH.
reabsorption of water from collecting
ducts.
intensity of vasoconstriction of efferent
arterioles.
active reabsorption of water in the
ascending limb of the loop of Henle.
active reabsorption of sodium ions in the
ascending limb of the loop of Henle.

In the absence of compensatory changes, a


drop in blood pressure results from

1.
2.
3.
4.
5.

Normal hemoglobin concentration is about 15


gm.ldl. blood and normal arterial oxygen
content is about 20 ml. aidl. blood. An
anemic individual breathing room air with a
hemoglobin concentration of 10 gmJdl. blood
is expected to have

inhibition of ACTH production.


inhibition of aldosterone secretion.
direct action on the synthesis of C21
corticoids.
none of the above.

vasoconstriction.
increased hematocrit.
increased stroke volume.
increased cardiac output.
decreased venous return.

1. normal arterial oxygen tension and normal


2.
3.

4.

62.

arterial oxygen content.


reduced arterial oxygen tension and
normal arterial oxygen content.
normal arterial oxygen tension and
reduced arterial oxygen content.
reduced arterial oxygen tension and
reduced arterial oxygen content.

66.

The chemical mediator at the neuroeffector


junction in sweat glands is

1.
2.
3.
4.
5.

atropine.
serotonin~
epinephrine.
acetylchol ine.
norepi neph rine.

The hemoglobin dissociation curve is shifted


to the right by

1. a decrease in temperature.
2. an increase in arterial Pcal.
3. an increase in arterial hydrogen ion
4.
5.

concentration.
both (1)and (3) above.
both (2) and (3) above.

67.

Hypothyroidism in an adult results in

1.
2.
3.
4.

myxedema.
thyrotoxicosis.
osteitis fibrosa cystica.
increased basal metabolic rate.

--

"

,
68.

72.

Carbon dioxide or bicarbonate is required in


the biosynthesisof fatty acids because

1.
2.
3.
4.
5.

1. the bicarbonate/carbonic acid buffer


2.

3.

4.
5.

69.

70.

73.

74.

Insulin
Cortisone
Vasopressin
Aldosterone
Parathyroid hormone

76.

Which of the following occurs in a normal


menstrual cycle?
1.
2.

3.
4.

Ovulation generally occurs 10 days from


the beginning of the cycle.
The ovum generally disintegrates or
becomes-nonviable if it is not fertilized
within four days.
The blood concentration of estradiol
increases as the corpus luteum develops.
The blood level of follicle-stimulating
hormone increases and the level of
luteinizing hormone decreases at the time
of ovulation.

-Mucin
Ammonia
Carbonate
Bicarbonate
Amino acids

The mechanism of systemic fluoride action in


reducing dental decay is most likely the result
of
1.
2.
3.
4.
5.

18

estrone.
progesterone.
luteinizing hormone.
follicle-stimulating hormone.
chorionic gonadotropic hormone.

Neutralization of acids by saliva results mainly


from which of the following salivary contents?
1-.
2.
3.
4.
5.

To mobil.ize lipid deposits


To enhance cell permeability to glucose
To decrease cell permeability to glucose
To conserve glucose by breaking down
amino acids

niacin.
glucose.
glycerol.
tyrosine.
methionine.

Development of ovarian follicles to the point


of ovulation is stimulated primarily by
1.
2.
3.
4.
5.

75.

urea.
ammonia.
glutamine.
creatinine.
aspartic acid.

Epinephrine is most closely related in structure


to
1.
2.
3.
4.
5.

Which of the following is a principal action of


insulin?
1.
2.
3.
4.

71.

system is very efficient at the pH optimum


of this sensitive enzyme system.
bicarbonate is a positive effector for this
system and favorably alters the
conformation of its enzymes.
carbon dioxide is incorporated into acetyl
coenzyme A forming malonyl coenzyme
A, an intermediate in the synthetic
process.
carbon dioxide is incorporated into
carbamyl phosphate, a reactive
intermediate in the synthetic process.
carbon dioxide provides an anaerobic
environment that prevents oxidation of the
sulfhydryl groups in the reactive sites of
the enzyme system.

Which of the following hormones facilitates


reabsorption of sodium in renal tubules?
1.
2.
3.
4.
5.

The nitrogenous product that is excreted in


high amounts after prolonged acidosis is

an increase in hardness of the tooth.


an inhibition of proteolytic enzymes.
deposition of CaF2 in areas of enamel
susceptible to bacterial penetration.
an increase in carbohydrate metabolism in
the oral cavity as a result of enzyme
inhibition.
a reduction in rate of solubilization of
enamel as fluoride content of the tissue
Increases.

..
77.

1.
2.
3.
4.

s.
78.

83.

The most abundant nonphospholipid


component of the cell membrane is
cholesterol.
deoxycholate.
prostaglandin.
macroglobulin.
triacylglyceride.

1.
2.
3.
4.
5.

4.

s.

84.

(a)
(b)
(c)
(d)
(e)
1.
2.
3.
4.
".
6.

Entropy
Enthalpy
Free energy
Potentialenergy

85.
Which of the following solutions has an
osmotic pressure different from all the others?

1.
2.
3.
4.
5.

81.

Which of the following proteins contain trace


elements essential for their biologic action?

intestines.
reabsorbed into the portal circulation and
reused.
removed from the circulation by the
kidneys and excreted in the urine.

What thermodynamic parameter is a measure


of randomness or disorder in a system?

1.
2.
3.
4.
80.

glycine.
glucose.
fructose.
glyceraldehyde.
None of the above

After functioning in the small intestines, the


largest portion of bile salts are

1. excreted in the feces.


2. reabsorbed into the central lacteal.
3. destroyed by bacteria in the large

79.

The arrangement of sugars into 0- and


L-configurations is based upon their
resemblance to 0- and l-

(a),
(a),
(a),
(b),
(b),
All

1.
2.
3.
4.

..
The function of rhythmic segmentation is
chiefly that of

(b) and (c)


(c) and (d)
(d) and (e)
(c) and (e)
(d) and (e)
of the above

Hydrolysis of sucrose by the enzyme sucrase


yields
(a)
(b)
(c)
(d)

1 M glucose
1 M sodium chloride
1 M potassium nitrate
1 N lithium iodide
1-N hydrochloric acid

Trypsin
Myoglobin
Thyroglobulin
Salivary amyla;;e
Carbonic anhydrase

5.

(a)
(b)
(b)
(b)
(c)

- galactose.
glucose.
fructose.
maltose.
and
only
and
and
and

(b)
(c)
(d)
(d)

1. macerating food.
2. mixing food with digestive secretions.
3. increasing the surface available for
4.

82.

absorption.
transporting material down the
gastrointestinal tract.

86.

Normal production of sound enamel and


dentin requires adequate amounts of which of
the following vitamins?
(a)
(b)
(c)
(d)
(e)

Which of the following describes the


movement of glycine molecules in an electric
field, at a pH of 6.06 (the isoelectric pH for
glycine)?
1.
2.
3.
4.

s.

Glycine molecules will not move.


All molecules will move to the anode.
All molecules will move to the cathode.
Half the molecules will move to the
anode and half to the cathode.
None of the above

1.
2.
3.
4.
s.
6.

(a),
(a),
(a),
(a),
(b),
(c),

A
B12
C
0
E
(b) and
(b) and
(b) and
(c) and
(d) and
(d) and

(c)
(d)
(e)
(d)
(e)
(e)

...
13.

17.

Right subclavian and right common carotid


arteries arise from the
1.
2.
3.
4.
5.

thoracic aorta.
ascending aorta.
arch of the aorta.
pulmonary artery.
brachiocephalic artery.

1.
2.
3.
4.

18.
14.

The number of roots that are formed is


determined by the

3.

mesocolon.
lesser omentum.
greater omentum.
gastrosplenic ligament.

4.

19.
15.

oral mucosa.
reduced enamel epithelium.
epithelial rests of Malassez.
the epithelial root sheath (Hertwig).

1. thickness of the cervical loop.


2. number of root sheaths developed by the

The common bile duct, the hepatic artery and


the portal vein are found grouped in the
1.
2.
3.
4.

When a tooth first erupts into the oral cavity,


the attachment epithelium is derived from

In the sketch below, the arrow labeled 0


points to the

enamel organ.
number of discontinuities developed in
the root sheath.
number of medial ingrowths of the
cervical loop.

In a routine demineralized histologic section


of an adult tooth, the area representing enamel
is generally optically clear because

1. enamel is very poor in organic matrix.


2. enamel is very brittle and breaks off
3.
4.

during sectioning.
ordinary stains do not stain the organic
matrix of enamel.
the mounting medium has the same
refractive index as enamel.

20.
A

The posterior cervical triangle is bounded


partly by
1.
2.
3.
4.

1.
2.
3.
4.
5.

21.

dental papi Ila.


dental sac (follicle).
inner enamel epithelium.
outer enamel epithelium.
bone of the crypt surrounding the tooth
germ.

The primary histologic characteristic of the


pancreas is
1.
2.
3.
4.

16.

The greater peritoneal sac communicates with


the lesser peritoneal sac by means of the

1.
2.
3.
4.
5.

22.

aortic hiatus.
inguinal canal.
epiploic foramen.
lesser pelvic aperture.
superior pelvic aperture.

follicles filled with secretion.


groups of special cells scattered among
glandular alveoli.
small groups of cells scattered between
follicles.
cells arranged in layers among which are
special cells.

The middle constrictor muscle of the pharynx


originates from the
1.
2.
3.
4.

trapezius and subclavius muscles.


splenius capitis and trapezius muscles.
trapezius and sternocleidomastoid
muscles.
the clavicle and the longus colli muscle.

hyoid bone.
thyroid cartilage.
pterygoid hamulus.
pterygomandibular raphe.

..
87.

(a)
(b)
(c)
(d)

(e)

1.
2.
3.
4.
5.

88.

1.
2.
3.
4.
5.

(d)
(e)

92.

93.

sectioning vagus nerves.


stimulating vagus nerves.
sectioning sympathetic nerves.
stimulating sympathetic nerves.

94.

1.
2.
3.
4.
5.

Which of the following is NOT a primary


(major) electrolyte of saliva?

Excessive use of fats by the body as a source


of energy during starvation or disturbances in
carbohydrate metabolism may lead to any of
the following conditions EXCEPT
1.
2.
3.
4.

Which Gf the following occur when active


transport is involved in the movement of a
solute across a biologic membrane?

(d)

ribose.
adenine.
deoxyribose.
acetic acid.
phosphoric acid.

1. Sodium
2. Calcium
3. Fluoride
4. Potassium
5. Bicarbonate

(a) only
(a) or (b)
(a) or (d)
(b) or (d)
(c) only

(a)
(b)
(c)

(a), (b) and (c)


(a), (b) and (d)
(a), (c) and (d)
(b), (c) and (d)
All of the above

Hydrolysis of nucleic acids is likely to yield all


of the following EXCEPT
1.
2.
3.
4.
5.

highly charged.
nonpolar in nature.
small in size (below 7.5 A in
diameter).

In an experimental animal, the heart rate can


be accelerated by

1.
2.
3.
4.
5.

Inhibitory postsynaptic potentials


Excitatory postsynaptic potentials
Generator potentials in pacinian
corpuscles
Endplate potentials at the
neuromuscular junction

(d)
only

(a) and (b)


(a) and (c)
(b) and (c)
All of the above

(a)
(b)
(c)
(d)

90.

(d)

In general, molecules that can penetrate a


biologic membrane with relative ease are

1.
2.
3.
4.

Which of the following are examples of local


(electrotonic or subthreshold) potentials?
(a)
(b)
(c)

cysteine.
hydroxyproline and hydroxylysine.
no sulfur-containing amino acids.
glycine, as nearly one-third the total
number of amino acid residues.
a high content of aromatic amino
acids.

(a), (b) and


(a), (c) and
(b) only
(b), (c) and
(b) and (d)

(a)
(b)
(c)

89.

91.

The amino acid composition of mammalian


collagen (Type I) is characterized by the
presence of

Expenditure of metabolic energy


A gain in free energy by the system
A unidirectional movement of the
solute
Movement of the solute against a
concentration gradient

95.

ketosis.
acidosis.
ketonuria.
alkalosis.

Which of the following enzymes does NOT


function in association with a membrane?

1. Permease
2. Na1'/K+ ATPase
3. Adenylate cyclase
4. Phosphofructokinase
5. Coenzyme Q reductase

(a) and (b)


(a), (c) and (d)
(b), (c) and (d)
(b) and (d) only
All of the above

20

....

96.

A substance that is actively secreted by the


kidney is helpful in measurement of all of the
following EXCEPT
1.
2.
3.
4.
5.

97.

99.

1.
2.
3.
4.
5.

renal blood flow.


renal plasma flow.
tubular maximum.
filtration fraction.
glomerular filtration rate.

stomach.
pancreas.
duodenum.
gallbladder.
urinary bladder.

Coupled respiration requires all of the


following EXCEPT
1.
2.
3.
4.
5.

ADP.
oxygen.
carbon dioxide.
an electron donor.
inorganic phosphate.

100.
98.

Gastrointestinal hormones act on each of the


following organs EXCEPTthe

Each of the following is a function of the liver


EXCEPT

The hypothalamus-pituitary complex controls


hormonal secretions of all of the following
EXCEPT

1. bile formation.
2. urea elimination.
3. steroid conjugation.
4. carbohydrate storage.
5. prothrombin synthesis.

1.
2.
3.
4.
5.

corpora lutea.
thyroid glands.
ovarian follicles.
parathyroid glands.
interstitial cells of the testes.

.
-

..

NATIONAL

BOARD DENTAL EXAMINATION PART I


ANSWER

KEY

BIOCHEMISTRY-PHYSIOLOGY- DEC. 1987

NO.

ANS.

1.

3
2
3
2
2

NO.

ANS.

NO.

ANS.

NO.

ANS.

26.
28.
29
30.

2
2
2
1
4

51.
52.
53.
54.
55.

3
4
3
4
1

76.
77.
78.
79.
80.

5
1
4
1
1

9.
10.

5
4
4
1
1

31.
32.
33.
34.
35.

1
2
2
2
4

56.
57.
58.
59.
60.

1
4
3
3
3

81.
82.
83.
84.
85.

2
1
4
4
3

11.
12.
13.
14.
15.

4
1
1
2
3

36.
37.
38.
39.
40.

2
2
3
5
2

61.
62.
63.
64.
65.

3
5
1
5
5

86.
87.
88.
89.
90.

4
4
3
3
5

16.
17.
18.
19.
20.

3
5
2
4
2

41.
42.
43.
44.
45.

2
2
4
3
1

66.

4
1
3
4
2

91.
92.
93.
94.
95.

5
4
3
4
4

21.
22.
23.
24.
25.

3
3
3
3
3

46.
47.
48.
49.
50.

4
1
4
2
2

2
2
4
4
4

96.
97.
98.
99.
100.

5
3
2
5
4

2.
3.

4.
5.
6.
7.

8.

27.

67.

68.
69.
70.
71.
72.
73.
74.
75.

PART I
MON DAY-P.M.
DECEMBER 1988

1.

Which of the following factors has the


greatest impact on prognosis of a skin
melanoma?

1.
2.
3.
4.
5.

2.

MICROBIOLOGY-PATHOLOGY (13)

Site of origin
Depth of invasion
Degree of pigmentation
Existence of ulceration
Size of surface area involved

The most common cause of a massive


hematemesis in alcoholics is

1.
2.
3.
4.
5.

7.

1.
2.
3.
4.
5.

8.

varices.

Mal rory-Weiss - syndrome.


acute hemorrhagic pancreatitis.

9.
3.

The chemotactic accumulation of


inflammatory cells that occurs at the sites
where immune complexes are deposited is
most probably the result of

1.
2.
3.
4.
5.

10.

4.

Removal of the capsule from an encapsulated


bacterium js likely to. result in

1.
2.
3.
4.
5.

5.

The major factor in the production of


inflammatory edema is

1.
2.
3.
4.

6.

loss of viability.
decreased generation time.
failure of the cell to gram stain.
increased susceptibility to mutation.
increased susceptibility to phagocytosis.

12.
Left-sided heart failure or shock may be
associated with

1.
2.
3.
4.

pneumocon iosis.
bronch iectasis.
pulmonary
pulmonary

edema.
emboli.

form spores.
withstand acid pH.
produce protoplasts.
undergo transformation.
grow at elevated temperatures.

The renal lesion most commonly


with benign hypertension is

1.
2.
3.
4.

venous obstruction.
arterial dilatation.
generalized kidney damage.
increased capillary permeability.

no living microorganisms.
a fusospirochetal complex.
an abundance of microorganisms.
only by-products of bacterial
metabolism.

Generally, resistance of a bacterium to killing


by heat is influenced by time and by
temperature employed in growth of the
culture. Another factor is the ability of the
organism to

1.
2.
3.
4.
5.

11.

degrading their flagella.


inactivating their ribosomes.
preventing the transpeptidation reaction.
breaking the bond between N-acetyl
glucosamine and N-acetyl muramic acid.

Although the exact mechanism of formation


of calculus is not understood, it is known
that the organic matrix of calculus in humans
includes

1.
2.
3.
4.

C5a.
IgA.
IgD.
rgE.
sensitized-lymphocytes.

Smallpox
Psoriasis
Cat-scratchfever
Recurrentvaricella
Infectious mononucleosis

Lysozyme can attack bacterial cells by

1.
2.
3.
4.

peptic ulcer.
acute gastritis.
esophageal

Which of the following diseases is


characterized by painful vesicles that occur
on the skin or a mucosal surface along the
distribution of a sensory nerve?

associated

renal atresia.
acute pyelonephritis.
chronic pyelonephritis.
arteri

0 Ioneph

roscleros

is.

The most likely diagnosis for a patient with


dysphonia, dysphagia, weight loss and a
history of heavy cigarette smoking is

1.
2.
3.
4.
5.

laryngitis.
tonsillitis.
laryngeal polyps.
carcinoma of the lungs.
carcinoma of the larynx.

,
13.

Two important factors for initiation of caries


by oral streptococci are

18.

1. production of protease and production


2.
3.

4.
5.

of acid.
production of dextranase and production
of soluble dextran.
production of collagenase and
production of hyaluronidase.
fermentation of mannitol and sorbitol
and production of protease.
synthesis of insoluble dextran and
production of glucosyltransferase.

A patient with dentinogenesis imperfecta has


a history of multiple fractures. Examination
reveals blue sclera. The most probable
diagnosis is
1.
2.
3.
4.

19.

achondroplasia.
Marfan's syndrome.
osteogenesis imperfecta.
Albers-Schonberg disease.

The primary function of SlgA is to

1. promote phagocytosis by monocytes.


2. prevent ingress of antigen through

14.

In order for a virus to infect a host cell, it


must first absorb to the cell surface. This
involves

1. phagocytic activity by the cell.


2. energy output on the part of the cell.
3. insertion of virally specified
4.
5.

15.

16.

21.

23.
17.

A benign neoplasm of the myometriumof


.

Calcium
Chloride
Potassium
Phosphorus
Acid phosphatase

The principal antibacterial action of the


tetracyclines is inhibition of
1.
2.
3.
4.
5.

Paget's disease of bone


Primary hyperparathyroidism
Secondary hyperparathyroidism
Breast carcinoma with bone metastasis
Prostatic carcinoma with bone metastasis

pleomorphism.
an enzyme that attacks penicillin.
production of a penicillin analogue.
a lack of mycolic acid in the cell wall.
the presence of a tough
lipopolysaccharide capsule.

Which of the following serum values is likely


to be elevated in a patient with
hyperparathyroidism?

1.
2.
3.
4.
5.

22.

mucosa.
activate complement in secretory fluids.
activate KINK cells in the gingival
crevice.

Resistance of Staphylococcus aureus to


penicillin is caused by
1.
2.
3.
4.
5.

The patient is a carrier.


The patient has acute disease.
The patient is in the convalescent stage.
The patient has recovered completely.
None of the above

An increased serum acid phosphatase level is


clinically significant and aids in the diagnosis
of which of the following conditions?

1.
2.
3.
4.
5.

20.

glycoproteins into the host cell


membrane.
a specific interaction between a viral
surface component and a specific
receptor on the cell membrane.
all of the above.

Laboratory results of a patient's serum show


presence of HBs and anti-HBs. Which of the
following can be concluded?

1.
2.
3.
A.
5.

3.
4.

DNA synthesis.
protein synthesis.
histone formation.
cell wall synthesis.
cell membrane function.

The most acceptable conditions for complete


sterilization by autoclaving are

the uterus is a

1.

1. myeloma.
2. fibroma.
3. leiomyoma.
4. myoblastoma.
5. rhabdomyoma.

2.
3.
4.

5 minutes at 275F. (135(.) and 15 Ibs.


of pressure.
15 minutes at 275F. (135(.) and 10 Ibs.
of pressure.
20 minutes at 250F. (121(.) and 1O.lbs.
of pressure.
20 minutes at 250F. (121(.) and 15 Ibs.
of pressure.

.
24.

The class of antibodies first detected in


serum after primary immunization is usually
1.
2.
3.
4.
5.

25.

26.

dysplasia.
anaplasia.
neoplasia.
metaplasia.
fibroplasia.

A disease caused by the interaction of


antigen and IgE on the surface of mast cells
with release of histamine is

1.
2.
3.
". 4.
5.

4.

serum sickness.
bronchial asthma.
pulmonary emphysema.
lupus erythematosus.
rheumatoid arthritis.

autoimmune.
bacterial infection.
hereditary enzyme deficiency.
premalignant diffuse hyperplasia.
secondary to hyperpituitarism.

Which of the following identifies the cell that


produces interleukin-1 and the cell affected
by interleukin-l, respectively?

1.
2.
3.
4.

33.

34.

fungi.
viruses.
parasites.
pyogenic bacteria.
tubercle bacilli.

Which of the following is diagnosed by


karyotyping?
1.
2.
3.
4.
5.

35~

T cell-B cell
Macrophage-T cell
Macrophage-monocyte
Dendritic cell-B lymphocyte

Patients with Bruton's agammaglobulinemia


principally suffer from infections caused by
1.
2.
3.
4.
5.

young people.
recent recipients of gamma globulin.
recent recipients of whole blood
transfusions.
persons with histories of recurrent
respiratory infections.

The etiologic basis of Hashimoto's thyroiditis


is

1.
2.
3.
4.
5.

32.

Osteoporosis
Multiple myeloma
Fibrous dysplasia
Osteogenesis imperfecta
Paget's disease of bone

A very high percentage of cases of hepatitis


A are found in
1.
2.
3.

29.

plasmin.
histamine.
complement.
bradykinin.
norepi neph rine.

Which of the following diseases is.


characterized by an extremely high level of
alkaline phosphatase, normal levels of
calcium and phosphorus, enlargement of the
skull, and an increased incidence of
osteogenic sarcoma?
1.
2.
3.
4.
5.

28.

31.

atrophy.
dysplasia.
metaplasia.
hypertrophy.
hyperplasia.

Production of bone in scar tissue occurs by


1.
2.
3.
4.
5.

27.

Long-standing gradual ischemia of an organ


or a tissue most likely results in
1.
2.
3.
4.
5.

IgA.
IgO.
IgE.
IgG.
IgM.

The principal chemical mediator located in


the granules of mast cells is
1.
2.
3.
4.
5.

30.

Phenylketonuria
Neurofibromatosis
Tay-Sachs disease
Turner's syndrome
Sickle cell anemia

Neoplasms of which of the following organs


have been shown to be hormonally
dependent?

1.
2.
3.
4.

Liver
Parotid
Prostate
Pancreas

..

36.

41.

Chronic passive congestion of the lung is


characterized by

1.
2.
3.
4.
S.

hyaline membranes.
chronic bronchitis.
giant cell arteritis.
interstitial infiltration of PMN's.
edema of alveolar walls and "heart
failure" cells.

The likelihood that oral bacteria play an


important role in gingival inflammation is
evidenced by
1.
2.

3.
4.

s.

37.

Use of vaccines for preventing clinical


symptoms after introduction of the virus is
most likely to be effective against

1.
2.
3.
4.

38.

42.

rabies.
influenza.
poliomyelitis.
herpes zoster.

4.

43.

Empyema
Pneumonia
Emphysema
Atelectasis
Bronchiectasis

A virus causing recurrent herpetic lesions is


never completely eliminated but assumes a
state of
1.
2.
3.
4.
S.

An oral lesion that may appear as an ulcer, a


nodule or a vegetative process and is often
mistaken for squamous cell carcinoma is a
manifestation of

The nuclear material of a bacterial cell is


typically characterized as a
1. single, double-stranded DNA molecule
2.

latency.
symbiosis.
mutualism.
commensalism.
none of the above.

3.
4.

45.
40.

culturing bacterial spores.


culturing the water reservoir.
thermocouples (temperature recording
devices).
indicators that change color at elevated
temperatures.

1. candidiasis.
2. trichinosis;
3. sporotrichosis.
4. histoplasmosis.

44.
39.

The effectiveness of autoclaving is best


determined by
1.
2.
3.

Which of the following is characterized by a


collapse of alveoli?

1.
2.
3.
4.
S.

an increase in salivary hyaluronidase.


an increased number of bacteria in
.saliva.
an increase in bacterial antibodies in
saliva.
appearance of new pathogenic strains in
the affected area.
a reduction in inflammation by removal
of bacteria.

enclosed within a nuclear membrane.


single, double-stranded DNA molecule
not confined within a nuclear
membrane.
rigid, DNA-protein complex enclosed
within a nuclear membrane.
rigid, DNA-lipopolysaccharide complex
not confined within a nuclear
membrane.

Patients receiving chemotherapy for leukemia


are particularly prone to develop oral

Which of the following is the most common


initial sign or symptom in patients with
malignant lymphoma?

1.
2.
3.
4.
S.

1. Pallor
2. Weight loss
3. Lymphadenopathy
4. Chronic infection
S. Unexplained fever

candidiasis.
actinomycosis.
blastomycosis.
histoplasmosis.
coccidioidomycosis.

>;

26

.
46.

1.
2.
3.
4.

47.

1.
2.
3.
4.
5.

49.

54.

2.
3.

4.
5.

55.

4.
j,

56.

51.

Cervicofacial actinomycosis is usually due to


1.

2.

3.
4.

poor aseptic technique during oral


surgery.
contamination of a trauma or surgical
site with spores.
contamination of a trauma or surgical
site with endogenous organisms.
spread of the organisms from a dermal
or thoracic infection.

rupture of a congenital aneurysm of the


circle of Willis.
brain abscess from necrosis of the
myocardium.
detachment of a bacterial embolus from
the pulmonic valve.
detachment of a mural thrombus from
the right ventricle. .
detachment of a mural thrombus from
the left ventricle.

Features of the herpes simplex virus type 1


virion surface include
1.
2.
3.

Brain
Heart
liver
Kidney
Adrenals

partially dominant.
autosomal dominant.
autosomal recessive.
sex-linked dominant.
sex-linked recessive.

Ten days after hospitalization for a large,


incapacitating myocardial infarct, a
50-year-old man suddenly develops paralysis
of the right side of his body. The best
explanation for his brain damage is
1.

Which of the following organs is the most


uncommon site for infarcts?
1.
2.
3.
4.
5.

Classic hemophilia is due to a deficiency of


normal Factor VIII which in turn is due to a
genetic deficiency that is
1.
2.
3.
4.
5.

bleeding.
perforation.
surgical complication.
malignant transformation.
obstruction from edema or scarring.

ampicillin.
lincomycin.
tetracycl ine.
streptomyci n.
erythromycin.

Decreased albumin
Elevated serum lipase
Decreased serum amylase
Elevated alkaline phosphatase

Generally, the antibiotic of choice for


prophylactic therapy covering dental
procedures in a patient with a heart valve
abnormality who is allergic to penicillin is

1.
2.
3.
4.
5.
50.

53.

Escherichia coli
Lactobacillus casei
Leptotrichia bucca/is
Staphylococcus aureus
Mycobacterium luberculosis

A complication of peptic ulcer disease that


accounts for the majority of deaths is

Which of the following laboratory results is


diagnostic of acute pancreatitis?
1.
2.
3.
4.

produce potent exotoxins.


are destructive for endothelial cells.
are particularly destructive to cutaneous
tissues.
are primarily neurotropic and cause
extensive damage in certain CNS
centers.

Which of the following bacteria has the


highest lipid content in the cell wall?

1.
2.
3.
4.
5.

48.

52.

Most rickettsial diseases produce severe


illness in humans because rickettsiae

no envelope.
an envelope synthesized de novo.
an envelope acquired by budding
through the cytoplasmic membrane.
an envelope acquired by budding
through the nuclear membrane.
none of the above.

Hemorrhagic infarction and necrosis are


characteristic of which of the following
fungal diseases?
1.
2.
3.
4.
5.
6.

Candidiasis
Actinomycosis
Cryptococcosis
Histoplasmosis
Coccidioidomycosis
Mucormycosis (Phycomycosis)

..

57.

The chemical nature of bacterial endotoxins


is a
1.
2.
3.
4.
5.

62.

I
I

Bacterial endotoxin

effect, in part, by
1.
2.
3.
4.

lipid.
protein.
complex carbohydrate.
lipopolysaccharide complex.
low molecular weight polypeptide.

5.

58.

63.

Detergents kill bacteria by interfering with


the function of the cell
1.
2.
3.
4.
5.

..

4.

ch romosome.

5.

59.

64.

Phage conversion is responsible for


1.
2.
3.
4.
5.

transduction of bacteria.
antigenic phase variation.
production of {J-galactosidase..
ability of Rhizobium species to fix
nitrogen.
production of erythrogenic toxin by
Streptococcus pyogenes.

An Rh-negative mother del ivered a normal


first child. Her second child developed
symptoms of erythroblastosis fetalis. Which of
the following can be concluded?
1.
2.
3.

4.
5.

61.

The father is Rh-negative.


The first baby is Rh-negative.
The first baby is a girl; the second is a
boy.
The mother has very high levels of
serum complement and anti-Rh IgE.
None of the above

Which of the following is most commonly


associated with development of
gastrointestinal cancer?

Which of the following is the most common


skin cancer in man?
1.
2.
3.
4.
5.

66.

Malignant melanoma
Basal cell carcinoma.
Squamous cell carcinoma
Sebaceous adenocarcinoma
Transitional cell carcinoma

Multiple drug resistance is related most


closely to

1. viruses.
2.
3.
4.

67.

A summer illness that produces vesicular


lesions of the uvula, anterior pillars and the
posterior pharynx is

1.
2.
3.
4.

marked cellularity of the tumor.


many mitotic figures in the tumor.
lymphocytic infiltration at the edge of
the neoplasm.
.a large number of blood vessels in the
neoplastic mass.
large areas of necrosis in the center of
the neoplastic mass.

1. .Amebiasis
2. VilIous adenoma
3. Meckel's diverticulum
4. Duodenal peptic ulcer
5. Peutz-Jeghers syndrome

65.
60.

stimulating production of IgA.


activating the complement cascade.
inactivating nonspecific Iymphokines.
releasing transfer factor from
B lymphocytes.
releasing vasoactive lipopolysaccharides
from mast cells.

The host response to a malignancy is best


reflected by
1.
2.
3.

wall.
capsule.
membrane.

ribosome.

exerts its pathologic

Which of the following organisms is most


frequently isolated from the oral cavity?
1.
2.
3A
4:
5.

influenza.
herpesvirus.
ECHO viruSe
coxsackievi ruSe

28

plasm ids.
transformation.
cell chromosomes.

Streptococcus sanguis
Streptococcus pyogenes
Streptococcus pneumoniae
Neisseria meningitidis
Staphylococcus aureus

I
.

.
68.

Which of the following characteristics of


enterobacteria is most important in hospital
infections?

73.

Which of the following are rarely, if ever,


acquired from another individual?
(a)
(b)
(c)
(d)

1. High degree of invasiveness


2. Ability to become resistant to antibiotics
3. Production of enzymes destructive to
4.

69.

1.
2.
3.
4.
5.
6.
7.

(a) and (b) only


(a), (b) and (c)
(a), (b) and (d)
(a) and (c) only
(b) and (c) only
(b) and (d) only
All of the above

The most common clinical consequence of


benign prostatic hyperplasia is
1.
2.
3.
4.
5.

70.

tissue
Ability to survive drying due to spore
formation

Blastomycosis
Sporotrichosis
Histoplasmosis
Neonatal candidiasis

74.
pain.
jaundice.
urinary tract obstruction.
red blood cell casts in urine.
progression to prostatic carcinoma.

growing gram-negative bacteria.


growing gram-positive bacteria.
nongrowing gram-negative bacteria.
nongrowing gram-positive bacteria.
sporulating gram-negative bacteria.

fibroblasts.
nerve fibers.
endothelial cells.
epithelioid cells.
giant cells.

- (a)
(b)
(c)
(d)
(e)

1.
2.
3.
4.
5.
6.

In organisms sensitive to penicillin, this


antibiotic shows the greatest bactericidal
activity against
1.
2.
3.
4.
5.

Granulation tissue typically contains

75.

(a)
(a)
(b)
(c)
(c)
(d)

and
and
and
and
and
and

Significant functions of polymorphonuclear


leukocytes in inflammation are

(a)
(b)
(c)

replication of new cells.


phagocytosis of bacteria.
elaboration of proteolytic
enzymes.
elaboration of antibodies.

(d)

71.

1.
2.
3.
4.
5.

Myasthenia gravis is caused by dysfunction of


the
1.
2.
3.
4.
5.

motor nerves.
smooth muscle.
sensory nerves.
skeletal muscle.
myoneural junction.

76.

(b)
(c)
(d)
(d)
(e)
(e)

(a)
(a)
(b)
(b)
(c)

and
and
and
and
and

(b)
(c)
(c)
(d)
(d)

IgEis involved in which of the following


conditions?

72.

(a)
(b)
(c)
(d)

Which antibody type is resistant to hydrolysis


by microbial proteolytic enzymes?
1.
2.
3.
4.

IgA
SlgA
IgE
IgG

1.
2.
3.
4.
5.

Hives
Hay fever
Pemphigus vulgaris
Lupus erythematosus

(a) and
(a) and
(a) and
(b) and
(c) and

(b)
(c)
(d)
(c)
(d)

..
23.

Pharyngotympanic (eustachian) tubes connect


the nasopharynx with the

1.
2.
3.
4.

24.

29.

1.
2.
3.
4.

outer ear.
middle ear.
.
semicircular canals.
vestibule of the inner ear.

30.
Bacteria are frequently ingested by

1.
2.
3.
4.
5.

Considering number and location, the


parathyroid glands are usually.

3.

4.

the thyroid gland.


multiple and embedded in the dorsum of
the thyroid gland.
multiple and just superior to the thyroid

gland.
5.

26.

27.

Upper head of the lateral pterygoid


Lower head of the lateral pterygoid
Deep head of the medial pterygoid
Superficial head of the medial pterygoid

The arterial supply of the submandibular gland


is from which branch of the externalcarotid
artery?
1.
2.
3.
4.

33.

Ascending pharyngeal
Superior thyroid
MaxiIlary
Facial

Which of the following blood elements is a


fragment of megakaryocytic cytoplasm?

1.
2.
3.
4.
5.

Platelet
Normoblast
Erythrocyte
Promyelocyte
Proerythroblast

mandibular first premolar.


maxillary first premolar.
maxillary first molar.
maxillary second molar.
maxillary third molar.

Which of the lymphoid tissues in the body are


considered subepithelial and nonencapsulated?

1. Thymus gland and lymph nodes


2. Lymph nodes and aggregated nodules
3. Pharyngeal tonsils and thymus gland
4. Peyer's patches and pharyngeal tonsils

35.
28.

aorta.
pulmonary vein.
pulmonary artery.
ductus arteriosus.
thebesian veins (venae cordis minimae).

The parotid duct pierces the buccinator muscle


opposite the

1.
2.
3.
4.
5.

34.

T-Iymphocytes.
B-Iymphocytes.
macrophages.
plasma cells.

In the fetus, blood enters>the common carotid


arteries by means of the

1.
2.
3.
4.
5.

not related to the thyroid gland.

A portion of which of the following muscles


has fibers inserting into the articular disk of
the temporomandibular joint?
1.
2.
3.
4.

32.

gastric artery.
splenic artery.
celiac artery (trunk).
superior mesenteric artery.

Most of the antibodies in the body that act


against bacterial antigens are produced by

1.
2.
3.
4.

1. single and dorsal to the thyroid gland.


2. single and embedded in the dorsum of

carotid sheath.
pterygomandibular raphe.
medial pterygoid muscle and its fascia.
stylopharyngeus muscle and its fascia.

The common hepatic artery is a branch of the


1.
2.
3.
4.

mast cells.
fibrocytes.
small lymphocytes.
basophilic leukocytes.
neutrophilic leukocytes.

31.
25.

The lateral boundary of the retropharyngeal


space at the level of the oropharynx is the

Most of the lymph is returned to the blood at


which of the following sites?
1.
2.
3.

4.

Right brachiocephalic vein


Left external jugular vein
junction of left internal jugular and
subclavian veins
junction of right internal jugular and
subclavian veins

77.

80.

Invasive staphylococcal infections of orofacial


structures are likely to result in

(a)
(b)
(c)
(d)
(e)

brain abscess.
osteomyelitis.
cavernous sinus thrombosis.
localized tissue abscesses.

(a)
(b)
(c)
(d)

1.
2.
3.
4.
5.

(a), (b)
(a), (b)
(a), (c)
(b), (c)
Any of

or (c)
or (d)
or (d)
or (d)
the above

1.
2.
3.
4.
5.

81.

78.

79.

(a)
(a)
(a)
(b)
(b)
(c)

and
and
and
and
and
and

(b)
(c)
(d)
(c)
(d)
(d)

82.

(a)
(b)
(c)
(d)
(e)

1.
2.
3.
4.
5.

Recent Heptavax@vaccination
Loss of cellular immunity defenses
Alteration of T-helperlT-suppressor
cell ratio in blood
Increased susceptibility to
opportunistic infections
Decreased immunoglobulin
concentration in peripheral
circulation

(a), (b) and


(a), (b) and
(a), (c) and
(b), (c) and
(c), (d) and

30

Thalassemia
Polycythemia vera
Erythroblastosis fetalis
Hereditary spherocytosis

are invasive.
readily metastasize.
exhibit mitotic figures.
are cured by early excision.
commonly occur in the oral cavity.

(a), (c) and (d)


(a), (d) and (e)
(b) and (c)
(b), (d) and (e)
(d) and (e) only

The secondary stage of syphilis may be


manifested in which of the following forms?
(a)
(b)
(c)
(d)
(e)

1.
2.
3.
4.
5.
6.

(d)
(e)
(e)
(d)
(e)

(c)
(e)
(d)
(e)
(e)

Basal cell carcinoma and squamous cell


carcinoma are similar in that both

1.
2.
3.
4.
5.

83.

Corynebacterium diphtheriae.
Clostridium tetani.
Hemophilus influenzae.

(a), (b) and (c)


(a), (b) and (d)
(a), (c) and (d)
(b), (c) and (d)
All of the above

(a)
(b)
(c)
(d)
(e)

Which of the following are associated with a


patient who has AIDS?

Bordetellapertussis.

Which of the following blood diseases are


red cell disorders?

1.
2.
3.
4.
5.

Impetigo
Dysentery
Food poisoning
Rheumatic fever

(a)
(b)
(c)
(d)

Brucella abortus.

(a), (b) and


(a), (c) and
(b), (c) and
(b), (d) and
(c), (d) and

(a)
(b)
(c)
(d)

Which of the following diseases are


commonly associated with pathogenic
staphylococci ?

1.
2.
3.
4,
5.
6.

Infants are routinely immunized against


diseases caused by

Hard chancre
Soft chancre
Maculopapular rash
Extensive CNS involvement
Mucous patches in the oral cavity

(a) and (b)


(a) and (d)
(b) and (c)
(c) only
(c) and (e)
(d) only

84. Ascites, which

is often a complication of
alcoholic liver disease, develops as a result of
(a)
(b)
(c)
(d)

1.
2.
3.
4.
5.
6.

85.

(a)
(b)
(c)
(d)

1.
2.
3.
4.
5.
6.

(a) only
(a) and (c) only
(a), (c) and (d)
(b) and (c)
(b) and (d)
All of the above

(a)
(b)
(c)
(d)
(e)

Abnormalitiesin cell growth that cause cells,


tissues or organs to be smaller than normal
include

cirrhosis.
esophageal varices.
portal hypertension.
decreased protein production by
the liver.

Cells of which of the followingretain a latent


capacity for mitotic division?

1.
2.
3.
4.
5.

87.

88.

liver
Bone marrow
Cardiac muscle
Salivary glands
Neurons

(a), (b) and


(a), (b) and
(a), (c) and
(b), (c) and
(b), (d) and

Right-sided heart failure affects the kidneys by


causing

1.
2.
3.
4.
...
J.

86.

(a) and (b) only


(a), (b) and (d)
(a) and (c) only
(a), (c) and (d)
(b), (c) and (d)
(b) and (d) only

(a)
(b)
(c)
(d)

(c)
(d)
(d)
(e)
(e)

metaplasia.
atrophy.
anaplasia.
hypoplasia.

renal hypoxia.
venous congestion.
retention of H2O and NaCI.
decreased glomerular filtration
rate.

(a), (b) and (c)


(a), (b) and (d)
(a), (c) and (d)
(b), (c) and (d)
All of the above

In healing of a fracture, which of the


following will prevent favorable
reconstruction and alignment of bone?
(a)
(b)

(c)
(d)
(e)

1.
2.
3.
4.
5.

Formation of pseudoarthrosis
Functional remodeling of the
procallus by osteoblasts and
osteoclasts
Formation of new bone at the site
of fracture
Presence of sequestrum
Organization of a hematoma at
the site of fracture

(a) only
(a) and (d)
(b) and (c) only
(b), (c) and (d)
(d) and (e)

89.

A soft tissue infection rather diffusely spread


along the mandible and into the floor of the
mouth would likely involve which of the
following organisms?
(a)
(b)
(c)
(d)

1.
2.
3.
4.
5.

Eikenella corrodens
Staphylococcus aureus
Streptococcus pyogenes
Peptostreptococcus anaerobius

(a), (c) and (d)


(b) and (c) only
(b), (c) and (d)
(b) and (d) only
All of the above

..

90.

Bacteria of which of the following genera


have a limited range of habitats in the oral
cavity?
(a)
(b)
(c)
(d)

1.
2.
3.
4.
5.
6.

--.

91.

(a)
(a)
(a)
(b)
(b)
(c)

95.

1.
2.
3.
4.
5.

Treponema
Bacteroides
Actinomyces
Streptococcus
and
and
and
and
and
and

(b)
(c)
(d)
(c)
(d)
(d)

Which of the following is NOT mediated by


immunoglobulins?

96.

Which of the following is NOT a direct cause


of left-sided heart failure?
1.
2.
3.
4.
5.

Which of the following will NOT promote

Anaphylaxis
Atopic allergy
Serum sickness
Arthus' reaction
Cpntact dermatitis

Emphysema
Hypertension
Aortic stenosis
Myocardial infarction
Rheumatic heart disease

fluid retention in the' interstitial


compartment?

1.
2.
3.
4.
5.

..,

92.

Each of the following may be seen in


multiple myeloma EXCEPT

1.
2.
3.
4.
5.

93.

5.

Breast
Tongue
Kidney
Thyroid
Prostate

Which of the following viruses is least likely


to be spread in the dental office?

1.
2.
3.
4.
5.
98.

99.

Rhinovirus
Hepatitis B
Herpes simplex virus

Production of MIF
Production of histamine
Activation of macrophages
Production of chemotactic factors
Protection against intracellular parasites

A patient with which of the following


diseases is least likely to show "clubbing" of
fingers and cyanotic nail beds?

1.
2.
3.
4.
5.
100.

HTLVIII
Adenovirus

Which of the following is NOT a function of


T lymphocytes in defense of the host?

1.
2.
3.
4.
5.

Splenomegaly
Necrotizing pharyngitis
Depressed heterophile titer
Abnormal lymphocytes in peripheral
blood
Marked increase in the number of
circulating lymphocytes

A primary tumor of which of the following


organs is least likely to give rise to skeletal
metastasis?

1.
2.
3.
4.
5.

97.

hypoproteinemia.
plasma cell neoplasia.
hypergammaglobul inemia.
light-chain proteinuria.
punched-out areas of bone.

Which of the following is NOT a feature of


infectious mononucleosis?
1.
2.
3.
4.

94.

Lymphatic obstruction
Increased capillary permeability
Decreased osmotic pressure of the blood
Increased hydrostatic pressure of the
blood
Decreased osmotic pressure of the
interstitial fluid

Polycythemia
Congenital heart disease
Chronic pulmonary disease
Congestive heart failure
Systemic lupus erythematosus

Which of the following species of


streptococci is usually NOT found in human
dental plaque?

1.
2.
3.
4.
5.

S. mutans
S. sanguis
S. pyogenes
5. sa/ivarius
S. mitior (5. mitis)

J
i~

..

NATIONAL BOARD DENTAL EXAMINATION PART I


ANSWER KEY
MICROBIOLOGY-PATHOLOGY

NO.

ANS.

NO.

ANS.

1.

2
3
1
5
4

26.
27.
28.
29.
30.

4
5
1
1
1

31.

2.
3.
4.
5.

DECEMBER

1988

NO.

ANS.

NO.

ANS.

51.
53.
54.
55.

3
2
5
5
4

76.
77.
78.
79.
80.

1
5
2
4
3

56.
57.
58.
59.
60.

6
4
3
5
5

81.
82.
83.
84.
85.

5
1
5
3
2

52 .

3
4
4
3
1

33.
34.
35.

2
2
4
4
3

14.
15.

4
5
5
4
1

36.
37.
38.
39.
40.

5
1
4
1
1

61.
62.
63.
64.
65.

4
2
3
2
2

86.
87.
88.
89.
90.

2
6
5
5
1

16.
17.
18.
19.
20.

5
3
3
2
2

41.
42.
43.
44.
45.

5
1
4
2
3

66.
67.
68.
69.
70.

2
1
2
3
2

91.
92.
93.
94.
95.

5
1
3
2
5

21.
22.
23.
24.
25.

1
2
4
5
2

46.
47.
48.
49.
50.

2
5
2
5
3

71.
72.
73.
74.
75.

5
2
2
2
3

96.
97.
98.
99.
100.

1
1
2
5
3

6.
7.
8.

9.
10.
11.
12.

13 .

32 .

..

DENTALANATOMYand OCCLUSION (14)

All test items relating to occlusion refer to a Class 1


canine and molar relationship unless otherwise
specified. Terms such as ilnormal" or ilideal" are
synonymous with the above definition.

1.

almost round.
divided into two canals.
flattened mesiodistally.
flattened faciolingually.
shaped much the same as that of a
maxillary central incisor.

6.

7.

3.

Right first mol?r


Leftfirst molar
Right second molar
Left second molar

The protrusive pathway of mandibular cusps


on maxillary posterior teeth is toward the

In an ideal intercuspal position, the


mesiolingual cusp of a permanent
mandibular second molar opposes th~

1. embrasure between maxillary first and


second molars.
2. distal fossa of the maxillary second
molar. .
3. lingual sulcus of the maxillary second

In the intercuspal position, the cusp tip of


the permanent maxillary canine is in direct
alignment with which anatomic feature of the
mandibular teeth?

molar.
central fossa of the maxillary second
molar.
mesial fossa of the maxillary. third molar.

4.

5.

1. Facial embrasure
2. Incisal embrasure
3. Interproximal space
4. Distal ridge of the cusp of the
5.

4.

mandibular canine
Mesial ridge of the facial cusp of the
mandibular first premolar

facial to the normal arch form.


lingual to the normal arch form.
mesial to the normal arch position.
distal to the normal arch position.

1. facial.
2. distal.
3. lingual.
4. mesial.
5. None of the above

Identify the permanent maxillary tooth


sketched below.

1.
2.
3.
4.

In cases of delayed resorption of primary


incisors, the permanent incisors may be
expected to erupt

1.
2.
3.
4.

A cross section at midroot of a permanent


mandibular central incisor is likely to show
that the pulp cavity is

1.
2.
3.
4.
5.

2.

5.

8.

Viewed anteriorly, which diagram represents


a chewing stroke?

Which of the following provides the most


reliable criterion for .differentiating permanent
mandibular central incisors from permanent
mandibular lateral incisors?

0
I

1. Difference in root length


2. Difference in rotation of the crown on
the root
Difference in ratio of crown length to
root length
4. Degree of parallelism between mesial
and distal surfaces of the tooth
5. Degree of slope of the inci.saledge
when viewed facially

1.

2.

3.

4.

5.

3.

1.
2.
3.
4.
5.

1
2
3
4
5

.j-,;
,I

...
9.

13.

On a mandibular first molar, the distofacial


groove serves as an escapeway for the
mesiolingual cusp of the maxillary first molar
during which of the following mandibular
movements?

1.
2.
3.
4.

In an ideal intercuspal position, the facial


cusp of the mandibular first premolar
contacts the
1.
2.

Protrusive
Centric slide
Working (laterotrusion)
Non-working (mediotrusion or balancing)

3.
4.
5.

10.

In an ideal intercuspal position, the


distolingual cusp of a permanent maxillary
first molar opposes the

14.

1.

central fossa of the mandibular first


molar.
2. lingual sulcus of the mandibular first
molar.
3. facial embrasure between mandibular
first and second molars.
4. mesial fossa of the mandibular second
molar.
5. mesial marginal ridge area of the
mandibular second molar.

I
r

In an ideal arrangement of teeth in the


intercuspal position, the mesiofacial cusp of
the permanent mandibular first molar would
be in contact with the maxillary first molar
1.
2.
3.
4.

15.

2.

Identify the cusp of the maxillary tooth


below that contacts the dot.

3.
4.
5.

16.
1.
2.
3.
4.
5.
6.

12.

Buccal cusp of second premolar


Lingual cusp of second premolar
Buccal cusp of first premolar
Lingual cusp of first premolar
Lingual surface of canine
None of the above

in the central fossa.


in the distal fossa.
on the obi ique ridge.
on the mesial marginal ridge area.

The lingual cusp of a mandibular first


premolar contacts the
1.

11.

mesial marginal ridge area of the


maxillary first premolar.
~
distal marginal ridge area of the
maxillary first premolar.
distal inner aspect of the lingual cusp of
the maxillary first premolar.
mesial outer aspect of the facial cusp of
the maxillary first premolar.
central groove of the maxillary second
premolar.

lingual embrasures of a maxillary canine


and a maxillary first premolar.
mesial incline plane of the lingual cusp
of a first premolar.
mesial marginal ridge of a maxillary first
premolar.
cingulum of a maxillary canine.
None of the above

Which of the following sketches illustrates


the distal view of a permanent maxillary first
molar?

The disk (meniscus) of the


temporomandibular joint is moved forward
principally by the

1. condyle.
2. articular capsule.
3. stylomandibular ligament.
4. medial pterygoid muscle.
5. lateral pterygoid muscle.

1.
2.
3.
4.

35

1
2
3
4

..
,
17.

22.

A primary mandibular second molar differs


from a permanent mandibular first molar in
that a,primary mandibular second molar
1.
2.
3.
4.
5.

1.
2.
3.
4.
5.

is darker in color.
is larger in size.
has a form peculiar to itself.
has a larger occlusal surface.
has roots that are more divergent.

23.
18.

19.

mesiolingual.
distolingual.
mesiofacial.
distofacial.
distal.

The embrasures surrounding the contact area


of an anterior tooth are

Viewed proximally, which of the following


teeth have the greatest axial inclinations
relative to the occlusal plane?

1. gingival, cervical, facial and lingual.


2. incisal, cervical, facial and lingual.
3. incisal, cervical and facial only.
4. cervical, lingual and facial only.

1.
2.
3.
4.

24.

The primary teeth that present the most


outstanding morphologic deviations from
permanent teeth are

1.
2.
3.
4.
5.

20.

The highest and sharpest cusp on a primary


mandibular first molar is the

central incisors.
lateral incisors.
canines.
first molars.
second molars.

~~
1f ~
.

25.

'",'

First molar
Second molar
First premolar
Second premolar

In an ideal arrangement of permanent


maxillary teeth, the narrowest incisal or
occlusal embrasure is the one between

1.
2.
3.
4.
5.
6.

-_._.
@}
: ::

..",

'...'
(..LJ

The roots of a primary mandibular second


molar are flared to allow space for the
developing tooth bud of which permanent
mandibular tooth?
1.
2.
3.
4.

Which of the following sketches illustrates


the occlusal view of a primary maxillary right
first molar?

Maxillary lateral incisors


Maxillary central incisors
Mandibular lateral incisors
Mandibular central incisors

central incisors.
central and lateral incisors.
lateral incisor and canine.
canine and first premolar.
first and second premolars.
second premolar and first molar.

..--

1.
2.
3.
4.
5.

21.

26.

1
2
3
4
5

The incisor indicated by the arrow in the


sketch below is a

When viewed from the proximal, which of


the following permanent teeth appears to be
aligned in its arch with the axial inclination
of its roots most nearly vertical?

1.
2.
3.
4.
5.

1.
2.
3.
4.
5.

Maxillaryfirst molar
Mandibular second molar
Maxillaryfirst premolar
Maxillarycentral incisor
Mandibular central incisor

36

primary maxillary left lateral.


primary mandibular right central.
primary mandibular right lateral.
permanent mandibular right lateral.
permanent mandibular left lateral.

~,
-,{

".:
!,

....
27.

1.
2.

3.

4.
5.

28.

From a proximal view, the incisal edge


is lingual to the long axis of the root.
From a proximal view, the crests of facial
and lingual cervicoenamel ridges are
directly opposite each other.
Mesial and distal contact areas are
located in the incisal third of the crown.
Mesial and distal contact areas are
directly opposite each other.
Mesial and distal contact areas are
approximately centered faciolingually.

On which of the following primary molars


would a prominent transverse ridge typically
occur?

1.
2.
3.
4.

32.

Which of the sketches below best represents


an occlusal view of a maxillary left second
premolar?

form dentin.
provide nutrition.
provide sensation.
assure root-end closure.
protect the periodontium.

1.
2.
3.
4.
5.

The sketch below of a cross section at the


cementoenamel junction is typical of which
permanent tooth?

33.

1.
2.
3.
4.
5.

Mandibular second premolar


Maxillary first premolar
Maxi Ilary molar
Mandibular molar
Number of different teeth and cannot be
identified

34.

4.

5.

1.
2.
3.
4.
5.

35.

Maxillary canine
Maxillary central incisor
Maxillary lateral incisor
Maxillary first premolar
Mandibular second premolar

37

1
2
3
4
5

central incisors.
central and lateral incisors.
lateral incisor and canine.
canine and first premolar.
first and second premolars.
second premolar and first molar.

provide spillways.
guide food toward the occlusal surface.
guide food over the free margin of the
gingiva.
prevent food particles from entering the
interproximal areas.
None of the above

The distal contact area of a permanent


maxillary canine is usually located at the

1.
2.
3.
4.

~
i!

A function of proximal contact areas of teeth


is to

1.
2.
3.

Which of the following teeth is most likely to


occasionally exhibit a lingual groove that
extends from the enamel onto the cemental.
area of the root?

In a normal, healthy mouth, the interdental


papilla that is the shortest cervicoincisally
(cervico-occlusally) is between mandibular

1.
2.
3.
4.
5.
6.

30.

Maxillary first molar


Maxillary second molar
Mandibular first molar
Mandibular second molar

The primary function of the dental pulp is to

1.
2.
3.
4.
5.

29.

31.

Which of the following characteristics is


common to all permanent incisors?

middlethird.

middle of the incisal third.


middle of the cervical third.
junction of incisal and middle thirds.

~-

.
.

36.

41.

The muscle of mastication that is primarily


responsible for synchronizing the movement
of the condyle and the articular disc is the

1. masseter.
2. temporalis.
3. internal pterygoid (medial pterygoid).
4. external pterygoid (lateral pterygoid).

1.

37.

38.

3.

gingival recession.
increased length of clinical crowns.
decreased length of anatomic crowns.
reduced interproximal embrasure spaces.

4.
5.

42.

Which axial surface of the crown of a


mandibular canine is almost parallel to the
long axis of the tooth?
1.
2.
3.
4.

,..

2.

Proximal contact of posterior teeth creates


wear patterns that eventually cause
1.
2.
3.
4.

-...

Which of the following teeth is most likely to


resist caries?
1.
2.
3.
4.

40.

L
L

mesial view of a permanent


first molar.
mesial view of a permanent
second molar.
mesial view of a permanent
third molar.
distal view of a permanent
first molar.
distal view of a permanent
second molar.

mandibular
mandibular
mandibular
mandibular

closing.
retruding.
protruding.
opening with translation.
opening without translation.

Which of the following sketches illustrates


the facial view of a permanent maxillary left
canine?

Maxillary central incisor


Mandibular canine
Mandibular first premolar
Maxillary lateral incisor

How does the greatest faciolingual


measurement of the mandibular canine
compare with its greatest mesiodistal
measurement?

1.
2.
3.
4.

mandibular

Contraction of the posterior fibers of the


temporalis muscle results in the mandible

1.
2.
3.
4.
5.

Mesial
Distal
Facial
Lingual

43.
39.

fL

The sketch below represents the

The faciolingual measurement is less.


The measurements are equal.
The faciolingual measurement is greater.
There is no definite relation.

38

1.
2.
3.
4.

1
2
3
4

~]
:i

4<

44.

In cross section, the root of a mandibular


canine is described as

48.

Which of the following sketches illustrates


the mesial view of a permanent mandibular
left lateral incisor?

1. roughly conical.
2. irregularly oval.
3. flattened in a mesiodistal direction.
4. broader mesiodistally on the lingual than
on the labial.

45.

1.
2.
3.
4.

first molars, lateral incisors, second


molars, canines.
lateral incisors, first molars, second
molars, canines.
lateral incisors, first molars, canines,
second molars.
lateral incisors, canines, second molars,
first molars.

49.

50.
When the teeth are in centric occlusion, the
position of the mandible in relation to the
maxilla is primarily determined by the
1.
2.
3.
4.

1.

'.
~
f;,

2.
3.
4.
5.
6.

Maxillary first molar


Mandibular first molar
Mandibular second molar
Mandibular first premolar
Maxillary second premolar
Mandibular second premolar

52.

slight.
maximum.
prematu reo
not present.

The permanent posterior tooth most likely to


have a pronounced concavity on the mesial
surface of its crown is the
maxillary first premolar.
maxillary second premolar.
mandibular first molar.
mandibular first premolar.
mandibular second premolar.

The non-working condyle performs which of


the following movements?

1.
2.
3.
4.
Which of the following permanent teeth has
its mesial marginal ridge located more
cervically than its distal marginal ridge?

When the mandible is in its physiologic rest


or postural position, contact of teeth is

1.
2.
3.
4.
5.

intercuspation of teeth.
presence or absence of diastemas.
tonus of muscles of head and neck.
ligaments of the temporomandibular'
joint.

51.

47.

1. 1
2. 2
3. 3
4. 4

1.
2.
3.
4.

46.

The usual order of appearance of the primary


teeth in the mouth is central incisors,

Straight forward
Downward, forward and lateral
Downward, forward and medial
Downward, backward and medial

The percentage of dentin that is organic is

1.
2.
3.
4.
5.

3-5%.
10-15%.
20-30%.
40-50%.
50-60%.

.,

36.

40.

In the fetal heart, the foramen ovale conneds


the
1.
2.

3.
4.
5.

pulmonary artery and the aorta.


right and the left atria, permitting the
of blood from right to left.
right and the left atria, permitting the
of blood from left to right.
right and the left ventricles, permitting
flow of blood from right to left.
right and the left ventricles, permitting
flow of blood from left to right.

1.
flow
2.
flow

3.

the

4.

the

41.
37.

Which of the following muscles receives its


motor supply from the facial nerve?

1.
2.
3.
4.
5.

Masseter
Temporal
Buccinator
lateral pterygoid
Tensorveli palatini

3.
4.

Identify the thalamus in the sketch below.

4.

43.

2.
1.
2.
3.
4.
5.

A
B
C
0
E

3.
4.

44.
39.

Second, third and fourth right posterior


intercostal veins drain from the right superior
intercostal vein into the

1.

azygos vein.
hemiazygos vein.
accessory hemiazygos vein.
left brachiocephalic vein.
left internal jugular vein.

3.
4.

mandibular incisors.
mandibular canines.
maxillary premolars.
mandibular second and third molars.

keratin.
a small amount of tissue fluid.
capillaries that course between cells to
approach the free surface.
none of the above.

underlying connective tissue of the gingiva


does not contain lymphoid cells.
epithelium of the epithelial attachment
does not have rete pegs.
epithelium of the gingiva is not
keratinized.
underlying connective tissue of the
epithelial attachment is less vascular.

The bony floor of the nasal cavity is formed by


the

2.

1.
2.
3.
4.
5.

myofibrils conneding with collagenous


fibrils of the tendon.
an abundance of reticular fibers in the
area of the jundion.
a continuity of connedive tissue sheaths
of the muscle with those of the tendon.
a special thickening of sarcoplasm that
unites with collagenous fibrils of the
tendon.

Normally, the free gingiva can be distinguished


from the epithelial attachment because the
1.

the union is

In the sulcular epithelium, small spaces


between cells in the stratum spinosum (prickle
cell layer) are normally filled by

1.
2.
3.

jundion,

Apical abscesses on certain teeth have a


marked tendency to produce cervical spread of
infection that occurs most rapidly in abscesses
of
1.
2.

42.
38.

At a muscle-tendon
made by

palatine process of the maxilla and the


vertical part of the palatine.
palatine process of the temporal and the
horizontal part of the palatine.
vomer and the vertical part of the
palati ne.
palatine process of the maxilla and the
horizontal part of the palatine.

..

53.

In the mandibular arch, the greatest lingual


inclination of the crown from its root is seen
in a permanent
1.
2.
3.
4.
5.

54.

canine.
third molar.
first premolar.
central incisor.
lateral incisor.

3.

mandibular central incisor.


mandibular lateral incisor.
mandibular canine.
maxillary central incisor.
maxillary lateral incisor.

4.
.

59.

60.

grooves.
shorter with multiple supplementary
grooves.

61.

57.

62.

mesiofacial cusp of first molar


Lingual cusp of second premolar and
mesiolingual cusp of first molar

Identify the mandibular movement in the


drawing above.

1.
2.
3.
4.
5.

In which of the following areas is the


alv~olar process the thinnest?

Lingualto the maxillary central incisors


Facialto the mandibular central incisors
Lingualto the 'maxillary canines
Lingualto the mandibular first molars
Facial to the maxillary second molars
lingual to the maxillary second molars

The pulp horns most likely to be exposed


accidentally in the preparation of a Class II
cavity in a maxillary first molar are

1.
2.
3.
4.

1. Facial cusps of premolars


2. Lingual cusps of premolars
3. Facial cusp of second premolar and
4.

mesiolingual anddistolingual.
mesiofacial and distolingual.
mesiolingual and distofacial.
mesiofacial and distofacial.
distofacial and distolingual.

1.
2.
3.
4.
5.
6.

In the sketch below, the arrows represent


possible contacting areas for which of the
following maxillary cusps?

molar

The occlusal outline of a maxillary first molar


contains two obtuse and two acute angles.
The acute angles are
1.
2.
3.
4.
5.

1. longer with less supplementary grooves.


2. shorter with less supplementary grooves.
3. longer with multiple supplementary

56.

Distal of second premolar and distal of


first molar
Mesial of first premolar and mesial of
second premolar
Distal of first molar and distal of second
molar
Mesial of first premolar and distal of first

2.

When compared with maxillary first


premolars, the central developmental groove
of maxillary second premolars is

4.

Which of the following surfaces of


permanent maxtllary teeth require special
consideration ,in adapting a matrix band
because of pronounced cervical concavities?
1.

A carious developmental pit is most likely to


be found in the lingual surface of a
1.
2.
3.
4.
5.

55.

58.

mesiofacialand mesiolingual.
mesiolingual and distolingual.
distolingual and distofacial.
distofacial and mesiofacial.

The drawing below is a cross section at the


cementoenamel junction of what permanent
tooth?

viewed from the incisal

GM

Protrusive
Right lateral (working side)
Right lateral (non-working side)
Left lateral (working side)
Left lateral (non-working side)

1.
2.
3.
4.

40

Maxillaryright canine
Mandibular right canine
Maxillaryright central incisor
Mandibular right central incisor

'iii'
:;~
,'.

-63.

Which of the following permanent teeth


would be expected to have the most variable
occlusal anatomy?

1.
2.
3.
4.
5.
6.

64.

68.

Maxillary first molar


Maxillary second premolar
Maxillary third molar
Mandibular first premolar
Mandibular first molar
Mandibular second molar

1.
2.
3.
4.
5.

Special consideration should be given in


preparing this tooth for endodontic therapy
because two pulp canals are usually found in
the

1.
2.
3.
4:
5.

1-2 years
3-4 years
5-6 years
None of the above

In comparison with the faciolingual


dimension of the crown of a permanent
mandibular first molar, the mesiodistal
dimension is
1.
2.
3.
4.

67.

2.
3.
4.
5.

71.

Primary mandibular first molar


Primary mandibular second molar
Permanent mandibular first molar
Permanent maxillary second molar

The ridges that together make up the oblique.


ridge of a permanent maxillary first molar
meet near the center of the occlusal surface
on a level with the
1.

slightly less.
slightly greater.
approximately twice.
approximately half.

Based upon morphology, a mesicrocclusal


(Class II) cavity preparation would be most
difficult in which of the following teeth?

1.
2.
3.
4.

Distal
Mesiofacial
Distofacial
Mesiolingual
Distolingual

7-8 years

70.
66.

Which pulp horn of a permanent mandibular


fi rst molar is the smallest?

1.
2.
3.
4.
).

Calcification of the roots of the primary


dentition is normally completed at what age?
1.
2.
3.
4.
5.

lingual root of a maxillary molar.


distal root of a mandibular first molar.
mandibular canine.
mandibular first premolar.
maxillary first premolar.

root system of a mandibular canine.


distofacial root of a maxillary molar.
distal root of a mandibular first molar.
mesial root of a mandibular first molar.
lingual root of a maxillary first premolar.

69.

65.

The cross-sectional view at midroot pictured


below is most probably that of a

tips of the mesiolingual and distofacial


cusps.
depth of central and distal fossae.
cusp of Carabelli.
marginal ridges.
central groove.

In removing calculus from the root trunk


area, which of the following anatomic
surfaces present the most difficulty?

1.
2.
3.
4.
5.

Mesials of mandibular second molars


Distals of mandibular second premolars
Distals of maxillary canines
Linguals of maxillary second premolars
Distals of maxillary first molars

1\
L...:.,--

L-

v-

72.

76.

The drawing below is a cross section at the


cervical line of what permanent tooth?

The mandibular movement indicated in the


drawing above is

1.
2.
3.
4.
5.

protrusive.
right lateral (working side).
left lateral (working side).
right lateral (non-working side).
left lateral (non-working side).

Ii

Mandibular central incisor


Mandibular first premolar
Mandibular canine
Maxillary lateral incisor

1.
2.
3.
4.

.......
~

77.
~
......

73.

1.
2.
3.
4.
5.

What muscle tissues form the floor of the


mouth in the lingual vestibule?

....

1.
2.
3.
4.
5.
6.

lingual
Stylohyoid
Buccinator
Mylohyoid
Sphenomandibular
Superior pharyngeal constrictor

78.

1-

74.

In a normal retrusive tooth contacting


movement of the mandible, which tooth
contacts the maxillary central incisor?

1.
2.
,3.
4.
5.

Canine
.
lateral incisor
Central incisor
Central and lateral incisors
None of the above

80.

1.
2.
3.
4.
po
:J.

facial cusps of premolars.


lingual cusps of premolars.
cusp of canine and facial cusp of first
premolar.
facial cusp of second premolar and the
mesiofacial cusp of first molar.
lingual cusp of second premolar and the
mesiolingual cusp of first molar.

42

Maxillary canine
Mandibular canine
Maxillary lateral incisor
Mandibular lateral incisor

Which of the following types of mucosa


characterizes attached gingiva?

1.
2.
3.
4.

Assuming occlusion and alignment are


normal, the arrows on the sketch below
represent the path taken by the

6-7 years
8-9 years
10-11years
12-13years
14-15 years

In the intercuspal position, which of the


following anterior teeth has the potential to
contact both anterior and posterior
antagonists?
1.
2.
3.
4.

79.

75.

At what age is a primary maxillary canine


usually exfoliated?

Free
Alveolar
Special ized
Masticatory

The height of contour,occlusocervically


located within the middle third of the

is

1. distal surface of a permanent central


incisor.
2. lingual surface of a permanent maxillary
first molar.
3. facial surface of a permanent mandibular
premolar.
4. facial surface of a permanent mandibular
first molar.

,.~

.J
.<
::
'j
~~
.J

81.

In an ideal intercuspal position, a permanent


mandibular lateral incisor opposes the

(a)

distal marginal ridge of a maxillary


central incisor.
.
mesial marginal ridge of a
maxillary lateral incisor.
distal marginal ridge of a maxillary
lateral incisor.
mesial marginal ridge of a
maxillary canine.
maxillary lateral incisor 2-3 mm.
cervically to the incisal edge.
maxillary lateral incisor at the

(b)
(c)
(d)
(e)
(t)

incisal edge.
1.
2.
3.
4.
5.

84.

(a) and (b)


(a), (b) and
(a), (b) and
(b) only
(c), (d) and

(a)
(b)
(c)
(d)
(e)
(t)

1.
2.
3.
4.
5.
6.

1.
2.
3.
4.
5.

Cingulum
Lingual cusp
Lingual ridge
Mesial and distal fossae
Prominent developmental grooves
Mesial and distal marginal ridges

1.
2.
3.
4.
5.

(a),
(a),
(b),
(b),
(d),

Dentin
Cementum
Bundle bone
lamina dura
Epithelial attachment
Periodontal ligament
(c)
(d)
(c)
(c)
(e)

and (d)
and (t)
and (e)
and (f)
and(t)

(a), (b), (d) and (e)


(a), (c), (d) and (t)
(a), (d), (e) and (t)
(b), (c), (e) and (t)
All of the above

Which of the following permanent anterior


teeth normally have their distal contact areas
located within the middle thirds of the
crowns incisogingivally?

When posterior teeth are in a crossbite


relationship, "'~,ich of the following cusps are
considered supporting cusps?
(a)
(b)
(c)
(d)

1.
2.
3.
4.

(b)
(d)
(t)
(d)
(e)
(t)

The tooth root is attached to the alveolar


process. What structures are included in this
attachment?
(a)
(b)
(c)
(d)
(e)
(f)

86.

83.

(a) and
(a) and
(b) and
(c) and
(c) and
(e) and

(e)

Which of the following are considered


normal lingual anatomy on a maxillary
canine?
(a)
(b)
(c)
(d)
(e)
(t)

Maxillary second premolar


Maxillary first molar
Maxillary second molar
Mandibular first premolar
Mandibular second premolar
Mandibular second molar

only
(e)
(t)

85.

82.

Which of the following permanent teeth most


frequently have three cusps?

Maxillary facial
Maxillary lingual
Mandibular facial
Mandibular lingual

(a) and
(a) and
(b) and
(b) and

(c)
(d)
(c)
(d)

(a)
(b)
(c)
(d)
(e)
(t)

1.
2.
3.
4.
5.
6.

Maxillary central incisors


Maxillary lateral incisors
Maxillary canines
Mandibular central incisors
Mandibular lateral incisors
Mandibular canines

(a), (b) and (c)


(b), (c) and (e)
(b), (c) and (t)
(c), (e) and (t)
(d), (e) and (t)
All of the above

..

87.

--

91.

The cusp of Carabelli occurs with sufficient


frequency to be considered normal in a
(a)

. (b)
(c)
(d)

""'--

The roots of mandibular first premolarsare


(a)
(b)
(c)
(d)

primary maxillary first molar.


primary maxillary second molar.
permanent mandibular first molar.
permanent maxillary first molar.

(e)
1.
2.
3.
4.

88.

(a)
(a)
(b)
(d)

(a)
(b)
(c)
(d)
(e)

1.
2.
3.
4.
5.

(a),
(a),
(a),
(b),
(c),

(b)
(b)
(d)
(d)
(d)

and
and
and
and
and

92.

(c)
(e)
(e)
(e)
(e)

In intercuspal position, a maxillary lateral


incisor opposes
(a)
(b)

1.
2.
3.
4.
5.

(c)
(d)

provide better retention for the

restoration.
(b)
(c)
(d)

1.
2.
3.
4.
5.

prevent food impaction.


stabi Iize the'"dental arch,
protect the periodontium.

(a)
(a)
(b)
(b)
(c)

the incisal edge of a mandibular


lateral incisor.
both mesial and distal marginal
ridges of a mandibular lateral
mclsor.
the distal marginal ridge of a
mandibular lateral incisor.
a mandibular canine.
and
and
and
and
and

(b)
(d)
(c)
(d)
(d)

(a) and (b)


(b) and (c) only
(b), (c) and (d)
(c) and (d) only
All of the above

Which of the following are reactions of the


dental pulp?
(a)
(b)
(c)
(d)

1.
2.
3.
4.
S.

(a) and (c)


(a), (d) and (e)
(b) and (e) only
(b), (d) and (e)

distolingual cusp.
mesiolingual cusp.
cusp of Carabell i.
mesiofacial cusp.
distofacial cusp.

Maintaining a positive contact area when


restoring the proximal surfaces of two
adjacent teeth. is essential to
(a)

90.

1.
2.
3.
4.

The primary cusp triangle on the occlusal


surface of a permanent maxillary molar is
formed by the

....

89.

and (c)
and (d)
and (d)
only

flattened faciolingually.
broader facially than lingually. .
frequently sharply curved distally.
usually free of marked distal
curvature.
frequently seen with slight
concave areas on mesial and distal
surfaces.

93.

A maxillary first premolar may be identified


by a

Formation of dentin
Transmission of pain stimuli
Formation of the mesenchyme of
the dental papilla
Production of a defensive reaction
when a tooth is exposed to
irritation

(a)
(b)
(c)
(d)
(e)

1.
2.
3.
4.
S.

(a) and (b) only


(a), (b) and (d)
(a), (c) and (d)
(b) and (d) only
All of the above

A A

(a),
(a),
(b)
(d)
(d)

marked mesial concavity in the


cervical area.
deep sulcus.
long central groove.
short central groove.
single pulp canal.
(b) and (c)
(c) and (e)
and (d)
only
and (e)

..
94.

Pronounced developmental grooves are


usually associated with embrasures between
permanent
(a)
(b)
(c)
(d)

maxillary central and lateral


incisors.
maxillary lateral incisors and
can meso
maxillary canines and first
premolars.
maxillary first and second

premolars.
(e)
(t)

"' (g)
(h)
1.
2.
3.
4.
5.
6.
7.
8.

95.

mandibular
premolars.
mandibular
premolars.
mandibular
first molars.
mandibular
molars.

(a)
(b)
(c)
(d)
(e)

1.
2.
3.
4.
5.

canines and first


first and second

Permanent central incisors


Permanent lateral incisors
Permanent canines
First premolars
Permanent first molars

(a) and
(a), (b)
(b), (c)
(b), (c)
(c), (d)

(b) only
and (c)
and (d)
and (e)
and (e)

second premolars and


first and second

98.
Distoli"ngual cusps of which of the following
posterior teeth may be completely absent?

1.
2.
3.
4.
5.
6.
7.

A 5-year-old child is treated with large doses


of tetracycline over a one year period. Which
of the following permanent teeth will least
likely show tetracycline staining?

(a) only
(b) only
(c) only
(c) and (e)
(d) only
(d) and (t)
(g) on Iy
(h) only

(a)
(b)
(c)
(d)
(e)

96.

97.

1.
2.
3.
4.
5.

Maxillary first molar


Mandibular first molar
Mandibular second molar
Maxillary third molar
Mandibular second premolar

(a) and
(a) and
(b) and
(c) only
(d) only
(d) and
(e) only

Which of the following permanent teeth is


least likely to have a divided pulp canal?
Maxillary central incisor
Maxillary first premolar
Mandibular central incisor
Mandibular lateral incisor
Mandibular first premolar

(b)
(c)
(d)

(e)

Which of the following permanent teeth


usually have oblique ridges?
(a)
(b)
(c)
(d)

1.
2.
3.
4.
5.

(a)
(b)
(b)
(d)
All

Maxillary premolars
Maxi Ilary molars
Mandibular premolars
Mandibular molars
and (c)
only
and (d)
only
of the above

99.

Which of the following fibers are NOT


periodontal ligament fibers?

1.
2.
3.
4.
5.
6.

Apical
Oblique
Principal
Transverse
Transseptal
Interradicular

NATIONAL

BOARD DENTAL EXAMINATION PART I


ANSWER KEY

DENTAL ANATO~ff & OCCLUSION

NO.

ANS.

1.
2.
3.
4.
5.

3
1
1
2
2

26.
27.
28.
29.
30.

6.
8.
9.
10.

4
1
4
4
5

31.
32.
33.
34.
35.

11.
12.
13.
14.
15.

6
5
1
4
5

36.
37.
38.
39.
40.

16.
17.
18.
19.
20.

4
5
2
4
3

41.
42.
43.
44.
45.

21.
22.
23.
24.
25.

3
1
2
4
1

46.
47.
48.
49.
50.

7.

1-

NO.

ANS.

not

DECEMBER

1988

NO.

ANS.

NO.

4
5
1
3
3

51.
52.
53.
54.
55.

3
3
3
5
4

76.
77.
78.
79.
80.

3
1
4
2

3
4
6
4
1

56.
57.
58.
59.
60.

2
3
4
2
2

81.
82.
83.
84.
85.

2
2
2
5
4

scored
4
1
2
3

61.
62.
63.
64.
65.

1
3
3
4
2

86.
87.
88.
89.
90.

3
3
4
3
2

4
2
1
2
3

66.
67.
68.
69.
70.

2
1
5
1
4

91.
92.
93.
94.
95.

4
2
1
4
6

1
4
3
4
1

71.
72.
73.
74.

5
1
4
5
1

96.
97.
98.
99.

2
3
1
5

75.

ANS.
""

,.
:
1

45.

Cells of the stratum granulosum of thick


stratified squamous epithelium
characteristically contain
1.
2.
3.
4.

46.

'Nhich of the following structures is formed


from the first branchial (visceral) arch?
1.
2.
3.
4.
).

melanin granules.
keratin granules.
keratohyalin granules,
granules that are organelles.

A major sensory innervation of the


temporomandibular joint is derived from the
1.
2.
3.
4.

51.

52.

great auricular nerve.


auriculotemporal nerve.
inferior alveolar nerve.
temporal branches of the facial nerve.

Lateral clefting of the lip results from the


failure of merging of
1.
2.
3.
~
"'t.

).

47.

Nasal
Vomer
Ethmoid
Pal.3tine
Sphenoid

53.

2.
3.
4.
5.

facial artery, the superior labial artery and


angular arteries.
lingual artery, the deep lingual artery and
internal nasal arteries.
occipital artery, the ascending palatine
artery and posterior nasal arteries.
maxillary arteryr the sphenopalatine artery
and posterior lateral nasal arteries.
superficial temporal artery, the transverse
facial artery and external nasal arteries.

The impulse-conducting mechanism in the


heart is composed of

54.

55.

Movement of the head about its vertical axis


(rotation to right or left) occurs chiefly at
which of the following joints?
1.
2.
3.
4.

Atlanto-axial
Spheno-occipital
Atlanto-occi pital
Third to seventh cervical vertebrae

phosphatases.
polypeptides.
orthophosphoric esters.
sulfated proteoglycans.

Tropocollagen is a protein molecule found in


1.
2.
3.
4.

56.

Osteocyte
Macrophage
Osteoblast
Mesothel ial cell
Small lymphocyte

The ground substance of hyaline cartilage is


basophilic because it contains
1.
2.,
3.
4.

1. autonomic nerve fibers.


2. bundles of collagenous fibers.
3. reticuloendothelial components.
4. modified cardiac muscle fibers.

50.

Which of the following cells is most likelyto


have an abundant amount of rough-surfaced
endoplasmic reticulum?
1.
2.
3.
4.
).

The external carotid artery contributes to the


blood supply of the nasal cavity by way of the
1.

49.

maxillary with mandibular processes.


maxillary processes with each other.
maxillary and medial nasal processes.
medial and lateral nasal processes.
palatine processes with the nasal septum.

The middle nasal concha is part of which of


the followingbones?
1.
2.
3.
4.
5.

48.'

Hyoid bone
Buccinator muscle
Stylopharyngeus muscle
Lateral pterygoid muscle
Superior parathyroid gland

collagen and elastic fibers only.


elastic and reticular fibers only.
collagen and reticular fibers only.
collagen, elastic and reticular fibers.

Hyaline cartilage differs from bone in that


hyaline cartilage
1.
2.
3.
4.

cells lie in lacunae.


may grow interstitially.
may grow appositionally.
can become more highly calcified.

'..
57.

62.

The most prominent functional component in


the tunica media of small arteries is
1.
2.
3.
4.
5.

elastic fibers.
smooth muscle.
striated muscle.
reticular fibers.
collagenous fibers.

During endochondral ossification of a long


bone of an extremity, the function of hyaline
cartilage is to
1.
2.
3.
4.

58.

The nucleolus produces

1. DNA.
2. transfer RNA.
3. ribosomal RNA.
4. messenger RNA.
5. both (1) and (2) above.

59.

63.

1.
2.
3.
4.

4.
5.

ovary.
kidney.
spleen.
gallbladder.
64.

60.

Most intrinsic muscles of the larynx receive


their motor innervation from
1.
2.
3.

An organ with a retroperitoneal location that


can be approached surgically without violating
the continuity of the peritoneum is the

In the drawing below of a medial view of the


right side of the mandible, which of the
following letters indicates the position of the
bulk of the sublingual gland?

65.

the laryngeal plexus.


the inferior (recurrent) laryngeal nerve.
the external branch of the superior
laryngeal nerve.
the internal branch of the superior
laryngeal nerve.
both external and internal branches of the
superior laryngeal nerve.

In the human embryo, during the third week


of development, the first branchial arch divides
to form the mandibular process and the

1.
2.
3.
4.
5.

increase strength of the developing bone.


provide a region where bone can grow in
length.
provide undifferentiated cells that will
become osteoblasts.
form cores of trabeculae that compose
most compact bone of the diaphysis.

hyoid arch.
thyrohyoid arch.
branchial pouch.
maxi Ilary process.
frontonasal process.

In a relaxed state, transitional epithelium can


be distinguished from stratified squamous
epithelium because transitional epithelium
contains

c
0
1.
2.
3.
4.

61.

1. stratum corneum.
2. flattened superficial cells.
3. dome-shaped superficial cells.
4. more visible ground substance.

A
B
C
D

66.

Which of the followingare mobilized when a


cell produces an excessive amount of protein?
1.
2.
3.
4.
5.

It is possible to distinguish histologically


between the stomach and the duodenum
because of the presence of
1. mucosal glands in the stomach only.
2. muscularis mucosa in the stomach only.
3. simple columnar epithelium lining the
stomach only.
4. submucosal glands in the duodenum only.
5. smooth muscle in the external
musculature of the duodenum only.

Lysosomes
Mitochondria
lipofuscin granules
Rough endoplasmic reticula
All of the above.

pfi

67.

Circular fibers are present in the

1.
2.
3.
4.

68.

72.

free gingiva and encircle the tooth.


free gingiva and encircle alveolar bone.
attached gingiva and encircle alveolar
bone.
periodontal ligament and encircle the
tooth.

Which of the following groups of periodontal


ligament fibers has a cementum-to-cementum
attachment?

1. Oblique
2. Transseptal
3. Free gingival
4. Interradicular
5. Dentoalveolar

1.
2.
3.
4.
5.

73.

crest

Which of the following is necessary for


cementum to be deposited during tooth.
development?

1.
2.
3.
4.

70.

75.

4.

sphenoid and zygomatic bones.


axial skeleton and the base of the skull.
axial skeleton and the flat bones of the
skull.
petrous portion of the temporal bones and
the parietal bones.

During intraoral injection to the mandibular


foramen, the needle passes through the
mucous membrane and the buccinator muscle
and lies

1.
2.
3.
4.
5.

lateral to the neck of the mandible.


medial to the pterygomandibular raphe.
lateral to the pterygomandibular raphe.
lateral to the medial pterygoid muscle.
medial to the superior constrictor muscle
of the pharynx.

Otic
Geniculate
Submandibular
Pterygopalatine
Trigeminal (semilunar)

The alveolar bone proper (cribriform


usually consists of
1.
2.
3.
4.

plate)

woven bone.
bundle bone only.
lamellar bone only.
bundle bone and lamellar bone.

Which of the following structures may be


found within the posterior mediastinum?

1. Heart
2. Phrenic nerve
3. Thoracic duct
4. Arch of the aorta
5. Recurrent laryngeal nerve

A disturbance in cartilage formation in a fetus


results .in deformities of the

1.
2.
3.

71.

The tooth must be erupting.


A bony socket must develop.
An apical vascular plexus must be present.
The continuity of the root sheath
(Hertwig) must be broken.

transversely.
anteroposteriorly.
anteromedially.
posteromedially.
None of the above

Cell bodies of taste fibers from the anterior


two-thirds of the tongue are located in which
of the following ganglia?

1.
2.
3.
4.
S.

74.

69.

The long axes of mandibular condyles intersect


at the foramen magnum indicating that these
axes are directed
.

76.

The optic tracts consist of axons from which of


the following cells?

1.
2.
3.
4.
S.

77.

Rod
Bipolar
Ganglion
Horizontal
All of the above

Gingiva is different from alveolar mucosa in


that gingiva has

1.
2.
3.
4.
5.

elastic fibers.
simple epithelium.
muscularis mucosa.
stratified epithelium.
high connective tissue papillae.

".
.

78.

1.
2.
3.
4.

79.

83.

The large mass of gray matter that bulges into


the floor and the lateral aspect of the lateral
ventricle is the

(a)
(b)
(c)
(d)

hypothalamus.
caudate nucleus.
substantia nigra.
amygdaloid complex.

1.
2.
3.
4.
5.
6.
7.

The large ascending bundle of fibers in the


medulla that is composed of second order
neuron fibers conveying proprioception and
discriminatory touch sensations to conscious
levels is the"
1.
2.
3.
4.

Which of the following increase with age in


dental pulp?

medial lemniscus.
reticular formation.
spinal trigeminal tract.
inferior cerebellar peduncle.

84.

(a) and
(a), (b)
(a) and
(a) and
(b) and
(b) and
(c) and

Which of the following forms of connective


tissues has a preponderance of amorphous
ground substance over fibers?
1.
2.
3.
4.
5.

81.

(b)

(c)

Loose
Cartilage
Reticular
Dense, regularly arranged
Dense, irregularly arranged

(d)

1.
2.
3.
4.

(a)
(a)
(b)
(b)

the formation of cell rests in the


periodontal ligament when the
sheath's functions have been
accomplished.
progressive involution, thereby
aiding in actual tooth movement
during eruption.
the absence of a stellate reticulum
and a stratum intermedium.
the absence of mitotic ability and,
thus, stretching as the root "grows.
and
and
and
and

(c)
(d)
(c)
(d)

In salivary glands, folds of the basal portion of


the cell membrane containing mitochondria
are characteristic of the
1.
2.
3.
4.

cells composing demilunes.


cells of the striated ducts.
cells of the intercalated ducts.
serous cells of the parotid gland.

85.

Obstruction of the cerebral aqueduct causes


enlargement of the
(a)
(b)
(c)

82.

(b) only
and (c)
(c) only
(d)
(c) only
(d)
(d)

The epithelial root sheath (Hertwig) is


characterized by
(a)

80.

Vascularity
Size of the pulp chamber
Number of collagen fibers
Calcification within the pulp

1.
2.
3.
4.
5.
6.

The lamina papyracea is located in the orbital

1.
2.
3.
4.

roof.
floor.
lateral wall.
medial wall.

third ventricle.
fourth ventricle.
lateral ventricle.

(a) only
(a) and (b)
(a) and (c)
(b) only
(b) and (c)
(c) only

II1II
~;

....
86.

What is the usual sequence of events in the


histogenesis of a tooth?
(a)

(b)
(c)
(d)

1.
2.
3.
4.
S.

89.

Deposition of the first layer of


enamel
Deposition of the first layer of
denti n
Elongation of inner enamel
epithelial cells
Differentiation of odontoblasts

(a)
(b)
(c)
(d)
(e)
(f)

1.
2.
3.
4.
5.
6.

a, b, c, d,
c, d, a, b
c, d, b, a
d, a, b, c
d, C,b, a

90.

87.

Which of the followingabdominal viscera are


retroperitoneal?

(a)
(b)
(c)
(d)
(e)

(d)
1.
2.
3.
4.
5.

Medial pterygoid muscle'


Lateral pterygoid muscle
Anterior fibers of the temporal
muscle
Posterior fibers of the temporal
muscle

(a) and (c) only


(a), (c) and (d)
(b), (c) and (d)
(b) and (d) only
All of the above

1.
2.
3.
4.
5.
6.

91.

(a),
(a),
(a),
(b),
(c),
All

1.
2.
3.
4.
5.

Which of the following articulate directly with


the body of the sternum?
(a)
(b)
(c)
(d)
(e)

1.
2.
3.
4.
S.

Clavicle
Eleventh rib
Manubrium
First rib
Xiphoid process

(a), (bYand (d)


(b), (d) and (e)
(c), (d) and (e)
(c) and (e) only
All of the above

92.

(c)
(c)
(d)
(e)
(e)
(e)

and
and
and
and
and
and

(d)
(e)
(f)
(f)
(f)
(f)

Sphenomandibular ligament
Medial pterygoid muscle
Middle meningeal artery
Ophthalmic nerve
Internal jugular vein
(b) and (c)
(b) and (d)
(c) and (e)
(d) and (e)
(d) and (e)
of the above

Which of the followingcell types may be


found in the periodontal ligament?,
(a)
(b)
(c)
(d)

88.

(b),
(b),
(b),
(c),
(d),
(d),

Which of the following strudures may be


found in the infratemporal fossa?

Which of the following muscles or muscle


fibers are elevators of the mandible?
(a)
(b)
(c)

(a),
(a),
(a),
(a),
(b),
(c),

Aorta
Liver
Kidney
Spleen
Pancreas
Suprarenal

Fibroclasts
Osteoblasts
Macrophages
Cementoblasts

(a), (b) and (c)


(a) and (c) only
(b) and (d)
(d) only
All of the above

Each of the following cell types secretes the


substance

1.
2.
3.
4.

with which it is paired

EXCEPT

Sertoli'scells-testosterone.
corpus luteum-progesterone.
alpha cells of the pancreas-glucagon.
chromaffincells of the suprarenalcatecholam

ine.

Part I
December 1985
1.

A section of the posterior lobe of the


hypophysis contains mainly
1.
2.
3.
4.

Anatom ic Sciences

7.

follicles.
alpha and beta cells.
chromophobes and chromophils.
unmyelinated nerve fibers.

The epithelial root sheath (Hertwig) is


essential to development of the root of a
tooth because it
1.

2.
3.

2.

The artery most commonly involved in stroke


is the
1.
2.
3.
4.

lenticulostriate.
anterior cerebral.
posterior cerebral.
middle meningeal.

4.

8.

In skeletal muscle, a triad consists of

1.

3.

4.

The fiber group in the periodontal ligament


that constitutes the main support of the tooth
against masticatory forces is the

2.

1.
2.
3.
4.
5.

4.

Nonarticular surfaces of the


temporomandibular joint are covered with
1.
2.
3.
4.

5.

4.

6.

5.

9.

10.

Heart
Phrenic nerve
Thoracic duct
Arch of the aorta
Recurrent laryngeal nerve

Ascending pharyngeal
Superior thyroid
Maxillary
Facial

The cytoplasm of osteoblasts stains intensely


with basic dyes. This is caused by the
1.
2.
3.
4.

11.

1.
2.
3.
4.
5.

The arterial supply of the submandibular


gland is from which branch of the external
1.
2.
3.
4.

Osteoblasts, osteoclasts, osteoid


Osteoblasts, osteoid, megakaryocytes
Perichondrium, osteoblasts, periosteal
bud
Periosteum, epiphyseal ossification
centers, osteoblasts

Which of the following structures may be


found within the posterior mediastinum?

a "T" tubule and associated


mitochondria.
a "T" tubule and invagination of the
sarcolemma.
a zone of attachment between adjacent
cells.
terminal cisternae and sarcoplasmic
reticulum.
terminal cisternae and a fingerlike
invagination of the sarcolemma.

carotid artery?

periosteum.
fibrocartilage.
elastic carti lage.
hyaline cartilage.

Which of the following elements are present


during development and growth of the
alveolar process?
1.
2.
3.

3.

apical.
oblique.
transverse.
horizontal.
transseptal.

remains as an essentiaJ constituent of the


periodontal ligament.
gives rise to cementoblasts that produce
cementum of the root.
gives rise to odontoblasts that lay down
denti n of the root.
molds the shape of the root and
stimulates differentiation of odontoblasts.

high ribonucleic acid .content.


high content of potassium ions.
increased number of mitochondria.
presence of acidophilic substances.

Which of the following structures crosses the


masseter muscle and pierces the buccinator
muscle?
1.
2.
3.
4.

Parotid duct
Masseteric nerve
Transverse facial artery
Zygomatic muscle branch of the facial
nerve

10

..

91.

All of the following structures are concerned

95.

with development of the tongue EXCEPT the

1.
2.
3.
4.
5.

92.

93.

96.

97.

structures may be found

globular dentin.
the neonatal line.
striae of Retzius.
contour lines of Owen.
odontoblastic processes.

Trachea
Bronchioles
Alveolar ducts
Primary bronchus

Which of the following is NOT a "principal"


collagenous fiber group of the periodontal
ligament?
1.
2.
3.
4.

Apical
Oblique
Horizontal
Gingivodental

Which of the following structures is NOT


found in the substance of the parotid gland?

1. Facial vein
2. Facial nerve'
3. External carotid artery
4. Superficial temporal artery
5. A branch of the great auricular nerve

Parathyroids
Adrenal cortex
Adrenal medulla
Anterior pituitary
Pancreatic islets (Langerhans)

All of the following


in dentin EXCEPT

1.
2.
3.
4.
5.

1.
2.
3.
4.

lingual nerve.
lingual artery.
hypoglossal nerve.
submandibular duct.

The secretion of which of the following


endocrine glands is NOT essential to life?

1.
2.
3.
4.
5.

94.

normally have cilia?


copula.
macula.
tuberculum impar.
second branchial arch.
third branchial arch.

All of the following structures lie superficial


to the hyoglossus muscle EXCEPT the

1.
2.
3.
4.

Epithelium of which of the following


structures of the respiratory system does NOT

98.

Submental lymph nodes receive lymphatic


vessels from all of the following EXCEPT
1.
2.
3.
4.
5.

mandibular molars.
the tip of the tongue.
the middle of the lower lip.
mandibular lateral incisors.
mandibular central incisors.

11

NATIONAL

BOARD DENTAL EXAMINATION

PART

ANSWER KEY

ANATOMIC SCIENCES - DEC. 1985

NO.

ANS.

NO.

ANS.

NO.

ANS.

NO.

ANS.

1.

4
1
2
1
1

26.
27.
28.
29.
30.

5
3
5
4
2

51.
52.
53.
54.
55.

1
4
1
1
5

76.
77.
78.
79.
80.

3
2
3
2
3

10.

3
4
5
4
1

31.
32..
33.
34.
35.

4
2
3
4
5

56.
57.
58.
59.
60.

5
4
4
3
4

81
82.
83.
84.
85.

4
2
2
2
3

11.
12.
13.
14.
15.

1
5
2
1
3

36.
37.
38.
39.
40.

2
4
3
5
1

61.
62.
63.
64.
65.

2
2
3
3
1

86.
87.
88.
89.
90.

1
6
4
4
4

16.
17.
18.
19.
20.

4
2
2
3
4

41.
42.
43.
44.
45.

3
3
4
3
2

66.
67.
68.
69.
70.

2
4
4
1
1

91.
92.
93.
94.
95.

2
2
3
3
3

21.
22.
23.
24.
25.

4
4
3
2
4

46.
47.
48.
49.
50.

3
3
2
1
3

71.
72.
73.
74.
75.

2
4
1
2
5

96.
97.
98.

4
1
1

2.
3.
4.

5.

6.
7.
8.
9.

.,~",.'
~~;f'

Part I
Decem
ber 1985
.

1.

The km value of an enzyme is numerically


equal to

1.
2.

3.
4.

2.

half the maximum velocity expressed in


molesll iter.
velocity of a reaction divided by
substrate concentration.
substrate concentration in moles/liter
necessary to achieve half the maximum
velocity of a reaction.
maximum velocity divided by half the
substrate concentration in moles
necessary to achieve maximum velocity.

The physiologically active form of vitamin D


produced in the kidney is

1.
2.
3.
4.
5.

...

12

Biochemistry-Physiology

6.

The presence of glucose in the urine proves


that the patient has

1.
2.
3.
4.

7.

8.

diabetes mellitus.
negative nitrogen balance.
exceeded his renal threshold for glucose.
excessive damage to a glomerulus of the
kidney:

Carbonic anhydrase in kidney tubular cells is


associated with reabsorption of

1.
2.
3.
4.

1,25-d ihydroxycholecalciferol.
25,.hydroxycholecalciferol.
7-dehydrocholesterol.
cholecalciferol.
ergosterol.

Exam

urea.
chloride.
carbohyd rate.
bicarbonate ion.

An impulse can travel from one nerve to


another in only one direction because the
1.
2.

synapse limits the direction of travel.


myelin sheath limits the direction of
travel.

3.

3.

Which of the following


reduction?

is an example of

-- -

1. -CH "'" CH- + HzO -CHz - CHOH2. R-COOH R-COO- + H +


3. -CHzCHz -CH == CH- + Hz
4. Cu+
Cu+z + e5. Fe+3 + e
Fe+z

--

"

4.

myoneural junction limits the direction


of travel.
4. nerve fiber permits conduction in only
one direction.
5." cell body must be stimulated before the
nerve fiber will conduct.

9.

1.
2.
3.
4.
5.

The ketone body acetoacetate is synthesized


inside mitochondria of hepatocytes by

1.
2.
3.
4.

5.

carboxylation of pyruvic acid.


transamination of. asp"artic acid.
cleavage of fJ-hydroxy-(3-methylglutaryl
CoA.
oxidative decarboxylation of
a-ketoglutarate.
oxidation of l-(3-hydroxybutyrate followed
by deacylation.

The intracellular, "second" messenger for


many peptide and polypeptide hormones is

10.

Human parotid saliva is believed to be


hyposmolar because

1.

2.
3.
4.

5.

Acids found in the citric acid (Krebs) cycle


are

1.
2.
3.
4.
5.

glutamic acid and succinic acid.


glucuronic acid and aspartic acid.
oxaloacetic acid and aspartic acid.
oxaloacetic acid and pyruvic acid.
oxaloacetic acid and a-ketoglutaric acid.

11.

AMP.
ATP.
cyclic AMP.
adenylate cyclase.
a cytoplasmic receptor.

water is reabsorbed by excretory ducts of


the gland.
acinar cells transport water more readily
than sodium ions.
striated duct cells produce excess water
and retain potassium.
reabsorption of water by striated duct
cells is less than reabsorption of sodium.

Which of the following is the major


contributor to colloid osmotic pressure?

1.
2.
3.
4.

Albumin
Alpha globulin
Beta globulin
Gamma globulin

.'

13

12.

The dominant factor controlling absorption of


iron from the gastrointestinal tract is
1.
2.
3.
4.

17.

1.
2.
3.
4.
5.

excretion of iron in the urine.


excretion of iron in the stools.
saturation of mucosal cells with iron.
concentration of the ferrous iron in bone
marrow, spleen and liver.

18.
13.

Spatial summation results from the


1.
2.
3.
4.

convergence of several afferent impulses


on' the same postsynaptic nerve soma.
repetitive firing of discharges (from the
same presynaptic term ina\), eventually
producing an action potential.
exhaustion of the stores of transmitter
substance at the synapse.
elaboration of some inhibitory substance,
such as y-aminobutyric acid, at the
synapse.

Visual purple, rhodopsin, is usually formed


in the

Aldosterone is normally associated with


partial regulation of which of the following
processes?
1.
2.
3.
4.
5.

19.

1.

1.
2.
3.
4.

15.

3.

Two atoms are considered isotopic if


their nuclei contain the same number of
neutrons.
their atomic numbers are the same, but
their mass numbers differ.
their mass numbers are the same, but
their atomic numbers differ.
one is a beta emitter, and the other an
alpha emitter.

The distribution of carbonate within dental


enamel follows the same surface to
dentinoenamel junction patterns as which of
the following?
1.
2.
3.
4.
5.

4.
5.

20.

Lead
Calcium
Fluoride
Strontium
None of the above

A deficiency of vitamin A in a developing


tooth most likely affects the
1.
2.
3.
4.

pulp.
enamel.
dentin.
cementum.

sterol.
glycerol.
lecithin.

sphingosine.

alcohols of high molecular weight.

At rest, the potential difference across the


membrane of which of the following is
greatest?
1.
2.
3.
4.

22.
16.

lack of effect of sex ho.rmones on


somatic tissue.
failure of the gonads to respond to
gonadotroph ins.
inability of the hypophysis to synthesize
gonadotrophins.
lack of hypothalamic stimulation of
gonadotrophin release.
inability of the hypophysis to respond tohypothalamic releasing hormones.

Neutral fats contain mixtures of one or more


fatty acids esterified with
1.
2.
3.
4.
5.

21.

Sodium balance
Gluconeogenesis
Lipid digestion
Protein degradation
Carbohydrate metabolism

The normal delay in sexual development


until puberty is attributed to

2.

14.

lens.
liver.
retina.
adrenals.
vitreous humor.

Odontoblast
Sinoatrial node
Skeletal muscle
Visceral smooth muscle

Antidiuretic hormone acts to


1.
2.
3.
4.

decrease renal filtration fraction.


increase storage capacity of the bladder.
decrease permeability of distoconvoluted
tubules and/or collecting ducts to water.
increase permeability of distoconvoluted
tubules and/or collecting ducts to water.

I "'t

ff

23.

The rapid movement of a substance across a


biologic membrane against a concentration
gradient requires
1.
2.
3.
4.
5.

24.

The main control over body temperature,


located in a portion of the central nervous
system, is the
1.
2.
3.
4.

30.

pons.
cerebellum.
hypothalamus.
medulla oblongata.

The second heart sound is related to


1.
2.
3.
4.
5.

31.

Mucin
Ammonia
Carbonate
Bicarbonate
Amino acids

excitation of atria.
opening of AV valves.
closure of AV valves.
opening of aortic valves.
closure of aortic valves.

In males, growth and development of


secondary sex organs are under direct control
of
1.
2.
3.
.~ 4.

Compared with hemoglobin A, the


substitution of a valine for a glutamic acid
residue in hemoglobin S results from
1.
2.
3.
4.

27.

29.

as free Fe + +.
as free Fe +++.
as ferritin.
associated with transferrin.
associated with ceruloplasmin.

Neutralization of acids by san'va results


mainly from which of the following salivary
contents?
1.
2.
3.
4.
5.

26.

lactic acid
Pyruvic acid
Glutaric acid
a-ketoglutaric acid
3-phosphoglyceric acid

The most abundant form of iron in human


blood plasma is found
1.
2.
3.
4.
5.

With glutamic acid as an amino group donor,


alanine can be synthesized directly from
which of the following acids?
1.
2.
3.
4.
5.

25.

that the substance be negatively charged.


participation of an energy-requiring
active transport system.
that the substance be readily soluble in
the lipid barrier of the membrane.
that the substance be carried across the
membrane by rapid influx of the solvent.
facilitated diffusion of the substance
aided by some binding system in the
membrane.

28.

32.

a genetic mutation.
irradiation of hemoglobin A.
proteolytic action in the liver.
exposure to low oxygen tension.

FSH and lH.


aldosterone.
progesterone.
testosterone.

The relative refractory period of a nerve


corresponds to
1.
2.
3.
4.

sodium permeability changes.


decreased potassium permeability.
increased potassium permeability.
threshold potential that is farther from
zero.

Where is norepinephrine stored?


1.
2.
3.
4.

At preganglionic
endings
At postganglionic
endings
At preganglionic
endings
At postganglionic
endings

sympathetic nerve

33.

The major center in the brain for autonomic


nervous system regulation is the

sympathetic nerve
parasympathetic nerve
parasympathetic nerve

1.
2.
3.
4.
5.

hypothalamus.
pituitary.
cerebral cortex.
vestibular nuclei.
floor of the fourth ventricle.

15

34.

Some sense organs, when stimulated


continuously by a constant stimulus, may
gradually lose their ability to continue to
respond. This phenomenon is known as

39.

1.
2.
3.

1. occlusion.
2. . summation.
3. adaptation.
4. facilitation.
5. sensory deprivation.

4.

40.
35.

4.
5.

3.

dilution of a buffer increases its pH.


pH is a function of temperature.
pH = pka when an acid is tenth
normal.
pH = pka when an acid is half
neutralized.
pH is independent of the dissociation
constant of the acid.

Prevents pernicious anemia


Allows adaptation to dim light
Acts as coenzyme in transamination
reactions
Prevents microcytic hypochromic anemia

Carbon monoxide decreases the amount of


1.
2.

The Henderson-Hasselbalch equation shows


that
1.
2.
3.

Which of the following is a function of


vitamin 86 (pyridoxal phosphate)?

4.

bicarbonate in the blood plasma.


carbonic anhydrase available in the
lungs.
oxygen that can be transported by
hemoglobin.
alveolar surface available for gaseous
. exchange'-

5.

41.

carbon dioxide that can be transported


by the blood.

The most likely cause of an increase in


filtration fraction is

36.

1.
2.
3.
4.
5.

In the absence of compensatory changes, a


drop in blood pressure results from
1.
2.
3.
4.

5.

vasoconstriction.
increased hematocrit.
increased stroke volume.
increased cardiac output.
decreased venous return.

42.
37.

DNA damage by ultraviolet light is due to

3.
4.

molecule.
frequent replacement in the DNA
molecule of purines by pyrimidines.
induction of dimerization by way of
covalent bonds between adjacent
thymine groups.

4.
5.

43.
38.

Sucrose is a glycoside of which of the.


following structures?

1.
2.
3.
4.

4-D-gl ucose-a-D-galactopy ranoside


4-D-g Iucose-fJ-D-galactopyranoside
a-D-gi ucopyranosido-fJ-D-fructofu ranos ide
fJ-D,..glucopy ra nos ido-fJ-D-fructofu ran os ide

elevation of ureteral pressure.


efferent arteriolar constriction.
afferent arteriolar constriction.
elevation of rerlaL-vein pressure.
elevation of plasma protein
concentration.

The two chains-of double stranded DNA are


so arranged that
1.
2.
3.

1. alkylation of the guanine in DNA.


2. excessive unwinding of the DNA

they are parallel.


heat does not affect interchain bonds.
hydrophobic aromatic nitrogen bases are
held close to each other.
purines and pyrimidines alternate along
each chain.
an adenine residue on one chain is
directly hydrogen bonded to. a cytosine
on the other chain.

A deficiency of rhodopsin is most likely


caused by decreased dietary intake of
1.
2.
3.
4.
5.

glucose.
adenine.
vitamin A.
tryptophan.
riboflavin.

"

, -

16

II

44.

49.

Removal of a molecule of water between the


carboxyl group of one amino acid and the
amino group of a second amino acid results
in formation of a

1.

the bicarbonate/carbonic
acid buffer
system is very efficient at the pH
optimum of this sensitive enzyme
. system.
2. bicarbonate is a positive effector for this
system and favorably alters the
conformation of its enzymes.
3. carbon dioxide is incorporated into
acetyl coenzyme A forming malonyl
coenzyme A, an intermediate in the
synthetic process.
4. ~ carbon dioxide is incorporated into
carbamyl phosphate, a reactive
intermediate in the synthetic process.
5. carbon dioxide provides an anaerobic
environment that prevents oxidation of
the sulfhydryl groups in the reactive sites
of the enzyme system.

1. zwitterion.
2. peptide bond.
3. hydrogen bond.
4. glycosidic bond.
5. hydrophobic bond.

45.

The primary effect of calcitonin


1.
2.
3.

4.

46.

is to

increase bone resorption.


inhibit bone resorption.
increase calcium absorption from the
intestine.
decrease calcium absorption from the
intestine.

The belief that the secretion of saliva is an


active process is supported by the
observation that

1.

2.
3.

4.
5.

50.

during secretion, there is vasodilation


within the gland.
during secretion, blood flow through the
gland is unchanged.
during secretion, blood flow through the
gland is decreased.
secretion continues even when the
pressure within the salivary duct is
higher than the blood pressure.
secretion stops if the pressure within the
salivary duct is the same as the blood
pressure.

48.

51.

Which of the following enzymes catalyzes


the formation of uric acid from purines?

1.
2.
3.
4.
5.

1.
2.
3.
4.
5.

myotatic reflex.
clasp-knife reflex.
monosynaptic reflex.
lateral inhibition.
reciprocal inhibition.

52.

equal to the clearance of urea.


equal to the clearance of inulin.
less than the. clearance of inulin.
greater than the. clearance of inulin.
equal to the glomerular filtration rate.

Energy for ATP synthesis is derived from the


electron transport system by which of the
following processes?

1.
2.
3.
4.
5.

Urease
Uricase
Xanthine oxidase
Aspartate transcarbamylase
Carbamyl phosphate synthetase

The process by wh ich extensor muscles


acting at a particular joint are inhibited by
stretch of the flexor muscle acting at the
same joint is

Any substance that is filtered by glomeruli


and secreted, but not reabsorbed, by renal
.tubules will have a renal clearance that is
1.
2.
3.
4.
5.

47.

Carbon dioxide or bicarbonate is required in


the biosynthesis of fatty acids because

Transam i nation

Aldolization
Reductive synthesis
Oxidative deamination
Oxidative phosphorylation

Which of the following statements is true


concerning fatty acid synthesis and fatty acid
breakdown?
1.
2.
3.
4.

Both are mitochondrial functions.


Both require phosphopantothenic acid.
Both share common electron donors and
acceptors.
Both share common enzymes for
oxidation and reduction reactions.

III

17

53.

The function of rhythmic segmentation is


chiefly that of

1.
2.
3.
4.

58.

The gallbladder is caused to contract by thE


hormone

1.
2.
3.
, 4.
5.

macerating food.
mixing food with digestive secretions.
increasing the surface available for
absorption.
transporting material down the
gastrointestinal tract.

59.
54.

1.
2.
3.
4.

Cytoplasm
Ribosomes
Membranes
Chromosomes

60.
55.

The partial pressure of oxygen in arterial


blood is lower than that in alveoli. The
condition suggested is

Lipids are most characteristic of which of the


following cellular components?
1.
2.
3.
4.

Which of the following is true of the T m


(melting temperature) of a given DNA double
helix?
1.
2.
3.
4.

Is a function of the base composition


Can be used to accurately predict its
molecular weight
Can be measured by observing the
change in chemical composition
Is higher if individual strands of the
DNA double helix are parallel rather
than anti parallel

5.

hyperventilation.
slow blood flow.
high blood pressure.
inadequate ventilation.
thickening of the alveolar membrane.

Which of the following tissues is primarily


responsible for formation of urea?
1.
2.
3.
4.

61.

gastri n.
secretin.
pancreozym in.
enterogastrone.
cholecystokinin.

Liver
Brain
Kidney
Muscle

.. ..".~

-.

In which of the followjOg situations is an


osmotic diuresis observed?

1.
2.
3.
4.

During diabetes mellitus


During water depdvation
Following ingestion of starch
Following injection of Pitressin@
.,-. .

56.

A submaximal direct stimulus to a skeletal


muscle causes

1.
2.
3.
4.
5.

62.
contraction of no fibers.
contraction of some fibers.'
partial contraction of all fibers in the
muscle.
less contraction than with a subminimal
stimulus.
more contraction than with a maximal
stimulus.

1.
2.
3.
4.
5.

63.
57.

Over a period of time, a man uses 4 liters of


oxygen and produces 3 liters of CO2, What
is his RQ for that period?
1.
2.
3.
4.

0.75
1.00
1.33
7.00

Which of the following causes some degree


of temporary alkalosis?

Hyperventilation
High fluid intake
Excessive smoking
Severe muscular effort
Ingestion of ammonium chloride

of the following enzymes is involvec


in the termination of neuromuscular

Which

transmission?

1.
2.
3.
4.
5.
6.

Choline acetylase
Monoamine oxidase
Methyltransferase
Adenylate cyclase
Acetylcholine esterase
Catecholam i ne-ortho-methyltransferase

II

64.

A muscle devoid of tonus is


1.
2.
3.
4.

65.

2.
3.
4.

66.

2.
3.
4.

5.

A carbon atom with four identical


groups attached to it
A carbon atom with four different groups
attached to it
A carbon with at least one carboxyl and
one amino group attached to it
A carbon atom that has two heavy
groups on one side and two light groups
on the other

70.

6.

71.

Potassium ion is both secreted and


reabsorbed by the nephron.
Antidiuretic hormone alters permeability
of the tubular cell membrane.
Glomerular filtration rate is determined
by plasma clearance of inulin.
Increased concentration of colloids in
plasma diminishes the formation of
fi Itrate.
The afferent arteriole is greater in
diameter than the efferent arteriole.

4.

73.
68.

Which of the following is most likely to


promote depolymerization of the ground
substance?
1.
2.
3.
4.
S.

hypoxia.
propylth iouraci I.
exogenous thyroid hormone.
high serum iodide concentration.
prolonged exposure to a cold
environment.
prolonged exposure to a warm
environment.

Increases.
decreases.
remains unchanged.

Which of the following vitamins can be


supplied to humans by the normal action of
intestinal flora?
1.
2.
3.
4.
5.

after translation.
in the mitochondrial matrix.
before formation of their respective
amino acyl-t-RNXs.
while proline or lysine is bound to the
peptidyl (P) site on the ribosome.

by

Following exercise, ventilation is in excess of


the metabolic requirement. During this
period, the level of lactic acid in blood
1.
2.
3.

Hydroxylation of proline and lysine during


collagen biosynthesis occurs
1.
2.
3.

on an RNA template.
on rough endoplasmic reticulum.
from methionine.
from progesterone.
from polyunsaturated fatty acids.

Thyroid secretion is stimulated

1.
2.
3.
4.
5.

72.
67.

cells

characterizes an

Which of the following statements supports


the observation that the glomerulus acts like
a simple physical microfilter?
1.

Prostaglandins are made within

1.
2.
3.
4.
5.

atonic.
spastic.
hypotonic.
hyperton ic.

Which of the following


asymmetric carbon?

1.

69.

A
D
E
K
None of the above

Some amino acids need not be present in


the diet of an animal because of the animal's
ability to synthesize the acids at an adequate
rate. A principal source of carbon for these
amino acids is

Cortisone
Collagenase
Chymotrypsin
Hyaluronidase
Ammonium phosphate

1.
2.
3.
4.

nucleic acids.
carbon dioxide.
metabolism of carbohydrates.
methylene folic acid derivatives.

19
~

74.

Distribution of fluids between intracellular


and extracellular compartments is mainly
achieved by
1.
2.

3.
4.
5.

79.

1. decreased
2.
increased
3. increased
4. . increased

bulk flow across the capillary wall.


unrestricted movement of sodium to
achieve osmotic balance.
unrestricted movement of water to
achieve osmotic balance.
unrestricted movement of both sodium
and potassium to achieve osmotic
balance.
movement of potassium out of the
intracellular compartment to achieve
osmotic balance.

pressu reo

80.
75.

76.

An electrocardiogram showing extra P waves


before each QRS complex indicates
1.
2.
3.
4.
5.

77.

muscle spindles.
gamma efferents.
pacinian corpuscles.
golgi tendon organs.

81.

fibrillation.
atrial bradycardia.
partial heart block.
ventricular tachycardia.
left bundle branch block.

flexor reflex.
stretch reflex.
crossed-extensor reflex.
continuous production of metabolic
products.

The air remaining in alveoli at the end of a


forced maximal expiration is the
1.
2.
3.
4.
5.

tidal volume.
vital capacity.
residual volume.
expiratory reserve volume.
inspiratory reserve volume:

In which of the following parts of the


circulatory system is blood pressure the
lowest?
1.
2.
3.
4.
5.

Aorta
Veins
Arterioles
Capillaries
Large arteries

82.

The plasma membrane can be best described


as
1.
2.
3.

78.

pH.
pH.
hemoglobin concentration.
r?'asma carbon dioxide partial

The tone of the masseter muscle is


maintained by the
1.
2.
3.
4.

The sensory organs concerned with


maintenance of skeletal muscle tonus are
1.
2.
3.
4.

Within physiologic limits, activity of the


respiratory center is decreased directly by

Carbohydrate is stored in the body


principally as

4.

1.
2.
3.
4.
5.

5.

glucose.
maltose.
sucrose.
glycogen.
glycosaminoglycans.

6.

a static trilamellar lipid-protein-lipid


sheath.
a static mosaic composed primarily of
lipids and proteins.
an unorganized zone between cytoplasm
and interstitial fluid.
a membrane whose composition is
essentially constant for all mammalian
cells.
a fluid mosaic composed of neutral fats
and glycoproteins.
a fluid mosaic composed primarily of
phospholipids, cholesterol and
glycoprotei

ns.

12.

In the Pain Gate Theory, a controller system


modulates sensory input so that there is a
selective and integrative action occurring
before impulses reach the first synapse for
onward transmission. The gate controller in
this system is the
1.
2.
3.
4.
5.

13.

and
and
and
and

palatal
parotid
sublingual
submandibular

Colloid in the usual thyroid follicle stains


1.
2.
3.
4.

19.

basophilic.
acidophilic.
positive for DNA.
positive for steroids.

In the photomicrograph below of the


interface of dental pulp and dentin, predentin
is indicated with the letter

Facial
Maxillary
External nasal
Greater palatine
Anterior superior alveolar

The alternate loosening and tightening of a


primary tooth that is about to be shed results
from
1.
2.
3.
4.

16.

18.

Perikymata
Enamel spindle
Gnarled enamel
Granular layer of Tomes

Which of the following arteries is most


concerned with the blood supply to the
upper lip?
1.
2.
3.
4.
5.

15.

Ebner's
Ebner's
Ebner's
Parotid

Of the structures seen on a ground section of


a permanent lateral incisor, which of the
following was formed first?
1.
2.
3.
4.

Which of the following adult salivary glands


are entirely serous?
1.
2.
3.
4.

14.

T-cell.
large A fiber.
small C fiber.
central control.
substantia gelatinosa.

17.

lack of a developing permanent


successor.
labial movement of the developing
permanent tooth.
alternate resorption and apposition of
cementum and bone.
alternate softening and hardening of
alveolar bone proper.

Which of the following structures can be


palpated by way of the external acoustic
meatus?
1.
2.
3.
4.

Zygomatic arch
Mandibular notch
Lateral pterygoid muscle
Posterior aspect of the mandibular
condyle

1.
2.
3.
4.

20.

A.
B.
C.
D.

The middle cranial fossa is separated from


the posterior cranial fossa primarily by the
1.
2.
3.
4.

crista galli.
groove for the transvere sinus.
lesser wing of the sphenoid bone.
petrous part of the temporal bone.

20

.'
83.

Functions of the hexose monophosphate


shunt include production of
(a)

The mechanism of fluoride action in


reducing dental decay involves

(b)
(c)
(d)

(a) only
(a), (b) and (e)
(a), (c) and (e)
(b) and (d)
(c) and (d)

1.
2.
3.
4.
5.
6.
87.

84.

(c)
(d)
1.
2.
3.
4.

(c)
(d)
(c)
(d)

88.

(a) on Iy
(a) and (b)
(a) and (c)
(b) only
(b) and (c)
(c) only
All of the above

The amino acid composition of mammalian


collagen (Type I) is characterized by the
presence of

(e)

The secretory product of which of the


following endocrine organs is controlled by a
pituitary tropic hormone?
(a)
(b)
(c)
(d)
(e)

1.
2.
3.
4.
5.
6.
7.

Adrenal cortex
Thyroid
Parathyroid
Thymus
Pancreas

(a) and (b) only


(a), (b) and (c)
(b) and (c) only
(b), (c) and (d)
(c) and (d) only
(c), (d) and (e)
(d) and (e) only

1.
2.
3.
4.
5.

89.

is."activated

epi neph rine.


glucagon.
insulin.

(a)
(b)
(c)
(d)

85.

(b)
(c)
(d)
(c)
(d)
(d)

Muscle glycogen phosphorylase


by

1.
2.
3.
4.
5.
6.
7.

increasing the force of contraction.


decreasing the force of
contraction.
increasing the rate of contraction.
decreasing the rate of contraction.

(a) and
(a) and
(b) and
(b) and

(a) and
(a) and
(a) and
(b) and
(b) and
(c) and

(a)
(b)
(c)

Ventricular muscle re~ponds to


norepinephrine by
(a)
(b)

an increase in remineralization of
incipient caries.
a reduction of solubility of enamel
as fluoride content increases.
an enhancement of plaque
accumulation.
an increase in carbohydrate
metabolism in oral bacteria.

(a)

glucuronic acid for heparin


synthesis.
NADPH for fatty acid synthesis.
NADP for pyruvate reduction.
D-ribose for nucleic acid synthesis.
ATP for anaerobic muscle
contraction.

(b)
(c)
(d)
(e)

1.
2.
3.
4.
5.

86.

cystei ne.
hydroxyproline and hydroxylysine.
no sulfur-containing amino acids.
glycine, as nearly one-third the
total number of amino acid
residues.
a high content of aromatic amino
acids.

(a), (b) and


(a), (c) and
(b) only
(b), (c) and
(b) and (d)

(d)
(e)
(d)
on Iy

Which of the following molecules is NOT


part of the mammalian cell membrane?
1.
2.
3.
4.
5.

Endotoxin
Cardiolipin
Plasmalogen
Proteoglycan
lipopolysaccharide

21

90.

Which of the following hormones does NOT


induce activation of adenylate cyclase?

1.
2.
3.
4.91.

Insulin
Glucagon
Epinephrine
Parathyroidhormone

Which of the following


monosaccharide?

1.
2.
3.
4.
J.

97.

\Vhich of the following statements is NOT


true concerning fluoride?

1.
2.
3.

NaCI.
H2CO3.
NaHCO3.
Na2HPO4.
NaH2PO4.

4.
:J.

Which of the following compounds does


NOT contain a high-energy bond?

1.
2.
3.
4.
5.
94.

It is excreted rapidly by the kidney.


It is deposited in calcified tissues.
It occurs only in the ionic form in
plasma.
It passes the placental barrier relatively
slowly.
.
At 1 ppm. in water, it is tasteless,
odorless and colorless.

ATP
Acetyl CoA
UDP-glucose
Glucose-6-phosphate
Phosphoenolypyruvate

Which of the following is NOr a cofactor or


a substance involved in hydroxylation of
proline during collagen biosynthesis?

1.
2.
3.
4.

95.

Amylose
Glucose
Fructose
Glyceraldehyde
Glucuronic acid

Chlorapatite
Fluorapatite
Hydroxyapatite
Calcium fluoride

All of the following function in buffer


systemsin the blood EXCEPT
1.
2.
3.
4.
5.

93.

is NOT a

Which of the following is least soluble in


solutions with low pH?

1.
2.
3.
4.

92.

96.

98.

Hydrolysis of nucleic acids is likely to yield


all of the following EXCEPT
1.
2.
3.
4.
J.

Ascorbate
Ergosterol
Ferrous ion
a-ketogl utarate

ri bose.
adenine.
deoxyribose.
acetic acid.
phosphoric acid.

Which of the following statements is NOT


true about ammonia and the a-amino group
of amino acids?
1.

2.

3.

4.

NH + 4 is formed from glutamine in the


kidney.
The amino group in carbamyl phosphate
is directly donated by aspartate in a
transamination reaction.
Cellular levels of ammonia must be
maintuined at low concentrations
because of its toxicity.
Glutamate dehydrogenase can catalyze
the formation of glutamate from
ammonia and a-ketoglutarate using
NADPH as a cofactor.

99.

Which of the following


involved in coagulation
1.
2.
3.
4.

Fe+2
Ca+2
Prothrombin
Fibrinogen

5.

Th rom boplasti

compounds
of blood?

is NOT

"
II

NATIONAL

BOARD

DENTAL

EXAMINATION

ANSWER

KEY

PART

BIOCHEMISTRY - PHYSIOLOGY

- DEC. 1985

ANS.

NO.

ANS.

NO.

ANS.

NO.

ANS.

4.
5.

3
1
5
3
5

26.
27.
28.
29.
30.

1
2
4
3
5

51.
52.
53.
54.
55.

5
2
2
3
1

76.
77.
78.
79.
80.

3
2
4
2
2

6.
7.
8.
9.
10.

3
4
1
3
4

31.
32.
33.
34.
35.

4
3
1
3
4

56.
57.
58.
59.
60.

2
1
5
5
1

81.
82.
83.
84.
85.

3
6
4
1
1

11.
12.
13.
14.
15.

1
3
1
2
5

36.
37.
38.
39.
40.

5
4
3
3
3

61.
62.
63.
64.
65.

1
1
5
1
2

86.
87.
88.
89.
90.

1
1
4
1
1

16.
17.
18.
19.
20.

2
3
1
4
2

41.
42.
43.
44.
I
45.

2
3
3
2
2

66.
67.
68.
69.
70.

4
1
4
5
5

91.
92.
93.
94.
95.

2
1
4
2
2

21.
22.
23.
24.
25.

3
4
2
2
4

46.
47.
48.
49.
50.

4
3
5
3
4

71.
72.
73.
74.
75.

2
4
3
3
1

96.
97.
98.
99.

1
3
4
1

NO.
1.
2.
3.

..
Part I
December 1986

1.

23

Two important factors for initiation


oral streptococci are

1.
2.
3.

of caries by

5.

:\
j

i.

The different types of Streptococcus


pneumoniae are distinguished by
1.
2.
3.
4.

production of protease and production of


acid.
production of dextranase and production
of soluble dextran.
production of collagenase and production

of hyaluronidase.
4.

Microbiology-Pathology Exam

J.

fermentation of mannitol and sorbitol and


production of protease.
synthesis of insoluble dextran and
production of glucosyltransferase.

8.

fermentation reactions.
distinct Hand 0 antigens.
distinct somatic proteins.
distinct capsular polysaccharides.
varying susceptibility to. specific
bacteriophages.

Which of the following most accurately


describes the antibacterial mechanism of
penicillin?

2.

3.

Which of the following viral-associated


enzymes is unique to RNA tumor viruses?

1.
2.

1.
2.
3.
4.

3.

Reversetranscriptase
DNA-dependent

9.

motile.
sensitive to HzOz.
insensitive to HzOz.
able to ferment lactate.
producers of Oz and H2O from HzOz.

Which of the following


prokaryotic cell?
1.
2.
3.
4.

4.

RNA polymerase

Anaerobic bacteria lacking catalase are


generally
12.
3.
4.
5.

4.

Neuramidase
Capping enzyme

is characteristic of a

A common oral manifestation


disease is
1.
2.
3.
4.

5.

10.

Mitochondrion
Nuclear membrane
Single chromosome
Endoplasmic reticulum

Multiple drug resistance is related most closely


to

1.
2.
3.
4.

1.
2.
3.
4.

11.

viruses.
plasmids.
transformati on.
cell chromosomes.

Which of the following is characterized by a


cumulative antimicrobial effect?

12.
3.
4.

1. Iodine
2. Alcohol
3. Cationic detergent
4. Acid glutaraldehyde
J. Chlorhexidine gluconate

12.

melanosis.
xerostomia.
'.-h~_<
glossodynia.
"cotton-wool" bone.
loss of lamina dura.

Hodgkin's disease.
multiple myeloma.
Burkitt's lymphoma.
acute lymphocytic leukemia.'
chronic myelogenous leukemia.

Osteomyelitis

J.

6.

of Addison's

The first human cancer that has been strongly


linked to a virus is

5.

5.

Inhibits DNA synthesis


Inhibits the terminal step in peptidoglycan
synthesis
Inhibits protein synthesis in growing
bacterial cells
.
Disrupts the integrity of the cell
membrane

is most commonly

caused by

Actinomyces bovis.
Borrelia vincentii.
Nocardia asteroides.
Staphylococcus aureus.
Mycobacterium tuberculosis.

Which of the following is elevated in the


serum of patients with prostate cancer?

1.
2.
3.
4.

Acid phosphatase
AI pha-fetoprotei n
Alkaline phosphatase
Carcinoembryonic
antigen

24
ff

13.

Immunosuppressed patients are particularly


prone to develop severe oral disease caused by

19.

1- Candida albicans.
2. Actinomyces israelii.
3. Pneumocystis carinii.
4. Streptococcus mutans.
5. Streptococcus salivarius.

14.

Tetany may result from hypofunction of which


of the following glands?
1.
2.
3.
4.
5.

15.

1.
2.
3.
4.
5.

20.

Thyroid
Hypophysis
Parathyroid
Adrenal cortex
Adrenal medulla

Which of the following is a significant effect of


pheoch romocytoma?

21.

The indiscriminate use of broad-spectrum


antibiotics is contraindicated because they
frequently
12.
3.
4.
5.

23.
17.

A positive skin test in an individual who has


had tuberculosis is an example of
1.
2.
3.
4.

18.

atopy.
autoimmunity.
hypersensitivity.
passive immunity.

Which of the following pneumoconioses is


most often associated with bronchogenic
carcinoma and mesothelioma in man?
1.
2.
3.
4.

Silicosis
Asbestosis
Anthracosis
Berylliosis

Hyperpituitarism
Cushing's disease
Hyperparathyroidism
Paget's disease of bone

lung.
liver.
heart.
brain.
kidney.

A patient is homozygous for a defective gene


that substitutes valine for glutamic acid on the
beta chain of globin. The most probable
diagnosis is
1.
2.
3.
4.

24.

pyemia.
toxemia.
bacteremia.
septicemia.
focal infection.

The ability of mammalian organs to regenerate


after injury is best demonstrated by the
1.
2.
3.
4.
5.

are extremely nephrotoxic.


cause psychogenic symptoms.
produce dependency reactions.
induce anaphylactoid reactions.
i.nterfere with indigenous biota.

infection.
malabsorption.
folate deficiency.
chronic blood loss.
a genetic abnormality.

Nephrolithiasis is most likely to result from


which of the following diseases?
1.
2.
3.
4.

22.

anemia is most often

As a result of dental prophylaxis,


microorganisms around teeth enter the
bloodstream. This condition is an example of
1.
2.
3.
4.
5.

1- Myxedema
2. Acromegaly
3. Glycosuria
4. Hypertension

16.

Microcytic hypochromic
due to

thalassemia major.
thalassemia minor.
sickle cell trait.
sickle cell disease.

Removal of the capsule from an encapsulated


bacterium is likely to result in
1.
2.
3.
4.
5.

loss of viability.
decreased generation time.
failure of the cell to gram stain.
increased susceptibility to mutation.
increased susceptibility to phagocytosis.

25

25.

A 65-year-old man who experiences urinary


retention or difficulty in voiding his bladder
most likely has

1.
2.
3.
4.

26.

31.

1.
2.
3.
-+.

carcinoma of the prostate.


benign prostatic hyperplasia.
malignant neoplasm involving the urethra.
bladder metastasis of bronchogenic
carcinoma.

The absence of cellular differentiation


characteristic of

is

:J.

32.

1. anaplasia.
2. metaplasia.
3. hyperplasia.
4. hypertrophy.
5. inflammation.

27.

The most characteristic finding in intestinal


malabsorption syndrome is

1.
2.
3.
4. .
5.

28.

1.
2.
3.
4.
5.

33.

In transduction, DNA is transferred from donor


cell to recipient cell by
1.
2.
3.
-+.

steatorrhea.

34.

hyaline membranes.
chronic bronchitis.
giant cell arteritis.
interstitial infiltration of PMN's.
thickening of alveolar walls and
hemosiderosis.

a plasmid.
an episome.
a bacteriophage.
purified nucleic acid.

The components in the cell wall of


'vlycobacteriaceae responsible for acid-fastness.
are
1.
J
3.
-+.

mucoproteins.
teichoic acids.
polysaccharides.
lipids and waxes.
magnesium ribonucleates.

A person with anti-A and anti-B isoagglutinins


in the serum belongs to blood group

35.
1.
2.
3.
4.

A.
B.
AB.
O.

Fat embolism
1.
2.
3.
,
~.

.J.

30.

fungi.
bacteria.
mycoplasmas.
rickettsiae.
spi rochetes.

Hypertensive heart disease is consistently


associated with which of the following
anatomic findings?

.J.

29.

and grow

1. Aortic stenosis
2. Tricuspid stenosis
3. Pericardial effusion
-+. Left ventricular hypertrophy
:J. Right ventricular hypertrophy

pain.
melena.
anorexia.
diarrhea.

Chronic passive congestion of the lung is


characterized by

Organisms that exhibit dimorphism


on Sabouraud's medium are

A cigarette-smoking patient notices a gradual


onset of progressive, labored breathing
characterized by prolonged, expiratory effort.
The most likely diagnosis is

1.
2.
3.
4.
:J.

bronchopneumonia.
chronic bronchitis.
pulmonary emphysema.
carcinoma of the lung.
left sided heart failure with pulmonary
congestion and edema.

36.

is most often a sequela of

fractu reo
infection.
contusion.
infarction.
deep laceration.

Which of the following viruses causes two


distinct diseases in different age groups?
1. Influenza
2. Measles
3. Smallpox
-+. Varicella
.J. Newcastle disease

26
..
37.
.

The requirement for an insect vector in the


transmission of human disease is a common
characteristic of all infectious agents included
in the genus
1.
2.
3.
4.
5.

38.

Brucella.
Bedsonia.
Treponema.
Mycoplasma.
Rickettsia.

44.

mutation.
conjugation.
translation.
transduction.
transformation.

45.

4.

46.
40.

Which of the following skin lesions is


generally considered precancerous?
1.
2.
3.
4.

41.

47.
Which of the following is produced by a
gram-positive, spore-forming anaerobic
badlJus?
1.
2.
3.
4.
5.

42.

Cytopathogenic effect in tissue culture cells is


used specifically for identification of certain
1.
2.
3.
4.
5.

48.

Colon
Larynx
Prostate
Pancreas

Multiple
Enlarged
Multiple
Indurated

ulcers
cervical lymph nodes
cutaneous abscesses with sinuses
nodules in the skin of the neck

Which type of chromosome is usually missing


from the cells of persons with Turner's
syndrome?
1.
2.
3.
4.
5.

Thrush
Gas gangrene
Blastomycosis
Actinomycosis
Undulant fever

Adenocarci noma
Bronchiolar carcinoma
Alveolar cell carcinoma
Squamous cell carcinoma
Anaplastic (oat cell) carcinoma

Which of the following are most characteristic


of cervicofacial actinomycosis?
1.
2.
3.
4.

Verruca vulgaris
Actinic keratosis
Intradermal nevus
Seborrheic keratosis

1-2 days.
1-2 weeks.
1-6 months.
1 year.
5 years.

Carcinoma of which of the following organs


has been shown to be hormonally dependent?
1.
2.
3.
4.

young people.
recent recipients of gamma globulin.
recent recipients of whole blood
transfusions.
persons with histories of recurrent
respiratory infections.

An autosome
Chromosome 21
A sex chromosome
A /Iring" chromosome
A Group A chromosome

A summer illness that produces nodular


lesions of the uvula, anterior pillars and the

posterior pharynx is
fungi.
viruses.
anaerobes.
spi rochetes.
spore-formers.

period for hepatitis B

Which of the following is the most common


type of lung cancer?
1.
2.
3.
4.
5.

A very high percentage of cases of hepatitis A


are found in
1.
2.
3.

The usual incubation


virus infection is
1.
2.
3.
4.
5.

Rough pneumococci grown in the presence of


DNA from smooth pneumococci develop
capsules. This is an example of
1.
2.
3.
4.
5.

39.

43.

1.
2.
3.
4.

influenza. .
herpesvirus.
ECHO virus.
coxsackievi rus.

.
27

49.

55.

Antiseptics differ from disinfectC\ntsin that


antiseptics
1.
2.
3.
4.

-\ patient with facial erythema, fever, Raynaud


phenomenon, joint pain and photophobia
probably has

steriIize.
kill pathogens.
are bacteriostatic only.
are appl ied to viable tissues.

1'.

.,
")

J.
J.

50.

Which of the following di"stinguishes a viral


infection from other microbial assaults?

56.
1.
2.
3.
4.
5.

51.

Necrosis
Chronicity
Interferon production
Intracellular infection
Need for an external vector

in which of the following is hyaluronidase


;nvolved?
i
i.

..,
3.

For the majority of individuals, the initial


infection with herpes simplex, virus results in
1.
2.
3.
4.
5.

57.

encephalitis as a young adult.


a dermal rash in childhood.
herpes labialis in puberty.
a subclinical disease.
genital herpes.

,.

.,
_.
J.
.l

Bacterial endotoxins may playa role in


development of periodontal disease due to
1.
2.

3.
4.
,..
J.

53.

58.

high amounts of endotoxin in plaque.


thE;ability of endotoxins to incite an
inflammatory response.
the fact that endotoxins are similar to
ground substance.
.
the presence of endotoxins in both
gram-positive and gram-negative bacteria.
none of the above.

54.

.,
5,

59.

5.

~
~.

60.

4.
5.

bone.

an increased number of bacteria in saliva.


an increase in salivary hyaluronidase.
an increase of neutralizing antibodies in
saliva.
new pathogenic strains in the involved
area.
a reduction of inflammatory states with
antibiotic treatment.

lipids.
haptens.
proteins.
carbohydrates.

7he most frequent complication following a


~;ngle transfusion of blood that was
':1adequateJy cross matched is
..

neural lesions.
genital lesions.
cervical carcinoma.
oral and ocular lesions.
none of the above.

The likelihood that oral bacteria play an


important role in gingival inflammation is
evidenced by
1.
2.
3.

subarticular

ligaments.
cartilage.

-\llergens that are usually responsible for


contact hypersensitivity are

5.

vilii.
synovia.
attached
articular

Herpes simplex virus type 1 is clinically most


often associated with
1.
2.
3.
4.

Arthus reaction
Shwartzman phenomenon
Spread of streptococcal infections
Localization of staphylococcal infections

7he earliest changes in rheumatoid arthritis


occur in the

J,

52.

scleroderma.
Paget's disease.
Sjogren's syndrome.
erythema multiforme.
systemic lupus erythematosus.

leukopenia.
infectious hepatitis.
altered clotting time.
agglutination of the recipient's
eryth rocytes.
agglutination of the donor's erythrocytes.

~heumatic fever may leave the heart seriously


camaged because of
1.
")

3.

primary infection of the heart valves with


Group A streptococci.
secondary infection of the heart valves
with Group A streptococci.
damage to the heart valves thought to be
a result of hypersensitivity to hemolytic
staphy Iococci.
damage to the heart valves thought to be
a result of hypersensitivity to Group A
streptococci.

28

"
61.

Opsonization may occur as a component of


the humoral immune response to
1.
2.
3.
4.
5. .

62.

63.

3.
4.

66.

69.

Neutralization and inactivation of bacterial


toxins are accomplished by

70.

Tetanus immunization
Injection of gamma globulin
Transfusion of immune blood
Placental transfer of antibody
Antibody produced during recovery from
measles

serve as a local inflammatory stimulus.


prolong antigen release to
immunocompetent cells in the region.
Both (1) and (2) above
Neither (1) nor (2) above

Diverticulosis
Villous adenoma
Pedunculated adenoma
Meckel's diverticulum
Duodenal peptic ulcer

Development of pseudomembranous colitis is


a major adverse effect of prolonged therapy
with
1.
2.
3.
4.
5.

71.

humoral immunity.
cellular immunity.
serotonin release.
Iymphokine synthesis.
delayed hypersensitivity.

'vVhich of the following is most commonly


associated with development of gastrointestinal
cancer?
1.
2.
3.
4.
5.

.
complexes.

The alum adjuvant employed in many vaccines


acts biologically in tissues to
1.
2.

68.

1.
2.
3.
4.
5.

Which of the following is an example of


naturally-acquired passive immunity in
humans?
1.
2.
3.
4.
5.

65.

pure lipids.
pure proteins.
pure carbohydrates.
mucoprotein complexes.
lipoprotein-polysaccharide

Which of the following cells are increased in


number in the bloodstream in allergy or
parasitic infection?
1.
Basophils
2.
Eosinophils
3.
Plasma cells
4. . f\typical lymphocytes
5.
Nucleated eryth rocytes

IgA
IgD
IgE
IgG
IgM

Most bacterial endotoxins are composed of


1.
2.
3.
4.
5.

64.

virulent Streptococcus pneumoniae.


intracellular microorganisms.
infections by viruses.
nucleic acids.
none of the .above.

Antibodies of which class are likely to be


associated with atopic allergy?
1.
2.
3.
4.
5.

67.

vancomycin.
clindamycin.
tetracycl ine.
streptomycin.
cephalosporin.

The aerosol produced during operative dental


procedures is likely to contain a predominance
of
1.
2.
3.
4.
5.

yeasts and spi rochetes.


spi rochetes and rod forms.
spirochetes and rickettsiae.
gram-positive microorganisms.
gram-negative rod forms and spi rochetes.

Recurrent herpes labial is occurs in people who


1.
2.
3.
4.

are hypersensitive to herpesvirus.


have been infected with herpesvirus and
who have antibodies against the virus.
have been infected with herpesvirus but
who fail to produce antibodies against the
virus.
None of the above

72.

Large wound defects heal by


1.
2.
3.
4.
5.

resolution.
regeneration.
wound contraction.
primary intention.
secondary intention.

29
73.

A lesion that is characterized microscopically


in the tissues by central necrosis surrounded
by macrophages, lymphocytes, plasma cells
and occasional giant cells is generally
classified as .

1.
2.
3.
4.
5.

74.

77.

80.

Immersion 'of instruments in cold disinfectants


for 30 minutes is expected to destroy
(a)
(b)
(c)
(d)

a sign is to disease.
a lesion is to disease.
a symptom is to disease.
pathogenesis is to disease.
an etiologic agent is to disease.

1.
2.
3.
4.
5.

81.

Russell bodies
Periarteritis nodosa
Aschoff bodies in the heart muscle
Monckeberg's sclerosis of the aorta

Clotting of blood is delayed in which of the


following conditions?
(a)
(b)

1.
2.
3.
4.

(e)
(0

Hyperparathyroidism
Granulocytic leukemia with
thrombocytopenia
During treatment with dicumarol
hepari n
Tetany
Alcoholic cirrhosis
Rickets
Microcytic anemia

(d)

lipping.
a pannus.
pseudoarthrosis.
Heberden's nodes.

(g)
1.
2.
3.
4.
J.

affects

lung.
heart.
liver.
spleen.
kidney.

Which of the following is the lymphoid cell


type probably responsible for initial immune
recognition of an antigen?

1. Neutrophil
2. Plasma cell
3. Immunoblast
4. B-Iymphocyte
5. T-Iymphocyte

streptococci.
hepatitis B virus.
bacterial spores.
staphylococci.

(a), (b) and (c)


(a) and (c) only
(a), (c) and (d)
(a) and (d) only
(b) and (c) only

(c)

Anoxia most seriously and immediately


the brain and the

82.

agent

formaldehyde.
ethylene oxide.
carbon dioxide.
hydrogen sulfide.

Inadequate immobilization after a bone


fracture is likely to result in

1.
2.
3.
4.
5.

78.

1.
2.
3.
4.

Which of the following histopathologic


findings during necropsy suggests that the
person died of, or at least had, rheumatic
fever?
1.
2.
3.
4.

76.

The most reliable gaseous sterilizing


available for dental instruments is

Leukocytosis is to acute appendicitis as

1.
2.
3.
4.
5.

75.

a cyst.
a phlegmon.
a granuloma.
an acute abscess.
an autoimmune disease.

79.

(a),(b)

and (e)

(b), (c) and (e)


(b), (c) and (f)
(b), (c) and
(d), (e) and

(g)
(g)

Which of the following


endogenous pigments?
(a)
(b)
(c)
(d)
(e)

Lead
Melanin
Hemosiderin
Bismuth
Bilirubin

1.
2.
3.
4.

(a),
(a),
(b),
(b),

,).
6.

(c), (d) and (e)


All of the above

,..

(b)
(d)
(c)
(c)

and
and
and
and

(c)
(e)
(d)
(e)

are examples of

or

21.

The normoblast is a developmental stage of

26.

Which of the following has no lymph sinuses


and is surrounded partly by connective tissue
and partly by epithelium, the latter forming
deep infoldings?

the

1.
2.
3.
4.

22.

3.
4.
5.

24.

27.

ventral surface of the brain stem.


ventral surface of the cerebral
hemisphere.
lateral surface of the cerebral
hemisphere.
anterior region of the medial surface of
the cerebral hemisphere.
posterior region of the medial surface or
the cerebral hemisphere.

1.
2.
3.
4.

28.

3.
4.
J.

29.

2.
3.
4.

dissolution without contribution to any


definitive structure.
dissolution with minor contribution to
ossification.
an active role in ossification of the
mandibular body.
an active role in ossification of the
alveolar process.

4.
J.

30.

1.
2.

4.

The long axes of mandibular condyles


intersect at the foramen magnum indicating
that these axes are directed
1.
2.
3.
4.
5.

transversely.
anteroposteriorly.
anteromedially.
posteromedially.
None of the above

31.

hyoid arch.
thyrohyoid arch.
branchial pouch.
maxillary process.
frontonasal process.

The sphenomandibular
to the

3.

25.

alveolar crest.
probe-depth measurement.
base of the histologic sulcus.
degree of periodontal health.
arrangement of supraalveolar fibers.

In the human embryo, during the third week


of development, the first branchial arch
divides to form the mandibular process and
the
1.
2.
3.

1.

stratum corneum.
flattened superficial cells.
dome-shaped superficial cells.
more visible ground substance.

The free gingival groove is related to the


1.
2.

The fate of Meckel's cartilage is believed to


be

Thymus
Peyer's patch
Lingual tonsil
Palatine tonsil
Pharyngeal tonsil

In a relaxed state, transitional epithelium can


be distinguished from stratified squamous
epithelium because transitional epithelium
contai ns

Tomes' fibers.
principal fibers.
haversian fibers.
Sharpey's fibers.
interstitial fibers.

The middle cerebral artery is distributed on


the
1.
2.

1.
2.
3.
4.
J.

e ryth rocyte.

When a tendon or a ligament is attached to


bone, the attaching fibers are
1.
2.
3.
4.
5.

23.

monocyte.
lymphocyte.
eosinophil.

ligament is attached

lesser wing of the sphenoid bone and


the neck of the mandible.
spine of the sphenoid bone and lingula
of the mandible.
spine of the sphenoid bone and the
angle of the mandible.
None of the above

After exertion, an athlete is puffing. Which of


the following paired muscles helps maintain
a wide airway through the larynx?
1.
2.
3.
4.

Cricothyroid.
Aryepiglottic
Thyroarytenoid
Posterior cricoarytenoid

30

83.

Edema may result from(a)


(b)
(c)
(d)
(e)

1.
2.
3.
4.
5.

increased
increased
increased
decreased
decreased

86.

sodium retention.
lymphatic drainage.
venule blood pressure.
capillary permeability.
plasma osmotic pressure.

(a)
(b)
(c)
(d)
(e)

(a), (b) and (d)


(a), (c) and (d)

1.
2.
3.
4.
5.
6.

(a), (c) and (e)


(b), (c) and (e)
(b), '(d) and

(e)

87.

84.

(d)
(e)

1.
2.
3.
4.
5.
6.
7.
8.

(a),
(a),
(a),
(a),
(a),
(b),
(b),
(c),

overproduction of a hormone.
direct invasion into a blood vessel.
compression of an adjacent vital
organ.
secondary spread to vital organs,
such as the liver.
hemorrhage following ulcerations of
an overlying mucosal surface.
(b)
(b)
(c)
(c)
(d)
(c)
(d)
(d)

and
and
and
and
and
and
and
and

1.
2.
3.
4.
5.

88.

1.
2.
3.
4.

(a)
(a)
(b)
All

production of Iymphokine.
direct cell-mediated cytotoxicity.
helper activity to trigger
B-Iymphocytes to divide and secrete
antibodies.
and (b)
and (c)
and (c)
of the above

(a),
(a),
(a),
(b),
All

(b)
(d)
(c)
(e)

Smoking
Hypertension
Diabetes mellitus
High serum cholesterol

1.
2.
3.
4.
5.
6.

of a duodenal peptic

(b)
and
(c)
and
(d)
(c)

only
(d)
only
(d)
only

Infectious mononucleosis
well-defined hematologic
of

1.
2.
3.
4.
5.
6.
7.

'

stenosis.
pernicious anemia.
mqJignant transformations.
perforation with peritonitis.

(a) and
(a), (b)
(a) and
(a), (c)
(a) and
(b) and

(a)
(b)
(c)

levels

(b) and (c)


(b) and (d)
(c) and (d)
(c) and (d)
of the above

Major complications
ulcer include
(a)
(b)
(c)
(d)

89.

(a)
(b)
(c)

and
and
and
only
and
only

(a)
(b)
(c)
(d)

(c)
(e)
(d)
(e)
(e),
(d)
(e)
(e)

Responses by T-Iymphocytes to viral infections


include

(a)
(a)
(b)
(c)
(c)
(d)

Hard chancre
Soft chancre
Maculopapular rash
Extensive CNS involvement
Mucous patches in the oral cavity

Which of the following are important etiologic


factors in arteriosclerosis?

Benign neoplasms may cause a serious threat


to a patient's well-being or, in some cases,
death by
(a)
(b)
(c)

85.

The secondary stage of syphilis may be


manifested in which of the following forms?

is characterized by a
picture that consists

atypical lymphocytes.
lymphocytic leukocytosis.
a positive heterophile test.

I
I

(a)
(a)
(a)
(b)
(b)
(c)
All

only
and (b)
and (c)
only
and (c)
only
of the above

'",

31

90.

Abnormalities in cell growth that cause cells,

95.

tissues or organs to be smaller than normal


include

\ \ hich of the following is least I


p~oduce acute abdominal sympt
1.

(a)
(b)
(c)
(d)
(e)

1.
2.
3.
4.
5.
6.

91.

..,
.).

-t

hyperplasia.
hypoplasia.
hypertrophy.

(a), (b),
(a), (b)
(a), (b)
(b) and
(b) and
(c) and

(c) and (d)


and (d) only
and (e)
(d) only
(e) only
(e)

96.

J.

5.
5.
5.
5.
5.

-l.
2.
3.
4.
5.

Trisomy 18
Down's syndrome
.., Neu rofi bromatosi s
5.
""T. Klinefelter's syndrome
I

97.

3.
4.

94.

98.

Delayed reaction
Immediate reaction
Passive transfer by serum
Participation of antibodies
Smooth muscle spasm with capillary
damage

:>.

Gallstones
Hepatitis A
Hemolytic anemia
Vitamin K deficiency
Carcinoma of the common bile duct

\\ lich of the following


\:amin D deficiency?
Rickets
Osteomalacia

3.
4.

Rachitic rosary
tv\etastatic calcification

\\ nich of the following


rTetastasize?
1.
2.
3.
.1.
J.

100.

does NC

2.

1.

Candida albicans
Actinomyces israelii
Histoplasma capsulatum
Blastomyces dermatitidis

jaundice does NOT result from which or the


following?
1.
2.
3.
4.

Treponema
Bacteroides
Actinobacillus
Fusobacterium
ivlycobacteria

is NOT associated with

Microorganisms of which of the following


species do NOT exhibit dimorphism?
l.
2.

\\ !lich of the following organisIT


li~e!y to be found among norma
flora of the gingival sulcus?
1.
..,
3.
.1.
J.

99.
93.

cannot t

karyotyping?

1.

mutans
sanguis
pyogenes
salivarius
mitior IS. mitis)

Which of the following


anaphylaxis?

Cholelithiasis
Acute pancreatitis
Carcinoma of the ampulla c
Carcinoma of the tail of the

\ \ ;,ich of the following

b.

Which of the following species of streptococci


is usually NOT found in human dental plaque?

1.
2.
3.
4.

92.

metaplasia.
atrophy.

is least Ii

Neuroblastoma
Chondrosarcoma
Epidermoid carcinoma
Basal cell carcinoma
Mucinous adenocarcinoma

r.emoptysis is characteristic
foilowing EXCEPT
l.
2.
3.
4.

:>.

of e,

emphysema.
tuberculosis.
lobar pneumonia.
pulmonary embolism.
bronchogenic carcinoma.

32

NATIONAL

BOARD

DENTAL

EXAMINATION

PART

ANS\AJER KEY

MICROBIOLOGY-PATHOLOGY

NO.
-

ANS.

NO.
-

ANS.

NO.

ANS.
-

5.

5
3
2
3
2

26.
27.
28.
29.
30.

1
5
5
4
3

51.
52.
53.
54.
55.

4
2
4
5
5

76.
77.
78.
79.
80.

3
2
5
2
4

6.
7.
8.
9.
10.

5
4
2
1
3

31.
32.
33.
34.
35.

1
4
3
4
1

56.
57.
58.
59.
60.

3
2
2
5
4

81.
82.
83.
84.
85.

2
4
3
4
4

11.
12.
13.
14.
15.

4
1
1
3
4

36.
37.
38.
39.
40.

4
5
5
1
2

61.
62.
63.
64.
65.

1
3
5
4
3

86.
87.
88.
89.

90.

5
5
5
7
4

16.
17.

5
3

2
2

66.
67.

18.

.3

68.

19.
20.

4
3

41.
42.
43.
44.
45.

4
3

69.
70.

2
2
1
2
2

91.
92.
93.
94.
95.

3
1
2
4
4

21.
22.
23.
24.
25.

3
2
4
5
2

46.
47.
48.
49.
50.

3
3
4
4
3

71.
72.
73.
74.
75.

4
5
3
1
3

96.
97.
98.
99.
100.

3
5
4
4
1

1.
2.
3.

4.

.",. '"',

1986

ANS.

NO.
-

-,

- DECEMBER

"

Part I
December 1986

33

All test items relating to occlusion refer to a Class 1


canine and molar relationshipunless otherwise
specified. Terms such as "normal" or "ideal" are
synonymous with the above definition.

1.

Dental Anatomy
5.

In the following drawing of mandibular teeth,


the base of the arrow represents a possible
contacting area and direction for the maxillary
cusp.

Assuming occlusion and alignment are normal,


the arrow on the sketch below represents the
path taken by the
Identify the cusp.
1.
2.
3.
4.
1.
2.
3.
4.
5.

mesiofacial cusp of a second molar.


mesiolingual cusp of a second molar.
distofacial cusp of a second molar.
distolingual cusp of a second molar.
mesiofacial cusp of a third molar.

6.

2.

Intercuspal position
Protrusive movement
-

lateral movement (working side)

lateral movement (non-working side)


:>. lateral protrusive movement

protrusive.
right lateral, working side.
left lateral, working side.
right lateral, non-working side.
left lateral, non-working side.

7.
3.

Indicate the mandibular movement.


1.
2.
3.
-+.

The-mandibular movement indicated is


1.
2.
3.
4.
5.

Mesiofacial cusp of a first molar


Mesiolingual cusp of a first molar
Distofacial cusp of a first molar
Distolingual cusp of a first molar

Assuming occlusion and alignment are normal,


the arrow on the sketch below represents the
path taken by the

Assuming occlusion and alignment are normal,


the arrow on the sketch below represents the
path taken by the

1.
2.
3.
4.
5.

4.

distofacial cusp of a first molar.


fifth cusp of a first molar.
mesiofacial cusp of a second molar.
mesiolingual cusp of a second molar.
distofacial cusp of a second molar.

The mandibular movement indicated is


1.
2.
3.
4.
5.

1.
2.
3.

protrusive.
right lateral, working side.
left lateral, working side.
right lateral, non-working side.
left lateral, non-working side.

1
""t.

:>.

8.

facial cusp of a first premolar.


lingual cusp of a first premolar.
facial cusp of a second premolar.
lingual cusp of a second premolar.
mesiolingual cusp of a first molar.

The mandibular movement indicated is


1.
2.
3.
-+.
:>.

protrusive.
right lateral, working side.
left lateral, working side.
right lateral, non-working side.
left lateral, non-working side.

..

34

.
9.

In the sketch below, the base of the arrow


represents a possible contacting area for which
of the following maxillary cusps?

14.

Which of the following primary teeth has a


distinctly prominent facial cervical ridge that
makes it uniquely different from other teeth?

1.
2.
3.
4.
5.
1.
2.
3.
4.
5.

10.

Lingual cusp of a second premolar


Distolingual cusp of a first molar
Mesiolingual cusp of a first molar
Mesiolingual cusp of a second molar
Distolingual cusp of a second molar

15.

1.
2.
3.
4.
5.

Cusp-fossa .~
Cusp-marginal ridges
Class I occlusion
All of the above
None of the above

Which of the following primary teeth has the


greatest faciolingual diameter of the crown?
1.
2.
3.
4."

The sketch represents which of the following


concepts of occlusion?

16.

2.
In a protrusive movement, the maxillary right
central incisor ideally occludes with which of
the following mandibular teeth?
1.
2.
3.
4.

Central incisors
Right central incisor only
Right lateral incisor only
Right central and lateral incisors

17.
12.

The tooth most likely to exhibit a lingual


groove that extends from the enamel onto the
cementa I area of the root is a permanent
1.
2.
3.
4.
5.

13.

1.
2.
3.
4.
5.

20%
30%
50%
80%
100%

and two developmental


and three developmental

Three mamelons
lobes

and two developmental

4.

Three mamelons
lobes

and four developmental

In an ideal intercuspal position (centric


occlusion), the cusp of a permanent
mandibular canine opposes the

4.

18.
Approximately what percentage of permanent
root formation is completed at the time the
tooth eru pts?

Two mamelons
lobes
Two mamelons
lobes

3.

1.
2.
3.

maxillary canine.
maxillary second molar.
maxillary central incisor.
maxillary lateral incisor.
mandibular second premolar.

Maxillary canine
Maxillary first molar
Maxillary second molar
Mandibular second molar

A permanent maxillary central incisor usually


has how many mamelons and developmental
lobes?
1.

11.

Maxillary canine
Mandibular canine
Mandibular first molar
Maxillary second molar
Maxillary central incisor

marginal ridges of maxillary premolars.


distal fossa of a max:llary first premolar.
occlusal embrasure between maxillary
canine and first premolar.
incisal embrasure between maxillary
canine and lateral incisor.

Which of the following jaw positions is


determined almost exclusively by tooth
contact?
1.
2.
3.
4.
5.
6.

Terminal hinge
Maximum opening
Postural position
Masticatory cycle
Intercuspal position
Retruded contacting

position

35

19.

A lingual pit is most common on which of the


following teeth?
1.
2.
3.
4.

24.

When an adult with normal occlusion moves


his mandible from right lateral relation to
centric occlusion, which cusp moves between
racial cusps of the maxillary rigH second
molar?

Maxillary canine
Mandibular canine
Maxillary lateral incisor
Maxillary central incisor

1.

Distofacial cusp of the f!1andibular first molar


.

Distal cusp of the mandibular first molar


Mesiofacial cusp of the mandibular
second molar

3.

20.

Number and arrangement of cusps, number


and location of roots, developmental grooves
and oblique ridges are similar in a primary
maxillary second molar and a
1.
2.
3.
4.
5.

-+.

Distofacial cusp of the mandibular


molar

second

J.

Mesiofacial cusp of the mandibular


molar

third

maxillary second premolar.


permanent maxillary first molar.
permanent mandibular first molar.
permanent maxi lIary second molar.
permanent mandibular second molar.

25.

Nhich of the following


,.

Root of a maxillary canine


Root of a mandibular canine

'"\

21 ..

'"\

J.

The dentition of a normally developed


6 Y2-year-old child usually consists of
1.
2.
3.
4.
5.

-'.

The ideal position and height of lingual cusps


of a mandibular first molar accommodate
which of the following?

,.
2.
3.
4.
S.

23.

Centric relation
Working movement
Protrusive position
Non-working movement
Maximum intercuspation

In carving an occlusal amalgam restoration in


a permanent mandibular second molar, which
of the following shapes is the most typical

groove pattern?
1.
2.
3.
4.
5.

Lingual root of a maxillary first molar


Lingual root of a maxillary second molar
Mesial root of a mandibular first molar

8 primary and 8 permanent teeth.


10 primary and 10 permanent teeth.
18 primary and 6 permanent teeth.
20 primary teeth.
permanent teeth -on Iy
26.

22.

is the longest root?

H
L
U
Y
+

Which of the following occurs when the


,::;istofacial cusp of a permanent mandibular
-ight first molar moves through the facial
~roove from the central fossa of the maxillary
-rght first molar?

-'.
-

27.

Direct lateral excursion to the left


Direct lateral excursion to the right
Direct protrusive mandibular excursion
Lateral-protrusive excursion to the left
Lateral-protrusive excursion to the r~ght

',yhich of the following permanent posterior


:eeth has a mesial marginal ridge that is
located more cervical than its distal marginal
ridge?
1.
2.
3.

Maxillary first premolar


Maxillary second molar
Mandibular first premolar
Mandibular second molar

36

28.

The sketch below is an occlusal view of a


segment of a normal dental arch. The tooth
indicated with an arrow is the primary

34.

From a proximal view, which of the following


permanent teeth tends to be positioned in the
arch with its axis most nearly vertical?
1.
2.
3.
4.
:>.

1.
2.
3.
4.
5.

29.

maxillary right first molar.


mandibular left first molar.
maxillary right second molar.
mandibular left second molar.
mandibular right second molar.

35.

Which of the following grooves separates cusp


ridges from marginal ridges?
1.
2.
3.
4.

A mandibular primate space is usually found


in the primary dentition between

1.
2.
3.
4.
5.

36.

central incisors.
central and lateral incisors.
lateral incisor and canine.
canine and first molar.
first and second molars.

The premolar that most frequently


central pit is the

1.
2.

1.
2.
3.
4.

31.

has a single

maxillary first.
maxillary second.
mandibular first.
mandibular second.

4.

37.

1.
2.
3.
4.
j.

maxillary central incisor.


maxillary lateral incisor.
mandibular central incisor.
mandibular first molar.

38.
32.

How soon after eruption of a permanent tooth


is the apex usually fully developed?
1.
2.
3.
4.

33.

horizontal and vertical overlap.


vertical and posterior cusp height.
horizontal overlap and posterior cusp
height.
intercondylar distance and free way space.
intercondylar distance and postural
vertical dimension.
.

Which of the following sketches illustrates the


mesial view of a permanent maxillary first
molar?

Immediately
3 months
2 or 3 years
5 or 6 years

A 1-year-old child is expected~ to have erupted


which of the following primary maxillary and
mandibular teeth?

1.
2.
3.
4.

have heavier root trunks.


tend to have less pronounced cervical
ridges.
have flatter facial and lingual surfaces
from occlusal to cervical ridges.
have thicker enamel compared with the
total bulk of crowns.

Anterior guidance is a result of

The first succedaneous tooth to erupt is a


1.
2.
3.
4.

Supplemental
Developmental
Mesiomarginal developmental
Marginal ridge developmental

Primary molars differ from permanent molars


in that primary molars

3.

30.

Maxillary canine
Maxillary lateral incisor
Maxillary central incisor
Mandibular lateral incisor
Mandibular central incisor

Incisors and canines


Incisors and first molars
Central incisors and canines
Mandibular central and lateral incisors
and maxillary central incisors

1
1.
2.
3.
4.

1
2
3
4

37

39.

In an otherwise normal arrangement, which of


the following teeth are most often in abnormal
relation and contact with adjacent teeth in the
same arch?

1.
2.
3.
4.
5.

40.

41.

Mandibular first premolars


Maxillary first molars
Mandibular second molars
Maxillary lateral incisors
Maxillary second premolars

cingulum of
fifth cusp of
middle lobe
distolingual
molar.

1.
2.
3.
4.

45.

a canine.
a maxillary molar.
of a mandibular canine.
cusp of a maxillary third

During nonmasticatory
usually

swallowing,

When viewed from the occlusal, the


mesiodistal width of the lingual portion of the
crown is generally greater than the mesiodistal
width of the facial portion of the crown in a
permanent
1.
2.
3.
4.

The lingual cusp of a mandibular first


premolar is similar in development to the
1.
2.
3.
4.

44.

teeth are

Which of the following muscles is the prime


mover in effecting a left working movement?
1.
2.
3.
4.

46.

4.

5.

42.

Mandibular molars only


Mandibular second premolars
Mandibular molars and maxillary first
premolars
Maxillary molars and maxillary first
premolars
Maxillary first and second molars and
some maxillary third molars

The largest and longest cusp in posterior teeth


is the
1.
2.
3.

4.

43.

Left lateral pterygoid


Left medial pterygoid
Right lateral pterygoid
Right medial pterygoid

Which of the following teeth in the permanent


dentition normally,have trifurcations?
1.
2.
3.

protruded.
in a working arrangement.
in contact in intercuspal position.
None of the above. This is a nonexistent
act.

maxillary first molar.


mandibular first molar.
maxillary second molar.
mandibular second molar.

mesiolingual cusp of a maxillary first


molar.
distolingual cusp of a maxillary first molar.
mesiofacial cusp of a mandibular first
molar.
distofacial cusp of a mandibular first
molar.

Contraction of the lateral pterygoid muscle


produces

1. initial upward closure of the mandible.


2. forward movement of the condyle from
3.

the articular fossa.


posterior displacement

of the condyle

from the articular eminence.


4.

final forceful closure of the molars


through a bolus of food.

47.

Which of the following jaw positions is


determined almost exclusively by the behavior
of the mandibular musculature?
1.
2.
3.
4.
5.

48."

Postural
Terminal
Intercuspal
Retruded contacting
Protruded contacting

Which of the following primary molars has an


occlusal surface that most often bears the
greatest resemblance to a premolar?
1.
2.
3.
4.

Maxillary first
Maxi Ilary second
Mandibular first
Mandibular second

38

49.

Which of the following roots is most likely to

55.

have a concavity on either or both its facial


and lingual surfaces?

1.
2.
3.
4.
5.
6.

50.

'In the diagram of Posselt's envelope of motion,


maximum intercuspal position is the

1.
2.
3.
4.
51.

1.
2.
3.
4.

56.

lowest point.
most anterior point.
most posterior point.
most superior point.

In which of the following molars is the mesial


fossa most distinctly separated from the
remainder of the occllJsal table by a transverse
ridge?
1.
2.
3.
4.

52.

Root of a maxillary second premolar


lingual root of a maxillary first molar
Mesial root of a mandibCJlar first molar
Distal root of a mandibular second molar
Mesiofacial root of a maxillary first molar
Distofacial root of a maxillary first molar

57.

:).

The oblique ridge on a permanent maxillary


first molar is reduced in height in the center of
the occlusal surface and is nearly level with
the
marginal ridge.
cusp tip of the distofacial cusp.
triangular ridge of the mesiofacial cusp.
None of the above

Because of the presence of a fissured groove,


cavity preparations most frequently need to be
extended from the occlusal surface to the
1.
2.
3.
4.
5.

In the sketch below, the arrows indicate which


of the following mandibular movements?

1.
2.
3.
4.

58.

54.

anodontia.
oligodontia.
microdontia.
hypodontia.

Maxillary first
Mandibular first
Maxillary second
Mandibular second

The glenoid fossa is an oval cavity or a


depression in the

1.
2.
3.
4.

triangular ridge of the mesiofacial cusp.


distal marginal ridge of a mandibular first
molar.
mesial marginal ridge of a mandibular
second molar.
distal marginal ridge of a mandibular
second premolar.

A developmental abnormality characterized by


the presence of fewer than the usual number
of teeth is
1.
2.
3.
4.

1." maxilla, just anterior to the malar process.


2. temporal bone, just anterior to the
auditory canal.
3. mandible, just posterior to the internal
oblique ridge.

53.

Occlusocervically, the height of the mesial


marginal ridge of a permanent mandibular first
molar is the same as the height of the

facial of maxillary molars.


facial of mandibular molars.
lingual of maxillary molars.
lingual of mandibular molars.
lingual of maxillary first premolars.

An anatomic feature that is most'likely


complicate root planing of a maxillary
incisor is
1.
2.
3.
4.

59.

Protrusive
Right lateral, working side
Left lateral, working side
Right lateral, non-working side
Left lateral, non-working side

to
lateral

a root bifurcation.
a mesial concavity.
an enamel projection.
a distolingual groove.

In the triangle formed by the projection of the


orifices of the canals of a maxillary first molar,
the
1.
2.
3.
4.
:).

angle at the distolingualcanal


is obtuse.
angle at the mesiofacial canal is obtuse.
line connecting mesial with distal is
longest.
line connecting mesial with lingual is
longest.
.
line connecting distal with lingual is
longest.

39

60.

Which of the following teeth have sharp


demarcations between pulp chambers and
. pulp canals?

1.
2.
3.
4.

65.

In a healthy state, the height of interproximal


alveolar bone is most directly related to the
1.

Mandibular canines
Maxillary lateral incisors
Maxillary first premolars
Mandibalar second premolars

convexity of facial and lingual surfaces of


the crown.
convexity of proximal surfaces of the
crown.
relationship of proximal contacts
occlusocervically.
relationship of proximal contacts
faciolingually.
relationship of cementoenamel. lines of
adjacent teeth.

2.
3.
-+.
J.

61.

A patient with a paralyzed right lateral


pterygoid muscle is instructed to open his
mouth wide. Which direction will the
mandible take on opening?
1.
2.
3.
4.

66.

To the left
To the right
Straight (without deviation to right or left)
None of the above. The mandible will not
move.

Which of the fol/owing covers the articulating


osseous structures of the temporomandibular
joint?
1.
2.
3.
4.
J.

62.

In an ideal intercuspal position, the cusp tip of


a permanent maxillary canine should contact

1.
2.
3.
4.
5.

63.

In the figure below, which number designates


retruded contacting po~itjon?
.

both mandibular canine and first


premolar.
the mandibular first premolar only.
the mandibular lateral incisor.

34

the mandibular canine only.


no other tooth.

From a facial view, the sketch' below is typical


of the pulpal outline of a permanent

~
1.
2.
3.
4.

64.

67.

Synovial membrane
Hyaline cartilage only
Vascular fibrous connective tissue
Dense avascular fibrous connective tissue
Dense avascular elastic connective tissue

maxillary canine.
mandibular canine.
mandibular first premolar.
maxillary central incisor.

Under normal conditions, the presence of


mamelons in a 14-year-old patient is indicative
of

1. fluorosis.
2. malnutrition.
3. malformation.
4. malocclusion.
5. enamel composition.

1.
2.
3.
4.
5.
6.
7.
8.

68.

1
2
3
4
5
6
7
8

Which of the following premolars frequently


has only one pulp horn?
1.
2.
3.
4.

Maxillary first
Mandibular first
Maxillary second
Mandibular second

.
.

32.

Which of the following


lacking in thin skin?

1.
2.
3.
4.
5.

33.

3.
4.

37.

38.

retention of the stellate reticulum.


adherence of the root sheath to the
dentinal surface.
a break in the root sheath before
formation of dentin.
an epithelial rest that lies in contact with
the dentin of the root.

The arrow in the sketch below points to the

Bifid tongue is a result of lack of fusion of


the

1.
2.
3.
4.
5.

basale
lucidum
spinosum
corneum
granulosum

Accessory root canals can be formed by


1.
2.

34.

Stratum
Stratum
Stratum
Stratum
Stratum

layers is totally

The greater peritoneal sac communicates


with the lesser peritoneal sac by means of
the

1.
2.
3.
4.
5.

39.

1.
2.
3.
4.

35.

41.

Cyclic DNA has been found in or associated


with which of the following organelles?

1.
2.
3.
4.
5.

3.

Centriole
Mitochondrion
Golgi apparatus
Rough endoplasmic reticulum
Smooth endoplasmic reticulum

4.
5.

42.

Dental sac
Dental papilla
Inner enamel epithelium
Outer enamel epithel i um
Epithelial root sheath (Hertwig)

The medial (internal) pterygoid muscle is


attached to the

1.
2.

elastic fibers.
simple epithelium.
muscularis mucosa.
stratified epithelium.
high connective tissue papillae.

thoracic aorta.
ascending aorta.
arch of the aorta.
pulmonary artery.
brachiocephalic artery.

Which of the following is the embryonic


precursor of cementoblasts?
1.
2.
3.
4.
5.

Gingiva is different from alveolar mucosa in


that gingiva has

1.
2.
3.
4.
5.

36.

abducens nucleus.
oculomotor nucleus.
motor nucleus of the trigeminal nerve.
motor nucleus of the hypoglossal nerve.

aortic hiatus.
inguinal canal.
epiploic foramen.
lesser pelvic aperture.
superior pelvic aperture.

Right subclavian and right common carotid


arteries arise from the

1.
2.
3.
4.
5.

40.

copula.
valleculae.
tuberculum impar.
lateral swellings.
None of the above

pterygoid hamulus.
lateral surface of the lateral pterygoid
plate.
medial surface of the lateral pterygoid
plate.
medial surface of the medial pterygoid
plate.
lateral surface of the greater wing of the
sphenoid.

Schwann's cells arise from

1.
2.
3.
4.

glial cells.
spongioblasts.
neural crest cells.
neuroepithel ial cells of the ependymal
zone.

40

.
69.

The facial surfaces of mandibular molars


compared with the anterior border of the
ascending ramus are located
1.
2.
3.
4.

70.

3.
4.
5.

71.

in the middle third.


at the junction of incisal and middle
thirds.
at the junction-ekervical
and middle
thirds.
.
cervical to the junction of incisal and
middle thirds.
incisal to the junction of incisal and
middle thirds.

5.

75.

76.

77.
72.

It is most difficult to obtain close adaptation of


the matrix band at the cervical for an amalgam
restorationc on the

zone of Weil.
dentinoenamel junction.
cementoenamel junction.
primitive epithelial layer.

Identify the permanent anterior tooth that most


frequently exhibits a bifurcated root and
identify the positions of the roots.
1.
2.
3.
4.
5.
6.

First molar
Second molar
First premolar
Central incisor

in the cervical third.


in the incisal third.
in the middle third.
at the junction of cervical and middle
thirds.
at the junction of middle and incisal
th irds.

In a developing tooth, the junction of the


dental papilla and the inner enamel
epithelium becomes the
1.
2.
3.
4.

The facial masticatory mucosa (attached


gingiva). is narrowest on which mandibular
tooth?
1.
2.
3.
4.

When viewed from the facial, the distal


contact of a maxillary canine is located
1.
2.
3.
4.

medial to the border.


lateral to the border.
continuous with the border.
None of the above. Great'individual
variation exists.

From a facial view, mesial and distal contact


areas of mandibular central incisors are located
1.
2.

74.

Maxillary canine-facial
and lingual
Maxillary canine-mesial
and distal
Maxillary lateral incisor~facial and lingual
Maxillary lateral incisor-mesial and distal
Mandibular canine-facial
and lingual
Mandibular canine-mesial
and distal

Sequence the following cusps of a permanent


maxillary second molar from largest to
smallest.

'

Mesiofacial
Distofacial
Mesiolingual
Distolingual

(a)

1.
2.
3.
4.
5.

mesial surface of a maxillary first


premolar.
distal surface of a maxillary first premolar.
mesial surface of a maxillary second
premolar.
distal surface of a mandibular second
premolar.
mesial surface of a mandibular first molar.

(b)
(c)
(d)

1.
2.
3.
4.
J.

73.

The sketch below represents the facial view of


a

as

78.

a,
a,
c,
c,
c,

b,
c,
a,
a,
d,

permanent mandibular right first molar.


permanent mandibular left first molar.
permanent mandibular right second molar.
permanent mandibular left second molar.
primary mandibular right second molar.

d
b
d
b
b

Arrange the three furcations of a maxillary first


molar from closest to farthest from the cervical
line.
Facial
Mesial
Distal

(a)
(b)
(c)

1.
2.
3.
4.
5.

c,
d,
b,
d,
a,

1.
2.
3.
4.
J.

a,
a,
b,
b,
c,

b,
c,
a,
c,
b,

c
b
c
a
a

"

,iI
.j

41

79.

Which of the following statements are


applicable to a maxillary second premolar?
(a)
(b)
.{c)
(d)
(e)
(f)

1.
2.
3.
4.
5.

83.

Usually one root is present.


The cusps may have vastly different
forms.
The root is extremely concave on
the mesial surface.
The crown is extremely concave on
the mesial surface.
There may be a groove on the
mesiolingual aspect of the crown.
Facial and lingual cusps are
approximately the same size.

(a), (c) and (f)


(a), (d) and (f)
(a), (e) and (f)
(a) and (f) only
(b), (d) and (e)

The mesiodistal measurement of the crown is


greater than the faciolingual measurement in
which of the following teeth?
(a)
(b)
(c)
(d)
1.
2.
3.
4.

84.

(a) and (b)


(a), (c) and (d)
(b) and (c)
(b) and (d)

Whid1. of the following are notable differences


between primary and' permanent dentitions?
(a)

80.

Which of the above statements are applicable


to a mandibular first premolar?
1.
2.
3.
4.

(a),
(a),
(a),
(b),

(b)
(c)
(e)
(c)

and
and
and
and

(b)

(e)
(e)
(f)
(f)

(c)

When viewed from the mesial or the distal,


the overall facial outline from cusp tip to root
apex of a mandibular canine. is
(a)
(b)
(c)
(d)

1.
2.
3.
4.
5.

82.

(a)
(a)
(b)
(b)
(d)

(f)

1.
2.
3.
4.
5.

made up of two arcs.


made up of one continuous arc.
different from the outline of a
maxillary canine.
very similar to the outline of a
'maxillary canin~.
and
and
and
and
only

(c)
(d)
(c)
(d)

(a),
(a),
(b),
(b),
(c),

"'"

Enamel is thinner in primary teeth


compared with bulk of the crowns.
Pulp cavities are larger in
permanent teeth.
Roots of permanent teeth are longer
and slimmer.
(b)
(e)
(c)
(c)
(d)

and
and
and
and
and

(c)
(f)
(d)
(e)
(e)
,

85.

The outline below of a pulp cavity indicates


that this tooth probably

The lingual surface of the crown of a


mandibular canine is smooth and
(a)
(b)
(c)

relatively flat in the fossa area.


poorly developed in the marginal
ridge area.
poorly developed in the cinguluf!1
area.

(a)
(b)
(c)
(d)

Cervical ridges of permanent teeth


are more pronounced.
Crowns of primary incisors are
shorter compared with mesiodistal
diameters.
Root trunks of primary teeth are less.

pronounced.
(d)
(e)

81.

Maxillary molar
Mandibular molar
Maxillary canine
Maxillary central and lateral incisors

1.
2.
3.
4.
5.
6.

(a) only
(a) and (b)
(b) only
(b) and (c)
(c) only
AIl of the above

1.
2.
3.
4.
5.

(a)
(a)
(a)
(b)
(b)

has three well-developed lobes.


has experienced trauma.
is that of an old person.
is that of a young person.
and
and
and
and
and

(b)
(c)
(d)
(c)
(d)

42

,'~
'r

86.

When occlusion and alignment of teeth are


normal, facets due to attrition develop in the
(a)
(b)

89.

;;,

The contact area, a self-protective feature of


the dentition, functions to
(a)
(b)
(c)
(d)

facioincisal of a maxillary lateral


incisor.
linguoincisal of a maxillary central
incisor.
facioincisal of a mandibular canine.
linguoincisal of a maxillary canine.
'

(c)
(d)

1.
2.
3.
4;

5.

(a), (b)
(a), (c)
(b) and
(b), (c)
(c) and

and
and
(c)
and
(d)

(e)

1.
2.
3.
4.
5.

(d)
(d)
only
(d)
only

90.
87.

In a normal occlusion, mandibular central


incisors contact maxillary incisors in which of
the following movements?

(a)

(b)

.....

88.

(a)
(b)
(c)
(d)
(e)
(f)

1.
2.
3.
4.
5.
6.
7.

(c)

(a) and (b) only


(a), (b) and (d)
(a) and (d) only
(b) and (c)
(c) only'
(d) only

Which of the following permanent teeth have


contact areas at approximately the same levels
cervicoincisally or cervico-occlusally, on the
mesial and the distal?
Maxillary central incisors
Mandibular central incisors
Maxillary canines
Mandibular canines
Maxillary molars
Mandibular molars

(a) and
(a), (c)
(a), (e)
(b), (d)
(b), (e)
(c) and
(e) and

(b)
and (e)
and (f)
and (f)
and (f)
(d)
(f) only

(a), (b) and (c)


(a), (d) and (e)
(b) and (d)
(c) and (d)
(e) only

Which of the following features are useful in


distinguishing a maxillary second premolar
from a mandibular second premolar?

(a) . Protrusive
(b)
lateral (working)
(c)
lateral (non-working)
(d)
lateral protrusive

1.
2.
3.
4.
5.
6.

(d)

1.
2.
3.
4.
5.

91.

prevent food impaction.


distribute occlusal forces.
protect mucosal tissues.
form embrasures.
stabilize the dental arch.

The tip of the lingual cusp of a


mandibular premolar is closer to the
lingual border of the crown.
The occlusal outline of a
mandibular premolar is more nearly
square.
The root of a mandibular premolar
is more circular in cross section.
Facial and lingual cusps of a
maxillary premolar are more nearly
the same height.

(a) and (b)


(a) and (d)
(b) and (c) only
(b), (c) and (df
All of the above

In an adult, which of the following occur


normally while swallowing?
(a)
(b)
(c)
(d)

1.
2.
3.
4.
5.

(a),
(a),
(a),
(b),
All

Masseter muscles contract.


The suprahyoid group of muscles
relaxes.
Teeth come into occlusal contact.
The tip of the tongue touches the
roof of the mouth.
(b) and (c)
(b) and (d)
(c) and (d)
(c) and (d)
of the above

?;~
.~
-2
oJ

"

43

92.

The design of a restored ocdusal surface is


dependent upon the
(a)
(b)
(c)
(d)

1.
2.
3.
4.
5.

contour
position
amount
rotating
amount
anterior

95.

of the articular eminence.


of the tooth in the arch.
of lateral shift in the
condyle,
of vertical overlap of
teeth.

(a)
(b)
(c)
(d)
(e)

1.
2.
3.
4.
5.
6.
7.

(a) and (c)


(b), (c) and (d)
(b) and (d) only
(c) and (d) only
All of the above

96.

93.

(c)
(d)
(e)

1.
2.
3.
4.
5.
6.

(a),
(a),
(a),
(b),
(b),
All

1.
2.
3.
4.
5.
6.

97.

1.
2.
3.
4.

lingual slope of the facial cusp.


facial slope of the lingual cusp.
facial slope of the facial cusp.
lingual slope of the lingual cusp.

(a), (b) and (c)


(a), (b) and (d)
(c) and (d)
All of the above

(e)

Maxillary central incisors


Maxillary first premolars
Maxillary first molars
Mandibular second premolars

Which of the following permanent anterior


teeth generally have a lingual cervical line
positioned JI'Loreapically than the facial
counterpart?

1.
2.
3.
4.
5.

98.

(b)
(c)
(d)

(a) and (b) on Iy


(a), (b) and (c)
(b) and (c) only
(b), (c) and (d)
(c) and (d) only,
All of the above

(a)
(b)
(c)
(d)
(e)
(t)

In carving the occlusal surface of a permanent


maxillary first molar, occlusal contact should
be anticipated on the
(a)
(b)
(c)
(d)

and
and
and
only
only
and
only

(a)
(b)
(c)
(d)

a sharper facial cusp outline.


a straighter mesial border, when
viewed facially.
contact areas located more incisally.
a more pronounced cingulum.
a cusp tip displaced lingually to a
midline faciolingually.
(b) and (c)
(c) and (d)
(d) and (e)
(c) and (e) .
(d) and (e)
of the above

(a)
(a)
(b)
(c)
(d)
(d)
(e)

Maxillary first molar


Mandibular first molar
Mandibular second molar -.
Maxillary third molar
Mandibular second premolar

Which of the following permanent teeth have


mesial concavities that require special attention
when removing calculus deposits?

A permanent mandibular canine can be


distinguished from a permanent maxillary
canine because, typically, the mandibular
canine has
(a)
(b)

94.

Distolingual cusps of which of the following


posterior teeth may be completely absent?

Maxillary central incisor


Mandibular central incisor
Maxillary lateral incisor
Mandibular lateral incisor
Maxillary canine
Mandibular canine

(a) and
(a), (e)
(b), (c)
(b) and
(e) and

(d)
and (t)
and (t)
(d)
(t) only

Which of the following muscles is NOT an


elevator of the mandible?
1.
2.
3.
4.

Temporal
Masseter
Medial pterygoid
lateral pterygoid

44

99.

Each of the following is a function of the


periodontal ligament EXCEPT
1.
2.
3.
4.
5.

sensory.
nutritional.
supportive via the fibers.
formative via its cellular elements.
maintenance of the epithelial attachment.
'M

45

NATIONAL BOARD DENTAL EXAMINATION

;',"

ANSWER

,.
'

DENTAL ANATOMY

PART

KEY

& OCCLUSION

DECEMBER

1986

'

it
,
0'.
'>
h

NO.

ANS.

NO.

ANS.
-

2
3
2
4
4

51.
52.
53.
54.
55.

2
2.
1
3
4

76.
77.
78.
79.
80.

5
3
3
4
1

31.

56.

81.

4
4
4
4

32.
33.
34.
35.

3
2
1
2

57.
58.
59.
60.

3
4
4
' 3

82.
83.
84.
85.

6
4
3
3

12.
13.
14.
15.

4
4
3
3
3

36.
37.
38.
39.
40.

3
1
1
4
1

61.
62.
63.
64.
65.

2
5
4
4
5

86.
87.
88.
89.
90

4
3
5
2
5

16.
17.
18.
19.
20.

4
4
5
3
2

41.
42.
43.
44.
45.

3
1
2
3

66.
67.
68.
69.
70.

4
4
2
1
5

91.
92.
93.
94.
95.

3
5
4
2
6

21.
22.
23.
24.
25.

3
2
5
4
1

46.
47.
48.
49.
50.

5
1
1
2
4

71.
72.
73.
74.
75.

3
1
1
3
2

96.
97.
98.
99.
100.

4
4
5

ANS.

NO.
-

2.
3.
4.
5.

3
2
3
5
2

26.
27.
28.
29.
30.

6.

7.

NO.

1.

8.
9.
10.
11.

ANS.

I'

:}

43.

Which of the following describes the manner


in which tissue fluid reaches the epithelium
of the skin?

1.
2.

3.
4.

44.

4.

49.

50.

The circulatory system of a near-term fetus


differs from that of an adult in that, in the
fetus, the
1.
2.
3.

51.

52.
4i.

Inorganic crystals in enamel have their long


axes parallel to the rods in the

4.

5.

centers of the bodies of the rods only.


periphery of the bodies of the rods only.
bodies of the rods and deviating
increasingly in the tails.
tails of the rods and deviating
increasingly from the centers of the
bodies to the periphery.
tails of the rods and deviating
increasingly from the periphery of the
bodies to the centers.

Smooth
Cardiac
Skeletal
Smooth
Cardiac

only
only
only
and cardiac
and skeletal

hyaline cartilage.
elastic cartilage.
fibrous cartilage.
fibrous connective tissue.

The papillary layer of the dermis differs from


the reticular layer in that the papillary layer
1.

1.
2.
3.

Maxillary nerve
Zygomatic nerve
lacrimal artery
Maxillary artery

The spheno-occipital synchondrosis in the


midline of the cranial base of a newborn
consists of

1.
2.
3.
4.

ductus venosus carries unoxygenated


blood.
ductus arteriosus carries fully oxygenated
blood.
foramen ovale allows blood to pass
primarily from the right atrium to the left
atrium.

relaxes.
moves inferiorly.
moves superiorly.
does not change position.
compresses thoracic viscera.

Peripherally located nuclei are found in


which of the following types of adult muscle
cells?

1.
2.
3.
4.
5.

Biceps brachii
Triceps brachii .
Brachioradialis
latissimus dorsi
Extensor digitorum communis

It
It
It
It
It

Which of the following structures passes


through the foramen rotundum?

1.
2.
3.
4.

Which of the following muscles serves as the


prime extensor of the forearm?

1.
2.
3.
4.
5.
46.

The genetic make-up of the individual


The nutritional state of the individual
Whether epiphyses of the long bones are
fused
Whether there is a concomitant excess
of thyroxine

During deep inspiration, the diaphragm


behaves in which of the following ways?

1.
2.
3.
4.
5.

In the condition of hypersecretion of growth


hormone, which of the following determines
whether gigantism or acromegaly results?

1.
2.
3.

45.

Arterioles are directly involved in tissue


fluid exchange with the epithelium.
Capillaries. are directly involved in tissue
fluid exchange with the epithelium.
Tissue fluid is obtained through the
ground substance of connective tissue
from arterioles.
Tissue fluid is obtained through the
ground substance of connective tissue
from capillaries.

48.

2.
3.

4.

contains larger blood vessels than the


reticular layer.
has a few elastic fibers, whereas the
reticular layer has no elastic fibers.
is composed of coarse collagenous
fibers, whereas the reticular layer is
composed of predominately fine
collagenous fibers.
is more finely constructed, whereas the
reticular layer contains coarser
collagenous fibers as a component.

6
.

53.

In an adult, cerebrospinal fluid can be


aspirated most safely by inserting the needle
between third and fourth lumbar vertebrae
because
1.
2.
3.
4.
5.

54.

the spinal cord does not extend below


lumbar 2.
the subarachnoid space does not extend
.
below lumbar 4.
there is more space between laminae of
these vertebrae.
there are no important nerves in this part
of the vertebral canal.
there is little danger of entering the
internal vertebral plexus at this level.

58.

1.
2.
3.
4.
5.

59.

The oral mucosa of the cheek differs from


the skin covering the outer surface of the lip
by possessing

60.

thoracic aorta.
musculophrenic arteries.
pericardiacophrenic arteries.
internal thoracic arteries.
superior epigastric arteries.

Which of the following cells contains a large


amount of rough-surfaced endoplasmic
reticulum?
1.
2.
3.
4.
5.

Ependymal cells constitute the tissue that

1. lines the ventricles of the brain.


2. lines the ventricles of the heart.
3. forms a part of the peripheral neuroglia.
4. covers nerve cell bodies in a ganglion.
55.

The upper five or six anterior intercostal


arteries are branches of the

Lymphocyte
Eosinophil
Plasma cell
Skeletal muscle cell
Reticuloendothelial cell

The facial process indicated with the letter A


in the sketch below of the head and neck
region of a human embryo gives rise to the

1. sweat glands in the submucosa.


2. groups of sebaceous glands associated
3.
4.
5.

56.

"".',
J
\
t
,

,.-.
,.~"

"..-"

Long bones of the skeleton increase in length


because of
1.
2.
3.
4.
5.

57.

with hair follicles.


dense, collagenous fibers that immovably
fix it to the orbicularis oris musCle.
a stratified, squamous, keratinized
epithelium with a thicker lamina propria.
a stratified, squamous, nonkeratinized.
epithelium with a thinner lamina
propria.

mitotic division of osteocytes.


mitotic division of osteoblasts.
resorption of primary bone by
osteoclasts.
appositional growth on the cartilaginous
epiphyseal plate.
interstitial growth in the cartilaginous
epiphyseal plate.

1.
2.
3.
4.

61.
Which of the following cells aid in the
immunologic defense of the body?
1.
2.
3.
4.

Mast cells
Giant cells
Neutrophils
Plasma cells

nasal septum.
nasal conchae.
primary palate.
secondary palate.

Polymerization of amino acids to form


procollagen filaments occurs
1.
2.
3.
4.
5.

on ribosomes.
in the golgi apparatus.
on the fibroblast surface.
in rough, endoplasmic reticulum.
after cleavage of the registration peptide.
,.'

r
';'1

:.

.i
(

62.

In salivary glands, folds of the basal portion


of the cell membrane containing
mitochondria are characteristic of the

68.

Multiple root formation in a tooth follows


unequal proliferation of the
1.
2.
3.
4.

1. cells composing demilunes.


2. cells of the striated ducts.
3. -cells of the intercalated ducts.
4. serous cells of the parotid gland.

63.

The cell body of a somatic afferent neuron is


found in the
1.
2.
3.
4.
5.

69.

1.
2.

dorsal horn of the spinal cord.


ventral horn of the spinal cord.
dorsal root ganglion of a spinal nerve.
ventral root ganglion of a spinal nerve.
ganglion of .the autonomic neryous
system.

3.
4.

2.
3.

4.

loose connective tissue and folds of


muscularis externa.
folds of submucosa that contain blood
vessels and Meissner's plexus.
a lamina propria that contains blood
vessels, nerves and lacteals.
connective tissue and two fairly
substantial layers of smooth muscle.

During growth of the mandible, resting lines


are seen in the cortical bone. This
phenomenon is a result of

70.

71.

membrane and the geniohyoid

The mylohyoid muscle functions to


1.
2.
3.
4.
5.

depress the hyoid bone.


raise the floor of the mouth.
approximate faucial pillars.
close the opening to the nasopharynx.
prevent food from entering the larynx.

Infections or neoplasms that spread by


lymphatics from the skin of the angle of the
mouth pass to the
1.
2.
3.
4.
5.

parotid nodes.
cavernous sinus.
pterygoid plexus.
submandibular nodes.
anterior auricular nodes.

facial artery.
maxiIlary artery.
internal carotid artery.
superficial temporal artery.

Eversion of valves of atrioventricular orifices


of the heart is prevented by papillary muscles
and
1.
2.
3.
4.
5.

73.

Thymus
Spleen
Lymph node
Peyer's patch
Pharyngeal tonsil

The pterygoid plexus and its tributaries are


the venous parallel of the
1.
2.
3.
4.

72.

67.

membrane only.
membrane and the genioglossus
.
membrane and the buccinator

In which of the following glands is the blood


supply most isolated from the parenchyma?
1.
2.
3.
4.
5.

1. bone apposition.
2. internal stress in bone.
3. calcification of cartilage.
4. eruption of permanent teeth.
5. endochondral bone resorption.
66.

mucous
mucous
muscle.
mucous
muscle.
mucous
muscle.

The cores of intestinal villi consist of


1.

65.

To expose the submandibular duct by an


intraoral approach, one must cut through

.~

64.

dental sac.
dental papilla.
epithelial rests.
epithelial diaphragm.

the terminal crest.


the ligamentum teres.
the ligamentum venosum.
chordae tendineae cordis.
trabeculae carneae cordis.

A lymph node is characterized by


1.
2.
3.
4.

containing medullary cords.


lacking afferent lymphatics.
having crypts lined with stratified
squamous epithelium.
having efferent lymphatics leaving at
multiple sites from the capsule.

li1
J
1
1J

74.

An organ with a retroperitoneal location that


can be approached surgically without
violating the community of the peritoneum is
the

80.

Considering number and location, the


parathyroid glands are usually
1.
2.

1. ovary.
2. kidney.
3. spleen.
4. gallbladder.

3.
4.
5.

75.

The component of bone tissue that gives


bone tensile strength is the

1.
2.
3.
4.
5.

76.

Cilia
Flagella
Microvilli
All of the above

Which of the following is found in the curve


of the duodenum?
1.
2.
3.
4.
5.

82.

Spleen
Cystic duct
left kidney
Head of the pancreas
Fundus of the stomach

Maxillary teeth are developed from which of


the following embryonic structures?
(a)
(b)
(c)
(d)

fibrocartilage.
hyaline cartilage.
elastic cartilage.
articular disks.

"1.
2.
3.
4.
5.

Arch I
Arch II
Globular process.
lateral nasal process

(a) and
(a) and
(b) and
(b) and
(c) and

(b)
(c)
(c)
(d)
(d)

Blood vessels in interdental papillae


anastomose freely with
1.
2.
3.
4.

79.

81.

Articular surfaces of most diarthrodial joints


are covered by
1.
2.
3.
4.

78.

elastic fibers.
calcified ground substance.
interconnecting canaliculi.
periosteal connective tissue.
collagenous fibrils of matrix.

Which of the following epithelial surface


variations serve primarily to increase the
functional surface area?
1.
2.
3.
4.

77.

single and dorsal to the thyroid gland.


single and embedded in the dorsum of
the thyroid gland.
multiple and embedded in the dorsum
of the thyroid gland.
multiple and just superior to the thyroid
gland.
not related to the thyroid gland.

periodontal vessels only.


interalveolar vessels only.
both periodontal and interalveolar
vessels.
none of the above.

A major tract connecting right and left


cerebral hemispheres is the

1. cingulum.
2. corpus callosum.
3. internal capsule.
4. superior occipitofrontal bundle.

83.

A sinusoidal arrangement of blood vessels is


found in the
(a)
(b)
(c)
(d)

hypophysis.
spleen.
kidney.
liver.

1. (a) and (b) only


2. (a), (b) and (d)
3. (a) and (c)
4. (b), (c) and (d)

84.

Which of the following organs have the


closest anatomic relationship to the right
kidney?
(a)
(b)
(c)
(d)
(e)
(t)

1.
2.
3.
4.
5.
6.

(a),
(a),
(a),
(b),
(c),
(d),

87.

(a)
(b)
(c)
(d)

Colon
Liver
Spleen
Stomach
Duodenum
Pancreas
(b) and
(b) and
(c) and
(d) and
(e) and
(e) and

(e)

1.
2.
3.
4.
5.
6.

86.

Nucleus
Lysosome
Golgi complex
Mitochondrion
Rough endoplasmic

1.
2.
3.
4.
5.

89.

undergo calcification.
form into cementicles.
become fibrous.
form cartilaginous nodules.

(a) and (b) only


(a), (b) and (c)
(b) and (c) only
(b), (c) and (d)
(c) and (d) only

Ligamentous remnants of the fetal circulatory


system persisting in the adult include

1.
2.
3.
4.
5.

90.
.

Clavicle
Eleventh rib
Manubrium
First rib
Xiphoid process

(a), (b) and (d)


(b), (d) and (e)
(c), (d) and (e)
(c) and (e) only
All of the above

(a)
(b)
(c)
(d)
(e)

The fate of the epithelial rests of Malassez is


that they may

1.
2.
3.
4.
5.

primary incisors
permanent canines
permanent first molars
permanent mandibular
.
of permanent first molars

Which of the following articulate directly


with the body of the sternum?
(a)
(b)
(c)
(d)
(e)

reticulum

(a), (b) and (c)


(a) and (c) only
(a) and (d) only
(a), (d) and (e)
(b), (c) and (d)
(b) and (e)

(a)
(b)
(c)
(d)

of
of
of
of

1. (a), (b) and (c)


2. (a), (b) and (d) only
3. (a), (b), (d) and (e)
4. (a), (c) and (d)
5. (b), (d) and (e) only
6. All of the above

(c)
(e)
(d)
(t)
(t)
(t)

Which of the following organelles have a


double-unit membrane?
(a)
(b)
(c)
(d)

Enamel
Enamel
Enamel
Dentin
incisors
Dentin

(e)

88.

85.

Neonatal lines are found in which of the


following?

(a),
(a),
(a),
(b),
(c),

"

ligamentum"
ligamentum
ligamentum
ligamentum
ligamentum

(b)
(b)
(d)
(c)
(d)

and
and
and
and
and

nuchae.
venosum.
arteriosum.
teres of the liver.
teres of the uterus.

(c)
(e)
(e)
(d)
(e)

Which of the following structures of the


infratemporal fossa is NOT found between
medial and lateral pterygoid muscles?

1.
2.
3.
4.

Lingual nerve
Inferio"r alveolar nerve
Inferior alveolar artery
Nerve to the masseter muscle

Anatomic Sciences

Part I

(11)

"

July 1982

"

1.

i
]

6.

adrenal gland secretes epinephrine?

Which of the following are particularly well


developed in cells synthesizing proteins for
secretion?

1.
2.
3.
4.

1. Lysosomes
2. Mitochondria
3. Lipofuscin granules

Whichof the following groups of cells in the

..

Medullary cells
Zona fasciculata cells
Zona glomerulosa cells
Zona reticularis cells
-

4.

Rough endoplasmic reticula

5. All of the above

2.

Whichof the following empty into the right


atrium?

7.

1. Pulmonary and bronchial veins


2. Superior vena cava, coronary sinus and
hemiazygos vein

3. Superior and inferior venae cavae and


4.

3.

1.
2.
3.
4.
5.

coronary sinus
Coronary sinus and pulmonary veins

Surgical excision of the parotid gland


endangers which of the following structures
in addition to the facial nerve?

8.

1. Hypoglossal nerve
2. Motor nerves to the muscles of mastication
3. External carotid artery and
auriculotemporal

nerve

accessory nerve

9.

Which.of the following connective tissue


cells is most concerned with defense against
bacterial invasion?

1.
2.
3.
4.
5.

Which of the following correctly describes


the activity of tooth-forming cells during
the last part of active eruption?

Motor fibers from the mandibular division.


of the trigeminal nerve are distributed to"
which of the following muscles?

1. Stylopharyngeus muscle only


2. Intrinsic muscles of the tongue
3. Musclesof mastication and anterior

Basophil
Eosinophil
Mast cell
Macrophage
Erythrocyte

belly of digastric only

4. Musclesof facial expression and


5.

5.

Parotid gland
Lingual nerve
Internal carotid artery
Pterygomandibular raphe
Tendon of the temporalis muscle

1. Ameloblasts are functioning actively.


2. Odontoblasts are functioning actively.
3. The secondary enamel cuticle is being
formed.
4. Cementoblasts have completed their
work and are no longer functional.

4. Lesser occipital nerve and spinal

4.

A needle for an inferior alveolar nerve


injection mistakenly passes posteriorly
at the levelof the mandibular foramen.
Which of the following structures is it
likely to contact?

Whichof the following elements are


present during development and
growth of the alveolar process?

10.

1. Osteoblasts, osteoclasts, osteoid


2. Perichondrium, osteoblasts,
periosteal bud
3. Osteoblasts, osteoid, megakaryocytes
4. Periosteum, epiphyseal ossification
centers, osteoblasts

Which of the following areas of the cerebral


cortex is concerned with the recognition of
painful stimuli from the teeth?

1.
2.
3.
4.

-1-

posterior belly of digastric


Musclesof mastication, anterior
belly of digastric, tensor tympani,
tensor veli palatini and mylohyoid

Precentral gyrus
Superior temporal gyrus
Postcentral gyrus
Cortex along the lips of the calcarine
fissure

Part I
July 1982

94.

Anatomic Sciences (11)

A layer of cells that seems to be essential


to enamel formation but does not actually
secrete the enamel is

98.

1. vestibular lamina.
2. stratum intermedium.
3. inner dental epithelium.
4. reduced dental epithelium.

95.

1.
2.
3.
4.
5.

closure offoramen ovale.


closure of the ductus venosus.
constriction of the ductus arteriosus.
constriction of the umbi Hcalarteries.
closure of the interventricular foramen.

An upward extension from the thyroid


gland may be identified as
1.
2.
3.
4.

96.

All of the following are immediate changes


that occur in the cardiovascular system at
birth EXCEPT

a remnant of the thyroglossal duct


a pyramidal lobe.
a muscular slip.
any of the above.

99.

All of the following are located in


the nasopharynx EXCEPT the

1.
2.
3.
4.

The osteon is a cylinder of compact bone


composed of

piriform recess.
eustachian canal.
pharyngeal tonsil.
pharyngeal recess.

1. basal laminae.
2. concentric lamellae.
3. interstitial lamellae.
4. circumferential lamellae.

97.

The nephron of the kidney contains all of


the following structures EXCEPT the

1.
2.
3.
4.
5.

collecting tubule.
capsule of Bowman.
descending limb of Henle.
distal convoluted tubule.
proximal convoluted tubule.

100.

Which of the following does NOT occur.


during contraction of the left ventricle of
a normal heart?
1.
2.
3.
4.
5.

The aortic semilunar valve opens.


Blood enters the coronary arteries.
The pulmonary semilunar valve opens.
The left atrioventricular valve closes.
The right atrioventricular valvecloses.

\'
t.

~i

:1
i
i
!

NATIONAL

BOARD DENTAL
A~~SWE

ANATOMIC

EXAMINATION

PART

KEY

SCIENCES

1982

JULY

"

NO.

ANS.

NO.

ANS.

NO.

ANS.

NO.

ANS.

1.
2.
3.
4.
5.

1
3
3
4
1

26.
27.
28.
29.
30.

3
2
2
4
5

51.
52.
53.
54.
55.

2
3
3
1
4

76.
77.
78.
79.
80.

1
1
1
1
3

6.
7.
8.
aj .

4
1
2
5

1 (',) .

31.
32.
33.
34.
35.

4
5
2
4
3

56.
57.
58.
59.
60.

2
4
2
4
2

81.
82.
83.
84.
85.

2
4
2
3
3

11.
12.
13.
14.
15.

1
3
1
3
3

36.
37.
38.
39.
40.

2
5
1
2
3

61.
62.
63.
64.
65.

5
2
3
1
1

86.
87.
88.
89.
90.

1
1
2
4
2

16.
17.
18.
19.
20.

.1
1
2
1
1

41.
42.
43.
44.
45.

3
1
2
1
3

66.
67.
68.
69.
70.

4
1
3
3
5

91.
92.
93.
94.
95.

1
2
3
2
4

21.
/ --

5
4
1

46.
47.
48.

3
4
2

71.
72.
73.

3
3
3

96.
97.
98.

2
1
5

2
2

49.
50.

4
1

74.
75.

4
5

99.
100.

1
2

"j

L..

."

J....-'
-< .

.;.
'"'.""

L.. . .

.
.

Part!

Biochemistry-Physiology

(12)

July 1982

1.

1.
2.
3.
4.
5.

2.

heparin.
hyaluronic acid.
keratan suIfate.
dermatan suIfate.
chondroitin sulfate.

In sickle cell anemia, a variation in an amino


acid of the hemoglobin is detected. This
substitution of valine for glutamic acid

1.
2.
3.
4.

results in no change in solubility.


results in no change in isoelectric pH.
is a result of a change in DNA coding.
has no noticeable effect on the O2
transport behavior of the erythrocyte.

Oxygen passes from alveolar air into


blood by

1.
2.
3.
4.
5.
3.

7.

The heteropolysaccharide
that prevents
formation of active thrombin is

8.

diffusion.
perfusion.
adsorption.
active transport.
facilitated transport.

The parasympathetic nervous system regulates

1.
2.
3.
4.
5.

heart rate.
venous tone.
arteriolar tone.
renal blood flow.
pu Imonary blood flow.

A fall in blood pressure causes


1.

increased activity of both vasoconstrictor


and cardioinhibitory centers.
2. decreased activity of both vasoconstrictor
and cardioinhibitory centers.
3. increased activity of the vasoconstrictor
center and decreased activity of the
cardioinhibitory center.
4. decreased activity of the vasoconstrictor
center and increased activity of the
cardioinhibitory center.

4.

1.
2.
3.
4.
5.

10.

chronic tetany.
a high blood calcium level.
retention of phosphates by the kidneys.
storage of excess minerals in the bones.
increased irritability of excitable tissues.

elastin.
keratin.
albumin.
collagen.
chondroitin.

A net water loss that resuIts from sweating


causes an increase in

1.
2.
3.
4.

urine formation.
antidiuretic hormone.
glomerular filtration.
tubular excretion of electrolytes.

Biologic oxidative-phosphorylation enzymes

. occur in the

cellprimarilyas

11.

1.
2.
3.
4.

components of the cell wall.


components of the nuclear membrane.
soluble enzymes of the cytoplasm.
enzymes partly in microsomes and
partly in the mitochondria.
5. highly organized multi-enzyme units in
the mitochondria.

6.

The most abundant protein (by weight) in


the human body is

Hyperparathyroidism is marked by

1.
2.
3.
4.
5.

5.

9.

The organic matrix of bone is composed


largely of collagen and

1.
2.
3.
4.

lipids.
citrate.
dextrans.
glycosaminoglycans.

A diet deficient in calcium will result in

12.

1. stimulation of the thyroid gland.


2. increased ability to cross-link fibrin.
3. production of calcitonin and a low
4.

Fasting for five hours leads to

1.
2.
3.
4.

blood calcium level.


production of parathyroid hormone
and bone resorption.

-12-

increased storage fat.


decreased liver glycogen.
elevated blood sugar level.
suppression of gluconeogenesis.

Biochemistry-Physiology

Part I
July 1982

13.

Colloid osmotic
important

pressure

of the blood is

19.

because it

Preganglionic fibers are exclusively

1.
2.
3.
4.
5.

1. aids in blood clotting.


2. nourishes blood cells.
3. prevents loss of erythrocytes from
the blood.

4. prevents excess loss of fluid from


capillaries.

5. prevents entrance of tissue fluid

20.

into capillaries.

14.

1.
2.
3.
4.
5.
15.

Release of glucocorticosteroid
controlled by

1.
2.
3.
4.
5.
16.

automaticity.
long chronaxie.
intercalated disks.
long refractory period.
protoplasmic bridges between fibers.

21.

to increase in magnitude.

4. the critical membrane potential of the

muscle fiber to move to a value nearer


the resting potential.

22.

glycerol.
acetyl CoA.
acetoacetate.
oxaJoacetate.
acetylcholine.

The end-product of glycolysis under


anaerobic conditions is

1.
2.
3.
4.
18.

1.
2.
3.
4.
5.

atmospheric pressure.
elasticity of blood capillaries.
bicarbonate content of the blood.
hemoglobin content of erythrocytes.
differentials in partial pressures of the
gases.

24.

synthesis of prothrombin.
activation of the Stuart factor.
regulation of calcium in the blood.
conversion of fibrinogen to fibrin.
transcriptional control for fibrinogen
synthesis.

Active transport systems generally involve


specific binding molecules that are

1.
2.
3.
4.
5.

lactic acid.
pyruvic acid.
acetoacetic acid.
oxaloacetic acid.

During respiration, carbon dioxide and


oxygen are exchanged at the junction of
blood and alveolar spaces and the junction
of blood and tissue. The actual diffusion of
the gasesis primarily controlled by the

The function of vitamin K is involved


directly with'

1.
2.
3.
4.
5.

23.
17.

Inhibition of acetylcholinesterase at a
neuromuscular junction causes

A common intermediate of metabolism'.of


carbohydrates, fatty acids and amino acids is

1.
2.
3.
4.
5.

intratubular pressure.
osmotic pressure of the blood.
hydrostatic pressure of the blood.
interstitial pressure of the tubules.

1. prolongation of the endplate potential.


2. increased liberation of acetylcholine
from neuronalterminals.
3. the resting potential of the muscle fiber

is

adrenocorticotropic hormone.
preganglionic adrenergic nerves.
postganglionic adrenergic nerves.
preganglionic cholinergic nerves.
postganglionic cholinergic nerves.

somatic.
adrenergic.
sympathetic.
cholinergic.
parasympathetic.

Energy for glomerular filtration is derived


from

1.
2.
3.
4.

Failure of cardiac muscle to exhibit


tetany results from its

(12)

lipids.
protei ns.
carbohydrates.
nucleic acids.
polyphosphates.

Sympathetic stimulation affects carbohydrate


metabolism because

1. peripheral tissues require epinephrine


to take up glucose.

2. sympathetic nerves to the pancreas


3.
4.

5.

regulate insulin release.


insulin cannot act on the liver in the
absence of epinephrine.
epinephrine increases liver
glycogenolysis.
None of the above

..

Part!
July 1982

25.

Biochemistry-Physiology

31.

In the absence of compensatory changes, a


drop in blood pressure results from

1.
2.
3.
4.
5.

vasoconstriction.
increased hematocrit.
increased stroke volume.
increased cardiac output.
decreased venous return.

Certain viruses have been isolated in


crystalline form and have been found to be

1.
2.
3.
4.
5.

27.

nucleotides.
polypeptides.
phospholipids.
scleroprotei ns.
nucleoproteins.

In the presence of a constant heart rate,


changes in blood pressure may be attributed
mainly to alterations in resistance offered by

33.

29.

intensity of the stimulus.


presence 'of a myelin sheath.
nature of the applied stimulus.
cross-sectional area of the axon.

34.

Glycosaminoglycans function as important


structural components of

1.
2.
3.
4.

According to the Henderson-Hasselbalch


equation, the pH of a buffer system depends
on the pK of the weak acid and the

35.

and weak acid.


Normally, the rate of the heart beat in a
human is determined by

36.

the bundle of His.


all cardiac muscle.
the sinoatrial node.
the cervical ganglion.
nerve impulses from the cardiac
center of the medulla.

inhibition of motility.
increase in HClsecretion.
secretion by pancreatic acini.
an increase in the rate of colonic evacuation.

Hydrolysis of thyroglobulin liberates a


number of iodinated compounds. Two are
considered thyroid hormones. These are

1.
2.
3.
4.

-14-

glycogen.
nucleic acids.
hyalu ron idase.
connective tissue.

Stimulation of the sympathetic nervous supply


to the gastrointestinal tract generally causes

1.
2.
3.
4.

acid present.

4. ratio of molar concentrations of salt

1.
2.
3.
4.
5.

The function of hypophyseal portal vessels


is to
drain blood from the adenohypophysis
into the median eminence.
2. supply blood to a small fraction of the
adenohypophysis.
3. carry hypothalamic releasing factors to
the adenohypophysis.
4. carry blood from the adenohypophysis
to the pars neriosa.

1. pK of the salt.
2. molar concentration of weak acid present.
3. molar concentration of salt of the weak

30.

adenine with guanine.


adenine with thymine.
cytosine with thymine.
cytosine with adenine.

1.

Velocity of nerve impulse conduction in


a sensory nerve fiber is related to

1.
2.
3.
4.

mutation.
replication.
translation.
transcription.

One essential feature of the


Watson-Crick-Wilkinsmodel for the double
stranded DNA molecule concerns the
nitrogenous base pairing on complementary
strands. This involves the specific pairing of
1.
2.
3.4.

1. veins.
2. arteries.
3. arterioles.
4. capillaries.

28.

The process by which genetic information


flows from RNA to protein is

1.
2.
3.
4.

32.
26.

(12)

tyrosine and thyronine.


tyrosine and diiodotyrosine.
thyroxin and triiodothyronine.
thyroxin and triiodotyrosine.

Part!
July 1982

37.

FSH exerts its action on

1.
2.
3.
4.
5.
38.

epididymis.
endometrium.
interstitial cells.
germinal epithelium.
juxtaglomerular apparatus.

1.
2.
3.
4.
5.

insulin.
glucagon.
glycogen.
secretin.
thiamine.

44.

45.

solubility only.
molecular weight only.
both solubility and molecular weight.
concentration gradient.
membrane pore size.

46.

in bone.
increased reabsorption of calcium
phosphate from bone.

1. axon reflexes.
2. spinal reflexes.
3. reflexes involving the precentral gyrus

47.

of the hemisphere.

Infusion of hypertonic NaCIsolution will

1. increase extracellular osmolarity only.


2. decrease intracellular osmolarity only.
3. decrease intracellular volume and

48.

increase extracellular volume.

4. increase both intracellular and

-15-

ferritin.
cytochrome.
hemoglobin.
transferrin.
hemosiderin.

Intracellular and interstitial body fluids


have similar
1.
2.
3..
4.

extracellular volumes.
5. decrease,both intracellular and
extracellular volumes.

axon reflexes.
acetylcholine.
spinal reflexes.
muscle metabolites.

Transport of iron in plasma is almost


entirely in the form of

1.
2.
3.
4.
5.

of the hemisphere.

4. reflexes involving the postcentral gyrus

pons.
medulla.
thalamus.
hypothalamus.
cerebral cortex.

Local autoregulation of blood flow in


skeletal muscle is determined chiefly by

1.
2.
3.
4.

A local, cold stimulus applied to' an extremity


but not leading to loweringof body
temperature evokes reflex shivering. This
is principally a result of

isoelectric point of the enzyme.


pH of most rapid reaction rate.
pH at which the enzyme is most soluble.
pH of most rapid denaturation of the
enzyme.

The center of the brain regulating


body temperature is in the

1.
2.
3.
4.
5.

The low serum phosphate level in


hyperparathyroidism is caused by

4.

42.

anaerobic glycolysis.
the tricarboxylic acid cycle.
the reductive fixation of CO2~
the hexose monophosphate shunt.
the Krebs-Henseleit (urea) cycle.

The optimum pH for an enzyme is the

1.
2.
3.
4.

1. decreased absorption of phosphate.


2. increased renal loss of phosphate.
3. increased deposition of calcium phosphate

41.

The final step in the complete metabolism


of fat is carried out by means of

The diffusion coefficient for the transfer of


each gas through the respiratory membrane
depends upon

1.
2.
3.
4.
5.
40.

43.

(12)

Alpha cells in the islands of Langerhans


are the source of

1.
2.
3.
4.
5.
39.

Biochemistry-Physiology

total osmotic pressures.


colloid osmotic pressures.
chloride ion concentrations.
potassium ion concentrations.

Biochemistry-Physiology

Part I
July 1982

49.

55.

Carotid body receptors are stimulated most


effectively by
1. elevated pH.
2. low oxygen tension.
3. high oxygen tension.
4. low carbon dioxide tension.
5. increased arterial blood pressure.

50.

The most important function of hydrochloric


acid in the stomach is

1.
2.
3.
4.
5.

51.

56.

destruction of bacteria.
neutralization of chyme.
activation of pepsinogen.
hydrolysis of dietary protein.
stimulation of pancreatic secretion.

tissue pressu reo


venous pressure.
capillary pressure.
arteriolar pressure.
tissue colloid osmotic pressure.

The amino acid that contributes to the


tertiary structure of a protein by causing
a bend when it occurs in the primary sequence
is

1.
2.
3.
4.
5.

leucine.
alanine.
proline.
tyrosine.
aspartic acid.

The countercurrent theory is used to explain


the functioning of

1.
2.
3.
4.
5.

52.

In relation to its effect upon movement


of water between vascujar and extravascular
space, plasma colloid osmotic pressure acts
in the same direction as

1.
2.
3.
4.
5.

the kidney.
nerve fibers.
muscle fibers.
the cerebral cortex.
the basilar membrane in the cochlea.

57.

1.
2.
3.
4.

absence of a neurilemma.
presence of spinal fluid.
absence of a myelin sheath.
impediment of scar tissue to growth.

58.

The plasma protein subfraction that contains


antibodies found bathing mucous surfaces
(mouth, bronchial passages, small intestine) is

1.
2.
3.
4.
5.

Regeneration of severed axons does not take


place in the central nervous system because
of the

(12)

IgA.
IgO.
IgE.
IgG.
IgM.

Under normal conditions, the resting potential


across a muscle cell membrane is lower than
the equilibrium potential for potassium
because

1. sodium permeability is equal to


potassium permeability.

53.

2. sodium permeability exceeds

The continuous flow of blood in arteries


during diastole is made possible by

1.
2.
3.
4.

potassium permeability.

3. the membrane has a low permeability


to the sodium ion.

the thoracic pump.


relaxation of arterioles.
the pumping action of skeletal muscle.
energy stored in arteries during systole.

4. the membrane has a low permeability


to the chloride ion.

59.
54.

Intrapleural pressure during normal


respiration is

1.
2.
3.
4.
5.

Two enzymes that have been postulated to play


very important roles in calcification are

1.
2.
3.
4.
5.

subatmospheric.
positive during expiration.
positive during inspiration.
zero at the end of expiration.
zero at the end of inspiration.

enolase and phosphorylase.


alkaline phosphatase and catalase.
pyrophosphatase and carbonic anhydrase.
pyrophosphatase and alkaline phosphatase.
carbonic anhydrase and alkaline
.

phosphatase.

-16'.

Part I
July 1982

60.

Biochemistry-Physiology

Free fatty acids are transported in blood


plasma as

1.
2.
3.
4.
5.

66.

sodium salts.
undissociated acids.
a complex with albumin.
a micelle complexed with bile salts.
a major constituent of plasma lipoproteins.

The discharge of impulses in small motor


(fusimotor) neurons innervating muscle
spindles serves to

1.
2.
3.
4.
5.

61.

diabetes.
loss of capacity to oxidize fat.
loss of capacity to store sugar.
increased sensitivityto insulin.

Most bound calcium is transported


as

1.
2.
3.
4.

63.

67.

in blood

calcium phosphate.
a globulin complex.
an albumin complex.
an associate with hemoglobin.

68.

pattern at the membrane.

charges within the cell's cytoplasm.

4. None of the above

69.

in potential spaces.
in the interstitial compartment.
in the intracellular compartment.
in the intravascularcompartment.
equally distributed among all compartments.

The major end-product of carbohydrate


ingestion is
xylose.
glucose.
mannose.
fructose.
galactose.

The rate of fluoride incorporation into


bones of an animal depends upon

1.
2.
3.
4.
5.
6.

In a state of water intoxication, the extra


volume of water is found primarily

1.
2.
3.
4.
5.

Electronegativity in livingcells requires that

3. negative charges outnumber positive

increased secretion of antidiuretic hormone.


'inhibition of the micturition reflex.
increased blood flow into the vasa recta.
increased permeability of sodium in the
proximal tubule and Henle's loop.
5. increased permeability of sodium in the
distal tubule and the collecting duct.

65.

in plasma.
in platelets.
in leukocytes.
in erythrocytes.
equally distributed between plasma
and erythrocytes.

1. a potential for charge separation exists.


2. biologic water assumes a crystalline

The main effect of aldosterone is to cause

1.
2.
3.
4.
5.

The greatest concentration of carbonic


anhydrase is found

1.
2.
3.
4.
5.

1.
2.
3.
4.

64.

inhibit extrafusal muscle contraction.


sustain extrafusal muscle contraction.
protect the muscle frpm excessivestrain
inhibit the discharge of impulses in
large motor fibers.
inhibit the discharge of impulses in
small motor fibers.

Removal of the anterior lobe of the pituitary


gland results in

1.
2.
3.
4.

62.

(12)

70.

age of the animal.


water-mineral ratio of the bone.
rate of bone remodeling or turnover.
amount of ionizable fluoride in the diet
all of the above.
only (2) and (3) above.

In mammals, the last stage of biologic


oxidation is accomplished with

1.
2.
3.
4.
5.

NAO+.
ubiquinone.
flavoprotein.
cytochrome c.
cytochrome oxidase.

Part I
..

Biochemistry-Physiology

(12)

July 1982

71.

Inhibition of lipolysis, stimulation of protein


synthesis and increased entry of glucose into
muscles and adipose tissues are biologic actions
of the hormone

1.
2.
3.
4.
5.

72.

Which of the following are energy-rich


phosphate carriers?
a)
b)
c)
d)
e)

insulin.
cortisol.
glucagon.
epinephrine.
testosterone.

1.
2.
3.
4.
5.

Phosphorylation of some enzymes by ATP


results in conversion of the enzyme from
an active to an inactive form. This is
illustrated by the conversion of
1.
2.
3.

75.

76.

pepsinogento pepsin.
trypsinogen to trypsin.
glycogen synthetase I to
glycogen synthetase D.

In untreated diabetes mellitus,


is related to

1.
2.
3.
4.

74.

a) and (b

a) and (d
b) and (c
c) and (e)
d) and (e)

Which of the following phospholipids


would most likely be found in the outer
lamella of the plasma membrane?
a) Sphingomyelin
b) Phosphatidylcholine
! c) Phosphatidylethanolamine

4. noneof the above.

73.

G lucose-6-phosphate
1,3-diphosphoglyceric acid
Creatine phosphate
3-phosphoglyceric acid
Adenosine monophosphate

1. a) and (b)
2. a) and (c)
3. l b) and (c)
4. Noneof the above

polyuria

the osmotic effect of glucosuria.


decreasedsecretion of antidiuretic
hormone.
increasedactive transport of sodium
by renal tubules.
the direct effect of insulin on tubular
reabsorption of water.

77.

Which of the following describe an


asymmetric model of membrane assembly?
(a) Some membrane proteins may have.
their N-terminal residuespredominately
on one side of the membrane.
(b) The polar head groups of the
phospholipids may be primarily
oriented toward one side of the
membrane.
(c) The membrane has an inverted
orientation.

During passage of an impulse across either


normal muscle or nerve fibers, which of
the following phenomena are exhibited?

1. a) and (b)
2. a) and (c)
3. ! b) and (c)
4. All of the above

(a) An electrical response


(b) A loss of impulse strength
caused by resistance
(c) An increase in consumption
of intracellular glucose
(d) An increase in permeability
of the cell mem brane
(e) An inflow of extracellular
sodium

78.

1. a and (b)
2. a, c) and d
3. a, ~d)and~e~
4. b), (c) and ~d)
5. c) and (e)
6. All of the above

A diet rich in tryptophan offsets a


deficiency of which of the following
vitamins?

1.
2.
3.
4.

Niacin
Thiamine
Riboflavin
Noneof theabove
,.

-18-

:tf!.ir.
':f~.
.~'-.~.;
j

Part I

Biochemistry-Physiology

July 1982

79.

The effect of a vitamin C deficiency on the


developing tooth is primarily on the
calcification of dentin and cementum. This
can best be explained by which of the
following statements?

84.

Protein content of enamel from mature


teeth is approximately what per cent of
enamel weight?

1.
2.
3.
4.
5.

1. This does happen, but the reason is


obscureo

2. Dentin is the most sensitive tissue in


the body to a vitamin C deficiency.

(12)

O.1-1%
5-1OOh
15-20%
25-30%
50-55%

3. Vitamin C influences the formation of


collagen, the organic matrix found in
dentin and cementum.
4. Vitamin C has metabolic interrelations
with other vitamins that have a greater
influence on dentin and cementum than
on enamel.
s. The vascular system is more important
to these tissues than to enamel and, in
an avitaminosisC, the vascular system
is subject to hemorrhage.

85.

Starting from the receptor, which of the


following best represents the arc of the
Hering-Breuer reflex?

1. Lung - vagus - medulla - vagus - bronchial


muscles

2. Carotid body -vagus - medulla - spinal


3.
4.

80.

1.
2.
3.
4.
5.
81.

Adenine
Inosine
Xanthine
Thyroxin
Pyridoxine

86.

Coupling
Allosterism
Modification
Microscopic reversibility

87.

1. Proteins are amino acids.


2. Proteins are polymers of dextrorotatory
3.

4.
5.

83.

monomers.
Proteins include all nitrogen compounds
of tissues.
Proteins are polymers formed by hydrogen
bonding between amino acids.
Proteins are polymers formed by amide
linkages between alpha-carboxyl groups
and alpha-amino groups.

Which of the following is a source of


transferable methyl groups in metabolism?

1.
2.
3.
4.

Valine
Choline
leucine
Isoleucine

88.

Henle's loop
Distal tubule
Proximal tubule
Collecting duct
Bowman's capsule

Which tissue is metabolically inert (not in


dynamic equilibrium with blood and
metabolites) ?

l.
2.
3.
4.
5.

The primary structure of proteins is best


described by which of the following statements?

In which segment of the nephron does


tubular fluid have the highest osmolality?

1.
2.
3.
4.
5.

Reactions that have unfavorable energetics


(i.e., +6GO) in metabolic pathways may be
driven to completion by which of the
following processes?

1.
2.
3.
4.
82.

5.

Which of the following is a vitamin?

cord - bronchial muscles


lung - vagus medulla - spinal cord
respiratory muscles
lung glossopharyngeal nerve - spinal
cord - bronchial muscles
Carotid body glossopharyngeal nerve medulla - spinal cord - respiratory
muscles

Bone
Dentin
Adipose
Connective
None of the above

By which of the following mechanisms does


the presence of fats in the small intestine
influence the rate of gastric emptying?

1. Fat in the small intestine hastens gastric


emptying through activity of the hormone,
enterogastrone.
2. Fat in the small intestine inhibits gastric
emptying through activity of the
hormone, enterogastrone.
3. Fat in the small intestine initiates the
enterogastric reflex that inhibits further
gastric emptying.
4. Fat in the small intestine initiates the
enterogastric reflex that stimulates
further gastric emptying.

Anatomic

Part I
July 1982

11.

1.
2.
3.
4.

12.

2.
3.
4.

14.'

(a)
b)
c)
1 d)
(e)

1.
2.
3.
4.
5.

A network of vascular spacesacts as a


hydrostatic cushion.
The entire submucosa consists of loose'
connective tissue containing compressible
ground substance.
The anterior zone contains spacesfilled
with fat while the posterior zone contains
nests of mucous glands.
The anterior zone consists of a dense
collagenous network while the posterior
zone contains compressible vascular
spaces.

17.

Cricoid' Thyroid
Arytenoid
Epiglottis
Secondtrachealcartilage

18.

The greater petrosal nerve contains which


of the following components?
(a)
(b)
C)
d)
1 e)'

1.
2.
3.
4.

Pars nervosa
Pars intermedia
Infundibular
stalk
Pars distalis of the hypophysis
None of the above

1.
2.
3.
4.

a),
b),
b),
c),

(b)
(c)
(d)
(d)

1.
2.
3.
4.
5.

and (c)
and (d)
-and (e)
and (e)

6.

-2-

(a) and (e)


(b) and (d)
(c) and (e)
All of the above

a)
b)
c)
d)
e)
f)

Mucous acini only


Relatively long secretory ducts
Mucous acini with serous demilunes
Serous acini only
Long intercalated ducts

Sympathetic fibers
General visceral afferent fibers
General somatic efferent fibers
Parasympathetic fibers
Special visceral efferent fibers

The noncellular
tissue include

The adult parotid gland can be distinguished


by which of the following features?

a)
b)
c)
! d)
(e)

Sphenomandibular ligament
Medial pterygoid muscle
Middle meningeal artery
Ophthalmic nerve
Internal jugular vein

1. a ' b) and (c)


2. a, b) and (d)
3. a, !c) and (e)
4. b), (d) and (e)
5. c), (d) and (e)
6. All of the above

19.
15.

c) and (d
d) and (e

Which of the following structures may be


found in the infratemporalfossa?

Which of th~ following contains the


hypothalamo-hypophyseal tract?

1.
2.
3.
4.
5.

Cell coat
Microfilaments
Reticular fibers
Golgi complex
Sharpey's fibers

a) and (b)
a) and (e)
b) and (C

a)
b)
c)
d)
e)

Which of the following cartilages will be


cut twice in a sagittal section through the
neck?

1.
2.
3.
4.
5.

Which of the following are attached to the


cell membrane?

Maturation of enamel
Formation of the dental sac
Formation of stellate reticulum
Differentation of the stratum intermedium

Which of the following is correct concerning


the structure of the palatal submucosa?
1.

13~

16.

Which of the following occurs during the


early stage of the eruption of a tooth?

Sciences (11)

components

in osseous

lacunae.
canaliculi.
osteocytes.
osteoblasts.
matrix substance.
hematopoietic elements.

a ' b) and (e)

a, b) and (f)
a, ! c) and (e)
b), (d) and (f)
c), (d) and (e)
c), (d) and (f)

,i

i
. :..}
,
f~
"
''>i
~'
". "
"'

""
"

,,,

,"
. :,~~-

,',
, ,

'..~:,

..

Biochemistry-Physiology

Part I
July 1982

89.

Which of the following portions of an ECG


is related to propagation of the cardiac
impulse between the SA node and the AV
node?

1.
2.
3.
4.
5.
90.

91.

96.

Which of the following is NOT a part of


accommodation for near vision?

Which of the following does NOT occur when


anesthetics are applied to a neural membrane?

4. Resting potential drops to a more negative


.

value.

97.

Which of the following is NOT involvedas


a cofactor in formation of acetyl CoA from
pyruvate?

1.
2.
3.
4.
5.

1. Thiamine
2. Riboflavin
3. Tocopherol
4. Calciferol
5. Pantothenic acid

92.

Detoxification
Gluconeogenesis
Formation of plasma protein.
Secretion of digestive enzymes
Regulation of blood sugar level

1. K+ flux remains unchanged.


2. Pores of the membrane become "frozen."
3. There is a Ca++ flux through the membrane.

Biotin
Thiamine
VitaminA
'Vitamin D
Riboflavin

A patient has cheilosis, angular stomatitis,


glossitis and red, itching eyes. The periodontal
surgery that was performed one week ago has
not healed properly. Deficiency of which of
the following vitamins is most likely?

Which of the following is NOT a function


of the liver?

1.
2.
3.
4.
5.

QRScomplex
PRinterval
QSinterval
QR interval
T wave

Which of the following vitamins is necessary


as a coenzyme in the initial steps of fatty
acid synthesis?

1.
2.
3.
4.
5.

95.

(12)

98.

1. Constriction of pupils
2. Convergence of eyeballs
3. Release of visual purple
4. Contraction of ciliary muscle

NAD+
FAD
Pyridoxine
Lipoic acid
Thiamine pyrophosphate

Which of the following statements concerning


collagen is INCORRECT?

1. Collagen has a trihelical structure.


2. The molecu jar weight of collagen
is above 100,000.

93.

3. Hydroxyproline is incorporated into


the molecule by tRNA.

Which of the following is NOT a reflex


mediated by the medulla oblongata?

4. Destruction of collagen can be caused


by collagenases.

1.
2.
3.
4.
5.

5. Collagen contains both hydroxyproline

Blinking
Coughing
Vomiting
Patellar
Swallowing

and hydroxylysine residues.

99.
94.

Whichof the followingagentsisNOT


likelyto be found in plasma?

Which of the following hormones exerts


the least effect on calcium metabolism
of bone tissue?

1.
2.
3.
4.
5.

1.
2.
3.
4.
5.

Thrombin
Fibrinogen
Prothrombin
Calciumion
Ascorbicacid

Androgen
Estrogen
Norepinephrine
Thyroid hormone
Parathyroid hormone

NATIONAL

BOARD

DENTAL EXAMINATION

PART

ANSWER KEY
BIOCHEMISTRY-PHYSIOLOGY

NO.

ANS.

NO.

ANS.

NO.

1.
2.
3.
4.
5.

1
1
3
2
5

26.
27.
28.
29.
30.

5
3
4
4
3

51.
52.
53.
54.
55.

6.
7.
8.
9.
10.

4
3
1
4
2

31.
32.
33.
34.
35.

3
2
3
4
1

11.
12.
13.
14.
IS.

4
2
4
4
1

36.
37.
38.
39.
40.

16.
17.
18.

2
1
r:J

41.

19.
20.

4
3

42.
43.
44.
45.

21.
22.
23.
24.
25.

1
1
2
4
5

46.
47.
48.
49.
50.

..

- JULY

.NS.

1982

NO.

ANS.

1
1
4
1
1

76.
77.
78.
79.
80.

1
1
1
3
5

56.
57.
58.
59.
60.

3
1
3
4
3

81.
82.
83.
84.
85.

I
5
2
1
3

3
4
2
3
2

61.
62.
63.
64.
65.

4
3
5
3

86.
87.
88.
89.

1
5
2
2

L.

90.

""

2
3

66.
67.

2
4

91.
92.

2
3

2
2
4

68.
69.
70.

1
5
5

93.
94.
95.

4
1
4

4
4
1
2
3

71.
72.
73.
74.
75.

1
3
1
3
3

96.
97.
98.
99.

4
3
3
3

Microbiology-Pathology

. Part!
December 1984

1.

Two important "post-streptococcal"


are

1.
2.
3.
4.
5.

diseases

7.

1.
2.
3.
4.

impetigo and osteomyelitis.


puerperal fever and anthrax.
scarlet fever and erysipelas.
pharyngitis and the common cold.
rheumatic fever and glomerulonephritis.

8.
2.

Squamous cell carcinoma usually metastasizes


by way of

1.
2.
3.
4.
3.

1.
2.
3.
4.
5.

4.

a teratoma.
an adenoma.
a carcinoma.
a sarcoma.
a hamartoma.

in adults

mumps.
measles.
nongonococcal urethritis.
ECHO virus meningoencephalitis.
parainfluenzal nasopharyngitis.

\
j

An infectious disease characterized by a


very hard, board-like swelling and draining
fistulas and caused by a branching,
gram-positive, microaerophilic, filamentous
microorganism is

1.
2.
3.
4.

impetigo.
smallpox.
candidiasis.
actinomycosis.

Hemorrhage is often difficult to curb in


patients with liver disorders because of
1.
2.
3.
4.
5.

5.

9.

purpura.
hematomas.
petechiae.
ecchymoses.

Orchitis is a serious complication


with

1.
2.
3.
4.
5.

the venous system.


the arterial system.
the lymphatic system.
aspiration into the lung.

A tumor composed of multiple tissues in


which there may be representatives of all
three embryonal layers is

Small pin-point hemorrhages that occur


on the skin are

hypoprothrombinem ia.
lack of bile pigments.
a deficiency of vitamin B12'
anemia that accompanies liver
dysfunction.
a deficiency of vitamin D caused by
impaired fat absorption.

A thrombus produced as a result of damage


to the ventricular endocardium is
characteristic of
1.
2.
3.
4.

10.

The naturalJeservoir for Rocky Mountain


spotted fever is

1.
2.
3.
4.
5.
6.

11.

vegetations.
mural thrombus.
postmortem clot.
occlusive thrombus.

lice.
mites.
ticks.
humans.
rabbits.'
mosquitoes.

The Lancefield streptococcal group most'


likely to contain strains pathogenic for
man is

1.
2.
3.
4.
5.

A.
B.
D.
E.
H.

'.;

6.

Osteoporosis, metastatic calcification, renal


stones, giant cell granulomas and increased
serum calcium level are manifestations of

1.
2.
3.
4.
5.

12.

The mechanism of action of penicillin is

1.
2.
3.
4.
5.

hypothyroidism.
hyperthyroidism.
hyperadrenalism.
hypoparathyroidism.
hyperparathyroidism.
-22-

prevention of RNA synthesis.


prevention of DNA polymerization.
inhibition of cell wall synthesis.
stimulation of phagocytic activity.
interference with protein synthesis.

13.

Production of a potent exotoxin is


characteristic of certain strains of

1.
2.
3.
4.
5.
14.

19.

1.
2.
3.
4.
5.

Salmonella typhi.
Clostridium tetani.
Hemophilusinfluenzae.
Streptococcus faecalis.
Neisseria meningitidis.

Functions of the complement system in


man include

20.

1. participation in inflammatory processes.


2. amplification of effects of specific antibody.
3. participation in lysis of bacteria and
4.
5.

A benign neoplasm of the myometrium of


the uterus is a

An injection of penicillin into a


penicillin-sensitized person may lead to
death due to

1.
2.
3.

erythrocytes.
all of the above.
none of the above.

4.
15.

an exotoxin.
an endotoxin.
ribonuclease.
a capsular antigen.
a flagellar antigen.

The type of maxillofacial bone graft having


the best chance of success is the

1.
2.
3.
4.

18.

heart and the liver.


brain and the meninges.
thymus and the appendix.
spleen and the lymph nodes.
upper and lower G.!. tract.

21.

~r.olonged administration of streptomycin


may resu It in damage to the

1.
2.
3.
4.

optic nerve.
facial nerve.
auditory nerve.
trigeminal nerve.

Shock during infection with gram-negative


bacteria is most likely caused by

1.
2.
3.
4.
5.

17.

absence of histamine.
a Shwartzman 's reaction.
release of enzymes from polymorphonuclear
leu kocytes.
constriction of bronchioli and drop in
blood pressure.

Most antibodies are produced in the

1.
2.
3.
4.
5.

16.

myeloma.
fibroma.
leiomyoma.
myoblastoma.
rhabdomyoma.

leu kem ia.


thalassem ia.
multiple myeloma.
Hodgkin's disease.
pernicious anemia.
sickle cell disease.

The major antiseptic value of soap lies in


its ability to

1.
2.
3.
4.
23.

isograft.
xenograft.
allograft.
autograft.

The disease characterized by radiolucent


bone lesions, anemia, hyperglobulinemia and
one or more bone marrow tumors containing
predominantly plasma cells is

1.
2.
3.
4.
5.
6.

22.

Dry heat destroys microorganisms primarily


by

1.
2.
3.
4.
5.

24.

kill microbes.
remove miCrobes.
inhibit microbial growth.
dehydrate microbial cells.

lysis.
oxidation.
precipitation of salts.
coagulation of protein.
acceleration of enzyme metabolism.

Tox0ids are most often prepared by treating

toxins with
1.
2.
3.
4.
5.

pepsin.
papain.
trypsin.
antitoxin.
formaldehyde.

25.

Of the following, the most common site of


a basal cell carcinoma is the

1.
2.
3.
4.
5.
26.

27.

1.
2.
3.
4.
28.

32.

Aschoff's giant cells.


Anichkov's myocytes.
fibrocytes in the myocardium.
hypertrophic myocardial fibers.
hemosiderin-laden macrophages in
alveo!i.

A lQ-year-old boy bled during tooth


extraction. Subsequently, he had multiple
bleeding into his joints, especially his
knees. Upon questioning, it was reported
that a maternal uncle and a male cousin
had had similar experiences. The most
probable diagnosis is a deficiency of Factor

VIII.
X.
XII.
XIII.

The most feared consequence of graft


therapy in a patient with an immunodeficiency
IS

33.

Left-sided heart failure or shock may be


associated

etiologically

with
;

1.
2.
3.
4.

i
i
i
i
I
Ii

pneumoconiosis.
bronchiectasis.
pulmonary edema.
pulmonary emboli.

I
i

34.

Aspiration of material from carious teeth


may cause

2.
3.
4.

35.
Hepatitis B surface antigen in a patient's
serum indicates that the patient is
recovered.
potentially infectious.
now an acceptable blood donor.
immune to subsequent exacerbations
of the disease.

DNA molecule
enclosed within a nuclear membrane.
rigid, DNA-protein complex enclosed
within a nuclear membrane.
single, double-stranded DNA molecule
not confined with in a nuclear membrane.
rigid, DNA-lipopolysaccharide complex
not confined within a nuclear membrane.

Mycotic organisms are characteristically


found microscopically within
reticuloendothelial cells in

1.
2.
3.
4.
5.
6.

36.

candidiasis.
aspergillosis.
mucormycosis.
histoplasmosis.
all of the above.
none of the above.

Miliary tuberculosis results from spread


of tubercle bacilli by way of

A number of fungi that commonly cause


superficial skin infections belong to the
genus

1.
2.
3.
4.
5.

1.
2.
3.
4.
5.

lymphatics.
Waldeyer's ring.
the bloodstream.
the urinary system.
bronchial passages. '

-24-

I
I

The nuclear material of a bacterial cell is


typically characterized as a

1. single, double-stranded

1.
2.
3.
4.

30.

leiomyoma.
papilloma.
rhabdomyoma.
leiomyosarcoma.
rhabdomyosarcoma.

1. autoimmunity.
2. immunoproliferation.
3. immune complex injury.
4. host versus graft reaction.
5. graft versus host reaction.

1. lu ng abscess.
2. tu bercu losis.
3. lobar pneumonia.
4. bronchopneumonia.
5. interstitial pneumonia.
29.

A benign tumor arising from voluntary


muscle is a

1.
2.
3.
4.
5.

tongue.
gingiva.
lower lip.
upper face.
oral mucosa.

Heart failure cells are

1.
2.
3.
4.
5.

31.

Aspergillus.
Blastomyces.
Histoplasma.
Coccidioides.
Trichophyton.

II
!
I
.

I
I
I

I
I

!
iI
i

37.

Long-standing chronic

renal insufficiency

44.

may produce

1.
2.
3.
4.
5.

1. thyroid hypoplasia.
2. thyroid hyperplasia.
3. parathyroid hypoplasia.
4. 'parathyroid hyperplasia.

38.

A bacterium well known for its large


polysaccharide capsuleis

1.
2.
3.
4.
5.

39.

45.

Clostridium tetani.
Staphylococcus aureus.
Hemophilus inf/uenzae.
Streptococcus pneumoniae.
Mycobacterium tuberculosis.

The predisposition
sensitization

of an individual
is characteristic of

40.

41.

atopic allergy.
Arthus reaction.
anaphylactic shock.
hyposensitive allergy.

diphtheroids.
staphylococci.
corynebacteria.
streptococcal hypersensitivity.
all of the above.

47.

4.

48.

43.

The type of infection commonly transmitted


by blood and blood-derived products is

humoral immunity.
cellular immunity.
non-specific anatom ic barriers.
none of the above.

49.

are not sporicidal.


are inactivated by soap.
do not kill gram-negative organisms.
have.a limited antimicrobial spectrum.

development of lysogeny.
absence of a demonstrable virus particle.
a slow increase in number of virus
particles.
all of the above.

The genetic ability of a bacterium to


grow in the presence of several antibiotics
is passed in vivo from one bacterium to
another by

1.
2.
3.
4.

cytomegalovirus.
type A hepatitis.
type B hepatitis.
herpes simplex keratitis.

Protection against encapsulated bacterial


pathogens is mediated mainly by

1.
2.
3.
4.

stromal connective tissue.


cells of the surface epithelium.
nuclei of the striated muscle cells.
cytoplasm of the striated muscle cells.
acinar cells of the glands of the tongue.

The eclipse stage of a viral infection is


characterized by

1.
2.
3.

Peptic ulcer most commonly occurs in the

1.
2.
3.
4.

a systemic Arthus reaction.

Submerging dental instrument~ for 15 minutes


in a cold disinfecting solution is unacceptable
as a sterilizing method because during this
interval cold disinfectants

1.
2.
3.
4.

1. Jejunum.
2. cardia of the stomach.
3. first part of the duodenum.
4. lesser curvature of the stomach.

42.

transplantation immunity.

to

The progression of impetigo in children


involvesstreptococci and

1.
2.
3.
4.
5.

anaphylactic shock.
angioneurotic edema.
cell-mediated immunity.

In amyloidosis of the tongue, the amyloid


is deposited primarily in the

1.
2.
3.
4.
5.

46.
1.
2.
3.
4.

Serum sickness syndrome is an example of

conjugation.
rarefaction.
transduction.
transformation.

B-Iymphocytes recognize specific antigens


by virtue of

1.
2.
3.
4.

C3 receptors.
cytophilic IgA.
membrane-bound immunoglobulin.
receptors for the Fc portion of Ig.

..
50.

The first clinical manifestation of herpes


simplex virus type 1 infection is usually

1.
2.
3.
4.

51.

Iron.
folic acid.
vitaminB12'
pyridoxine.
pantothenic acid.

(a)
(b)
(c)
(d)
(e)
(f)

53.

(a)
(b)
(c)
(d)
(e)
1.
2.
3.
4.
5.

55.

disuse.
pressure.
loss of innervation.
chemical stimulation.
decreased nourishment.
overstimulation with hormones.

(a)
(b)
(c)
(d)
(e)

1.
2.
3.
4.
5.
6.

(a), (b) or (c)


(a), (c) or (d)
(a), (c) or (e)
(b), (d) or (e)
Any of the above

Growth of which of the following neoplasms

(a)
(b)
(c)
(d)
1.
2.
3.
4.
5.
6.

56.

Glioma
Breast carcinoma
Prostatic carcinoma
Renal cell carcinoma

(a) and (b)


(a) and (c)
(a) and (d)
(b) and (c)
(b) and (d)
(c) and (d)

Whichof the following elements are usual


constituents of an inflammatory infiltrate?
(a)
(b)
(c)
(d)
(e)

(a), (b), (c) and (d)


(a), (b), (c) and (e)
(a), (b), (d) and (f)
(a), (c), (e) and (f)
(b), (d), (e) and (f)
(c), (d), (e) and (f)

Malignant bone tumors likely to be encountered


in children or young adults include

infarction of the myocardium.


proliferation of myoc~rdial fibers.
fibrosis of the myocardium.
stenosis of the mitral valve.
conductive changes.

is often influenced by hormones?

Causes of atrophy include

1.
2.
3.
4.
5.
6.

Thrombotic occlusion of a coronary artery


may result in

perleche.
gingivostomatitis.
keratoconjunctivitis.
involvement of the genital tract.

Pernicious anemia is best treated with

1.
2.
3.
4.
5.

52.

54.

Fluids
Emboli
Neutrophils
Macrophages
Multinuclear giant cells

1. (a) and (c) only


2. (a), (c) and (d)
3. (a), (c) and (e)
4. (b), (d) and (e)
5. (b) and (e) only
6. All of the above

57.

Leukocytosis is apt to be present in which


of the following conditions?

myeloma.
metastatic carcinoma.
Ewing's sarcoma.
osteogenic sarcoma.
liposarcoma.

(a)
(b)
(c)
(d)
(e)

(a), (b) and (c)


(a) and (c) only
(b), (c) and (d)
(b) and (d) only
(c) and (d) only
(c), (d) and (e)

1.
2.
3.
4.
5.

-26-

Acute abscess
Agranulocytosis
Osteomyelitis
Leukoplakia
Leukopenia

(a) and (b)


(a) and (c)
(b) and (c)
(c) and (e)
(d) and (e)

58.

Which of the following therapeutic agents


are classedas broad-spectrum antibiotics?
(a)
(b)
(c)
(d)
(e)
1.
2.
3.
4.
5.
6.
7.

59.

Tetracycline
Chloramphenicol
Dihydrostreptomycin
Penicillin
Isoniazid

63.

Salk vaccine
Tetanus toxoid
Sabin's oral vaccine
All of the above, -

Hyperglycemia, glycosuria, hyperlipemia


and ketonuria are characteristic of which
of the following conditions?

1.
2.
3.
4.
5.

Hypothyroidism
Lipoid nephrosis
Diabetes mellitus
Letterer-Siwe disease
Hand-SchUller-Christian

disease

Which of the following reactions require


(a)
(b)
(c)
(d)

1.
2.
3.
4.
5.

65.
pathologic

changes

66.

Mumps
Influenza
Scarlet fever
Herpessimplex
Whooping cough

(a ' (b) and (d)


(a and (c)
(a and (e)
(b), (c) and (e)
(b), (d) and (e)

67.

Gonococcus
Enterococcus
Streptococcus
Staphylococcus

Microorganisms of which of the following


genera are most predominant in the normal
oral flora?

1.
2.
3.
4.
5.

are viral diseases?

Nystatin
Bacitracin
Penicillin
Tetracycline
Griseofulvin

Acute osteomyelitis is most frequently


caused by which of the following
microorganisms?
1.
2.
3.
4.

(a), (b) and (e)


(a), (c) and (e)
(b) and (c)
(b), (d) and (e)
(c) and (d)

(a)
(b)
(c)
(d)
(e)

Which of the following antibiotics is


effective in treating oral candidiasis?

1.
2.
3.
4.
5.

Fatty degeneration
Hydropic degeneration
Autolysis
Infarction
Fatty metamorphosis

Which of the following

1.
2.
3.
4.
5.

64.

Anaphylaxis
Arthus reaction
Contact dermatitis
Erythroblastosis fetalis

Which of the following


are irreversible?

1.
2.
3.
4.
5.

(a), (b) and (c)


(a), (b) and (d)
(a), (c) and (d)
(b), (c) and (d)
All of the above

(a)
(b)
(c)
(d)
(e)

61.

Inactivated microorganisms are used in


the manufacture of which of the following?

1.
2.
3.
4.

(a) and (b) only


(a), (b) and (d)
(a), (c) and (e)
(a) and (e) only
(b), (c) and (d)
(c) and (d) only
All of the above

prior sensitization?

60.

62.

Bacillus
Streptococcus
Lactobacillus
Staphylococcus
Corynebacterium

What is the fastest, safest and most


effective method to sterilize a metal
impression tray with a solder that melts
at a temperature above 175c.?

1.
2.
3.
4.
5.

Filtration
Autoclaving
Dry heat sterilization
Soaking in 2% gluteraldehyde
Ethylene oxide sterilization

..

68.

Herpangina is a pathologic condition


associated with which of the following
viruses?

74.

1.
2.
3.
4.

1. Myxcvirus
2. Adenovirus
3. Herpesvir,us
4. Coxsackie virus

75.
69.

Aerobic microbial metabolism is characterized


by which of the following end-products?

Whichof the following cells are more


abundant in chronic inflammation than in
acute inflammation?
l.
2.
3.
4.
5.

76.

Plasma cells
Eosinophiis
Neutrophils
Normoblasts
Basophils

Osteomalacia
Osteopetrosis
Paget's disease of bone
Osteogenesis imperfecta

Which of the following is a significant effect


of pheochromocytoma?
l.
2.
3.
4.

1. Fatty acids
2. lactic acid
3. Hydrogen and pyruvate
4. Carbon dioxide and water
70.

A patient with which of the following diseases


is predisposed to develop osteosarcoma?

Myxedema
Acromegaly
Glycosuria
Hypertension

A high titer of serum heteroph ile antibodies


is found in patients with which of the
following conditions?

l.
2.
3.
4.
5.

Hepatitis A
Hepatitis B
Actinomycosis
Herpes zoster
Infectious mononucleosis

I
I

I
I

71.

Which of the following is a very serious


complication of an infant delivered of
a mother with gonorrhea?

l.
2.
3.
4.
5.
72.

73.

Following injury, which tissue restores


functional capacity by hypertrophy?

l.
2.
3.
4.
5.

78.

79.

Liver
Adrenal cortex
Cardiac muscle
Peripheral nerve
Skin of the trunk

Exudation
Congestion
Transudation
Proliferation

Renin is found in which of the following

organs?
l.
2.
3.
4.
5.

Viruses
Rickettsiae
Streptococci
Staphylococci

-1

In chronic, granulomatous inflammation,


which of the following processes is most.
likely to predominate?
1.
2.
3.
4.

Mucor
Candida
Brucella
Treponema
Aspergillus

Abscess formation is particularly characteristic


of infections with which of the following
microorganisms?

1.
2.
3.
4.

77.

Toxemia
A perforated palate
Hutchinson's incisors
Congenital gonorrhea
Ophthalmia neonatorum

Which of the following microorganisms is


most likely to be cultured from chronic,
bilateral ulcerations at the corners of the
mouth?

l.
2.
3.
4.
5.

Liver
Kidney
Pancreas
Adrenal gland
Thyroid gland

-28-

80.

Which of the following diseases is caused


by an agent that produces neither exotoxins
nor endotoxins?

86.

1.
2.
3.
4.
5.

1. Cholera
2. Syphilis
3. Gonorrhea
4. Brucellosis
5. Gas gangrene

81.

Which of the following is the most common


skin cancer in man?

87.

1. Malignant melanoma
2. Basal cell carcinoma
3. Squamous cell carcinoma
4. Sebaceous adenocarcinoma
5. Transitional cell carcinoma
82.

Specific chemotherapy is useful for treating


which of the following conditions?

1.
2.
3.
4.
5.

83.

88.

Which of the following agents is most


often associated with fatalities following
influenzal infections?

1.
2.
3.
4.
5.

85.

89.

1.
2.
3.
4.
5.

Anthrax
Syphilis
Influenza
Dysentery
Gas gangrene

Which of the following is an example of


natural passive immunity?

90.

Mitral stenosis
Cardiac cirrhosis
Cardiac tamponade
Constrictive pericarditis
Granulomatous lung disease~.

Which of the following may result from


chronic bronchitis?

1.
2.
3.
4.
5.

91.

Treatment with antibiotics


Injection of gamma globulin
Vaccination with dead organisms
Vaccination with live organisms
Placental transfer of mother's antibody

Which of the following is most likely to


cause a sudden arrest of heart function?

1.
2.
3.
4.
5.

Influenza virus
Streptococcus mitis
Streptococcus pyogenes
Staphylococcus aureus
Hemophilus influenzae

Microorganisms characteristic of which of


the following infections require a specific
receptor site to infect a host?

Which of the following factors determines


whether a patient develops acromegaly or
gigantism?

1.
2.
3.
4.
5.

1. Ascites
.2. Jaundice
3. Hepatomegaly
4. Esophageal varix
84.

Hemoglobinopathies
Lipid storage diseases
Glycogen storage diseases
Uric acid metabolism diseases
Mucopolysaccharide storage diseases

1. Sex
2. Age at onset of the tumor
3. Amount of available calcium
4. Degree of function of the tumor

Measles
Hepatitis
Herpangina
Herpes conjunctivitis
Infectious mononucleosis

Which .of the following sequelae of fatty


nutritional cirrhosis is most likely to
result in exsanguination?

Gaucher's, Niemann-Pick and Tay-Sachs are


genetic diseases of which of the following
metabolic classes?

Cor pulmonale
Increased airway resistance
Metaplasia and dysplasia of respiratory
epithelium
All of the above
Only (1) and (3) above

Which of the following acids is an


intermediate in biosynthesis of the cell
wall?

1.
2.
3.
4.
5.

Uridylic
L-glutamic
Thymidylic
N-acetylmuramic
Alanyl diaminopimelic

Part I
July 1982

Anatomic Sciences (11)

'1
)
'j

.;,

20.

Mineral-poor ectodermal structures in enamel


include

25.

The normaLgingival sulcus is bounded by the

1.
a)
b)
c)
d)

tufts.
lamellae.
spindles.
odontoblastic processes.

2.
3.

1. a) and (b)
2. b and (c) only
3. bt {e}and (d)
4. l c) and (d) only
5. All of the above

21.

1.
2.
3.
4.
5.
6.

26.

a, b,
a, b,
a, b,
a, d,
b), (c)
c), (d,

c and d)
c and
e and
e and ~
d) and (e)
e) and (f)

muscles.

3. palatoglossus and palatopharyngeal


muscles.

4. None of the above

27.

nuclear membrane.
Golgi complex.
central bodies.
chromatin.

The pillars of the fauces are formed by


mucosa and the underlying

1. palatoglossus and styloglossus muscles.


2. palatoglossus and stylopharyngeus

upper lip.
tip of the tongue.
mandibular incisors.
anterior floor of the mouth.
middle portion of the lower lip.
lateral portion of the lower lip.

The diploid number of chromosomes is


perpetuated in somatic cells by a process of

1.
2.
3.
4.

The cell component that is genetically


continuous from one cell generation to
the next is
1.
2.
3.
4.

23.

5.

The submental lymph nodes receive


drainage from the

a)
b)
c)
d)
e)
f)

22.

4.

tooth surface and epithelial


covering of attached gingiva.
tooth surface and epithelial
covering of free gingiva.
epithelial covering of free and
attached gingiva.
free gingivalgroove and
mucogingivaljunction.
tooth surface and gingival
lamina propria.

28.

meiosis.
mitosis.
amitosis.
cytokinesis.

The space between the jaws into which the


teeth erupt is provided by growth at the

1. alveolar processes.
2. mandibular condyles.
3. intermaxillary suture and the mental
symphysis.

The combined layers of stomodeum ectoderm


and foregut endoderm comprise the

4. sphenomaxillary, the frontomaxillary


and the zygomaticomaxillary sutures.

1.
2.

buccopharyngeal membrane.
first pharyngeal arch.
.

3. mandibularprocess.
4. primitivepalate.

29.
24.

The function of the posterior lobe of the


hypophysis involves the storage and release of

DNA is found principally in

1.
2.
3.
4.
5.

a nucleolus.
the nucleus.
the cytoplasm.
the mitochondria.
the cell membrane.

1. thyrotropic hormone and gonadotropic


hormone.
2. thyrotropic hormone and vasopressin
(ADH)~
3. oxytocin and gonadotropic hormone.
4. vasopressin (ADH) and oxytocin.

,.

II

92.

Carcinoma of which of the following tissues


has been associated with ingestion of food
contaminated with Aspergillus?

1.
2.
3.
4.

93.

97.

Which of the following does NOT apply to


pulmonary emphysema?

1.
2.
3.
4.
5.

Lip
Lung
Liver
Colon

May be reversible
Is generally bilateral
Is more common in males
May lead to cardiac failure Is a significant public health problem

An infarct is most frequently characterized


by what type of necrosis?
1. Fatty
2. Caseous
3. Gangrenous
4. Coagulative
5. Liquefactive

98.

1.
2.
3.
4.
5.

;o,~ -",

94.

Which of the following is NOT aproperty


of conjugation?

Which of the following is an acidogenic


genus found in deep dental caries that
often increases significantly in the saliva
during periods of-caries activity?

Requires cell-to-cell contact


Is an energy-dependent process
Depends on the presence of pili
Requires flagella for pair formation
Involves transfer of single-stranded DNA

1. Rothia
2. Candida
3. Actinomyces
4. LactobaciLlus

99.
95.

Which of the following is the most pronounced

1.
2.
3.
4.
5.

effect on the oral microflora of a reduction


in rate of salivary flow?
1. Shift toward a more aerobic microflora
2. Shift toward a more acidogenic
3.

4.

96.

Which of the following is NOT a histologic


criterion of malignant growth?

Aplasia
Anaplasia
Pleomorphism
Hyperchromatism
Abnormal mitosis

microflora
Significant increase in number of oral
bacteria
Significant decrease in number of oral
bacteria

100.
Which of the following renal diseases is
autosomal dominant?

The form of bacterial gene transfer that


is least susceptible to DNase and does
NOT require cell-to-cell contact is

1. Renal adenoma
2. Polycystic kidney
3. Glomerulonephritis
4. Secondary amyloidosis

1.
2.
3.
4.

-30-

transition.
conjugation.
transduction.
transformation.

NATIONAL

BOARD

DENTAL

ANSWER

EXAMINATION

PART

- DECEMBER

1984

KEY

MICROBIOLOGY-PATHOLOGY

No.

Ans.

No.

Ans.

No.

Ans.

-No.

Ans.

1.

2.
3.

3
1
1
2

26.
27.
28.
29.
30.

5
1
1
2
3

51.
52.
53.
54.
55.

3
2
5
3
4

76.
77.
78.
79.
80.

5
3
4
2
2

9.
10.

5
3
1
4
3

31.'
32.
33.
34.
35.

3
5
3
3
4

56.
57.
58.
59.
60.

2
2
1
5
5

81.
82.
83.
84.
85.

2
4
4
4
3

11.
12.
13.
14.
15.

1
3
2
4
4

36.
37.
38.
39.
40.

5
4
4
1
2

61.
62.
63.
64.
65.

1
1
3
1
4

86.
87.
88.
89.
90.

2
2
5
3
4

16.
17.
18.
19.
20.

2
4
3
3
4

41.
42.
43.
44.
45.

3
3
1
5
1

66.
67.
68.
69.
70.

2
2
4
4
1

91.
92.
93.
94.
95.

4
3
4
4
2

21.
22.
23.
24.
25.

3
2
4
5
4

46.
47.
48.
49.
50.

1
2
1
3
2

71.
72.
73.
74.
75.

5
2
4
3
4

96.
97.
98.
99.
100.

2
1
4
1
3

4.
5.
6.

7.
8.

Part I
becember 1984

All test items relating to occlusion refer to a Class 1


canine and molar relationsh ip unless otherwise
specified. Terms such as "normal" or "ideal" are
synonymous with the above definition.

1.

Dental Anatomy

5~

Assuming occlusion and alignment are


normal, the arrows on the sketch below
represent the path taken by

Assuming occlusion and alignment are normal,


the arrow on the sketch below represents
the path taken by a
1.
2.
3.

1.
2.
3.
4.
5.
2.

4.
mesiofacial cusp of a second molar.
mesiolingual cusp of a second- molar.
distofacial cusp of a second molar.
distolingual cusp of a second molar.
mesiolingual cusp of a third molar.

6.

4.

1.

protrusive.
right lateral; working side.
left lateral; working side.
right lateral; non-working side.
left lateral; non-working side.

Assuming occlusion and alignment are normal,


the arrow on the sketch below represents
the path taken by a

mesiofacial cusp of a second molar.


mesiolingual cusp of a second molar.
distofacial cusp of a second molar.
distolingual cusp of a second molar.
mesiofacial cusp of a third molar.

protrusive.
right lateral; working side.
left lateral; working side.
right lateral; non-working side.
left lateral; non-working side.

. Identify the cusp of the tooth in the


intercuspal position and the mandibular
movement indicated by the arrow in the
drawing below.

MAXILLARY
CUSP
(a) Lingual cusp of a second premolar
(b) Facial cusp of a first premolar
(c) Lingual cusp of a first premolar
MOVEMENT
(d) lateral movement (working side)
(e) lateral movement (non-working side)
(f) Protrusive movement

The mandibular movement indicated in the


drawing above is

1.
2.
3.
4.
5.

The mandibular movement indicated in the

2.
3.
4.
5.

7.

1.
2.
3.
4.
5.

facial cusps of premolars and mesiofacial


cusps of molars.
facial cusps of premolars and distofacial
cusps of molars.
lingual cusps of premolars and mesiolingual
cusps of molars.
lingual cusps of premolars and distolingual
cusps of molars.

drawing above is.

The mandibular movement indicated in the


drawing above is

1.
2.
3.
4.
5.

3.

and Occlusion

l.
2.
3.
4.
5.
6.
7.
8.

protrusive.
right lateral; working side.
left lateral; working side.
right lateral; non-working side.
left lateral; non-working side.

-32-

(a) and (e)


(a) and (f)
(b) and (e)
(b) and (f)
(c) and (d)
(c) and (e)
(c) and (f)
None of the above

.
8.

Which of the sketches below best represents a


facial view of a permanent mandibular right
canine?

13.

Mesiolingual grooves may be found on


wh ich of the following permanent teeth?

1.
2.
3.
4.

14.

9.

1.
2.
3.
4.
5.

15.
Mesial of the maxillary
Distal of the maxillary
Mesial of the maxillary
Distal of the maxillary

canine
canine
lateral incisor
lateral incisor

Flattened root
More pronounced lingual fossa
Thinner incisal ridge
Sharper proximal incisal angles

Maxillary central incisor


Mandibular central incisor
Mandibular lateral incisor
Mandibular second premolar
Mandibular first molar
Mandibular second molar

The drawing below represents a cross section


at midroot of which of the following roots?

l.
2.
3.
4.
5.
6.

16.

17.

2.
3.

1.
2.
3.
4.

Maxillary first
Maxillary second
Mandibular first
Mandibular second

Lingual to the maxillary central incisors


Facial to the mandibular central incisors
Lingual to the maxillary can ines
Lingual to the mandibular first molars
Facial to the maxillary second molars
Lingual to the maxillary second molars

In periodontal structures, normal interdental


osseous architecture is influenced to the
greatest extent by which of the following?
1.

Which of the premolars is usually the


smallest?

Mandibular canine
Maxillary canine
Distofacial of maxillary molar
Lingual of maxillary molar
Mesial of mandibular molar
None of the above

In which of the following areas is the


alveolar process the th innest?

l.
2.
3.
4.
5.
6.

On which of the following permanent teeth


is it most difficult to distinguish between
mesial and distal aspects?

1.
2.
3.
4.
5.
6.

12.

Maxillary lateral incisor


Maxillary canine
Maxillary first premolar
Mandibular central incisor
Mandibular first molar

Which of the following features of a


permanent maxillary lateral incisor aids in
distinguishing it from. a mandibular lateral
incisor?
1.
2.
3.
4.

11.

Which of the following permanent teeth


has the greatest variation in crown size and
form?

In the intercuspal position, the mesial cusp


ridge of a permanent mandibular canine
opposes which of the following?

1.
2.
3.
4.

10.

Mandibular second molars


Mandibular first premolars
Maxillary first molars
Maxillary canines

4.

Proximal cementoenainel junctions of


adjacent teeth at the same level
Convex coronal facial contours located
in the gingival thirds
Convex coronal lingual contours located
in the middle thirds
Occlusal tables of posterior teeth being
50-60 per cent of the faciolingual overall
widths of the teeth

ff

18.

Deep concavities in cervical areas occur on


which axial surfaces of permanent maxillary
teeth?

23.

1. Mesial surfaces of canines and of first

1. From the dentinoenameHunction

molars

2. Mesial surfaces of central incisors and


3.
4.

19.

1.
2.
3.
4.

2.

of first premolars
Distal surfaces of second premolars
and of second molars
Mesial surfaces of first premolars and
distal surfaces of first molars

When viewedocclusally, a primary mandibular


second molar closely resembles which of
the Jollowing mandibular teeth?

In mesiodistal cross section, what is the


direction of enamel rods in the cervical
third of the crown of a permanent maxillary
first molar?

3.
4.
5.

24.

Permanent second molar


Permanent first molar
Second premolar
Primaryfirst molar

Generally, contact areas between posterior


teeth are located in which directions from
the center of the proximal surface?

1. Facially and occlusally


2. Facially and gingivally
3. Lingually and gingivally
..4.

20.

Which of the following permanent .teeth is


most likely to have a distal coronal concavity
that may pose special problems in placing
a matrix?

25.

1. Maxillary first premoLar


2. Maxillary second premolar
3. Maxillary first molar
. 4.
5.
6.

21.

outward, they incline apically.


From the dentinoenamel junction
outward, they slope occlusally.
From the dentinoenamel junction
outward, they run horizontally.
They blend smoothly onto the root.
None of the above. Enamel rods are
not evident in this region.

Lingually and occlusally

The transverse groove of the oblique ridge


of a permanent maxillary molar connects
which of the following pits?
,. Distal and distofacial
2. Central and distofacial
3. . Central and facial
4. Central and distal
.,5. Central and mesial
6. Central and lingual

Mandibular first premolar


Mandibular second premolar
Mandibular first molar

Which of the following permanent teeth


has a mesiodistal measurement greater
lingually than facially?

1.
2.
3.
4.
5.

Maxillarycanine
Maxillaryfirst premolar
Maxillaryfirst molar
Mandibular first premolar
Mandibular first molar

26.

The sketch below represents a cross section


at midroot of which of the following
permanent teeth?

viewed from the apex

22.

1.
2.
3.
4.
5.
6.
7.

Which of the following muscles is the prime


mover in effecting a left working movement?

1.
2.
3.
4.

Left lateral pterygoid


Left medial pterygoid
Right lateral pterygoid
Right medial pterygoid

Mandibular left canine


Mandibular right canine
Maxillary left lateral incisor
Maxillary right lateral incisor
Maxillary left first premolar
Maxillary right first premolar
Maxillary left second premolar
I-'
I

-34-

"
_
b
~
:i

.
27.

Which of the following is correct with respect


to the roots of a permanent maxillary first
molar?

32.

Which of the following groups of fibers


of the periodontal ligament is most resistant
to forces along the long axis of a tooth?

1. The palatal root is flattened and concave


2.
3.
4.

28.

on its mesial and distal surfaces.


The distofacial root Is flattened and
concave on its mesial and distal surfaces.
The palatal root is located closer to the
mesial.
The palatal root has a concave lingual
su rface.

1.
2.
3.
4.

33.

Which of the sketches below best represents


an occlusal view of a permanent maxillary
left third molar?

Which of the following rQots of permanent


teeth,is typically very thin mesiodistally,
much wider faciolingually and concave on

both mesial and distal surfaces?

1.
2.
3.
4.
5.
'6.

Distofacial root of a maxillary first molar


lingual root of a maxillary first molar
Mesial root of a mandibular first molar
Distal root of a mandibular first molar
Root of a maxillary central incisor
Root of a mandibular first premolar

1
34.

2.
Which line angle area is most acute when
a permanent maxillary second moiar is
viewed occlusally?

3.

4.
1. Mesiofacial
2. Distofacial
3. Mesiolingual
4. Distolingual

5.

35.

30.

Whichof the following primary teeth differ


most from the permanent teeth that replace
them?

Which of the following teeth have their long'


axes most nearly perpendicular to the
horizontal plane when the teeth are in
maximum intercuspation?

1.
2.
3.
4.

Mandibular incisors
Maxillary and mandibular molars
Maxillary and mandibular canines
Maxillary and mandibular premolars

Jj
.~

;."

-35-

5 .

Mesiofacial cusp of a maxillary first


molar
Mesiolingual cusp of a maxillary second
molar
Lingual cusp of a maxillary second

premolar

Mesiolingual cusp of a maxillary first


molar
Distolingual cusp of a maxillary first
molar

Which of the numbers on the sketch below


represents the habitual rest position?

1 2

1. Molars
2. Canines
3. lateral incisors
4. Central incisors

31.

In lateral excursion, which of the following


cusps passes between the lingual cusps of
a mandibuJar right first molar if the
mandible is moving to the right?

1.
29.

Apical
Oblique
Horizontal
Alveolar crest

..

36.

Which of the following are characteristics


of a permanent mandibular second molar?

39.

(a) Facial and lingual grooves form


right angle's with the central groovc.
(b) It typically has four cusps of
approximately equal size.
(c) The occlusal outline is hexagonal
or pen tagonaL
(d) The roots are closer together than
those of a permanent mandibular
first molar and are inclined more
distally.

1.
2.
3.
4.
5.

37.

(a)
(b)
(c)
(d)
1.
2.
3.
4.
5.
6.

38.

(a)
(b)
(c)
(d)
(e)
(f)
(g)

1.
2.
3.
4.
).

(a), (b) and (d)


(a) and (c)
(b), (c) and (d)
(c) and (d) only
All of the above

Which of the following premolars might


have a central pit or fossa?

Which three mandibular teeth are so aligned


that, when viewed from the occlusal, a
straight line may be drawn that will bisect
all con tact areas?

40.

Maxillary first
Maxillary second
Mandibular first
Mandibular second

Central incisor
lateral incisor.
Canine
First premolar
Secone: premolar
First molar
Second molar

(a), (b) and (c)


(c), (d) and (e)
(d), (c) and (f)
(e), (f) and (g)
Noneof the above combinations

Which of the following anterior teeth are


characterized by having distal contact areas
in a more apical position than mesial contact
areas?
(a)
(b)
(c)
(d)

(a) and (b)


(a) and (cr
(a) and (d)
(b) and (c)
(b) and (d)
(c) and (d)

1.
2.
3.
4.
5.

Maxillary central incisor


Maxillary lateral incisor
Mandibular centra! incisor
Mandibular lateral incisor

.,I
t

(a), (b) and (c)


(a), (b) and (d)
(a), (c) and (d)
(b), (c) and (d)
All of the above

When waxing a hypothetical ideal occlusion,


which of the following factors must be
incorporated into the design?
(a) Relate guiding (non-supporting)
cusps to interproximal areas or
developmental grooves.
(b) Relate supporting cusps to marginal
ridges and fossae.
(c) Decrease supporting cusp heights
as one progresses posteriorly.
(d) Overlap guiding (non-supporting)
cusps facial to mandibular teeth
and lingual to maxillary teeth.

1.
2.
3.
4.
5.
6.
7.

41.

Caries susceptibility is positively correlated


with which of the following areas of teeth?
(a)
(b)
(c)
(d)
(e)

(a) only
(a), (b) and(c)
(a), (c) and (d)
(b) only
(c) only
(c) and (d) only
All of the above

1.
2.

3.
4.
5.
6.

-36-

Cervical thirds
Summits of cusps
Crests of contour
Proximal surfaces
Pits and fissures

(a) and (c)


(a), (d) and (e)
(b), (c) and (d)
(c) and (e)
(d) and (e) only
All of the above

.;;:

.
42.

Mesial and distal contact areas are at


approximately the same level, cervicoincisally
or cervico-occlusally, in which.of the following
permanent teeth?
(a)
(b)
(c)
(d)
(e)

1.
2.
3.
4.
5.
6.

45.

(a)
(b)
(c)
(d)

Maxillary central incisors


Mandibular central incisors
Maxillary canines
Mandibular canines
_Maxillary second premolars

1.
2.
3.
4.

(a) and (b)


(a) and (c)
(b) and (d)
(b) and (e)
(c) and (d)
None of the above

46.

43.

When viewed from the incisal, the tip of the


cusp of a maxillary canine with reference to
the center of the crown is normally

lingual.
facial.
mesial.
distal.

(a) and (c)


(a) and (d)
(b) and (c)
(b) and (d)

Primary teeth differ from permanent teeth


in that primary teeth have
(a)
(b)
(c)
(d)

whiter crowns.
more pronounced cervical ridges.
comparatively larger pulp cavities.
comparatively larger and slimmer
roo ts.
(e) longer crowns compared with their
mesiodistal diameters.

Which of the following occur normally


during the act of swallowing?
(a) Masseter muscles contract.
(b) The suprahyoid group of muscles
relaxes.
(c) Teeth come into occlusal contact.
(d) The tip of the tongue touches the
roof of the mouth.

1.
2.
3.
4.
'5.

1. (a), (b) and (c)


2. (a), (b) and (d)
3. (a), (c) and (d)
4. (b), (c) and (d)
5. All of the above

(a), (b) and (c) only


a), b)-'(c) and (d)
a), b), (c) and (e)
la)J d) and (e)
(c), id) and (e)

44.

Design for restoring a complete and


functional occlusal surface depends on the

47.

When viewed from the mesial or the distal,


the overall facial outline from cusp tip
to root apex of a mandibular canine is

(a) position of the tooth in the arch.


(b) amount and direction of lateral
shift in the working condyle.
(c) contour of the articular eminence.
(d) amoun t of vertical overlap of the
anterior teeth.

1. (a), (b) and (d)


2. (a) and (d) only
3. (b) and (c)
4. (b) and (d) only
5. All of the above

(a) made up of two arcs.


(b) made up of one continuous arc.
(c) different from the outline of a
maxillary canine.
(d) very similar to the outline of a
maxillary canine.

1.
2.
3.
4.
5.

(a) and (c)


(a) and (d)
(b) and (c)
(b) and (d)
(d) only

..

48.

In comparison with a mandibular canine, a


maxillary canine

51.

(a)
(b)
(c)
(d)

is shorter.
has a shorter crown.
has a more pronounced cingulum.
has a less pr'onounced developmental
depression on the mesial of the root.
(e) has a cusp tip more nearly centered
over the root when viewed from
the facial aspect.

1.
2.
3.
4.
5.
6.

(a), (d) and (e)


(a) and (e) only
(b), (c) and (d)
(b), (c) and (e)
(c) only
All of the above

1.
2.
3.
4.

52.

Changes in the shape of the pulp chamber of


a tooth may be related to
(a)
(b)
(c)
(d)
(e)

1. (a), (b), (c) and (d)


2. (a), (b) and (e)
3. (a) and (c) only
4. (a), (c), (d) and (e)
5. (a), (d) and (e) only
6. (b), (c), (d) and (e)

54.

4.
5.

1.
2.
3.
4.

permanent maxillary first molar.


permanent maxillary second premolar.
permanent maxillary lateral incisor.
primary maxillary first molar in a
9-year-old child.

downward only.

downward and forward.

1. central fossa of the mandibular first

3.

If a root apex were forced into the maxillary


sinus during a surgical procedure, it would
probably be that of a

backward.
forward only.

In the intercuspal position, the mesiolingual


cusp of a permanent maxillary first molar
rests in the

2.

50.

. pulp canal.
pulp chamber.
mental foramen.
apical>foramen.
mandibular foramen.

When the incisal edges of anterior teeth are


placed in end-to-end contact, the condyles
of the mandible have moved

1.
2.
3.
4.

age.
sex.
attrition.
dental caries~
restorative dental treatment.

medial to the border.


lateral to the' border.
continuous with the border.
None of the above. Great individual
variation exists.

The radicular pulp is continuous with


tissues of the periapical area via the

1.
2.
3.
4.
5.

53.
49.

The facial surfaces of mandibular molars


compared with the anterior border of the
ascending ramus are located

55.

molar.
central fossa of the mandibular second
molar.
distal triangular fossa of the, mandibular
first molar.
mesial triangular fossa of the mandibular
second molar.
None of the above

The lingual surfaces of mandibular first


and second premolars differ in that, for
the second premolar, the lingual

1.
2.
3.
4.

surface is narrower mesiodistally.


surface has no more than one cusp.
cusp is lower and less developed.
surface is much wider mesiodistally and
the mesiolingual cusp is higher.

,!
1
J
j

56.

In the drawing below, erupted teeth of


maxillary and mandibular left halves of
a mixed dentition are shaded. The
approximate age of the child is

59.

In anesthetizing the mandibular teeth, local


infiltration is most likely to be effective in
the

l.
2.
3.
4.
5.

I~~~~ ~J

l!J!!1

~Aj*i
j~~'

molar area.
canine area.
incisor area.
premolar area.
None of the above. Local infiltration
is equally effective throughout the arch.

60.

""

When in its proper position relative to the


plane of occlusion, the crown of a mandibular
second molar inclines

l.
2.
3.
4.

distally and facially.


mesially and facially.
mesially and lingually.
distally and lingually.

~,~~~,,~~
61.

1.
2.
3.
4.
5.
6.

l.
2.
3.
4.
5.

2 years.
5 years.
6 years.
8 years.
11 years.
12 years.

'.

62.

57.

1.

resorbed only.
deposited only.
resorbed and deposited intermittently.
.
neither resorbed nor deposited. This is
a static period.

63.

58.
,
';.'

When a permanent mandibular canine is


distal to the maxillary canine during full
closure of the mandible, the condition
is regarded as a

1. ClassI occlusion.
2. Class II occlusion.
3. Class III occlusion.

material

in

less than 50 per cen t.


50 per cent.
87 per cent.
93 per cent.
more than 95 per cent.

The maxillary tooth having the greatest


statistical variation in root alignment is the

2.
3.
4.
5.
6.
7.
8.

During eruption of permanent teeth, alveolar


bone is

1.
2.
3.
4.

The percentage of inorganic


fully developed enamel is

central incisor.
lateral incisor.
canine.
first premolar.
second premolar.
first molar.
second molar.
third molar.

A distinguishing characteristic of a maxillary


first premolar that identifies it as right or
left is the

l.
2.
3.
4.
5.

height of the lingual cusp.


mesial inclination of the lingual cusp.
flattened area beneath the contact on
the distal surface.
greater length of the distal cusp ridge
on the facial cusp.
greater length of the mesial cusp ridge
on the lingual cusp.

Part I

Anatomic

Sciences (11)

July 1982

30.

31.

36.

revealsminute openings which represent

Smooth muscle fibers differ from striated


muscle fibers in that the smooth muscle
fibers are

1.
2.
3.
4.
5.

1.
2.
3.
4.

The cribriform plate (alveolar bone proper)


areasof osteoclastic activity.
regions of hematopoietic activity.
resorption sites of the spongiosa.
attachment sites of Sharpey's fibers.
regions for passageof vascular and
nerve elements.

37.

The parotid duct pierces the buccinator


muscle opposite the

1. mandibular first premolar.


2.
3.
4.
5.

maxillary
maxillary
maxillary
maxillary

5.
32.

Intramembranous

1.
2.
3.
4.
5.

bone formation

mitosis of osteocytes.
hypertrophy of osteocytes.
transformation
of osteoclasts to
chondrocytes.
transformation
of osteocytes to
chondrocytes.
transformation
of osteoblasts to

38.

34.

inserts into the coronoid process


of the mandible.
2. lateral to the zygomatic arch and
inserts into the coronoid process
of the mandible.
3. medial to the zygomatic arch and
. inserts into the medial side of the

35.

4.

great auricular nerve.


auriculotemporal nerve.
inferior alveolar nerve.
temporal branches of the facial nerve.

39.
The foramen rotundum

1.
2.
3.
4.

transmits the

40.

parotid nodes.
submentalnodes.
submandibularnodes.
anteriorauricularnodes.
superficialcervicalnodes.

perineurium.
myelinated axons.
nerve cell bodies.
loose connective tissue.

After the tooth is formed, the dental


papilla remains asthe

1.
2.
3.
4.
-4-

angle of the ramus.


lateral to the zygomatic arch and
inserts into the medial side of the
angle of the ramus.

In the spinal cord, white matter consists


mainly of

1.
2.
3.
4.

maxillary artery.
lacrimalartery.
zygomaticnerve.
maxillary nerve.

Lymph from most of the dental and


periodontal tissuesdrains initially
into the

1.
2.
3.
4.
5.

The temporalis muscle originates along the


side of the skull and passes

1. medial to the zygomatic arch and

A major sensory innervation of the


temporomandibular joint is derived from the
1.
2.
3.
4.

metaphysis of developing bone.


resting lines of certain bones.
cartilage of the intervertebral disks.
the nodes of Ranvier of myelinated
nerve fibers.
the muscle fibers of the myocardium.

involves

osteocytes.

33.

demonstrably single nucleated.


incapable of hypertrophic changes.
multinucleated and of indefinite length.

The intercalated disks, if present,


constitute one of the distinguishing
characteristics of

1.
2.
3.
4.

first premolar.
first molar.
second molar.
third molar.

longer.

dentin.
dental sac.
dental pulp.
dental follicle.

64.

In normal Class I interarch arrangement,


wear facets on the lingual surface of a
permanent maxillary canine can be caused
by contact with the mandibular

l.
2.
3.
4.

69.

l.
2.
3.
4.

canine and lateral incisor.


canine and first premolat.
first premolar only.
canine only.
70.

65.

6.
12.
24.
32.

':1

The sketch below represents the

The primary function of the dental pulp is to

l.
2.
3.
4.
5.

66.

!he total number of cingula in each dentition


IS

form dentin.
provide nutrition.
provide sensation.
assure root-end closure.
protect the periodontium.
l.
2.
3.
4.

When proximal surfaces of adjacent teeth


diverge from an area of contact, an embrasure
is formed

l. lingually only.
2. lingually and facially only.
3. lingually and occlusally only.
4. lingually, facially, occlusally and
cervically.

71.

distal view of a maxillary first molar.


mesial view of a maxillary first molar.
mesial view of a maxillary second molar.
distal view of a maxillary third molar.

The permanent teeth that are expected in


the mouth of a 10-year-old child include

1. all succedaneous teeth only.


2. all succedaneous teeth and first-molars.
3. central and lateral incisors, first premolars
and first molars.

4. central and lateral incisors, maxillary


67.

5.
Viewed from the facial, the mesial outline
of a permanent mandibular canine, from
the contact area to the apex of the root, is
1.
2.
3.
4.

convex.
concave.
irregular.
reJativelystraight.

72.

Compared with its permanent successors,


a primary incisor has

l.
2.
3.
4.

68.

Pulp horns most likely to be exposed in a


cavity preparation of a newly erupted
permanent mandibular first molar are the

1.
2.
3.
4.
5.
6.

73.

mesiofacial and mesiolingual.


mesiolingual and distolingual.
distofacial and distolingual.
distofacial and mesiofacial.
distofacial and distal.
mesiofacial and distal.

more rounded incisal angles.


more pronounced marginal ridges.
a shorter root length than crown
length.
a wider mesiodistal diameter than
cervicoincisal length of crown.

The sum of the mesiodistal widths of the


primary molars in anyone quadrant compared
with their permanent successors is

l.
2.
3.
4.
5.

-40-

canines and first molars.


central and lateral incisors, canines and
first premolars.

2-4 mm. less.


1 mm. less.
the same.
1 mm. greater.
2-4 mm. greater.

ri
~:
i.o
~~
Pi
J'
'fl

.
74.

79.

In an ideal intercuspal position, the cusp


tip of a permanent maxillary canine should
contact

1. both the mandibular canine and the


2.
3.
4.
5.

In comparison with a permanent mandibular


first molar, a primary mandibular second
molar

1.
'2.
3.

first premolar.
the mandibular first premolar.
the mandibular lateral incisor.
the mandibular canine.
no other tooth.

4.
5.

75.

A divided pulp canal is most likely to occur

in the

1.
2.
3.
4.
5.

76.

root of a maxillary canine.


root of a mandibular canine.
root of a maxillary central incisor.
lingual root of a maxillary first molar.
distofacial root of a maxillary second
molar.

81.

82.

'",
1;

~j

mesial aspect.
distal aspect.,
facial aspect.
lingual aspect.
dentinoenamel junction.

In the mandibular arch, the greatest lingual


inclination of the crown from its root is
seen in the. permanent

spongy layer.

2. cortical plate.
3. cribriform plate.
4. None of the above

78.

one.
two.
three.
four.
five.

The cervical line on adult teeth has the


greatest depth of curvature toward the
incisal

1.
2.
3.
4.
5.

marginal ridges.
cusp tip of the distofacial cusp.
triangular ridge of the mesiofacial cusp.
None of the above.

The most external layer of the alveolar


process is collectively known as the

,.

At eruption, the number of pulp horns


in a permanent mandibular first molar is
usually

1.
2.
3.
4.
5.

The oblique ridge on a permanent maxillary


first molar is reduced in height in the
center of the occlusal surface and is nearly
level with the

1.
2.
3.
4.

77.

80.

does not have a distal cusp.'


hasa larger distofacial cusp.
has roots positioned facially and
lingually.
has a more prominent distal marginal
ridge.
has a more prominent facial cervical
ridge.

1.
2.
3.
4.
5.

Calcification of a permanent first molar begins

1. at 4 months in utero.
2. at birth.
3. 3-4 months after birth.
4. , 0-' 2 months after birth.
5. 6-7 years after birth.

83.

The immediate side shift (Bennett movement)


influences the

1.
2.
3.
4.

-41-

canIne.
third molar.
first premolar.
central incisor.
lateral incisor.

mesiodistal position of cusps.


faciolingual position of cusps.
position of the central fossa.
depth of the distal fossa.

.'
84.

When the mouth is open, there is more


interocclusal distance anteriorly than
posteriorly because of the

88.

In the intercuspal position, the oblique


ridge of a permanent maxillary first molar
opposes the

1.

1. curve of Spee.
2. shape of the articular eminence.
3. anterior inclination of the teeth.
4. rotary nature of the opening"-closing

interproximal area between permanent


mandibular first and second molars.
developmental groove between the
mesiofacial and distofacial cusps of
the permanent mandibular first molar.
developmental groove between the
distofacial and distal cusps of the
permanent mandibular first molar.
developmental groove between the
mesiolingual and distolingual cusps of
the permanent mandibular first molar.

2.

movement.

3.

4.

85.

89.

i"

The height of contour of the facial surface


of a permanent mandibular first molar is
Ioca ted

apically in the cervical third of the


crown.
2. from the dentinoenamel junction
occlusally in the cervical third of the
crown.
3. in the same direction as in a permanent
tooth in the cervical third, but not in
the incisal or occlusal thirds of the
crown.
4. None of the above. They are diffused
with a gnarled appearance.

1. in the cervical third.


2. at the center of the middle third.
3. at the junction of occlusal and middle
4.

90.

4.

The highest and sharpest cusp on a


primary mandibular first molar is the

""

91.

faciolingually.
lingual to the center of the crown
faciolingually.

The patient who has a true or pseudo-Class


III (Angle) occlusal relationship has

1.
2.
3.
4.

The periodontal ligament in the region of


a permanent mandibular canine tends to
become thinner with age, probably as the
result of

1.
2.
3.
4.

th irds.
at the junction of cervical and middle
th ird s.

1. mesiolingual.
2. distolingual.
3. mesiofacial.
4. distofacial.
5. distal.

The mesial contact area of a permanent


maxillary first molar, when viewed from
the mesial, is usually

1. centered on the crown faciolingually.


2. centered on the crown occlusocervically.
3. facial to the center of the crown

87.

'Enamel rods in a primary tooth extend

1. from the dentinoenamel junction

86.

92.

deposition of cementum and bone.


downgrowths of gingival epithelium.
gingival cysts formed from cell rests.
reversal of function of horizontal and
oblique fibers.

A developmental abnormality characterized


by the presence of fewer than the usual
number of teeth is
1.
2.
3.
4.

-42-

normal vertical and horizontal overlaps


of anterior teeth.
excessive posterior cus"p height.
a steep incisal guidance.
no incisal guidance.

anodontia.
oligodontia.
microdontia.
dens in dente.

i!.

%
""

.
93.

The smallest permanent tooth in the mouth


ISa

1.
2.
3.
4.
5.

94.

I
i

mandibular canine.
mandibular lateral incisor.
mandibular central incisor..

maxillarylateralincisor.

maxillary central incisor.

97.

98.

*
-,:

10
;l

-43-

sensory.
nutritional.
supportive via the fibers.
formative via its cellular elements.
maintenance of the, epithelial attachment.

Of the following permanent teeth,


which is least likely to have two roots?

l.
2.
3.
4.
5.

fossae on Iy.
grooves and embrasures.
fossae and marginal ridges only.
fossae, marginal ridges and embrasures.

a fourth root.
the lingual root.
the distofacial root.
the mesiofacial root.

Each of the following is a function of


the periodontal ligament EXCEPT

l.
2.
3.
4.
5.

an increase in occlusal vertical dimension.


an increase in horizontal overlap.
a decrease in vertical overlap.
all of the above.

In the intercuspal position, facial cusps


of mandibular teeth occlude with

1.
2.
3.
4.

A permanent maxillary first molar


occasionally has a fourth canal that is
most often located in

1.
2.
3.
4.

In the dentulous mouth, placing the


mandible in the retruded path of closure
usually results in

1.
2.
3.
4.

95.

96.

Maxillary canine
Mandibular canine
Maxillary first premolar
Mandibular first premolar
Maxillary second premolar

NATIONAL

BOARD
ANSWER

DENTAL

EXAMINATION

PART

KEY

DENTAL ANATOMY - DECEMBER 1984

Ans.

RQ..... Ans.

No.

Ans.

Ans.

No.
-

4
4
3
5
4

26.
27.
28.
29.
30.

6
4
3
1
1

51.
52.
53.
54.
55.

1
4
4
1
4

76.
77.
78.
79.
80.

1
2
2
5
5

9.
10.

4
8
3
4
2

31.
32.
33.
34.
35.

4
2
2
4
5

56.
57.
58.
59.
60.

4
3
2
3
3

81.
82.
83.
84.
85.

1
3
1
4
2

11.
12.
13.
14.
15.

2
3
2
1
c:
..J

36.
37.
38.
39.
40.

1
6
7
4
2

61.
62.
63.
64.
65.

5
8
2
2
1

86.
87.
88.
89.
90.

3
1
3
1
1

16.
17.
18.
19.
20.

2
1
4
2
3

41.
42.
43.
44.
45.

2
4
3
5
3

66.
67.
68.
69.
70.

4
4
1
2
1

91.
92.
93.
94.
95.

4
2
3
4
3

21.
22.
23.
24.
25.

3
3
1
1
4

46.
47.
48.
49.
50.

2
3
4
4
1

71.

3
4
5
5
2

96.
97.
98.

4
5
1

No.
1.
2.
3.
4.

5.
6.
7.

8.

"

'

72.
73.

74.
75.

Part I
July 1982

41.

The esophageal branches of the right vagus


nerve in the lower thorax are found mainly
on the

1.
2.
3.
4.

42.

44.

1.
2.
3.
4.
5.

45.

48.

49.

50.

51.

1. a ligament connecting the liverto the

52.

inferiorly only.
superiorly only.
anteriorly and superiorly.
posteriorly and inferiorly.
posteriorly and superiorly.

Ependymal cells constitute the tissue that


lines the ventricles of the brain.
lines the ventricles of the heart.
forms a part of the peripheral neuroglia.
covers the nerve cell body in the ganglion.

The apical foramen of a fully developed


permanent tooth is usually lined by

1.
2.
3.
4.

spine.
3. anterior superior iliac spine and the
pubic tubercle.
4. anterior and posterior superior iliac
spines.

The ligamentum arteriosum is

In a double vertical fracture through the


mental foramina, muscle action will cause
the small fragment to move

1.
2.
3.
4.

1. pubic tubercle and the iliac tubercle.


2. symphysis pubis and the inferior iliac

46.

...

In the human kidney, a renal papilla


projects directly into the

1.
2.
3.
4.
5.

sternal angle or just below.


seventh cervical vertebra.
su prasternal notch.
xiphoid process.
first rib.

The inguinal ligament runs between the

soft palate.
buccal gingiva.
.lingualgingiva.
sublingual mucosa.
oral surface of lip.

1. ureter.
2. minor calyx.
3. major calyx.
4. ~enal pyramid.

mesocolon.
lesser omentum.
greater omentum.
gastrosplenic ligament.

The trachea bifurcates into right and left


primary bronchi at the levelof the

The thinnest epithelium of the oral cavity


is found on the.

1.
2.
3.
4.
5.

foramen ovale.
foramen rotundum.
inferior orbital fissure.
superior.orbital fissure.

The common bile duct, the hepatic artery


and the portal vein are found grouped in the

1.
2.
3.
4.

47.

left wall of the esophagus.


right wall of the esophagus.
posterior wall of the esophagus.
None of the above

The motor division of the trigeminal nerve


leavesthe skuII through the

1.
2.
3.
4.

43.

Anatomic Sciences (11)

dentin.
cementum.
epithelial rest cells.
the epithelial diaphragm.

The portion of the hard palate located


directly posterior to the maxillary central
incisors is derived from the

anterior abdominal wall.

2. the remnant of a fetal channel connecting


the right atrium to the left atrium.
3. a fibrous remnant of a fetal channel
connecting the left pulmonary artery
to the aorta.
4. none of the above.

1.
2.
3.
4.
5.

first branchial arch.


second branch ial arch.
median nasal process.
lateral nasal process.
palatal processes of the maxillary
processes.

..

Part I
July 1982

53.

Anatomic

58.

The epithelial root sheath (Hertwig) is the


apical continuation of

1. the inner enamel epithelium only.


2. the outer enamel epithelium only.
3. the inner and outer enamel epithelium.
4. the stellate reticulum and stratum

""",

Pain in the temporomandibular joint is


transmitted by the

1. auriculotemporal nerve in the articular

5.

2.

intermedium.
all four layers.

3.
4.

54.

surfaces of the mandible and temporal


bones.
auriculotemporal nerve in the capsule
and periphery of the disk.
maxiHary branch of the trigeminal in the
retrodiscal pad and capsule.
maxillary branch of the trigeminal in
the meniscus and the retrodiscal pad.

Retraction of the mandible is achieved by

1.
2.
3.

posterior fibers of the temporal is muscle.


medial and lateral pterygoid muscles.
lateral pterygoid and deep portion of
masseter

59.

digastric muscles.

The histologic feature that most readily


distinguishes the aorta is the presenceof

1.
2.
3.
4.

Prickle cells are found in

1.
2.
3.
4.

muscles.

4. lateral pterygoid, geniohyoid and

55.

Sciences (11)

60.

a clearly visible external elastic membrane.


distinct vasa vasorum in the tunica media.
a relatively wide and distinct tunica
adventitia.
a tunica media composed primarily of
elastic membranes.

ski n derm is.


sweat glands.
peripheral nerve tissue.
stratified squamous epithelium.

In the fetal heart, the foramen ovale


connects the

1. pulmonary artery and the aorta.


2. right and the left atria, permitting the
flow of blood from right to left.

3. right and the left atria, permitting the


flow of blood from left to right

4. right and the left ventricles, permitting'


the flow of blood from right to left.

5. right and the left ventricles, permitting


the flow of blood from left to right.

56.

The difference between capillaries and


sinusoids is that capillaries

1.
2.
3.

4.

contain blood whereas sinusoids contain


plasma.
have a constant lumen and a complete
endothelial lining whereas sinusoids
are irregular, tortuous tubes.
have a muscular wall that can contract
and shut off the circulation whereas
sinusoids do not.
are restricted to muscular tissue only
whereas sinusoids are widely distributed
in every organ.

61.

1.
2.
3.
4.
5.

62.
57.

The cross striations


rods represent

1.
2.
3.

4.

characteristic

The structure first formed by the tooth bud


that remains in evidence in the formed
tooth is the

of enamel

Spleen, thymus and lymph nodes are similar


in that they all

1.
2.
3.
4.
5.

Hunter-Schreger bands.
the junctional complex of ameloblasts.
the area where enamel rods branch.
incrementalgrowth pattern.

primary cuticle.
secondary cuticle.
inner enamel epithelium.
cementoenamel junction.
dentinoenamel junction.

filter blood.
contain lymphocytes.
have a medulIa and a cortex.
serveas filters for tissue fluid.
have afferent and efferent lymphatic
vessels.

-61~.

','

'f'

't

'I
~
I

Part I

63.

An endocrine gland of ectodermal origin in


the abdomen is the

1.
2.
3.
4.
5.
64.

69.

liver.
corpus luteum.

medullaof the adrenal.

'

cortical portion of the adrenal.


interstitial cells of the testis.

The lingual artery passes

66.

70.

Histologically, dentin of the root is


distinguished readily from crown dentin
by the presence of

1.
2.
3.
4.
5.

The superior~ orbital fissure is located


between the

1. greater and lesserwings of the sphenoid.


2. greater wing of the sphenoid and the
maxilla.
3. lesserwing of the sphenoid and the
palatine bone.
4. greater wing of the sphenoid and the
pterygoid process of the sphenoid.
5. pterygoid process of the sphenoid and
the maxilla.

71.

Irritability is defined as the

72.

palatini.

pharyngeal constrictor.

1.
2.
3.
4.
5.

1.
2.
3.
4.
5.

mesothelium.
endothelium.
lowcolumnarepithelium.
ciliatedsquamousepithelium.
stratifiedsquamousepithelium.

Reichert's cartilage.
a complete cartilage model.
intramembranous bone development
all of the above.
none of the above.

Cleft lip results from the failure of merging of

1.
2.
3.
4.
5.

74.

'

Development of the body of the mandible


involves
'

1. fibrinogen.
2. blood cells.
3. lymphocytes.
4. plasma colloids.
5. plasma crystalloids.

consistsof

Two muscles that prevent food from entering


the nasopharynx are the

4. musculus uvulae and the inferior

Serum is blood plasma without

The lining of the terminal bronchiole

odontoblasts.
mantle dentin.
reparative dentin.
contour lines (Owen).
granular layer (Tomes).

1. styloglossus and the palatoglossus.


.2. tensor tympani and the stylopharyngeus.
3. tensor veti palatini and the levator veli

73.

68.

.'

spleen.
pineal gland.
mammary glands.
corpus albicans. .
pancreatic acini.

'

1. ability of a cell to contract when


stimulated.
2. property of protoplasm which enables a
cell to move away from a painful stimulus.
'3. property of a stimulated nerve cell
exhibiting electrical activity.
4. property of protoplasm responsible for
a cell's being sensitiveto a stimulus.

67.

The hypophysis has direct hormonal control


over the

1.
2.
3.
4.
5.

1. deep to the hyoglossus muscle.


2. superficial to the mylohyoid muscle.
3. su'l'erficial to the digastric muscle.
4. between digastric and mylohyoid muscles.

65.

.-

Anatomic Sciences (11)

July 1982

maxillary with mandibular processes.


maxillary processes with each other.
maxillary and medial nasal processes.
medial and lateral nasal processes.
palatine processes with nasal septum.

The articular disk of the temporomandibular


joint consists of

1. an outer fibrous layer and an inner


synovial layer.

2. areolar tissue covered by dense fibrous


3.
4.

connective tissue.
an outer layer of mesothelium and an
inner layer of calcified cartilage.
dense fibrous connective tissue which
may be associated with chondrocytes.

':~'~f
:r~'
.

Part I

Anatomic Sciences (11)

July 1982

75.

Productionof bile takes place in

80.

1. the hepatic duct.


2. the gallbladder.
3. von Kupffer's cell.
4. the common bile duct.

5. none of the above.

76.

The initial epithelial attachment joining


the gingivato the tooth arises directly from

1.
2.
3.
4.

81.
Electron microscopy has revealed that each
cilium is girdled by the cell.membrane and
that it consists of filaments. The typical
arrangement of these structures is

Lymphoid tissue of the spleen has the


function of filtering
1.
2.
3.
4.

1. nine double circumferential and two


single centraUy located microtubules.

oral mucosa.
cervical loop epithelium.
reduced enamel epithelium.
the epithelial root sheath (Hertwig).

lymph only.
blood only.
tissue fluid.
both lymph and blood.

2. nine single circumferential and two

double centrally located microtubules.

3. nine single circumferential and two

82.

single centrally located microtubules.


4. nine double circumferential and two
double centrally located microtubules.

77.

78.

1. hyaline cartilage.
2. typical perichondrium.
3. a thin layer of synovial membrane.
4. a dense layer of coUagenousconnective
tissue.

Sensory loss in the skin overlying the


parotid gland cou Id be caused by damage
to the

1.
2.
3.
4.

83.

great auricular nerve.


greater occipital nerve.
zygomaticotemporal nerve.
transverse cervical nerve.

Endochondral ossification is a procedure


in the development of bone in which

1.
2.
3.
4.

The connective tissue sheath that surrounds


the muscle as a whole and is synonymous
with the gross anatomical deep fascia is the

1.
2.
3.
4.
5.

The articulating surface of the adult


mandibular condyle is covered by

84.

epimysium.
endomysium.
perimysium.
periosteum.
perichondrium.

hyaline cartilage is transforme.d into bone.

calcifiedcartilageis replacedby bone.

osteocytes of perichondrium form bone.


osteons with cartilage lamellae are formed.

The junctional epithelium of the dentogingival


attachment is characterized by

1. orthokeratinization.
2. the presence of rete pegs.
3. hemidesmosomes in the ceUsfacing the
tooth surface.

4. all of the above.


5. none of the above.

79.

The epiploic foramen is bounded anteriorly


by the

1.
2.
3.
4.

free border of the lesser omentum.


free border of the greater omentum.
peritoneum over the inferior vena cava.
peritoneum at the beginningof the
duodenum.
5. peritoneum on the ~audate lobe of the
liver.

85.

One important collateral circulation of


the hepatic portal system is via the

1.
2.
3.
4.

renalveins.
mesentericveins.
esophagealveins.
externaliliacveins.

-8".
.~.

,..

Part I
July 1982

86.

Anatomic Sciences (11)

As it crosses the first rib, the subclavian


vein lies

1.
2.
3.
4.
5.

90.

The photomicrograph
section of

anterior to the anterior scalene muscle.


posterior to the anterior scalene muscle.
between the anterior and middle scalene
muscles.
between the middle and posterior
scalene muscles.
posterior to the posterior scalene
muscle.

-.
-..,.

below is a ground

""'::"'-"
-',

"1JIIf" ,<
87.

~,'"

" '7:~:--,~,

In the photomicrograph
below of a ground
section of a tooth, the large, dark area
represents

~
"

1.
2.
3.
4.

"\

'}

woven bone.
lamellar bone.
primary cementum.
secondary cementum.

. ....

91.

The lateral boundary of the retropharyngeal


(prevertebral) spaceat the level of the
oropharynx is the

1.
2.
3.
4.
5.
l.a
dead tract.
2. sclerotic dentin.
3. decalcified dentin.
4. a contour line (Owen).

carotid sheath.
pterygomandibular
raphe.
medial pterygoid muscle and Hs fascia.
stylopharyngeus muscle 'and its fascia.
middle pharyngeal constrictor and its

fascia.

92.

'

A terminal web delimits the odontoblast from


the odontoblastic process. This aggregation of
microfi laments is found

1. intracellularly at the basal aspectof the


88.

The nasolacrimal duct drains into the

1.
2.
3.
4.

middle meatus.
inferior meatus.
superior meatus.
sphenoethmoidal

2.
3.
4.

recess.

5.

89.

The auditory tube and cavities of the


middle ear are derived from the

1.
2.
3.
4.
5.

first branchial arch.


second branchial arch.
third branchial arch.
first pharyngeal pouch.
second pharyngeal pouch.

93.

cells.
intracellularly inserting into the
junctional complex of the cell.
extracellularly projectin~ into the
predentin (Korffs fibers).
extracellularly connecting the junctional
complexes of adjacent cells.
extracellularly being continuous with
fibers present in the cell-free zone of
Weil.

The primary source of cranial connective


tissue cells is the

1.
2.
3.
4.
5.

ectoderm.
endoderm.
ectomesenchyme.
primary follicle.
None of the above

...

S-ar putea să vă placă și